You are on page 1of 829

CONCEPTUAL

ADVANCED MATHEMATICS
For Secondary Schools Form V

MPL
i

PUBLISHER P. L. MAYOMBYA
CONCEPTUAL
ADVANCED MATHEMATICS

For Secondary Schools

Form V

i
ii
CONCEPTUAL
ADVANCED MATHEMATICS
For Secondary Schools
Form V
Author
Petro L Mayombya

Edited by:
Peter Amos Kasunzu
Petro L Mayombya

iii
Published by: Petro L Mayombya
P.o Box 14
Bagamoyo, Tanzania
Tel: +255765365925
Email: mayombyap@gmail.com

Text Petro L Mayombya


First Edition 2020

ISBN 978 9976 59 707 3


Author: Petro L Mayombya
Typeset: Petro L Mayombya
Pictures: Petro L Mayombya
Art work: Petro L Mayombya
Page layout: Petro L Mayombya
Cover design: Petro L Mayombya

All Rights Reserved®. No part of this publication may be reproduced, stored in


any a retrieval system or transmitted in any form or by any means, electronic,
mechanical, photocopying, recording, scanning or otherwise, except under the
terms of the Copyright and Designs.

iv
Table of Contents
ACKNOWLEDGEMENT ...............................................................................................x
PREFACE...................................................................................................................... xi
CALCULATING DEVICES ...........................................................................................1
1.1 CALCULATOR ....................................................................................................1
1.2 COMPUTER .......................................................................................................15
1.2.1 BASIC COMPUTER OPERATIONS ..........................................................18
1.2.2 COMPONENTS OF A COMPUTER SYSTEM..........................................21
SETS ..............................................................................................................................29
2.1 SPECIFICATION OF SETS ...............................................................................29
2.2 CARDINAL NUMBER OR NUMBER OF ELEMENTS ..................................31
2.3 SUBSET ..............................................................................................................32
2.4 POWER SET .......................................................................................................33
2.5 UNIVERSAL SET ..............................................................................................33
2.6 BASIC OPERATIONS OF SETS .......................................................................33
2.7 VENN DIAGRAM ..............................................................................................38
2.8 REPRESENTATION OF SET BY USING NUMBER LINE ............................40
2.9 THE LAWS OF ALGEBRA IN SETS................................................................41
2.10 SET WORD PROBLEMS .................................................................................50
LOGIC ...........................................................................................................................61
3.1 SENTENCE AND STATEMENT ......................................................................61
3.2 LOGICAL CONNECTIVES ...............................................................................63
3.3 TRUTH TABLES FOR SIMPLE AND COMPOUND STATEMENTS ...........69
3.4 Tautologies, contradictions and equivalent statements .......................................73
3.5 Converse, contrapositive and inverse of a given statements ...............................73
3.6 LAWS OF ALGEBRA OF PROPOSITION .......................................................76
3.7 VALIDITY OF ARGUMENT ............................................................................79
3.8 ELECTRICAL NETWORKS..............................................................................81
COORDINATE GEOMETRY I....................................................................................92
v
4.1 THE DISTANCE BETWEEN TWO POINTS....................................................92
4.2 THE MIDPOINT BETWEEN TWO PINTS.......................................................93
4.3 THE ANGLE BETWEEN TWO LINES ............................................................94
4.4 PARALLEL AND PERPENDICULAR LINES .................................................95
4.5 THE EQUATION OF PERPENDICULAR BISECTOR ....................................97
4.6 THE PERPENDICULAR SHORTEST DISTANCE FROM A POINT TO A
LINE ..........................................................................................................................99
4.7 THE EQUATIONS OF ANGLES BISECTOR ................................................103
4.8 DISTANCE BETWEEN TWO PARALLEL LINES........................................105
4.9 RATIO THEOREM...........................................................................................107
4.10 LOCUS ............................................................................................................112
4.11 CIRCLES .........................................................................................................116
4.11.1 EQUATIONS OF THE CIRCLES ...........................................................116
4.11.2 POINT(S) OF INTERSECTIONS OF A LINE AND A CIRCLE ...........129
4.11.3 TANGENTS .............................................................................................130
4.11.4 INTERSECTION OF TWO CIRCLES ....................................................143
4.11.5 CONCENTRIC CIRCLES .......................................................................147
4.11.6 CIRCLE THROUGH THE POINT OF INTERSECTION OF THE GIVEN
TWO CIRCLES...................................................................................................148
4.11.7 RADICAL AXIS (COMMON CHORD) OF TWO CIRCLES................149
FUNCTIONS ...............................................................................................................175
5.1 IMPORTANT TERMS USED IN FUNCTIONS..............................................175
5.2 GRAPHS OF FUNCTIONS ..............................................................................176
ALGEBRA ..................................................................................................................202
6.1 INDICES AND LOGARITHMS.......................................................................202
6.2 PARTIAL FRACTIONS ...................................................................................209
6.3 SERIES ..............................................................................................................219
6.4 PROOF MATHEMATICAL INDUCTION ......................................................242
6.5 ROOTS OF A POLYNOMIAL FUNCTIONS .................................................274

vi
6.5.1 ROOTS OF QUADRATIC EQUATIONS .................................................275
6.5.2 ROOTS OF CUBIC EQUATIONS ............................................................284
6.6 REMAINDER THEOREM ...............................................................................289
6.7 INEQUALITIES................................................................................................299
6.8 MATRICES .......................................................................................................309
6.9 BINOMIAL THEOREM ...................................................................................317
TRIGONOMETRY .....................................................................................................346
7.1 TRIGONOMETRIC RATIOS ...........................................................................346
7.2 SPECIAL ANGLES ..........................................................................................347
7.3 TRIGONOMETRIC IDENTITIES ...................................................................351
7.4 ELIMINATION OF PARAMETER .................................................................354
7.5 COMPOUND ANGLE FORMULA .................................................................357
7.6 DOUBLE ANGLE FORMULA ........................................................................363
7.7 TRIPLE ANGLE FORMULA...........................................................................366
7.8 HALF ANGLE FORMULA ..............................................................................368
7.9 t-FORMULA .....................................................................................................371
7.10 R-FORMULA..................................................................................................373
7.11 FACTOR FORMULAE ..................................................................................392
7.12 ANGLE OF THE TRIANGLE ........................................................................401
7.13 TRIGONOMETRIC EQUATIONS ................................................................406
7.14 INVERSE OF TRIGONOMETRIC FUNCTIONS .........................................413
7.15 RADIANS AND SMALL ANGLES ..............................................................419
LINEAR PROGRAMMING .......................................................................................448
8.1 Graphical Solutions ...........................................................................................449
8.2 Transportation Problems ...................................................................................456
DIFFERENTIATION ..................................................................................................464
9.1 DERIVATIVE BY FIRST PRINCIPLE ...........................................................465
9.2 POWER RULE ..................................................................................................479
9.3 PRODUCT RULE .............................................................................................481
vii
9.4 QUOTIENT RULE ...........................................................................................483
9.5 CHAIN RULE ...................................................................................................486
9.6 DERIVATIVE OF TRIGONOMETRIC FUNCTIONS ...................................489
9.7 DERIVATIVE OF INVERSE TRIGONOMETRIC FUNCTIONS ..................507
9.8 DERIVATIVE OF HYPERBOLIC FUNCTIONS ...........................................512
9.9 DERIVATIVE OF NATURAL LOGARITM AND EXPONENTIAL
FUNCTIONS ...........................................................................................................523
9.10 DERIVATIVE OF IMPLICITY FUNCTIONS ..............................................530
9.11 HIGHER DERIVATIVE .................................................................................531
9.12 DERIVATIVE OF PARAMETRIC FUNCTIONS .........................................533
9.13 PARTIAL DERIVATIVE ...............................................................................536
9. 14 APPLICATION OF DIFFERENTIATION ....................................................539
INTEGRATION ..........................................................................................................595
10.1 GENERAL FORMULA FOR INTEGRATION .............................................595
10.2 STANDARD INTEGRAL ..............................................................................597
10.3 INTEGRATION BY USING FACTOR FORMULA .....................................608
10.4 INTEGRATION BY CHANGING OF VARIABLES ....................................612
10.5 INTEGRATION OF FUNCTION AND ITS DERIVATIVE .........................613
10.6 INTEGRATION WHICH INVOLVES PARTIAL FRACTIONS ..................618
10.7 INTEGRATION BY USING TRIGONOMETRIC IDENTITIES ..................621
10.8 INTEGRATION BY USING TRIGONOMETRIC INVERSE CONCEPTS .631
10.9 INTEGRATION BY USING HYPERBOLIC INVERSE CONCEPTS .........652
10.10 INTEGRATION BY USING DIFFERENT TECHNIQUES AND
SUBSTITUTION ....................................................................................................666
10.11 INTEGRATION BY PART ..........................................................................683
10.12 INTEGRATION BY REDUCTION FORMULA .........................................704
10.13 APPLICATION OF INTEGRATION ...........................................................714
10.13.1 AREA .....................................................................................................714
10.13.2 LENGTH ................................................................................................727

viii
10.13.3 VOLUME OF SOLIDS OF REVOLUTION .........................................734
ANSWERS .................................................................................................................788
SET ..............................................................................................................................788
LOGIC .........................................................................................................................788
COORDINATE GEOMETRY I..................................................................................789
FUNCTIONS ...............................................................................................................793
ALGEBRA ..................................................................................................................793
TRIGONOMETRY .....................................................................................................797
LINEAR PROGRAMMING .......................................................................................803
DIFFERENTIATION ..................................................................................................804
INTEGRATION ..........................................................................................................809
INDEX.........................................................................................................................814
BIBLIOGRAPHY .......................................................................................................817

ix
ACKNOWLEDGEMENT
I express sincere appreciation to my family most especially my lovely wife, E.David,
my daughter (Naomi), my incomparable mom, N. Kazungu, my beloved sisters,
Salome, Winfrida and Josephine.

I am extremely thankful to my colleagues and mathematician from various schools for


their useful suggestions and ideas, most especially;

1. Mr. Christian Haule (Marian Girls High School)

2. Mr. Peter Amos Kasunzu (Eagles High School)

3. Mr. Solomon & Mr. Zufko (Marian Boys High School)

4. Mr. Didas Boniphase & Mr. Samson (AHMES)

5. Mr. Telemu Majigwa (Feza Boys’ High School)

6. Mr. Musa Kisinza (Thomas More Machrina)

7. Mr. Masebo and Mr. Francis (Tusiime Secondary School)

8. Sir Kiteddy (EKU345 ACADEMY)

9. Mr. Ally Abdallah (ALLAI ACADEMY)

10. Mr. E. Zablon and S. Minule (Barbro Johannson Model Girls Sec. School)

11. Mr. Patrick Clement (Kwemaramba Secondary School)

12. Mr. Majallah, R (Kibaha High School)

13. Mr. Steven Rutabingwa (St. Joseph Cathedral School)

14. Mr. Yessaya Joseph Mnyambi (St. Mary’s Mazinde Juu Secondary School)

15. Mr. kasmiri George (Kisimiri High School)

I am extremely thankful to various students who have given their valuable


suggestions for the preparation of this book, most especially Marian High Schools
particularly form six (2016/17, 2017/18, 2018/19 . I sincerely acknowledge their
great contribution.

I am really very grateful to our Manager, Head of School and other members of the
staff for making the project successful.

Lastly, I thank the Almighty God for enabling the successful completion of this book.

Suggestions for further improvements from the readers will be thankfully


received and will be duly incorporated.
x
PREFACE
This publication is designed to provide accurate and authoritative
information in regard to the subject matter covered. It is sold on the
understanding that the Publisher is not engaged in rendering professional
services. If professional advice or other expert assistance is required, the
services of a competent professional should be sought.

In addition, major effort has been made to make the text suitable for a wider
range of students. Consideration has been made to reduce unnecessary
complexity especially with vivid examples. Particular care has been taken with
introduction to each topic, some sections have been recognized, sub headings
added. It is hoped that these changes will improve the readability while
retaining the vigor and depth of the text.

It is also, my hopeful that, the book will be resourceful to all teachers and other
professionals specialised and interested in Mathematics.

Petro L Mayombya

Tel: +255765365925

E-mail: mayombyap@gmail.com

Bagamoyo, Tanzania

January 2020

xi
Chapter One
CALCULATING DEVICES
Calculating device is an electronic device used to solve mathematical
expression, eg calculator, computer etc.

1.1 CALCULATOR
Calculator is an electronic device used to carry out evaluation of
mathematical expression.

Types of calculators
There are several types of calculators but the ones used mostly are;
(i) Scientific calculators
(ii) Non-scientific calculators
(iii) Programmable calculators
(iv) Non-programmable calculators

Scientific calculator is an electronic device used for computation of


mathematical expression. It is the recommended calculator to be used in
Advanced Level Syllabus.
Keys of scientific calculator
Scientific calculator has two main keys which are;
(a) Unmarked on top keys these are keys which have no label on top, eg
(SHIFT, ALPHA, MODE, ON). Unmarked keys are sometimes
called activation keys, i.e. SHIFT, ALPHA, MODE.

(b) Marked on top keys these are keys which have label on top, eg (all
keys except unmarked keys).

Marked on top keys are categorized into two groups which are key
caps and alternate keys
Key caps these are keys labeled with white colour on top of the key.
Alternate keys these are keys labeled beside the key caps.

Modes of Scientific calculator


Scientific calculator has several modes but the most applicable modes in
these level are as follows
(i) CMPLX-stand for COMPLEX mode
(ii) SD-stand for Statistical Data mode
(iii) EQN-stand for EQUATION mode
(iv) MAT-stand for MATRIX mode
1
(v) VCT-stand for VECTORS mode
(vi) Rad-stand for Radian mode

Note: Fix-stand for Decimal places


Sci-stand for Significant figure

Basic symbols of scientific calculator are;


CALC -mean CALCULATE
 dx -mean INTEGRATE with respect to x
x 1 -mean Reciprocal of a number
a b c -mean to express a number in mixed fraction
d
-mean to express a number in proper fraction.
c
-mean Square root
x 2 -mean any number exponent 2
 -mean power
log -mean Logarithm
In -mean Natural logarithm i.e. log to base e, loge 
, , , -mean degree, minutes, seconds.
hyp -mean hyperbolic function
n
Pr -mean n permutation of r
n
C r -mean n combination of r
DEL -mean DELETE
Ans -mean Answer
 mean equal sign

(A) STATISTICAL DATA


Statistical data are data which can be organized and analyzed in a useful
manner but calculating devices evaluate the following values
(i)  x - sum of values
(ii)  x - sum of square of values
2

(iii) n - number of values (data)


(iv) x - mean (arithmetic mean)
(v)  n - population standard deviation
(vi)  n1 - sample standard deviation

2
Procedures used to evaluate Statistical Data

(a) Express your calculator in Statistical Data (SD) Mode


Mode  SD 
(b) Input data into your calculator x; f  M  where x and f are
class mark and frequency respectively
 x ,  x and n
2
(c) Press SHIFT  S - SUM  in order to determine

(d) Press SHIFT  S - VAR  in order to determine x,  x and S x


NOTE
  x implies  n
 S x implies  n1
 The symbol  has been used as followed by, not implication

Example 1
By using scientific calculator find  x ,  x, n, 
2
n and x
Values x  6 100 14 18 22

Frequency  f  1 3 2 5 4

Answers
 x 33984
2

 x  512
n  15
 n  33.174
x  34.133

Example 2
By using scientific calculator find  x ,  x, n, 
2
n and x of the
following data 2,2,8,8,8,4,10,10,10 .
Answers

3
 x 516
2

 x  62
n9
 n  3.143
x  6.89

Alternative
We can present the data into a table before evaluation
Values x  2 8 4 10

Frequency  f  2 3 1 3

Answers
 x 516
2

 x  62
n9
 n  3.143
x  6.89

Example 3
By using scientific calculator find  x ,  x, n, 
2
n and x
Interval 4 8 9 13 1418 19 23

Frequency  f  5 3 1 2

Note: Determine first the class mark of each interval then evaluate

Interval 4 8 9 13 1418 19 23

Values x  6 11 16 21

Frequency  f  5 3 1 2

4
Answers
 x 1681
2

 x  121
n  11
 n  5.64
x  11

(B) TRIGONOMETRY
Trigonometry is the study of angles with corresponding sides of a right
angled triangle.
Trigonometric ratios are ratios of sides of right angled triangle, these
ratios are divided into two categories
Main trigonometric ratios
opp
(i) sin 
hyp
Adj
(ii) cos 
hyp
opp sin
(iii) tan   or tan  
Adj cos
Reciprocal of main trigonometric ratios
1
(i) cosec 
sin
1
(ii) sec 
cos
1 cos
(iii) cot  or cot 
tan  sin

Inverse of trigonometric functions


Inverse trigonometric functions are functions
like sin 1 x , cos1 x, tan 1 x , cos ec 1 x, sec1 x and cot1 x

5
Relationship between normal and inverse trigonometric functions
1 1 1
(i) sec x  cos  
 x
1 1  1 
(ii) cosec x  sin  
 x
1 1  1 
(iii) cot x  tan  
 x

Procedures used to evaluate trigonometric problems


(a) Normal trigonometric functions are evaluated when your calculator
it’s in normal mode
eg sin 72 , cos185 , tan 90 , sec30 , cos ec45 , cot 60 etc
(b) Inverse trigonometric functions and trigonometric functions with
angle in radian form are evaluated when the calculator it’s in radian
mode
eg sin 1 0.5, cos1 0.866 ,

1 1 1      
tan 1, sec 0.32, cosec 2, sin , cos , tan  etc
3 6 2

Example 4
By using scientific calculator evaluate the following and leave the
answer in three decimal places
(i) sin 24  sec10
(ii) cos 21   tan 7
3

cos43 sin 1 0.2


(iii)
tan 513
2 tan 5  tan 5
(iv)
tan9 
(v) sec1 4  sec1 

Answers
(i) 1.193
(ii) 0.406
(iii) 0.056
(iv) 9.769
(v) 2.565

6
Example 5
By using non programmable scientific calculator evaluate the following;
(i) 47 2835  69  2 radians (Leave your answer in two decimal
places)
(ii) 1011 12   (Leave your answer in two decimal places)
(iii) 24 20  75 radians (Leave your answer in three significant figures)
(iv) 99  811889

Answers
(i) 70.03
(ii) 3.32
(iii) 7.54  101
(iv) 100.4193842

Example 6
1
sin 0.44418
By using scientific calculator evaluate 5 leave the answer in 3
decimal places.
Answer 3.235

Example 7
sin 2
 1  3     
By using scientific calculator evaluate sin    cot1 


 2 
  2   
leave the answer in 3 decimal places.
Answer 1.592

(C) HYPERBOLIC FUNCTIONS


Hyperbolic functions are functions which are defined in terms of
exponential functions.
Note;
All hyperbolic functions are evaluated when the calculator is in it’s
normal mode

7
Example 8
By using scientific calculator evaluate the following and leave the
answer in three decimal places
(a ) cosh 3
(b) sinh 1.5
(c) tanh 2
Answers
(a) 10.068
(b) 2.129
(c) 0.964

Example 9
Evaluate sinh 1 In3  sinh 1  14  by using scientific calculator, leave the
answer into 3 decimal places.
Answer 1.197

Example 10
By using non programmable scientific calculator find the value of
2
 coshIn 23   tan 1203 
 
 
1
correct to 3 decimal places.
 sin 72  cos 0.8192 
Answer 0.022

(D) POLYNOMIAL EQUATIONS


Quadratic equations and polynomial with degree three can be evaluated
by using calculator.
General format are ax 2  bx  c  0 and ax3  bx 2  cx  d  0

Procedure Mode  EQN  Degree


Example 11
By using scientific calculator solve x 2  2 x  3  0
Answer x  3, x  1

Example 12
By using scientific calculator solve x 2  2 x  8  0
Answer x  4, x  2
Example 13
By using scientific calculator solve x 3  2 x 2  x  2  0
Answer x  2, x  1, x  1

8
(E) SIMULTANEOUS EQUATIONS
Simultaneous equations is a system of equations which consist of two or
more equations. The number of unknown depends on the number of
equations.
General format of simultaneous equations
System of Equations with two unknowns
a1 x  b1 y  c1
a2 x  b2 y  c2

System of Equations with three unknowns


a1 x  b1 y  c1 z  d1
a2 x  b2 y  c2 z  d 2
a3 x  b3 y  c3 z  d 3

Procedure Mode  EQN  Unknowns 

Example 14
By using scientific calculator solve the system of equations
x  2y  3
2 x  6 y  15
Answer x  4.8, y  0.9

Example 15
By using scientific calculator solve the system of equations
x  y  2z  4
x  2 y  3z  2
3x  y  2 z  4
Answer x  1, y  1, z  1

Example 16
By using scientific calculator solve the system of equations
x  4 y  3z  3
x  10 y  7 z  13
x  2y  z  3
Answer x  2, y  1, z  3
9
Example 17
By using scientific calculator solve the system of equations
1 4 3
   3
x y z
1 10 7
   13
x y z
1 2 1
  3
x y z
Answer x  1 2 , y  1, z  1 3

(F) LOGARITHMS
Logarithms are categorized into two main types which are
(a) Common logarithms these are logarithms to base ten, eg log10 x
(b) Natural logarithms these are logarithms to base "e" ,
x
eg loge or Inx

Properties of logarithms
1. logc  logc  logc
ab a b

a
2. logc b  logc  logc
a b

an
 n logc
a
3. logc
log a
4. logb 
a

log b
log a
5. loga  1
a

log a
1
6. logb 
a
b
loga
Note; In and log use the same properties but replace the word log by
In

Example 18
In5  sin 1  12 
By using scientific calculator evaluate leave the

log5   cos60 
answer in 3 decimal places.
Answer 0.966
10
Example 19
 
By using scientific calculator evaluate sec1  In 2394
tan   leave the
 log3 3

answer in 3 decimal places.
Answer 1.391

Example 20
2.1In 4  0.2 sin 2 5
By using scientific calculator evaluate leave the
7.1log 3
answer in 3 decimal places.
Answer 0.859

Example 21
By using scientific calculator evaluate log 27  4 cos 6 leave the
e 6
 In 7
answer in 3 decimal places.
Answer 0.586

Example 22
e log 3  log 5
By using scientific calculator evaluate leave the
In 23
answer in 3 decimal places.
Answer 0.838

Example 23
4
log
Evaluate e In 0.5
 51 by using scientific calculator, leave the
sec 5
answer into 3 decimal places.
Answer 0.718

Example 24
2
 tan 1 3.4 
Evaluate   by using scientific calculator, leave the answer
 e 2 log  2 
 e 
into 3 significant figures.
Answer 5.77 103

11
(G) MATRIX
Matrix is the rectangular representation of numbers in rows and columns
In matrix it’s possible to apply scientific calculator to evaluate the
following values
(a) Determinant of a matrix
(b) Inverse of a matrix
(c) Transpose of a matrix
(d) Multiplication of a matrix

Procedures
(i) Express your calculator in Matrix mode Mode  MAT 
(ii) Then press SHIFT  MAT 

Example 25
1 1 2 
  1
Given that A  1  1 1  , find A , A , A , A and A
T 2 3

1 1 1 
 
Answers
A 2
  1 0.5 1.5 
1
 
A   0  0.5 0.5 
1  1 
 0
 1 1 1
 
AT   1  1 1
 2 1 1
 
 4 2 5
 
A2   1 3 2 
 3 1 4
 
11 7 15
 
A3   6 0 7 
 8 6 11
 

12
Example 26
 3  2 2  1 5 0
   
Given that A   0  1 1  and B   3 1 7  , find
  4 6 0   4  2 0
   
A , B and AB
Answers
A  18
B  126
AB  2268
  11 9  14 
 
AB    7  3  7 
 14  14 42 
 

(H) INTEGRATION
Integration mean continuous summation of data
Scientific calculator is capable to integrate definite integral. These are
integration of the form
b

 F ( x)dx where a and b are lower and upper limits respectively


a

Procedure;
b
By using scientific calculator evaluate  F ( x)dx
a

How to input these data  F ( x), a, b


Example

 x sin3x 
3
Evaluate 2
 2 dx by using scientific calculator
2
Answer 0.913639

Example 27
e2

x
2
Evaluate Inxdx by using scientific calculator
1

Answer 224.24

13
Example 28
1
8
1
Evaluate  1 x
0
2
dx by using scientific calculator

Answer 0.124355

Example 29
1
xe x
Evaluate 0.11  x dx by using scientific calculator
Answer 0.58788

Example 30
2

 sec
1
Evaluate x 2 dx by using scientific calculator
2

Answer 40.554

(I) SUMMATION  
This is the discrete summation of data for example
b

 f ( x)  f (a)      f (b)
x a

Example 31
3
By using scientific calculator evaluate  e 1  1  InxInx in 4
x 2
Inx

decimal places.
Answer 14.2639

Example 32
3
By using scientific calculator evaluate  e In
x 1
x
1  x in 3 decimal

places.
Answer 18.923

14
Example 33

 e 
2
By using scientific calculator evaluate
x
 tan x in 3 decimal places.
x 0

Answer 10.480

Example 34

 e  tan 1 x  in 3 decimal
1
x
By using scientific calculator evaluate
x 0

places.
Answer 4.504

(J) PERMUTATION AND COMBINATION


Permutation is the arrangement of objects in a specific order Pr  
n

Combination is the selection of objects in which matter does not matter


n
 
n
C r or  
r
Example 35
By using scientific calculator evaluate P3  C2
7 4

Answer 1260

1.2 COMPUTER
What is a computer?
A Computer is an electronic device that accepts input, processes the data
and instructions, produces output from the processing that is useful and
meaningful and stores the results for future use.

A Computer is a device or set of devices that work under the


control of a stored program, automatically accepts and processes data
to provide information.
Computers process data to create information. Data is a collection of
raw unprocessed facts, figures, and symbols. Information is data that is
organized, meaningful, and useful.

How does a computer know what to do?


(a) It must be given a detailed set of instructions that tell it exactly what
to do.
(b) These instructions are called a computer program, or software.
15
Types of computers
(a) Personal computers (PCs)
 Desktop computers
 Notebook (laptop) computers
 Tablet PCs
(b) Handheld computers
 PDA (personal digital assistant)
 MP3 players
 Cell phones
(c) Mainframe computers
(d) Supercomputers

(a) Personal computer


Designed to meet the computer needs of an individual.
Provides access to a wide variety of computing applications, such as
word processing, photo editing, e-mail, and internet. Examples
1. Desktop computers
2. Notebook (laptop) computers
3. Tablet PCs

Desktop computer Laptop Tablet PCs

(b) Handheld computers


Also called a PDA (Personal Digital Assistant).
A computer that fits into a pocket, runs on batteries, and is used
while holding the unit in your hand.

16
Typically used as an appointment book, address book, calculator,
and notepad.

(c) Supercomputers
Are the largest and fastest of computers, and can process an
enormous volume of data.
Are used for highly calculation-intensive tasks such as problems
including weather forecasting, climate research, oil and gas
exploration, molecular modeling (computing the structures and
properties of chemical compounds) and physical simulations (such
as simulation of airplanes in wind tunnels

(d) Mainframe Computer


Large expensive computer capable of simultaneously processing
data for hundreds or thousands of users.
Used to store, manage, and process large amounts of data that need
to be reliable, secure, and centralized.
The Mainframe computers can be accommodated in large air-
conditioned rooms because of its size

17
1.2.1 BASIC COMPUTER OPERATIONS
Basically, a computer performs five major operations or functions
irrespective of their size and make.
These are:
(i) It accepts data or instructions by way of input
(ii) It stores data
(iii) It can process data as required by the user
(iv) It gives results in the form of output, and
(v) It controls all operations inside a computer.

INPUT
This is the process of entering data and programs into the computer
system.
The data or instructions you type into the computer are called input
Input is any data or instructions entered into the memory of a computer.
It may be data, commands, user response etc. Whatever is input into the
computer, has to be changed into machine language (binary numbers 0s
and 1s) in which computer understand.

18
Data is the raw facts given to the computer. Data is a collection of
unorganized items that can include words, numbers, pictures, sounds,
and video.

Input types
Programs: A program is a series of instructions that tells a computer
how to perform the tasks necessary to process data into information
Commands: A command is an instruction given to a computer
program. These are special codes or key words that the user inputs to
perform a task, like RUN "ACCOUNTS". These can be selected from a
menu of commands like "Open" on the File menu, “Login” in e-mail.
They may also be chosen by clicking on a command button.
User response is an instruction issued by users to the computer by
responding to a question posed by a computer program to the computer's
question, such as choosing OK, YES, or NO or by typing in text, for
example the name of a file.

STORAGE
Data has to be fed into the system before the actual processing starts. It
is because the processing speed of Central Processing Unit (CPU) is so
fast that the data has to be provided to CPU with the same speed.
Therefore the data is first stored in the storage unit for faster access and
processing. This storage unit or the primary storage of the computer
system is designed to do the above functionality. It provides space for
storing data and instructions.
The storage unit performs the following major functions:
All data and instructions are stored here before and after processing.
Intermediate results of processing are also stored here.

19
PROCESSING
Processing refers to the way that a computer manipulates data. A
computer processes data in a device called the Central Processing Unit
(CPU).
The Central Processing Unit (CPU) takes data and instructions from the
memory and makes all sorts of calculations based on the instructions
given and the type of data provided. It is then sent back to the storage
unit.
OUTPUT
The result of the computer processing your input is referred to as output
and also referred to as Information.
An output is the process of producing results from the data for getting
useful information. Again the output is also stored inside the computer
for further processing.

Forms of Output
Computer has two forms of outputs, hardcopy and softcopy
Hardcopy is an information that has been recorded into tangible
medium i.e. you can touch it while a Softcopy is an output that is
displayed on a computer screen or produced in a voice form

CONTROL
The manner how instructions are executed and the above operations are
performed.
Controlling of all operations like input, processing and output are
performed by Control Unit. It takes care of step by step processing of all
operations inside the computer.

20
What makes a computer powerful?
Computers are powerful for a variety of reasons. They operate with
amazing speed, reliability, and accuracy. Computers can store huge
amounts of data and information.
Also, computers allow users to communicate with other users or
computers.
(a) Speed - Most computers carry out billions of operations a single
second. The world's fastest computer can perform trillions of
operations in one second.
(b) Reliability and Consistency - The electronic components in
modern computers are dependable because they have a low failure
rate. The high reliability of components enables the computer to
produce consistent results.
(c) Accuracy - Computers process large amounts of data and generate
error-free results provided the data input is correctly and the
instructions work properly. If data is inaccurate, the resulting
output will be incorrect
(d) Storage - Many computers store enormous amounts of data and
make this data available for processing anytime it is needed.

1.2.2 COMPONENTS OF A COMPUTER SYSTEM


The components of a Computer system fall into two categories;
Software and hardware.
Computer hardware is the term used to describe the physical
components of the system; the system unit (system box), monitor,
keyboard, mouse, etc.
Computer software is the term used to describe the set of instructions
(computer programs) that define the actions the computer performs

21
Computer Software
A personal computer has something installed on it called software
Software is a set of computer programs (instructions) that determine the
computer behavior and the actions it performs. Usually the instructions
are written in such a language that the machine can understand. Such
languages is what is know as programming languages
The most common type of software found on any personal computer is
the Operating System, which provides a user interface and some basic
commands so users can actually do something on a computer.
There are two main classifications of software that all programs fit
under namely an application software and system software.

Application Software: This is a program designed for end-users to


perform a special task. It is a set of computer programs that helps a
person carry out a task. It enables you to perform specific computer
tasks, such as document production, spreadsheet calculations, and
database management.
System software manages the fundamental operations of your computer
These are software programs that control the computer system. System
software includes the Operating systems (OS), BIOS and Device
drivers. It behaves like a firmware, interpreting user commands to be
performed by the machine.

22
COMPUTER HARDWARE
Hardware is everything physically present in a computer: the keyboard,
mouse, the screen, and the case where the actual computer resides all
constitute hardware. The term hardware refers to the computer’s
physical components.

Hardware needs software to work: The term software, or program,


refers to the set of instructions that directs the hardware to accomplish a
task. A personal computer processes data that is digital, which means it
is in one of two states: positive (on), 1 and non-positive (off), 0. This
technology is called binary.

Examples of hardware

Inside the system case, there's even more hardware that makes up a
basic computer. The Central Processing Unit (CPU) resides on a circuit
board called the motherboard and can be loosely described as the
"brains" of the computer
Other chips work with the CPU; for example, to keep track of things,
the computer has memory chips called RAM, which stands for Random
Access Memory. Memory/RAM is an area of a computer that holds data
that is waiting to be processed, stored, or output.

23
PARTS OF A COMPUTER HARDWARE
Computer Hardware is used for four basic functions: input, output,
processing, and storage
(i) An input device is a component, such as a mouse or keyboard, which
allows the user to enter in data and issue command.
(ii) An output device is a component, such as a monitor or printer, which
displays the results of processing data.
(iii)The processing device is used to process data
(iv) A storage device is used to store data either temporarily or permanent.

The Central Processing Unit (CPU)


The faster the processor, the more expensive the computer. Which
processor should you select?

The best-known and most widely used CPU at the moment is the ‘Intel
Pentium’ processor.

Components of a CPU
CPU consist of two parts namely the Control Unit and the
Arithmetic/Logic Unit
The Control Unit
The control Unit controls the whole computer system by performing the
following functions:
(i) Directs and coordinates all operation called for by the program
(ii) Activates the appropriate circuits necessary for inputs and output
devices.
(iii)Causes the entire computer system to operate in an automatic
manner.
24
The figure below shows the two parts of the CPU (CU and ALU).
Memory holds data and instructions temporarily at the time the program
is being executed. The CPU interacts closely with memory, referring to
it for both instructions and data.

The control unit does not execute the instructions itself; rather, it directs
other parts of the system to do so.

The Arithmetic/Logic Unit


The ALU contains the electronic circuitry that executes all arithmetic
operation and Logic operations, which are usually comparing
operations. The ALU is where all the arithmetic and logical operations
are carried out. There are 4 types of arithmetic operations that the CPU
can perform: Addition, Subtraction, Multiplication and Division.

25
The ALU is able to compare numbers, letters, or special characters and
take alternative course of action.
Basically, the logical operations of the CPU test for 3 conditions:
Equal To - The CPU simply compares 2 values and tests for equality.
Greater Than - The CPU will compare 2 values to see if value a is
greater than value b.
Less Than - The CPU compares 2 values to see if value a is less than
value b.
There are complex combinations of logic as well.

Logical Operations

DIFFERENT COMPUTER PACKAGES USED IN CALCULATIONS


What is a computer packages?
A "computer package" is the collection of hardware and software you
need to have a working system.

(a) Microsoft Excel


Microsoft Excel provides a grid interface to organize nearly any
type of information. The power of Excel lies in it's flexibility to
define the layout and structure of the information you want to
manage. Basic tasks require no special training, and Excel allows
you to work with text, numbers, and date information in a relatively
open and unstructured way.

26
Major uses for Excel
Excel is used widely in any financially-related activity. The ability
to create new spreadsheets where users can define custom formulas
to calculate anything from a simple quarterly forecast to a full
corporate annual report makes Excel highly appealing. Excel is
also used widely for common information organization and
tracking like a list of sales leads, project status reports, contact lists,
and invoicing. Finally, Excel is a useful tool for scientific and
statistical analysis with large data sets. Excel's statistical formulas
and graphing can help researches perform variance analysis, chi-
square testing, and chart complex data.

How Excel Works


An Excel document is called a Workbook. A workbook always has
at least one Worksheet. Workseets are the grid where you can store
and calculate data. You can have many worksheets stored inside a
workbook, each with a unique worksheet name.

Worksheets are laid out in columns (vertical) and rows


(horizontal). The intersection of any given row and column is a
cell. Cells are really where you enter any information. A cell will
accept a large amount of text, or you can enter a date, number, or
formula. Each cell can be formatted individually with distinct
border, background color, and font color/size/type.

Excel Formulas
You can create simple and complex formulas in Excel to calculate
just about anything. Inputs to a formula may be other cells, the
results of other formulas, or just straight-forward math (5*2+3).
Excel includes a formula library for calculating things like Net
Present Value (NPV), standard deviation, interest payments over
time, and other common financial and mathematic formulae.
Excel's formula bar includes a feature to help you search for a
formula you need, and also helps you select the appropriate cells in
your workbook to calculate the formula.

27
(b) SPSS
SPSS (Statistical Package for the Social Sciences) is a widely used
program for statistical analysis in social science. It is also used by
market researchers, health researchers, survey companies,
government, education researchers, marketing organizations, data
miners, and others. The original SPSS manual (Nie, Bent & Hull,
1970) has been described as one of "sociology's most influential
books" for allowing ordinary researchers to do their own statistical
analysis.[4] In addition to statistical analysis, data management (case
selection, file reshaping, creating derived data) and data
documentation (a metadata dictionary is stored in the datafile) are
features of the base software.
(c) Matlab
MATLAB (matrix laboratory) is a multi-paradigm numerical
computing environment and proprietary programming language
developed by MathWorks. MATLAB allows matrix manipulations,
plotting of functions and data, implementation of algorithms,
creation of user interfaces, and interfacing with programs written in
other languages, including C, C++, C#,
Although MATLAB is intended primarily for numerical
computing, an optional toolbox uses the MuPAD symbolic engine,
allowing access to symbolic computing abilities. An additional
package, Simulink, adds graphical multi-domain simulation and
model-based design for dynamic and embedded systems.
(d) Maple
Maple is math software that combines the world’s most powerful
math engine with an interface that makes it extremely easy to
analyze, explore, visualize, and solve mathematical problems. With
Maple, you aren’t forced to choose between mathematical power
and usability, making it the ideal tool for both education and
research.
Maple has over 5000 functions covering virtually every area of
mathematics, including calculus, algebra, differential equations,
statistics, linear algebra, geometry, and much more. Here are just a
few of the many areas of mathematics Maple can handle:
(e) Mathematica
Mathematica is an interactive computational software package for
performing numerical, graphic, and algebraic calculations.
Developed and maintained by Wolfram Research, Mathematica
incorporates a high-level programming language that lets you
define your own procedures.

28
Chapter Two
SETS
Definition of a set
A set is a well-defined collection of objects or entities that have a
common property.

Elements of set
The objects of a set are called members or elements of the set. For
example, the set of all colours of the rainbow has 7 elements or
members, namely, violet, indigo, blue, green, yellow, orange and red.

Notation and representation of sets


A set is denoted by a capital letter, such as A, B, C ,   . The elements of
a set are always denoted by lower-case letters, such as a, b, c,   .
If a is an element of the set A , then we write a  A .
If a is not an element of the set A , then we write a  A .

Sets are always described or represented by giving a name of the set and
enclosing its elements in curly brackets   or braces.

2.1 SPECIFICATION OF SETS


Sets can be specified into three (3) ways
(a) Statement form
(b) Roster/Listing method
(c) Builder/Rule method

(A) STATEMENT FORM


Statement form is the form which state the elements of a set in words.
eg. A set of even numbers less than ten
A  Even number less than ten

(B) ROSTER METHOD


Roster method is a way which state elements by listing.
Eg. A set prime numbers greater than 5 and less than 17.
B  7,11,13

29
(C) BUILDER NOTATION
A set with too many elements to list or with as yet unknown elements is
described by the set-builder method.
The set of all real numbers lying between 0 and 2 is given by
A  x : 0  x  2 or A  x | 0  x  2
In this method, usually x , is used to denote a typical element of the set.
The colon (:) or the vertical bar (|) is read as “such that”.

TYPES OF SETS
(a) Null (Empty) set is a set with no elements,   or 

(b) Finite set is a set with countable number of elements


eg A  2,3,5,6,9 , B  a, e, i, o, u

(c) Infinite set is a set with uncountable number of elements


A  1,2,3,4... , B  x : x  

(d) Singleton set is a set of only one element


eg A  2

(e) Equal sets are two or more sets which have the same elements
eg, A  1,2,3, B  3,1,2
Then set A is equal to set B .

(f) Equivalent sets are two or more sets which have the same number of
elements
eg, A  1,2,3,4,5, B  a, e, i, o, u
Since n A  nB  , then set A and set B are equivalent.

(g) Joint (or Overlapping) sets are sets which have at least one common
elements
Eg Consider set A  1,2,3,4, and B  2,4,6, since elements 2,4
are both in set A and set B then they are joint sets.
That is;

30
(h) Disjoint sets are sets which have no common elements.
Eg Consider set A  1,2,3 and B  a, b, c since there is no
element in common then set A and set B are disjoint sets.
That is;

2.2 CARDINAL NUMBER OR NUMBER OF ELEMENTS


Cardinal number is total number of elements contained in a certain set.
Cardinal number is denoted by n( A) or A for the given set A.

Example 1
If A  1,2,3, a, b, c find the number of element.
Solution
n( A)  6

Example 2

If B  x : x  z


, x  7 find n(B )
Solution
B  1,2,3,4,5,6
n( B )  6

31
2.3 SUBSET
Subset is a set in which all its element is contained to the universal set,
or is a set in which all its element is contained in another set.
If A and B are two sets, if every elements of A is an element of B ,
then A is a subset of B .

Types of subsets
There are two types of subset which are;
(i) Proper subset
(ii) Improper subset

(i) Proper subset


Proper subset is a set in which all its elements are contained to
another set.
Proper subset is denoted by 
For example A  B or B  A
(ii) Improper subset
Two sets are said to be improper subset if the elements in each set
are equal, this mean that all element of A are in B and all elements
of set B are in set A .
Proper subset is denoted by 
For example A  B or B  A

Example 3
Determine which set of the following is a subset to the
other, A  1,2,3, a, b, c and B  2, b
Solution
Since all element of set B is contained in set A,
 B A

Propertities of subsets
(a) Empty set is a subset of all sets
(b) Every set is a subset of itself
(c) For equal sets each set is a subset of the other
Note. Formula used to find the number of subset is 2n
where n is a number of elements

32
2.4 POWER SET
Power set is a set of all possible subset of a given set.

Power set of set A is denoted P ( A)

Example 4

Find the power set of A  1,2,3

Solution

 P A   , 1, 2, 3, 1,2, 1,3, 2,3, 1,2,3

2.5 UNIVERSAL SET


All sets under investigation in any one application are assumed to be
subsets of a common larger set known as the Universal set. For
example, if we are talking of counting numbers, or rational numbers, or
irrational numbers, then the universal set is the set of real numbers. If
we are talking about candidates who passed an examination, then the
universal set is the set of candidates who sat for that examination.

Universal set is denoted by  or 

2.6 BASIC OPERATIONS OF SETS


(a) Union of sets 
The union of two sets, A and B , denoted by A  B , is the set of all
elements which belong either A or B , or to both A and B . Thus,
A  B  x | x  A or x  B . In everyday language, the phrase
“either Peter or Paul” means Peter or Paul, but not both Peter and
Paul. However, the word or used in the context of the union of two
sets is the inclusive or , in the sense that, it includes the case when
x belongs to both.
Example 5
If A  1,2,3,4 and A  3,4,5,6, find A  B
Solution
A  B  1,2,3,4,5,6

33
(b) Intersection of sets 
The intersection of two sets, A and B , denoted by A  B , is the set
of all elements which belong to both A and B . Thus,
A  B  x | x  A and x  B .

Example 6
If A  1,2,4 and A  1,3,4,5, find A  B
Solution
A  B  1,4

NOTE: If two sets A and B have no element in common, then their


intersection is a set which does not have a single element and
therefore, it is the empty set. In such a case, we write A  B   .
The two sets are then said to be disjoint.

(c) complement of a set


Recall that, in a particular context, all sets under consideration are
subsets of a well-defined universal set  . The complement of a
set A , denoted by Ac , is the set of all elements which belong to  but
which do not belong to A .

Example 7
If   1,2,3,4,5,6 and A  1,3,5. Find A
Solution
A  2,4,6

Properties of complement of a set



(a)  A  A
(b)    
(c) A  A  
(d) A  A  
(e)    
34
(d) Relative complement or difference of set
The difference between set A and set B , denoted by either A  B
or A \ B , or A  B , is the set of all elements of set A that are not
elements of set B .

Example 8
If A  1,2,3,4,5,6 and B  2,3,5. Find A  B
Solution
A  B  A  B  1,4,6
A  B  1,4,6

(e) Symmetric difference of two sets


Let A and B be two given sets. Symmetric difference between set A
and B are all elements which are either in A or B but not in both.
Symmetric difference of two sets A and B is denoted by
AB or A  B .These two sets are said to be symmetric difference
if and only if AB   A  B   B  A

Example 9
Given A  1,2,4,7 and B  3,4,7,8. Find AB
Solution
AB   A  B   B  A
AB   A  B  B  A
AB  1,2   3,8
 AB  1,2,3,8

(f) Cartesian product of two sets

These are the all ordered pairs a, b  of elements a  A and b B


where A and B are the given sets and is denoted by A B .
In symbolic form A  B  a, b : a  A, b  B

35
NOTE: The elements of the cross product of two sets is a set of all
possible ordered pairs of the form ( a, b) , whose first component a is an
element of the first set and the second component b is an element of the
second set.
If three set are under consideration A, B and C

A  B  C  a, b, c  : a  A, b  B, c  C

Number of elements of A B =No. of elements of A  No. of elements


of B
Alternatively:
By using tree diagram

 A  B  a1  b1 , a1  b2 , a1  b3 , a2  b1 , a2  b2 , a2  b3 , a3  b1 , a3  b2 , a3  b3 

Example 10

If A  1,2,3 and B  a, b, c . Find;

(a) A B
(b) B  A
Solution
(a) A  B  1, a , 1, b, 1, c , 2, a , 2, b, 2, c , 3, a , 3, b, 3, c 

(b) B  A  a,1, a,2, a,3, b,1, b,2, b,3, c,1, c,2, c,3

36
Alternative

(a) A  B  1, a , 1, b, 1, c , 2, a , 2, b, 2, c , 3, a , 3, b, 3, c 

(b) B  A  a,1, a,2, a,3, b,1, b,2, b,3, c,1, c,2, c,3

Example 11

If A  1,2, B  a, b and C  3,4. Find A  B  C

Solution

 A  B  C  1, a,3, 1, a,4, 1, b,3, 1, b,4, 2, a,3, 2, a,4, 2, b,3, 2, b,4

37
2.7 VENN DIAGRAM
A Venn diagram is a pictorial representation of sets. Sets are represented
by areas enclosed in a rectangular region in the plane. The rectangular
region represents the universal set U and each enclosed area represents
one of its subsets.
Example 12
Represent the following in Venn diagram

(a) A B

(b) A  B

(c) A  B
A  B  A  B

38
(d) B  A
B  A  B  A

(e)  A  B 

(f)  A  B

(g) AB
AB   A  B   B  A   A  B  B  A

39
2.8 REPRESENTATION OF SET BY USING NUMBER LINE
SETS INTERVALS
An interval is a set of real numbers. For example, the set of all numbers
x satisfying 0  x  1 is an interval which contains the numbers 0 and 1
as well as all real numbers that lie between them. Other examples of
intervals are the set of all real numbers R , the set of all negative real
numbers, and the empty set.
Notation
Intervals that include the endpoints

The interval of numbers between a and b , including a and b , is denoted


by [ a , b ] or by a  x  b . The two numbers a and b are called the
endpoints of the interval.

Intervals that exclude the endpoints


To indicate that one of the endpoints is to be excluded from the set, the
corresponding square bracket can be either replaced with a parenthesis,
or reversed. Thus, in set builder notation,
(a, b)  x  R | a  x  b
[a, b)  x  R | a  x  b
(a, b]  x  R | a  x  b
[a, b]  x  R | a  x  b
Infinite endpoints
In both styles of notation, one may use an infinite endpoint to indicate
that there is no bound in that direction.
Specifically, one may use a   or b   (or both). For example,
(0,) is the set of all positive real numbers, and (,) is the set of
real numbers.

40
Note
An open interval does not include its endpoints, and is indicated with
parentheses. For example (0,1) means greater than 0 and less than 1 .

A closed interval includes its endpoints, and is denoted with square


brackets. For example [0,1] means greater than or equal to 0 and less
than or equal to 1.
Example 13
Given A  x : x  , x  1 and B  x : x  ,  3  x  4 , using a
number line find;

(a) A  B (b) A  B
Solution

(a) A  B

(b) A  B

2.9 THE LAWS OF ALGEBRA IN SETS


The three basic set operations of union, intersection and
complement defined on Set Operations, obey certain algebraic rules
which are collectively known as “the laws of the algebra of sets” or
simply as “set identities”.

(1) Idempotent law


 A A  A
 A A  A

41
(2) Identity law
 A   
 A   A
 A   A
 A   

(3) Commutative law


 A B  B  A
 A B  B  A
(4) Associative law
 A  B  C    A  B  C
 A  B  C    A  B  C
(5) Distributive law
 A  B  C    A  B   A  C 
 A  B  C    A  B   A  C 
(6) De-Morgan law
  A  B   A  B
  A  B   A  B

(7) Modulus law


 A   A  B  A
 A   A  B  A
(8) Properties of complement of a set
 A  A  
 A  A  

  A  A
   
   
(9) Idempotent law
   
    
    
    
NB: These laws are used in simplification and proving sets.

42
Example 14

By using the basic properties of sets simplify A   A  B 

Solution

A   A  B .......... .......... .......... .......... ..given


 A      A  B .......... .......... .......... .....identity law
A    B .......... .......... .......... ..... ........di stributive law
A   .......... .......... .......... ..... .......... .......ide ntity law
A.......... .......... .......... .......... .......... .......ide ntity law

Example 15
By using the basic properties of sets simplify  A  B   B  A

Solution

 A  B   B  A.......... .......... .......... .......... .......given


 A  B  B  A.......... .......... .......... .......... .....set difference
 A  A  B  B.......... .......... .......... .......... .....associative law
   .......... .......... .......... .......... .... .......... .......... complement law
 .......... .......... .......... .......... ............. .......... .......ide mpotent law

Example 16

By using the basic properties of sets simplify  A  B    A  B 

Solution

 A  B    A  B .......... .......... .......... .......... ...given


 A  B    A  B.......... .......... .......... .......... ...by set difference
 A  B   A  B.......... .......... .......... .......... ...Demorgan law
 A  A  B.......... .......... .......... .......... ............d istributive law
  B.......... .......... .......... .......... ............. .........c omplement law
B.......... .......... .......... .......... ............. .......... .......ide ntity law

43
Example 17


By using the basic properties of sets simplify A   B  C  A 
 

Solution

  
A  B  C  A .......... .......... .......... .......... .......... .......... ...given
 
A  B  C  A.......... .......... .......... .......... .......... .......... ......Demorgan's law
A  B  C   A.......... .......... .......... .......... .......... .......... ......associative law
A  A  B  C .......... .......... .......... .......... .......... .......... ......commutative law
A  A B  C .......... .......... .......... .......... .......... .......... ......associative law
  B  C .......... .......... .......... .......... .......... .......... .......... .....complement law
 .......... .......... .......... .......... .......... .......... .......... ..... .......... .....ident ity law

Example 18

By using the basic properties of sets simplify  A  B   A  B  B

Solution
 A  B    A  B  B.......... .......... .......... .......... .......... ...given
 A  B    A  B   B  B .......... .......... .......... .......... .....distributive law
 A  B    A  B    .......... .......... .......... .......... ..... .........c omplement law
 A  B    A  B .......... .......... .......... .......... ..... .......... .......ide ntity law
 A  B   A  A  B   B.......... .......... .......... .......... ..... .distributive law
B  A  A  A  B   B.......... .......... .......... .......... ..... .commutative law
B   A  A A  B  B .......... .......... .......... .......... ..... .associative law
B  A A  B.......... .......... .......... .......... ..... .......... .......ide mpotent law
A  B A  B.......... .......... .......... .......... ..... .......... .......commutative law
A  B.......... .......... .......... .......... ..... .......... .......... .......... .....idempotent law

44
Example 19

Use the laws of algebra to simplify the following  A  B    A  B 
  
Solutions

 A  B    A  B  .......... .......... .......... .......... .......... ...given
 
 A  B    A  B  .......... .......... .......... .......... .......... ........Demorgan' s law
 A  B    A  B .......... .......... .......... .......... .......... ........set difference
 A  B    A  B .......... .......... .......... .......... .......... ..........De morgan' s law
 A  A  B.......... .......... .......... .......... .......... ............ ......dist ributive law
  B.......... .......... .......... .......... .......... ............ .......... ......complement law
B.......... .......... .......... .......... .......... ............ .......... .......... ..identity law

Example 20
Use the laws of algebra to simplify the following  A  B   B  A
Solutions
 A  B   B  A.......... .......... .......... .......... .......... given
 A  B  B  A.......... .......... .......... .......... ....... .set difference
 A  A  B  B.......... .......... .......... .......... ....... .commutative law
   .......... .......... .......... .......... .......... .......... ...... .complement law
 .......... .......... .......... .......... .......... .......... ...... .......ide mpotent law

Example 21
Use the laws of algebra to simplify the following P  Q  P  Q  P  Q
Solutions
P  Q   P  Q   P  Q .......... .......... .......... .......... ..given
P  Q   P  Q   P  Q .......... .......... .......... .......... .commutative law
P  Q   P  Q .......... .......... .......... .......... ........... ......idempotent law
P  P  Q.......... .......... .......... .......... ........... .......... ....distributive law
  Q.......... .......... .......... .......... ........... .......... .......... ...complement law
Q.......... .......... .......... .......... ........... .......... .......... .........i dentity law

45
Example 22
Use the laws of algebra to simplify the following A   A  B
Solutions
A   A  B .......... .......... .......... .......... ......... given
 A      A  B .......... .......... .......... .......... ..identity law
A    B .......... .......... .......... .......... ...........d istributive law
A   .......... .......... .......... .......... ............ .......ide ntity law
A.......... .......... .......... .......... ........ .......... .......ide ntity law

Example 23
Use the laws of algebra to simplify the following
A   A  B B   A  B
Solutions
A   A  B   B   A  B.......... .......... .......... .......... .......... ..given
 A  A   A  B   B  A  B  B.......... .......... .......... .....distributive law
   A  B   B  A   .......... .......... .......... .......... ............. .complement law
 A  B   B  A.......... .......... .......... .......... ............. .......... ......iden tity law
 A  B    A  B .......... .......... .......... .......... ............. .......... ......commutative law
 A  A  B.......... .......... .......... .......... ............. .......... .......... ....distributive law
  B.......... .......... .......... .......... ............. .......... .......... .......... ...complement law
B.......... .......... .......... .......... ............. .......... .......... .......... .........i dentity law

Example 24
If A and B are non-empty sets simplify A   A  B B   A  B
Solution
A   A  B  B   A  B.......... .......... .......... .......... ..given
 A  A   A  B  B  A  B  B.......... .......... .....distributive law
   A  B  B  A   .......... .......... ..... .......... .........c omplement law
 A  B   B  A.......... .......... ..... .......... .......... .......... ....identity law
 A  B    A  B .......... .......... ..... .......... .......... .......... ....commutative law
 A  A  B.......... .......... .............. .......... .......... .......... ....distributive law
  B.......... .......... .... .......... .......... .......... .......... .......... ...complement law
B.......... .......... .... .......... .......... .......... .......... .......... .......... identity law

46
Example 25
Use the laws of algebra to prove the following
 A  B  B  A   A  B   A  B
Solution
Consider L.H.S
 A  B   B  A.......... .......... .......... .......... .......... .......... .given
 A  B  B  A.......... .......... .......... .......... .......... ........ by set difference
 A  B  B   A  B  A.......... .......... .......... .......... ..distributive law
 A  B   B  B    A  A  B  A.......... .......... .....distributive law
 A  B        B  A.......... .......... ..... .......... ........co mplement law
 A  B   B  A.......... .......... ..... .......... .......... .......... ....identity law
 A  B   B  A .......... .......... ..... .......... .......... .......... ....by set difference
 A  B   B  A.......... .......... ..... .......... .......... .......... .......Demorgan' s law
 A  B    A  B .......... .......... ..... .......... .......... .......... .......commutative law
  A  B    B  A   A  B    A  B 

Example 26
Use the laws of algebra to simplify the following
 A  B   A  B   A  B   A  B
Solutions
 A  B    A  B   A  B    A  B.......... .......... .......given
A  B  B  A  B    A  B.......... .......... ....... ........di stributive law
 A      A  B    A  B.......... .......... ....... .......... ......complement law
A   A  B    A  B.......... .......... ....... .......... .......... .....ident ity law
 A  A   A  B   A  B.......... .......... ....... .......... .....distributive law
   A  B   A  B.......... .......... ....... .......... .......... .....complement law
 A  B    A  B.......... .......... ....... .......... .......... .......... ...identity law
A  B  B.......... .......... ...... .......... .......... .......... .......... ..distributive law
A  .......... .......... ...... .......... .......... .......... .......... .......... complement law
.......... .......... ...... .......... .......... .......... .......... .......... ......iden tity law

47
Alternative
 A  B    A  B   A  B    A  B.......... .......... .......given
 A  B    A  B   A  B    A  B   .......... .......... ..Demorgan's law
   
 A  B    A  B   .......... .......... ............ .......... .......... complement law
 A  B    A  B   .......... .......... ............ .......... .......... complement law
.......... .......... ............ .......... .......... .......... .......... .......... ..identity law

Example 27
Use algebra laws to simplify  A  B  B  A   A  B
Solution
 A  B  B  A   A  B.......... .......... .......... ..given
 A  B   A  B   A  B.......... .......... .......... ..commutative laws
A  B  B   A  B.......... .......... .......... ...........d istributive laws
 A      A  B.......... .......... .......... ............ ........co mplement laws
   A  B.......... .......... .......... ............ ........ .......... identity laws
 .......... .......... .......... .......... ........ .......... ........ .......... identity laws
Example 28
Use algebra laws of set to simplify  A  B  A
Solution
 A  B  A.......... .......... .......... .......... .....given
 A  A  B  A.......... .......... .......... .......distributive law
  B  A.......... .......... .......... ....... .........c omplement law
 .......... .......... .......... ....... .......... .......... ....identity law

Example 29
Use algebra laws of set to prove that  A  B    A  B   A
Solution
Consider L.H.S
 A  B    A  B .......... .......... .......... ........given
 A  B   A  B .......... .......... .......... ........ Demorgan's law
A  B   B .......... .......... .......... ........ .........d istributive law
A   .......... .......... .......... ........ .......... ........co mplement law
A.......... .......... .......... ........ .......... .......... .....ident ity law

  A  B    A  B   A

48
Example 30
Use algebra laws of set to prove that A A  B  A  B
Solution
Consider L.H.S
A A  B .......... .......... .......... .......... .......... .......given
A   A  B    A  B   A.......... .......... .......... ...by difinition of 
 A   A  B     A  B   A.......... .......... ......... by definition of set difference
 
A   A  B   A  B   A.......... .......... ......... Demorgan's law
 A  A   A  B  A  A  B  A.......... .distributive law
   A  B    B  A.......... ........... .........c omplement law
 A  B   .......... ........... .......... .......... .......... ...identity law
 A  B.......... ........... .......... .......... .......... ........id entity law
 A  B .......... ........... .......... .......... .......... .........b y definition of set difference
 A A  B    A  B 

Exercise 2.1
1. Use algebraic laws to simplify the following
(a) A   A  B B   A  B
(b)  A  B   A  B    A  B   B 
 
(c) B  A   A  B

2. Use algebraic laws to simplify the following


(a)  A  B   B B   A  B
(b)  A  B   A  A  B 
(c) A  BA
(d)  A / B  B  A  B  A

3. Use algebraic laws of sets to prove the following


(a)  A  B    A  B  
(b) A   A  B   A  B
(c)  A  B    A  B    A  A
 
(d)  A  B    A  B   A

49
2.10 SET WORD PROBLEMS
Set word problem is a mathematical exercise where significant background
information on the problem is presented as text rather than in mathematical
notation. As word problems often involve a narrative of some sort, they are
occasionally also referred to as story problems and may vary in the amount of
language used.

Example 31
The dormitory students of Marian were asked whether they had a dictionary or
thesaurus in their rooms. The result showed that 650 students has a dictionary.
150did not have a dictionary, 175 had a thesaurus and 50 had neither a
dictionary nor a thesaurus. Find the number of students who
(a) Live in the dormitory
(b) Have both a dictionary and a thesaurus
(c) Have only a thesaurus
Solution
Let D= Students had Dictionary, nD  650
T= Students had Thesaurus, nT   175

(a) Students living in the dormitory  800 students


(b) Have both dictionary and a thesaurus  75 students
(c) Students having only thesaurus  100 students

Example 32
In talents show (or competition), a school awarded medals in different
categories. 36 medals in dance, 12 medals in dramatics and 18 in music. If
these medals amounted to a total of 45 winners and only 4 winners got medals
in all the three categories, how many received medals in exactly two of these
categories?
Solution

50
Let, medals in dance be A, n( A)  36
Medals in dramatics be B, n( B)  12
Medals in music be C , n(C )  18
n A  B  C   45
n A  B  C   4

n A  B  C   n A  nB   nC   n A  B   n A  C   nB  C   n A  B  C 


45  36  12  18  n A  B   n A  C   nB  C   4
45  70  n A  B   n A  C   nB  C 
n A  B   n A  C   nB  C   70  45
n A  B   n A  C   nB  C   25
 25 students received exactly two medals

Example 33
Out 130 students at a certain school, 10 study Physics and Mathematics while
28 study neither of the two subjects. Those who study Physics only are three
times as many as those who study mathematics only. How many students study
mathematics?
Solution
Let, students studying mathematics only be M , n( M )  x
Students studying Physics only be P, n( P)  3x
nP  M   10
nP  M   10

51
From, nP  M   nP   nM   nP  M 
130  28  3x  10  x  10  10
102  4 x  10
4 x  92
x  23
Students studying math  10  x  10  23  33
 Students studying math  33

Example 34
In a group of 60 students, 23 play football, 15 play tennis and 20 play
basketball. 7 play football and tennis, 5 basketball and tennis, 4 play football
and basketball and 15 do not play any of these games. Find the number of
students who play.
(a) All the three games
(b) Football but not basketball
(c) Football and basketball but not tennis
Solution
Let F  students playing football, nF   23
T  students playing tennis, nT   15
B  students playing basketball, nB   20
  60 students
n F  T   7
n B  T   5
n F  B   4

nF  T  B   15

52
nF  B  T   nF   nT   nB   nF  T   nF  B   nB  T   nF  B  T 
60  15  23  15  20  7  4  5  x
45  42  x
x3
(a) 3 students play all games
(b) 19 students play football but not basketball
(c) 30 students play football and basketball but not tennis

Example 35
A group of students consists of 17 girls and 15 boys. 22 of them play handball
while 16 play basketball. Among the boys 12 play handball, 11 play basketball
and 10 play both games. Among the girls, 3 do not play either of the games
(a) Summarize the given information by using Venn diagram
(b) How many students play either handball or basketball
(c) How many girls play both games
Solution

53
Let G  Girl, nG   17
B  Boy, nB   15
H  Students playing Handball, nH   22
B  Students playing Basketball, nB   16
H b  Boys playing Handball, nH b   12
Bb  Boys playing Basketball, nBb   11
Boys playing both games is 10
Girls who play neither games is 3

Consider Girls
n H  B   n H   n B   n H  B 
17  3  10  5  x
14  10  5  x
x 1
(b) Number of students playing either handball or basketball is 27
(c) Number of girls playing both games is 1

Example 36
In a certain collage, there are 20 students who are girls and 20 students who are
boys, 15 students study mathematics and 16 students study chemistry. 10 girls
study mathematics and 11 boys study chemistry. If 8 girls and 7 boys study
neither mathematics nor chemistry calculate
(a) The number of students studying both subjects mathematics and chemistry
(b) Number of students who study both subjects mathematics but not chemistry
(c) Number of students who study only one subject
(d) Number of students who study at least one subject
Solution

54
Number of girls  20
Number of boys  20
15 students study mathematics
16 students study chemistry
10 girls study mathematics
11 boys study chemistry
8 girls study neither subject
7 boys study neither subject

Consider Boys
nM  C   nM   nC   nM  C 
20  7   5  11 y
13  16  y
y 3
Also consider Girls
nM  C   nM   nC   nM  C 
20  8  10  5  x
12  15  x
x3
(a) 6 students study both subjects
(b) 9 students study mathematics but not chemistry
(c)19 students study only one subject
(d) 25 students study at least one subject

55
Example 37
In a class of a certain school there are 15 girls who like dancing, 5 girls are
pretty and 6 girls who are intelligent. Every pretty girl likes dancing, 3
intelligent girls do not like dancing. If 2 girls are both pretty and intelligent
(a) Present the above information in the Venn diagram
(b) Find the number of girls in the class
(c) Find the number of girls who are pretty but not intelligent
Solution
(a) Venn diagram

(b) Number of girls  9  3  2  1  3  18girls


(c) Pretty but not intelligent  3 girls

Example 38
A survey of 500 students taking one or more course in physics, chemistry and
mathematics during one academic year revealed the following numbers of
students in the indicated subjects. Physics and chemistry are 83, chemistry and
mathematics are 63, physics and mathematics are 217, mathematics are 295,
chemistry 186 and physics 329. Draw a Venn diagram to show the information
given above and hence calculate the number of students taking physics or
mathematics but not chemistry.
Solution

56
nP  C  M   nP   nC   nM   nP  C   nP  M   nC  M   nP  C  M 
500  329  186  295  83  217  63  x
x  53
Number of students taking physics or mathematics but not chemistry  82  (217  x)  68
Number of students  82  (217  53)  68  314
 Number of students taking physics or mathematics but not chemistryis 314

Example 39
At Misasi Secondary School, 48 students are taking Physics, Chemistry or
Mathematics. Those who are taking Mathematics and Chemistry are one
seventh of those whose are taking Mathematics, those who are taking
Mathematics only are double as many as those taking Physics only. All those
who are taking Chemistry are also taking Mathematics and Physics, those who
are taking Chemistry is two third of those who are taking Physics and
Mathematics. 35 students are taking Mathematics, those who are taking
Mathematics and Physics is one quarter of those who are taking Physics,
Chemistry or Mathematics.
(a) Display the information above in a Venn diagram
(b) Find the number of students taking;
(i) Just Chemistry
(ii) At least two subject
(iii)Mathematics
(iv) Physics or Chemistry but not Mathematics.
Solution

57
(a) Venn Diagram

(b) From the Venn diagram above;


a  b  c  d  e  f  g  48.......... .......... .........( i )
b  c  e  f  35.......... .......... .......... .......... ....( ii)
a  b  d  e  P.......... .......... .......... .......... ......( iii)
d  e  f  g  C.......... .......... .......... .......... ......( iv)
Then,
M C  1
7 35  5
e  f  5.......... .......... .......... .......... .......... .......( v)
PM  1
4 48  12
b  e  12.......... .......... .......... .......... .......... ......( vi)
C  23 12  8
from d  e  f  g  C but g  0
d 58
d 3
nC   8
a  d  g  48  35
a  d  g  13
nM   2nP only
nM   210
nM   20

58
a  b  c  48  8
a  b  c  40
From b  e  12 but b  10
e  12  b  12  10  2
e2
e f 5
f 3
a  b  d  e  25
 Number of students taking;
i  Just Chemistry is 0
ii At least two subject is 18
From, b  d  e  f  18
iii Mathematics is 35
iv  Physics or Chemistry but not Mathematics is 13
From, a  d  f  13

Example 40
100 people were asked about three brands of soft drinks called A, B and C. The
results shows that 18 like A only, 23 like A but not B, 26 like A, 8 like B and C,
48 like C, 8 like A and C and 54 like one and only one of the drinks.
Find how many
(i) People like B?
(ii) People don’t like any of the drinks?
Solution
Consider the diagram below;

59
18  48  5  3  5  x  54
x 1
(i )nB   5  3  0  1  9
 9 people like B only.

ii  Don' t like any of the drinks  100  26  35  5  1  33 people

Exercise 2.2
1. A certain poultry in Bagamoyo which produces three types of chicks had
its six month report which revealed that out of 126 of its regular customers,
65 bought broilers, 80 bought layers and 75 bought cocks, 45 bought layers
and cocks, 35 bought broilers and cocks, 10 bought broilers only, 15
bought layers only and 6 bought cocks only. 6 of the customers did not
show up
(a) How many customers bought all three products?
(b) How many customers bought exactly two of the farm’s products?
2. In a class of 58 students, 30 are boys and the rest are girls. 29 of all
students study mathematics while 27 of study physics. Among the girls, 3
study both mathematics and physics, 9 girls study mathematics only and 4
girls study neither mathematics nor physics. Among the boys 5 do not
study either of these two subjects.
(a) Summarize the information using the Venn diagram
(b) How many students study both subjects?
(c) How many boys study physics only?

60
Chapter Three
LOGIC
Introduction

Logic is a systematic approach of determining whether a given statement is true


or false.

Logic is the mathematical concepts used to draw a conclusion whether the


given statement is true or false.

Logic deals with reasoning and judgement of statement whether to be true or


false, judgement is only applied for statements.

Sometimes logic is called mathematical Reasoning.

Mathematical reasoning is the study of reasoning through the use of


mathematical symbols.

Application of Logic

(i) It is used in computer programming and designing of machines.


(ii) It is used in electric circuit and in entertainment (Logic puzzles).
(iii) To weigh pros and cons of the decision.
(iv) The power of reasoning differentiate humans from animal.
(v) The successful lawyer can win a case in a court by strong reasoning.

3.1 SENTENCE AND STATEMENT

Sentence is a basic unit of language that express a complete thought.


This is a set of word that is complete in itself, it can be in terms of questions,
exclamations or commands, prediction e.t.c.

Examples
(i) Amina is the most beautiful girl in the class.
(ii) Tanzanians are the best in pure mathematics.

61
Statement is a group of words which can be judged to be true or false but not
both.
Example
 Naomi is beautiful
 Mayombya likes Mathematics
 Eva is tall but Salome is fat
 Two is greater than four
 Dodoma is a capital city of Tanzania.
 Every square is a rectangle .

NB. Every statement is sentence but not every sentence is a statement,

Proposition is a representative letter in which it carries two meanings of either


being true or false eg p, q, r, s etc
Proposition is a sentence that is either true or false but not both.
Types of logical statements
Logical statements are categorized into two types which are
(i) Simple logical statement
Simple logical statement is a single logical statement or is a statement which is
complete in its meaning and does not require the connective words like “and,
but, then,…”
Example
 Obama is active
 Kikwete is competent
 Four is even number
 Peter is writing
 Malema is speaking

(ii) Compound logical statement


Compound statement is the combination of two or more simple statements, or is
the statement which is joined by the connective words like “and, but, then,…”
Example
 Joseph is thin and Joyce is charming
 Joyce is writing and Peter is reading
 The sun is bright and hot
 Mathematics is simple and interesting subject
Truth Value is the truth or falsity of a statement.
62
If the statement is true then its truth value is TRUE or T and if the statement is
false then its truth value is FALSE or F.
Truth table is the table which used to show the validity of the compound
statement, or is a table used to determine specific truth values of a given logical
statement.

3.2 LOGICAL CONNECTIVES


(a) Logical connectives in a compound statement
These are connectives in logic which are used in joining simple statements to
form compound statements, or Logical connective are connectives used to
connect simple logical statements to make up logical compound statement.

Logical connectives are categorized as follows


(i) Conjunction ( )
Conjunction is the connective which connect two or more simple statements
and compound statement formed is said to be true if and only if all propositions
are true. Connective words which stands for conjunctions are and, but, both,
while, when, e.t.c

Conjunction is denoted by 
Example 1
The sky is blue and the grass is green.
Let p  The sky is blue
q  The grass is green
Connective is ' and'  
Symbolically is p  q

Example 2
Draw the truth table of p  q
Solution
p q pq

T T T

T F F

F T F

F F F

63
Example 3
Draw the truth table of  p  q   r
Solution
p q r pq  p  q  r
T T T T T

T F T F

T F T F F

T F F F F

F T T F F

F T F F F

F F T F F

F F F F F

(ii) Disjunction ( )
Disjunction is the connective in which the statement is said to true if at least
one proposition is true.
Connective words which stands for Disjunctions are or, either or, etc.

Disjunction is denoted by 

Example 4
Robert is reading or writing
Let p  Robert is reading
q  Robert is writing
connective is ' or'  
Symbolically is p  q

64
Example 5
Draw the truth table of p  q
Solution
p q pq

T T T

T F T

F T T

F F F

(iii) Conditional (Implication) ( )


Condition is the connective in which one proposition implies another
proposition.
Connective words which stands for Condition are if….then…, only if..,
necessary for.., therefore…, sufficient for.., implies, necessary, whenever,
condition, etc
eg. If p and q are two simple statements
In conditional probability this can be expresed as one of the following statements,
(i) p implies q i.e. p  q 
(ii) If p, then q
(iii) p is necessary for q
(iv) q whenever p
(v) A necessary condition of p is q
(vi) A sufficient condition for q is p

 The compound statement p  q is said to be false if p is true and q is false


otherwise is true.

65
Example 6
If you study hard then you will pass an examination.
Solution
p  You study hard
q  You will pass an examination
Connective word is " If....then....."
 If p then q  p  q

Example 7
Draw the truth table of p  q
Solution
p q pq

T T T

T F F

F T T

F F T

(iv) Bi-condition (Double implication ) ( )


The logical statement is said to be true if and only if all proposition are either
true or false.
Connective words which stands for Bi-condition are “if and only if, sufficient
and necessary for, etc
Bi-condition or double implication is denoted by 

Example 8
You can take the flight if and only if you buy a ticket.
p  you can take the flight
q  you buy a ticket
Connective word is "iff "
 p iff q  p  q

66
Example 9
Draw the truth table of p  q
Solution
p q pq

T T T

T F F

F T F

F F T

(v) Negation
This is the opposite of the stated statement, and denoted by ~
If p is a given statement, its negation is ~ p

Example 10
 Peter is clever  p 
 Peter is not clever ~ p 

Consider the table below


Statement Negation
p ~p
q ~q
pq ~  p  q

T F
F T

67
OTHER LOGICAL CONNECTIVES
This involves two common connectives namely as
(a) Exclusive disjunction
A mathematical statement is said to be exclusive disjunction if the results of the
truth table is said to be true if one of them is true but not both otherwise is false.
The exclusive disjunction of p and q is denoted by pq read as either
p or q but not both of them.
Example 11
Draw the truth table of pq
Solution
p q pq

T T F
T F T
F T T
F F F

(b) Joint denial


Joint denial is the connective in which the truth table results is said to be true
only if both are false otherwise false.
Joint denial is denoted by 
The joint denial of p and q is written as p  q .
p  q read as neither p nor q

Example 12
Draw the truth table of p  q
Solution
p q pq

T T F
T F F
F T F
F F T

68
3.3 TRUTH TABLES FOR SIMPLE AND COMPOUND STATEMENTS
Simple and compound statements can be expressed in symbolic form,
Let p = first proposition, q  second proposition, r  third proposition etc ,
then connect the symbols by using the specified connectives. Now from
symbolic logic connectives its possible to draw the truth table.
Example 13
Change into symbolic logical form and draw the truth table of the statement
“If I am clever then I understand logic”
Solution
Let P= I am clever
q= I understand logic
In symbolic form p  q
Truth table
p q pq

T T T

T F F

F T T

F F T

Example 14
Change into symbolic logical form and draw the truth table of the statement
“If today is Monday then tomorrow is Tuesday”
Solution
Let P= today is Monday
q= tomorrow is Tuesday
In symbolic form p  q
Truth table
p q pq

T T T

T F F

F T T

F F T

69
Example 15
Change into symbolic logical form and draw the truth table of the statement
“If I like Mathematics or I study then I will pass”
Solution
Let P= I like mathematics
q= I study
r= I will pass
In symbolic form  p  q   r
Truth table
p q r p  q  p  q  r

T T T T T

T T F T F

T F T T T

T F F T F

F T T T T

F T F T F

F F T F T

F F F F T

Example 16
Change into symbolic form the statement “If my brother stands first in the class,
I will give him a phone. Either he stands first or I was out of station. I didn’t
give my brother a phone this time. Therefore I was out of station”
Solution
Let P= He stands first in the class
q= I will give him a phone
r= I was out of station
 Symbolic logical form  p  q    p  r  ~ q  r

70
Equivalent of propositions
Two or more propositions are said to be equivalent if they have the same truth
value on the truth table.

Example 17
Determine whether the sentences p  q and q  p are equivalent.
Solution
p q pq q p

T T T T

T F T T

F T T T

F F F F

From the table above the column for p  q is the same as the column for q  p .
 The given propositions are equivalent ie. p  q  q  p  .

Example 18
Determine whether the sentences p  q and ~ p  q are equivalent.
Solution
p q ~ p p  q ~ pq

T T F T T

T F F F F

F T T T T

T F F T T

From the table above the column for p  q is the same as the column
for ~ p  q .
 The given propositions are equivalent ie. p  q ~ p  q  .

71
Example 19
Determine whether the sentences p  q and  p  q   q  p  are
equivalent.
Solution
p q pq q p pq  p  q   q  p 
T T T T T T

T F F T F F

F T T F F F

T F F T T T

From the table above the column for p  q is the same as the column
for  p  q   q  p  .
 The given propositions are equivalent  p  q    p  q   q  p 

Example 20
Determine whether the sentences p  q and ~ p  q   ~ q  p  are
equivalent.
Solution
p q ~p ~q ~ pq ~ q p pq ~ p  q   ~ q  p 

T T F F T T T T

T F F T F T F F

F T T F T F F F

T FT FT T T T

From the table above the column for p  q is the same as the column
for ~ p  q   ~ q  p  .
 The given propositions are equivalent  p  q   ~ p  q   ~ q  p 

72
3.4 Tautologies, contradictions and equivalent statements
(i) Tautology
A compound statement is said to be tautology if it contains all True values
(T) in the last column of its truth table.

(ii) Contradiction
A compound statement is said to be contradiction if it contains all False
values (F) in the last column of its truth table.

(iii)Logical contigency
A compound statement is said to be contigent if it contains at least one T
and F in each row of its truth table.

(iv) Fallacy
Fallacy is the incorrect argument in logic or incorrect reasoning

3.5 Converse, contrapositive and inverse of a given statements


We can define the converse, the contrapositive and the inverse of a conditional
statement. We start with the conditional statement “If p then q .”

 The converse of the conditional statement is “If q then p .”


 The contrapositive of the conditional statement is “If not q then
not p .”
 The inverse of the conditional statement is “If not p then
not q .”

We will see how these statements work with an example. Suppose


we start with the conditional statement “If it rained last night, then
the sidewalk is wet.”

 The converse of the conditional statement is “If the sidewalk is


wet, then it rained last night.”
 The contrapositive of the conditional statement is “If the
sidewalk is not wet, then it did not rain last night.”
 The inverse of the conditional statement is “If it did not rain last
night, then the sidewalk is not wet.”

73
(i) Converse of the conditional statement
Converse is the reverse of conditional statement.
If p  q is the conditional statements then its converse is q  p .
i.e.
Statement p  q
Converse q  p

(ii) Contrapositive of the conditional statement


Contrapositive is the opposite measuring converse of conditional statement.
If p  q is the conditional statements then its contrapositive is ~ q ~ p .
i.e.
Statement pq
Converse q p
Contrapositive ~ q ~ p

(iii) Inverse of the conditional statement


Inverse is the converse of the contrapositive.
If p  q is the conditional statements then its inverse is ~ p ~ q .
i.e.
Statement pq
Converse q p
Contrapositive ~ q ~ p
Inverse ~ p ~ q

Example 21
Write the converse, contrapositive and inverse of the statement “If Juma is a
poet, then he is poor”
Solution
Converse “If Juma is poor, then he is a poet”
Contrapositive “If Juma is not poor, then he is not a poet”
Inverse “If Juma is not a poet, then he is not poor”

Example 22
Write the converse, contrapositive and inverse of the statement “If it rains, then
they cancel school.”
Solution
Converse "If they cancel school, then it rains."
Contrapositive "If they do not cancel school, then it does not rain."
Inverse “If it does not rain, then they do not cancel school.”
74
Example 23
Write the converse, contrapositive and inverse of the statement “If two angles
are congruent, then they have the same measure.”
Solution
Converse “If two angles have the same measure, then they are congruent.”
Contrapositive “If two angles do not have the same measure, then they are not
congruent.”
Inverse “If two angles are not congruent, then they do not have the same
measure.”

Example 24
Write the converse, contrapositive and inverse of the statement “If a
quadrilateral is a rectangle, then it has two pairs of parallel sides.”
Solution
Converse “If a quadrilateral has two pairs of parallel sides, then it is a
rectangle.”
Contrapositive “If a quadrilateral does not have two pairs of parallel sides, then
it is not a rectangle.”
Inverse “If a quadrilateral is not a rectangle, then it does not have two pairs of
parallel sides.”

Example 25
Write the converse, contrapositive and inverse of the statement “If a person is
18 years old, then he is a legal adult.”
Solution
Converse “If a person is a legal adult, then he is 18 years old.”
Contrapositive “If a person is not a legal adult, then he is not 18 years old.”
Inverse “If a person is not 18 years old, then he is not a legal adult.”

75
3.6 LAWS OF ALGEBRA OF PROPOSITION
The following are laws of algebra used to simplify propositions;

(i) Commutative law


 pq q p
 pr  r p

(ii) Associative law


 p  q  r    p  q   r
 p  q  r    p  q   r

(iii)Distributive law
 p  q  r    p  q    p  r 
 p  q  r    p  q    p  r 

(iv) Identity law


 p T  p
 p T T
q  F  F
q F  q

(v) Compliment law


 p~ p F
 p ~ p T

(vi) De-Morgan’s law


 ~  p  q  ~ p  ~ q
 ~  p  q  ~ p  ~ q

(vii) Idempotent law


 p p  p
q q  q

(viii) Modulus law (absorption law)


 p   p  q  p
 p   p  q  p

76
Example 26
Use the algebra propositions to simplify the following expression
 p  q   p  q
Solutions
 p  q    p  q .......... .......... .......... .......... ..given
 p  q   q  p    p  q .......... .......... ......by defition
~ ~ p  q   ~ q  p   ~ p  q .......... .......... ..by defition
~ ~ p  q  ~ ~ q  p  ~ p  q .......... .......... Demorgan's law
~ ~ p  q   ~ p  q  ~ ~ q  p .......... .......... commutative law
T  ~ ~ q  p .......... .......... .......... .......... .......... ..complement law
T .......... .......... .......... .......... .......... .......... .........identity law

Example 27
Determine whether the following proposition is a tautology
 p ~ q   q  r   p  r
Solution
 p ~ q   q  r   p  r.......... .......... ..given
~ ~ p ~ q   q  r   p   r.......... .......... by definition
~ ~ p ~ q   p  q  r   r.......... .......... commutative law
~ ~ p  p   ~ q  p   q  r   r.......... ..distributive law
~ F  ~ q  p   q  r   r.......... ............ complement law
~ ~ q  p   q  r   r.......... ............ ......iden tity law
~ ~ q  p  ~ q  r   r.......... ............ ....Demorgan' s law
q ~ p   ~ q  ~ r   r.......... ............ ....Demorgan' s law
q ~ p   ~ q  r   ~ r  r .......... ........dist ributive law
q ~ p   ~ q  r   T .......... ............ .....compliment law
q ~ p   ~ q  r .......... ........... .......... ...identity law
q ~ q   ~ p  r .......... ........... .......... ...commutative law
T  ~ p  r .......... ........... .......... .......... ..complement law
T .......... ........... .......... .......... .......... .........i dentity law

77
Example 28
Simplify the following using laws  p   p  q  ~ p
Solution
 p   p  q  ~ p.......... .......... .......... .given
~  p   p  q  ~ p.......... .......... .......... by definition
~ p  ~  p  q  ~ p.......... .......... .......... Demorgan's law
~ p  ~ p ~ q  ~ p.......... .......... ........ Demorgan's law
~ p  F   ~ p ~ q  ~ p.......... .......... identity law
~ p  F  ~ q  ~ p.......... .......... .......... distributive law
~ p  F  ~ p.......... .......... .......... .......... identity law
~ p ~ p.......... .......... .......... .......... ........identity law
~ p.......... .......... .......... .......... .......... .......idempotent law

Example 29
Simplify by using law of algebra  p  q   q
Solution
 p  q   q.......... .......... .......given
~  p  q   q.......... .......... ....... by definition
~ p ~ q   q.......... .......... .....Demorgan law
~ p  ~ q  q .......... .......... .....associative law
~ p  T .......... .......... .......... .....complement law
T .......... .......... .......... .......... ....identity law

Example 30
Simplify by using law of algebra  p   p  q  ~ p
Solution
 p   p  q  ~ p.......... .......... ..given
~  p   p  q  ~ p.......... .......... .by definition
~ p  ~  p  q  ~ p.......... ......... Demorgan law
~ p  ~ p ~ q  ~ p.......... ......Demorgan law
~ p  F   ~ p ~ q  ~ p........identity law
~ p  F  ~ q  ~ p.......... .........distributive law
~ p  F  ~ p.......... .......... .........identity law
~ p ~ p .......... .......... .......... .....identity law
~ p.......... .......... .......... .......... .....idempotent law

78
Example 31
Simplify the following proposition by deductive reasoning  p  q   ~ q  p 
Solution
   
p  q  ~ q  p .......... .......... ....given
~  p  q  ~ ~ q  p .......... ......... by definition
~ p  ~ q   q  ~ p .......... ......... Demorgan's law
~ p  ~ p   ~ q  q .......... ......... commutative law
~ p  ~ q  q .......... .......... ......... idempotent law
~ p  F .......... .......... .......... ......... complement law
F .......... .......... .......... .......... .......identity law

Exercise 3.1
1. Simplify the expression  p  q   q by using algebra laws
2. Simplify the following by using algebra laws ~  p  q   ~ p  q 

3.7 VALIDITY OF ARGUMENT


An argument is the combination of different compound statements (or
premises), or an argument is a sequence of premises (or propositions) together
with its conclusion.
i.e. P1  P2  P3      Pn   Conclusion

Validity means valid or invalid of logical statement.


The argument can be valid (Tautology) invalid (Contradiction/fallacy)
VALID ARGUMENT
A valid argument is an argument which is always true, or an argument which
the truth of its premises entire the truth of its conclusion.

Principle of Validity
The premises represented by P1  P2  P3      Pn are joined using connective
“and” or “  ” and the conclusion is given by words like “Thus” or “Therefore”
and denoted by “  ” an implication.
i.e. P1  P2  P3      Pn   Conclusion

79
Example 32
Check if the following argument is valid “If I am clever then I understand
Logic. I don’t understand Logic. Therefore I am not clever”
Solution
Let p  I am clever
q  I understand Logic
Compound statement  p  q  ~ q  ~ p
p q ~ p ~ q p  q  p  q  ~ q  p  q  ~ q ~ p

T T F F T F T

T F F T F F T

F T T F T F T

F F T T T T T

 Hence the argument is valid because it is TAUTOLOGY

Alternative
 p  q  ~ q ~ p
~  p  q  ~ q  ~ p By definition
~ ~ p  q  ~ q  ~ p By definition
~ ~ p  q   q ~ p De Morgan' s law
 p  ~ q   q ~ p De Morgan' s law
 p  q   ~ q  q  ~ p Distributive law
 p  q   T  ~ p Complement law
 p  q  ~ p Identity law
q  p  ~ p Commutative law
q   p ~ p  Associative law
qT Complement law
T Identity law
 Hence the argument is valid because it is TAUTOLOGY

80
Example 33
Determine whether the following proposition is true or not
 p  ~ q   q  r   p  r by using laws.
Solution
 p  ~ q   q  r   p  r.......... .......... ......... given
~  p  ~ q   q  r   p   r.......... .......... ........ definition
~  ~ p  ~ q   q  r   p   r.......... .......... ......definition
~  ~ p  ~ q   p  q  r   r.......... .......... ......commutative law
~ ~ p  p   ~ q  p   q  r   r.......... .......... distributive law
~ F  ~ q  p   q  r   r.......... .......... .......... .complement law
~ ~ q  p   q  r   r.......... .......... .......... .......identity law
~ ~ q  p  ~ q  r  r.......... .......... .......... .....demorgan's law
q ~ p   ~ q  ~ r  r.......... .......... .......... .....demorgan's law
q ~ p   ~ q  ~ r   r .......... .......... .......... .....associative law
q ~ p   ~ q  r   ~ r  r .......... .......... ........distributive law
q ~ p   ~ q  r   T .......... .......... .......... .......complement law
q ~ p   ~ q  r .......... .......... .......... .......... ....identity law
q ~ q   ~ p  r .......... .......... .......... .......... ....commutative law
T  ~ p  r .......... .......... .......... .......... .......... ....complement law
T .......... .......... .......... .......... .......... .......... .......... .identity law

Exercise 3.2
1. Test the validity of the following arguments p  ~ q , ~ r  q , r  ~ p
2. Test the validity of the following argument p  ~ q, r  q, q Ⱶ ~ r

3.8 ELECTRICAL NETWORKS


This is the application of logic statement to check the flow of current
from one terminal to another.
That is

81
Point to note in electrical networks
(i) The current will flow from T1 to T2 only if the switch is closed and
not otherwise.
(ii) The current may not flow from T1 to T2 only if the switch is open
and not otherwise.
(iii) The flow of current is denoted by T while not flow of current is
denoted by F .
(iv) Negation always implies opening of switch and its vice versa is
true.

Connection of Switches
There are two connections of switches namely;
(a) Series connection
(b) Parallel connection

(a) Series connection


Series connection is the connection at which two or more switches
are connected in a single line.
Example

In series connection the flow of current is possible if and only if all


switches are closed.
Series connection use conjunction   connective.
That is;

Logically is represented by p  q

Logically is represented by p  q  r

Logically is represented by p  q  ~ r no flowing of current.

82
(b) Parallel connection
Parallel connection is the connection in which two or more
switches are connected in a side way.
Series connection use disconjunction   connective.

Parallel connection of two switches

Logically is represented by p  q

Parallel connection of three switches

Logically is represented by p  q  r

In parallel connection the current may flow only if at least one of


them is closed.
Note:
For the case of complex switches (both series and parallel
connections) apply brackets to symbolize.
Example 34
Construct the network for  p  q   r
Solution

83
Example 35
Construct the network for  p  q   r
Solution

Example 36
Construct the network for p  q  r   s
Solution

Example 37
Construct the network for p  q  r   s
Solution

Example 38
Construct the network for  p  q   r  s 
Solution

84
Example 39
Construct the network for p  q
Solution
p  q  ~ p  q   ~ q  p 

Alternative

Example 40
Draw the electrical network for the proposition ~  p  q   ~ p  q 
Solution
~  p  q   ~ p  q 
~  p  q   ~ p  q   ~ p ~ q   ~ p  q ........De Morgan' s law

Alternative electrical network

Example 41
Use the laws of algebra of proposition to simplify the statement
q   p ~ q   r  q  and hence draw the corresponding simple
electrical network.
Solution

85
q   p  ~ q   r  q .......... ......given
q  p   q ~ q   r  q ......... distributive law
q  p   T  r  q .......... ........ .compliment law
q  p   r  q .......... ........ .......ide ntity law
q  p   q  r .......... ........ .......commutative law
q   p  r .......... ........ .......... .....distributive law

Example 42
Use the laws of algebra to simplify the statement
p  q  r  ~ q   ~ p ~ r  and hence draw the corresponding simple
electrical network.
Solution
p  q  r  ~ q   ~ p  ~ r .......... .......... .......... ......given
p  q  r   q  ~ q   ~ p  ~ r .......... .......... ......... distributive law
p  q  r   F  ~ p  ~ r .......... .......... .......... ........ complement law
p  q  r   ~ p  ~ r .......... .......... .......... ........ ......iden tity law
 p  q  r   ~ p  ~ r .......... .......... .......... .......... ......asso ciative law

Alternative electric circuit

86
Example 43
Simplify the logical proposition below and draw its corresponding electrical
network r  ~ p  q   p
Solution
r  ~ p  q   p .......... .......... .......... ......given
r   p  q   p .......... .......... .......... .......... by definition
r   p  p   q  p .......... .......... .......... ..distributive law
r   p  q  p .......... .......... .......... .......... idempotent law
r   p  T   q  p .......... .......... .......... ...identity law
r   p  T    p  q .......... .......... .......... ...commutative law
r   p  T  q .......... .......... .......... ...........di stributive law
r   p  T .......... .......... .......... ............. ......iden tity law
r  p.......... .......... .......... ............. .......... ....identi ty law

Example 44
Determine the compound statement in words from the table below. Hence
draw electric diagram for the statement.
M N PM , N 

T T F
T F F
F T T
F F F
Solution
Circle T from P in the truth table
~M N

Alternative

87
Example 45
Draw a simple network diagram corresponding to the statement of the
following truth table
x y z p

T T T F
T T F F
T F T
T F F T
F T T F
F T F T
F F T T
F F F T

Solution
On circling F
~ x ~ y ~ z   ~ x ~ y  z   x ~ y ~ z .........given
~ x ~ y   ~ z  z   x ~ y ~ z ......... .......... .....distributive law
~ x ~ y   F   x ~ y ~ z ......... .......... .......... .....complement law
~ x ~ y   x ~ y  ~ z ......... .......... .......... .......... ..identity law
~ y ~ x   ~ y  x ~ z ......... .......... .......... .......... ..commutative law
~ y  ~ x  x ~ z ......... .......... .......... .......... .......... distributive law
~ y  ~ x  x   ~ x  ~ z ......... .......... .......... .......... distributive law
~ y  F  ~ x  ~ z ......... .......... .......... .......... .......... complement law
~ y  ~ x  ~ z ......... .......... .......... .......... .......... .......ide ntity law

88
Example 46
A statement J is formed from P, G and M . Write J in most simplified
form and use it to draw an electric circuit which will allow the current to
flow whenever J is true
P G M J

T T T T
T T F T
T F T
T F F T
F T T T
F T F F
F F T F
F F F F

Solution
P  G  M   P  G  ~ M   P ~ G  M   P ~ G  ~ M   ~ P  G  M .......... given
P  G  M   P  G  ~ M  P ~ G  M   P ~ G  ~ M  ~ P  G  M .......given
P  G   M  ~ M  P ~ G   M  ~ M  ~ P  G  M .......... .......... distributive law
P  G   T  P ~ G   T  ~ P  G  M .......... .......... .......... .......... .......complement law
P  G   P ~ G  ~ P  G  M .......... .......... .......... .......... .......... .......... ..identity law
P  G ~ G  ~ P  G  M .......... .......... .......... .......... .......... .......... .........d istributive law
P  T  ~ P  G  M .......... .......... .......... .......... .......... .......... .......... .......... complement law
P  ~ P  G  M .......... .......... .......... .......... .......... .......... .......... .......... ........id entity law
P ~ P   P  G  M .......... .......... .......... .......... .......... .......... .......... .........d estributive law
T  P  G  M .......... .......... .......... .......... .......... .......... .......... .......... .........c omplement law
P  G  M .......... ......... .......... .......... .......... .......... .......... .......... .......... .......ide ntity law

89
Example 47
Draw the electrical diagram corresponding to the statement ~  p  q   q  p 
Solution
~  p  q   q  p .......... .......... .......... .......... ...given
~ p ~ q   q  p .......... .......... .......... .......... Demorgan's law

Example 48
Draw a simple network for q   p ~ q   r  ~ p 
Solution
q   p  ~ q   r  ~ p .......... .......... .......... .....given
q  p   q ~ q   r  ~ p .......... .......... ..... .distributive law
q  p   T   r  ~ p .......... .......... ..... .......... .complement law
q  p   r  ~ p .......... .......... ..... .......... ........id entity law
q  p   r   q  p  ~ p.......... .......... ........di stributive law
q  p   r   q   p ~ p .......... .......... ........as sociative law
q  p   r   q  T .......... .......... .......... ........co mplement law
q  p   r   T .......... .......... .......... .......... ......iden tity law
q  p  r.......... .......... .......... .......... .......... .....ident ity law

90
Example 49
Draw a simple network for s p, q, r  having the truth table shown below.
p q r s
T T T T
T T F F
T F TT
T F F F
F T T F
F T F F
F F T F
F F F F
Solution
Circle T from the compound statement s from the table
 p  q  r    p  ~ q  r .......... .......... .......... ......... given
 p  r  q    p  r  ~ q .......... .......... .......... ......... commutative law
 p  r   q ~ q .......... .......... .......... .......... .......... distributive law
 p  r   T .......... .......... .......... .......... .......... .......... complement law
 p  r .......... .......... .......... .......... .......... .......... .....ident ity law

Exercise 3.3
1. Simplify the following by using laws, then draw a circuit diagram
 p  q   ~  p  r   q  r 
2. Write in symbolic form the compound statement which represents the
following circuit diagram, where T1 and T2 are terminals

3. Draw the electrical circuit network of p  q  r   r  s 

91
Chapter Four
COORDINATE GEOMETRY I
Coordinate geometry is the branch of mathematics which deal with study of
points in the plane.
4.1 THE DISTANCE BETWEEN TWO POINTS
Distance between two points is the length between two points.
Consider the figure below which show the distance between two given points in
the x y plane .

fig 4. 1

Apply Pythagoras Theorem to ABC above,


2 2 2
AB  AC  BC
AB  x2  x1    y 2  y1 
2 2 2

AB  x2  x1 2   y2  y1 2
The length between A and B is given by x2  x1 2   y2  y1 2
The distance formula between two points is AB  x2  x1 2   y2  y1 2

Example 1
Find the distance between points A( 2,2) and B (6,1)
Solution
From distance formula AB  x2  x1 2   y2  y1 2
AB  6  22  1  (2)2  4 2  32  25  5 unit
 The distance between points is 5 unit

92
4.2 THE MIDPOINT BETWEEN TWO PINTS
Midpoint is point which divide two points into two equal parts (i.e. is a point
which bisect two given points)
Consider the figure below which has two points A and B

fig 4. 2

The above triangles are similar (i.e. AMP is similar with PNB)
AM MP AP
  but AP  PB
PN NB PB
AM MP
 1
PN NB
AM MP
 1 and 1
PN NB
AM x  x1
Consider  1, 1
PN x2  x
x1  x2
x  x1  x2  x , 2 x  x1  x2 , x 
2
MP y  y1
Also consider  1, 1
NB y2  y
y1  y 2
y  y1  y 2  y , 2 y  y1  y 2 , y 
2
 x  x2 y1  y 2 
The midpoint P x, y    1 , 
 2 2 

x x y y 
Midpoint formula Px, y    1 2 , 1 2 
 2 2 

93
Example 2
Find the midpoint between town A1,4 and town B7,2
Solution
 x  x2 y1  y 2 
From midpoint formula P x, y    1 , 
 2 2 
1 7 4  2   8 6 
P  x, y    ,    ,   4,3
 2 2  2 2
 The midpoint between towns is 4,3

4.3 THE ANGLE BETWEEN TWO LINES


The angle between two lines is an angle subtended by two intersecting lines.
Consider the figure below which show angle between given two lines
L1 and L2  in the xy plane,

fig 4. 3

From the figure above sum of interior angles is equal to exterior angle
  1   2
   2  1 .......... .......... .......... .......... .....( i )
Apply tan both sides to equation (i)
tan   tan  2  1 
tan  2  tan 1
tan   but tan 1  m1 slope of L1 , tan  2  m2 solpe of L2
1  tan 1 tan  2
m2  m1
tan   where m2  m1
1  m1m2
 m2  m1 
Formula used to find angle between lines   tan 1  
 1  m1 2 
m

94
Example 3
Find the acute angle between the lines 3 x  2 y  8  0 and x  5 y  7  0 .
Solution
Determine first the slopes of each equation
3 x  2 y  8  0, m2  3
2

x  5 y  7  0, m1  1
5

 m  m1 
From   tan 1  2 
 1  m1m2 
1  2  5

3 1
  tan   
 1   
1 3 
5 2 
  tan 1 1  45
 The angle between the lines is 45

4.4 PARALLEL AND PERPENDICULAR LINES


(a) Parallel lines
Parallel lines are line which does not intersects and the angle between them is
equal to zero (i.e. no angle between them).
Condition for two lines to be parallel is shown below,

fig 4. 4

95
m2  m1
From angle formula tan   but   0  for parallel lines
1  m1 m2
m2  m1
tan 0  
1  m1 m2
m2  m1
0
1  m1 m2
m2  m1  0
Condition for parallel lines m1  m2
Example 4
Find the equation of the line parallel to the line 3 x  2 y  7  0 and passing
through point 1,2
Solution
Given 3 x  2 y  7  0, m1  32
But m1  m2  3
2

The equation which has m2  32 and pass through 1,2


From y  m x  x0   y0
y 3
2 x  1  2
2 y  3x  3  4
 3x  2 y  1  0

(b) Perpendicular lines


Perpendicular lines are lines which are orthogonal to each other. These lines
makes right angle to each other.
Consider the sketch below which shows the orthogonal lines

fig 4. 5

96
m2  m1
From angle formula tan   but   90 for orthogonal lines
1  m1m2
m2  m1
tan 90 
1  m1m2
sin 90 m  m1
 2
cos90 1  m1m2

1 m2  m1

0 1  m1m2
1  m1m2  0
Condition for orthogonal lines m1m2  1

Example 5
Find the equation of the line perpendicular to the line 3 x  4 y  2  0 and
passing through point 1,3
Solution
Given 3 x  4 y  2  0, m1  34
But from m1m2  1, m2   43
The equation which has m2   43 and pass through 1,3
From y  m x  x0   y0
y   43  x  1  3
3 y  4 x  4  9
 4x  3y  5  0

4.5 THE EQUATION OF PERPENDICULAR BISECTOR


The equation of perpendicular bisector is an equation which divides two given
points into two equal parts. This equation passes through the midpoint of the
given points and it is perpendicular to the line joining the given two points.

97
Consider the sketch below which represent the above statements.

fig 4. 6

Procedures
(a) Determine the slope of required equation by using the condition m1m2  1
(b) Determine the midpoint of a line AB
(c) Finally we have a slope and one point, assume another point as ( x, y ) then
find the required equation.

Example 6
Find the equation of the perpendicular bisector of AB where A and B are the
points A3,4 and B1,2
Solution
Consider the figure below in which point P2,1 is the midpoint of AB

98
The midpoint between points A and B is Px, y   P2,1
The slope of a line joining AB is m1  3
Line AB is perpendicular to the required line, m1m2  1
Therefore m2   13
The required equation has m2   13 and pass through P2,1
From y  m2 x  x0   y0
y   13 x  2  1
 The equation of bisector is x  3 y  5  0

4.6 THE PERPENDICULAR SHORTEST DISTANCE FROM A POINT TO


A LINE

The perpendicular distance from point to a line is the shortest distance


measured from a point to a line.

Consider the figure below;

fig 4. 7

We need to find the shortest distance formula from P to a line ax  by  c  0

99
Let the required distance PB  d
Consider the triangle PQB
PQ x2  x1
cos  
PB d
x2  x1  d cos .......... .......... .......... .......... .......... .......... .......... .......... .......... .(i )
QB y2  y1
sin   
PB d
y2  y1  d sin  .......... .......... .......... .......... .......... .......... .......... .......... .......... ..(ii)
Point x2, y2  lies on the line ax  by  c  0, thereforethe point satisfy the equation,
ax2  by2  c  0
ax1  d cos   b y1  d sin    c  0
ad cos  bd sin   ax1  by1  c  0
 ax1  by1  c 
d .......... .......... .......... .......... .......... .......... .......... .......... ........( iii)
a cos  b sin 
The line joining points P and B is perpendicular to a line ax  by  c  0
Let the slope of line PB be m1  tan  .......... .......... .......... .......... .......... .......... ...( iv)
a
Let the slope of line ax  by  c  0 be m2   .......... .......... .......... .......... .......... .(v)
b
From the perpendicularity concepts m1m2  1
a
 tan   1
b
b
tan  
a
b
Now express tan   in term of cos and sin 
a
1  tan   sec 
2 2

b2 1
1 2 
a cos2 
a
cos  .......... .......... .......... .......... .......... .......... .......... .......... .......... ....( vi)
a 2  b2
b sin  b
Also from tan   , 
a cos a
b
sin   cos .......... .......... .......... .......... .......... .......... .......... .......... .......... ........( vii)
a

100
Substitute equation (vi) into equation (vii)
b a 
sin    
a  a 2  b 2 
b
sin   .......... .......... .......... .......... .......... .......... .......... .......... .......... .....( viii)
a2  b2
Substitute equations (vii) and (viii) into equation (iii)
 ax1  by1  c 
d
 a   b 
a   b 
2   2 2 
 a b   a b 
2

 ax1  by1  c 
d
 a2 b2 
  
2 
 a b a b 
2 2 2

 ax1  by1  c 
d
 a2  b2 
  
2 
 a b 
2

 ax1  by1  c   ax  by1  c  ax1  by1  c


d   1 

 a b 2 2
 a b 
2 2
a2  b2
ax1  by1  c
d
a2  b2
ax1  by1  c
d
The shortest distance formula a2  b2

Example 7
Find the shortest distance from point P2,3 to the line 4 x  3 y  3  0 .
Solution
4x  3y  3
d from  x, y   2,3
4   3
2 2

4(2)  3(3)  3 20
d  4
25 5
 The shortest distance is 4 units

101
Example 8
Find the shortest distance from point B3,4 to the line 2 x  y  3  0 .
Solution
2x  y  3
d from  x, y   3,4 
2   1
2 2

2(3)  4  3 1 1 5
d   
5 5 5 5
5
 The shortest distance is units
5

Example 9
Determine the ratio in which the line 3 x  y  9  0 divides the line segment
joining points 1,3 and 2,7 .
Solution
By using shortest distance formula
3x  y  9
d1  from 1,3
32  12
3(1)  3  9 3 3
d1    .......... .......... .........( i )
32  12 10 10
3x  y  9
d2  from 2,7 
3 12 2

3(2)  7  9 4 4
d1    .......... .......... .........( ii)
32  12 10 10
Divide equation (i) by equation (ii)
d1 3

d2 4
d1 : d 2  3 : 4
 The line 3x  y  9 divide the line segment in the ratio 3 : 4

102
Example 10
If the perpendicular distance of the point 1,1 from the straight line
3x  4 y  h is 0.2 units, find the possible values of h .
Solution
ax  by  c
From d 
a2  b2
3x  4 y  h
d from  x, y   1,1
32  4 2
3(1)  4(1)  h
d
25
7h
d
5
1
But d  0.2 
5
1  7  h 

5 5
1   7  h 
h  6 and h  8
 The possible values h are 6 and 8

4.7 THE EQUATIONS OF ANGLES BISECTOR


Equation of angle bisector is an equation which divide the given angle into two
equal angles. When two lines intersect together form four angles and each two
angles are equal (i.e. opposite angles are equal). Consider the sketch below
which represent above statements. Let L1 and L2 be two intersecting lines and
lines L3 and L4 are two required lines which bisects the given angles.

103
fig 4. 8

Let L1 be a1 x  b1 y  c1  0
L2 be a2 x  b2 y  c2  0
By shortest dustance formula,

a1 x  b1 y  c1
d1  .......... .......... .......... .....( i )
a1  b1
2 2

a2 x  b21 y  c2
d2  .......... .......... .......... ..(ii)
a2  b2
2 2

Since the required line bisect the angle then d1  d 2

a1 x  b1 y  c1 a2 x  b21 y  c2

a1  b1 a2  b2
2 2 2 2

a1 x  b1 y  c1  a xb y c 
The equations of bisectors are   2 21 2 

a1  b1
2 2  a
2
 b
2 
 2 2 

Note: To determine acute or obtuse angle bisector consider the following;


Step1; Check whether the constant terms of the equations i.e. c1 and c2 are
positive, if not make them positive.
Step2; If a1a2  b1b2  0 then   for obtuse angle and   for acute angle.
If a1a2  b1b2  0 then   for obtuse angle and   for acute angle.

104
Example 11
Find the equations of bisectors of the angles between the lines 6 x  7 y  11  0
and 2 x  9 y  3  0 .
Solution
ax  by  c Ax  By  C
From 
a 2  b2 A2  B 2
6 x  7 y  11 2x  9 y  3

6 2  (7) 2 22  92
6 x  7 y  11  2x  9 y  3 
  
85  85 
6 x  7 y  11  2 x  9 y  3
When is (), 6 x  7 y  11  2 x  9 y  3
 4 x  16 y  14  0
When is (), 6 x  7 y  11  2 x  9 y  3
8 x  2 y  8  0
 The equations of angles bisector are 2 x  8 y  7  0 and 4 x  y  4  0

4.8 DISTANCE BETWEEN TWO PARALLEL LINES


Consider the figure below

fig 4. 9

105
If the lines a1 x  b1 y  c1  0 and a2 x  b2 y  c2  0 are parallel then;
a1  a2  a.......... .......... .......... .......... .......... .......... ......( i)
b1  b2  b.......... .......... .......... .......... .......... .......... ......( ii)
c1  c2 .......... .......... .......... .......... .......... .......... .......... .(iii)

c2  c1 c1  c2
Distance, d  or d 
a2  b2 a2  b2

Note: If c1  c2 then d  0

Example 12
Find the length between lines 3 x  4 y  3  0 and 3 x  4 y  7  0
Solution
c2  c1 7  (3) 10 10
Distance, d     2
a 2  b2 32  4 2 25 5
 The length between lines is 2 units

Example 13
Find the distance between lines 15x  8 y  30  0 and 15x  8 y  21  0 .
Solution
c1  c2  30  21  51
From, d     3  3
a2  b2 152  8 2 17
 The distance between parallel line is 3 units

Alternative

By using shortest distance formula from the intercepts of any line.


Consider the figure below;
Consider 15x  8 y  30  0, x  intercept is 2,0
Consider the figure below;

106
Then find the distance from point 2,0 to a line 15x  8 y  21  0
ax  by  c 15x  8 y  21
From d   from 2,0
a2  b2 152  8 2
51
d  3 units
289
 The distance between line is 3 units

4.9 RATIO THEOREM


Ratio Theorem is the theorem used to determine the internal and external
division of a line joining two points. The two points can be divided internally or
externally.

Internal and External Division


(a) Internal Division
A line segments can be divided internally in the ratio m:n as shown below,

fig 4. 10

107
From the figure above the two triangles are similar, ACP ~ PDB
AC CP AP
 
PD DB PB
x  x1 y  y1 m
 
x2  x y 2  y n
x  x1 m y  y1 m
 and  .......... .......... .......... .......... .....( i )
x2  x n y2  y n
x  x1 m
Consider 
x2  x n
n x  x1   m x2  x 
nx  nx1  mx2  mx
mx  nx  mx2  nx1
m  n x  mx2  nx1
mx2  nx1
x .......... .......... .......... .......... .......... .......... .....( ii)
mn
y  y1 m
Also consider 
y2  y n
n y  y1   m y2  y 
ny  ny1  my2  my
my  ny  my2  ny1
m  n  y  my2  ny1
my 2  ny1
y .......... .......... .......... .......... .......( iii)
mn
Therefore internal division P x, y  is given by

x, y    mx2  nx1 , my2  ny1 


 mn mn 

Internal Division x, y    mx2  nx1 , my2  ny1 


 mn mn 

Example 14
Find the coordinates of a point which divide the line segment joining the points
A 2,3 and B4,3 internally in the ratio 1 : 2
Solution

108
Given that x1 , y1    2,3, x2 , y 2   4,3 and m : n  1 : 2
 mx  nx1 my 2  ny1 
From internal division formula, x, y    2 , 
 mn mn 

x, y    1(4)  2(2) , 1(3)  2(3)   0,1


 1 2 1 2 
 The coordinate which divide point A and B internally in the ratio 1 : 2 is 0,1

(b) External Division


Consider the figure below which show external division of the given two points

fig 4. 11

From the figure above the two triangles are similar, ACP ~ BDP
AC CP AP
 
BD DP BP
x  x1 y  y1 m
 
x  x2 y  y 2 n
x  x1 m y  y1 m
 and  .......... .......... .......... .......... .......... ......( i )
x  x2 n y  y2 n
x  x1 m
Consider 
x  x2 n
m x  x2   n x  x1 
mx  mx2  nx  nx1
mx  nx  mx2  nx1
m  n x  mx2  nx1
mx2  nx1
x .......... .......... .......... .......... .......... .......... .......... ......( ii)
mn

109
y  y1 m
Also consider 
y  y2 n
m y  y 2   n y  y1 
my  my 2  ny  ny1
my  ny  my 2  ny1
m  n  y  my2  ny1
my 2  ny1
y .......... .......... .......... .......... .......... .......... .......... ......( iii)
mn
The external division P( x, y ) is given by

x, y    mx2  nx1 , my2  ny1 


 mn mn 

External Division x, y    mx2  nx1 , my2  ny1 


 mn mn 

Example 15
Find the coordinates of a point which divide the line segment joining the points
A3,2 and B5,7  externally in the ratio 1 : 2
Solution
Given that  x1 , y1   3,2,  x2 , y 2   5,7  and m : n  1 : 2
 mx  nx1 my 2  ny1 
From external division formula, x, y    2 , 
 m  n mn 

x, y    1(5)  2(3) , 1(7)  2(2)   1,3


 1 2 1 2 
 The coordinate which divide point A and B externally in the ratio 1 : 2 is 1,3

TRISECTION POINTS OF A LINE SEGMENT


Trisection points are points which divide the line joining two points into three
equal parts.

Procedures
(a) The first point is obtained by taking internal division in the ratio 1: 2
(b) The second point is obtained by taking internal division in the ratio 2 : 1

110
Example 16
Find the two points of trisection of the line joining points A0,4 and B6,4
Solution
Given that A0,4 and B6,4 
 mx  nx1 my2  ny1 
From internal division formula x, y    2 , 
 mn mn 
1st point occurs when line AB is divided internally in the ratio 1 : 2
 1(6)  2(0) 1(4)  2(4) 
  x, y    ,   2,4 
 1 2 1 2 
2 st point occurs when line AB is divided internally in the ratio 2 : 1
 2(6)  1(0) 2(4)  1(4) 
  x, y    ,   4,4 
 2 1 2 1 
 The trisection points of a line AB are 2,4 and 4,4 

Example 17
Find the two points of trisection of the line joining points A 6,2 and
B12,8
Solution
Given that A 6,2 and B12,8
 mx  nx1 my2  ny1 
From internal division formula x, y    2 , 
 mn mn 
1st point occurs when line AB is divided internally in the ratio 1 : 2
 1(12)  2(6) 1(8)  2(2) 
  x, y    ,   0,4 
 1 2 1 2 
2 st point occurs when line AB is divided internally in the ratio 2 : 1
 2(12)  1(6) 2(8)  1(2) 
  x, y    ,   6,6 
 2 1 2 1 
 The trisection points of a line AB are 0,4  and 6,6 

111
4.10 LOCUS
Locus is the set of all points whose location is determined by stated conditions.

If a point moves on a plane satisfying some given geometrical condition then


the path trace out by the point in the plane is called its locus. By definition, a
locus is determined if some geometrical condition are given. Evidently, the co-
ordinate of all points on the locus will satisfy the given geometrical condition.
The algebraic form of the given geometrical condition which is satisfy by the
co-ordinate of all points on the locus is called the equation to the locus of the
moving point. Thus, the co-ordinates of all points on the locus satisfy its
equation of locus: but the co-ordinates of a point which does not lie on the
locus, do not satisfy the equation of locus. Conversely, the points whose co-
ordinates satisfy the equation of locus lie on the locus of the moving point.

Example 18
Find the equation to the locus of a moving point which is always equidistant
from the points (2, -1) and (3, 2).
Solution
Consider the figure below;

From distance formula d  x2  x1 2   y2  y1 2


d x  22   y  12 .......... .......... .......... .......... ......( i)
d x  32   y  22 .......... .......... .......... .......... ......( ii)
Equate equation (i) and (ii)
x  22   y  12  x  32   y  22
x  22   y  12  x  32   y  22
 The locus is a straight line x  3 y  4

112
Example 19
A and B are two given point whose co-ordinates are  5,3 and 2,4
respectively. A point P moves in such a manner that PA : PB  3 : 2 . Find the
equation to the locus traced out by P.
Solution
Consider the figure below;

From PA : PB  3 : 2
PA 3

PB 2
2 PA  3PB.......... .......... .......... .......... .......... .......... .......... .......... .......... .....( i )
PA  x  52   y  32 .......... .......... .......... .......... .......... .......... .......... .(ii)
PB  x  22   y  42 .......... .......... .......... .......... .......... .......... .......... (iii)
Substitute equations (ii) and (iii) into equation (i)
2 x  52   y  32  3 x  22   y  42
4x  5   y  3   9x  2    y  4  
2 2 2 2

 The locus is acircle 5 x 2  5 y 2  76x  48 y  108  0 with centre 7.6,4.8

Example 20
The sum of the distance of a moving point from the points c, o  and  c, o  is
always 2a units. Find the equation to the locus of the moving point.
Solution
Consider the figure below;

113
From distance formula d  x2  x1 2   y2  y1 2
d1  x  c 2  y 2 .......... .......... .......... .......... .......... .......... ......( i)
d2  x  c 2  y 2 .......... .......... .......... .......... .......... .......... .....( ii)
Since d1  d 2  2a
x  c 2  y 2  x  c 2  y 2  2a
x  c 2  y 2  2a  x  c 2  y 2
x  c 2  y 2  4a 2  4a x  c 2  y 2  x  c 2  y 2
x 2  2cx  c 2  y 2  4a 2  4a  x  c   y 2  x 2  2cx  c 2  y 2
2

4a 2  4a  x  c   y 2  4cx  0
2

a 2  cx  a x  c   y 2
2

a 2

 cx   a 2 x  c   y 2
2 2

a 4  2a 2cx  c 2 x 2  a 2 x 2  2a 2cx  a 2c 2  a 2 y 2
a 4  c 2 x 2  a 2 x 2  a 2c 2  a 2 y 2
a 2

 c 2 x 2  a 2 y 2  a 4  a 2c 2
 
 The locus is a a 2  c 2 x 2  a 2 y 2  a 4  a 2c 2

Example 21
Find the equation of the curve that is the locus of all points equidistant
from the line x  3 and the point 3,0 .
Solution
The point Px, y  is at equidistant from line x  3  0 and point 3,0
d1  x  32  y 2 .......... .......... .......... .......... .......... .......... .......... .....( i)
d 2  x  3 .......... .......... .......... .......... .......... .......... .......... .......... .......... (ii)

x3  x  32  y 2
x  32  x  32  y 2
y 2  12x
 The locus is a parabola y 2  12x

114
Example 22
A gun bullet moves so that its distance from town P is equal to its distance
from town M . If the location of the points P and M are represented by on the
Cartesian plane as the points 2,1 and 0,4 respectively, determine the locus
of the points traced by the motion of the bullet.
Solution
From distance formula d  x2  x1 2   y2  y1 2
Distance from P is d  x  22   y  12 .......... .......... ......( i)
Distance from M is d  x 2   y  4  .......... .......... .......... ...(ii)
2

Equate equation (i) and (ii)


x  22   y  12  x 2   y  42
x  22   y  12  x 2   y  42
 The locus is a straight line 4 x  10 y  11  0

Exercise 4.1
1. The slope of a line is twice the slope of another line. If the tangent of acute
1
angle between them is 3 , find the slope of the lines.
2. Find the locus of a point which is at equidistant from points 1,1 and 2,3
3. A point Px, y  is twice as far from the point 3,0 as it is from the line
x  5  0 . Find the locus of Px, y  .
4. Show that the line joining the points 2,3 and  5,1 is parallel to the
line joining the points 7,1 and 0,3 and perpendicular to the line
joining 4,5 and 0,2 .
5. A point moves so that the sum of its distance from points ae,0 and
x2 y2
 ae,0 is 2a . Prove that the locus is given by 2  2  1 where
a b
2 2

b  a 1 e . 2

115
4.11 CIRCLES
A circle is a locus of point which moves in a plane such that it is always at
equidistant from the fixed point.
The fixed point is called the centre point of the circle and the fixed distance is
called the radius of the circle.
Consider the figure below

fig 4. 12

From the point above c is a centre, r is radius and P( x, y ) is any point on the circumference of a circle.

4.11.1 EQUATIONS OF THE CIRCLES


Equations of the circles are categorized into many forms but according to
A-level syllabus we shall pass through three main forms;
(i) Equation of the circle with centre at the origin i.e. x  y  r
2 2 2

(ii) Equation of the circle with centre not at the origin


i.e. x  a   y  b  r 2
2 2

(iii)Equation of the circle passing through three points


i.e. x 2  y 2  2 gx  2 fy  C  0

(I) EQUATION OF THE CIRCLE WITH CENTRE AT THE ORIGIN


Consider the circle with centre at the origin as shown below

fig 4. 13

116
From the figure above consider triangle CPA

From CPA
2 2 2
CA  AP  CP
x2  y2  r 2
Equation of a circle with centre at origin x2  y 2  r 2
Example 23
Write down the equation of the circle whose centre is at the origin and the
radius is 3 unit.
Solution
Given that r  3
From x 2  y 2  r 2
x 2  y 2  32
 x2  y2  9

Example 24
Find the centre and radius of the circle x  y  2
2 2

Solution
Given that x 2  y 2  2 compare with x 2  y 2  r 2
 The centre of the given circleis 0,0  and r  2

Example 25
Find the centre and radius of the circle 3x 2  3 y 2  54
Solution
Given that 3 x 2  3 y 2  54
x 2  y 2  18 compare with x 2  y 2  r 2
 The centre of the given circleis 0,0  and r  3 2

117
Example 26
Write down the equation of the circle whose centre is at the origin and the
1
radius is 2 unit.
4

Solution
Given that r  2
1
4

From x 2  y 2  r 2
x2  y2  2
1
2

 x2  y2  2

(II) EQUATION OF THE CIRCLE WITH CENTRE NOT AT THE ORIGIN


Consider any circle whose centre is not at origin as shown below;

fig 4. 14

From the figure above consider triangle CPA

From CPA
2 2 2
CA  AP  CP
x  a 2   y  b2  r 2
The equation of the circle with centre not at the origin x  a2   y  b2  r 2

118
Example 27
Write down the equation of the circle whose centre is 2,3 and the radius is 4
unit.
Solution
Given that a, b   2,3 and r  4
From x  a    y  b   r 2
2 2

 x  2   y  3  4 2
2 2

Example 28
Write down the equation of the circle whose centre is  2,1 and the radius is 5
unit.
Solution
Given that a, b   - 2,1 and r  5
From x  a    y  b   r 2
2 2

 x  2   y  1  52
2 2

Example 29
Write down the equation of the circle whose centre is  4,3 and the radius is
7 unit.
Solution
Given that a, b    4,3 and r  7
From x  a    y  b   r 2
2 2

 x  4    y  3  7
2 2

Example 30
Find the centre and radius of the circle x  6   y  4  25
2 2

Solution
Given that x  6   y  4  25 compare with x  a    y  b   r 2
2 2 2 2

 The centre of the given circleis 6,4 and r  5

119
Example 31
Find the centre and radius of the circle x  1   y  2  3
2 2

Solution
Given that x  1   y  2  3 compare with x  a    y  b   r 2
2 2 2 2

 The centre of the given circleis  1,2 and r  3

Example 32
   2
Find the centre and radius of the circle x  2  y  3  7  2

Solution
  
2

Given that x  2  y  3  7 compare with x  a    y  b   r 2
2 2 2

 The centre of the given circleis  


2 , 3 and r  7

Example 33
Find the centre and radius of the circle x 1   y  k   k where k  .
2 2 2

Solution
Given that x  1   y  k   k 2 compare with x  a    y  b   r 2
2 2 2 2

 The centre of the given circleis 1, k  and r  k

Example 34
Find the centre and radius of the circle x  n   y  2n  16 where n   .
2 2

Solution
Given that x  n    y  2n   16 compare with x  a    y  b   r 2
2 2 2 2

 The centre of the given circleis n,2n  and r  4

120
Example 35
Show that for the point whose coordinates are given by
x  3 cos  2, y  3 sin  4 is a circle.
Solution
Given that x  3 cos  2, y  3 sin  4
x2
x  3 cos  2, cos  .......... .......... .......... .........( i )
3
y4
y  3 sin  4, sin  .......... .......... .......... .......... (ii)
3
Substitute equations (i) and (ii) into cos2   sin 2   1
 x2  y4
2 2

    1
 3   3 
  x  2    y  4   32
2 2

(III) EQUATION OF THE CIRCLE PASSING THROUGH THREE


POINTS
Consider the equation of the circle with the given centre and radius
x  a2   y  b2  r 2
From  x  a    y  b   r 2
2 2

x 2  2ax  a 2  y 2  2by  b 2  r 2
x 2  y 2  2ax  2by  a 2  b 2  r 2  0
Let a   g , b   f
x 2  y 2  2 gx  2 fy  g 2  f 2  r 2  0
Also let C  g 2  f 2  r 2
x 2  y 2  2 gx  2 fy  C  0
Equation of the circle passing through three points x 2  y 2  2 gx  2 fy  C  0

Note: Centre of the circle is given by a, b   g , f 


Radius, r  g 2  f 2  C where C is aconstant number.

121
Example 36
Find the centre and radius of the circle x 2  y 2  2 x  4 y  4  0
Solution
Given that x2  y 2  2x  4 y  4  0
Compare with x 2  y 2  2 gx  2 fy  c  0
g  1, f  2 and c  4
Centre   g , f    1,2 
Radius r   g 2  f 2  c  12  2 2  4  1
 The centre of the circleis  1,2  and its radius is 1

Example 37
Find the centre and radius of the circle x 2  y 2  6 x  8 y  0
Solution
Given that x2  y 2  6x  8 y  0
Compare with x 2  y 2  2 gx  2 fy  c  0
g  3, f  4 and c  0
Centre   g , f   3,4 
Radius r   g 2  f 2  c  (3) 2  4 2  5
 The centre of the circleis 3,4  and its radius is 5

Example 38
Find the centre and radius of the circle x 2  y 2  4 x  3  0
Solution
Given that x 2  y 2  4x  3  0
Compare with x 2  y 2  2 gx  2 fy  c  0
g  2, f  0 and c  3
Centre   g , f   2,0 
Radius r   g 2  f 2  c  2 2  0 2  (3)  7
 The centre of the circleis 2,0  and its radius is 7

122
Example 39
Find the centre and radius of the circle 3x 2  3 y 2  18x  12 y  15  0
Solution
Given that 3x 2  3 y 2  18x  12 y  15  0
x2  y 2  6x  4 y  5  0
Compare with x 2  y 2  2 gx  2 fy  c  0
g  3, f  2 and c  5
Centre   g , f   3,2
Radius r   g 2  f 2  c  32   2  5  2 2
2

 The centre of the circleis 3,2 and its radius is 2 2

Example 40
Find the equation of the circle passing through points
1,5, 6,2 and  2,2 .
Solution
Given points are  1,5, 6,2 and  2,2
From the standard equation of the circle passing through t hree given points
x 2  y 2  2 gx  2 fy  c  0. The given points satisfies x 2  y 2  2 gx  2 fy  c  0
At point  x, y    1,5
2 g  10 f  c  26.......... .......... .......... .......... .......... .......... .......... .(i )
At point  x, y   6,2
12g  4 f  c  40.......... .......... .......... .......... .......... .......... .......... (ii)
At point  x, y    2,2 
4 g  4 f  c  8.......... .......... .......... .......... .......... .......... .......... .....( iii)
Solve equations (i),(ii) and (iii) simulteneously
2 g  10 f  c  26

12g  4 f  c  40
4 g  4 f  c  8

g  3, f  2, c  12
Substitute the values of g  3, f  2, c  12 into x 2  y 2  2 gx  2 fy  c  0
 The equation of the circleis x 2  y 2  6 x  4 y  12  0

123
Additional Concepts of Equations of the circles
(a) Equation of the circle with two points as end’s of the diameter.
Consider the figure below which show the circle with a diameter joining two
points

fig 4. 15

Suppose Ax1 , y1  and Bx2 , y2  are the end’s points of the diameter, the
equation of the circle can be obtained through the following steps;
(i) Determine the centre by using midpoint formula,
x x y y 
i.e. C a, b   1 2 , 1 2 
 2 2 
(ii) Determine the radius by using distance formula,
 1 
i.e.  r  AC  BC  AB 
 2 
(iii)Finally substitute the above values into x  a    y  b  r
2 2 2

Alternatively
Consider the figure below

P x, y is any point on the circle.


since AP is perpendicular to PB then the product of their slopes is equal to  1
i.e. M AP  M PB  1

124
Example 41
Find the equation on the line joining the points  3,4 and 1,2 as the end
points of the diameter.
Solution
Consider the figure below

  3 1 4  2 
Centre a, b    ,    1,3.......... .......... ..(i )
 2 2 
Radius, r   3  12  4  22  5.......... .......... .(ii)
1
2

From x  a    y  b   r 2
2 2

 The equation of the circleis x  1   y  3  5


2 2

Example 42
If AB is a diameter of a circle and the coordinates of A and B are x1 , y1 
and x2 , y2  respectively, show that the equation of the circle is given by
x  x1 x  x2    y  y1  y  y2   0 .
Solution
Consider the figure below;

Line AP is perpendicular to line PB, therefore m1m2  1


 y  y1  y  y2 
    1
 x  x1  x  x 2 

 y  y1  y  y2   x  x1 x  x2 
 x  x1 x  x2    y  y1  y  y2   0

125
(b) Equation of the circle with two points on a circle and a line passing through
a centre of a circle.
Consider the figure below which show the circle passing through two given
points and a line passing through a centre of a circle,

fig 4. 16

Steps used to determine the equation of a circle


(i) Substitute points Ax1 , y1  and Bx2 , y2  into x 2  y 2  2 gx  2 fy  C  0 , then
two equations in term of g , f and C will be obtained.
(ii) The centre of a circle satisfy the linear equation, therefore the third
equation in term of g , f and C will be obtained.
(iii)Solve the three equations obtained simultaneously and finally substitute the
values of g , f and C into x 2  y 2  2 gx  2 fy  C  0 .

Example 43
Find the equation of the circle passing through points 1,1 and  1,2 whose
centre lies on the line x  2 y  1  0
Solution
Consider the figure below;

126
The given two points satisfies the equation x 2  y 2  2 gx  2 fy  c  0
At point 1,1
2 g  2 f  c  2.......... .......... .......... .......... .......... .......... .......... .(i )
At point  1,2 
2 g  4 f  c  5.......... .......... .......... .......... .......... .......... .......... .....( ii)
The centre  g , f  satisfy the line x  2 y  1  0
 g  2 f 1  0
g  2 f  1  0.......... .......... .......... .......... .......... .......... .......... ........( iii)
Solve equations (i), (ii) and (iii) simulteneous
2 g  2 f  c  2

2 g  4 f  c  5
g  2 f  1  0

g  52 , f   107 , c   75
Substitute the value of g , f and c into x 2  y 2  2 gx  2 fy  c  0
x 2  y 2  2 52 x  2 107 y  75  0
 The equation of the circleis 5 x 2  5 y 2  4 x  7 y  7  0

(c) Equation of the circle with the given centre and tangent line
Tangent is a straight line which touches the circle at only one point.
Suppose a line ax  by  c  0 is a tangent to the circle
x 2  y 2  2 gx  2 fy  C  0
Consider a tangent line to the circle as shown below

fig 4. 17

127
Steps used to determine the equation of a circle
(i) Determine the radius by shortest distance formula,
Ax  By  C
r  CP  from (a, b)
A B2 2

(ii) The centre is given and radius we have from (a) substitute them into
x  a2   y  b2  r 2 or x 2  y 2  2 gx  2 fy  C  0

Example 44
Find the equation of the circle whose centre is at the origin and line
x  4 y  5  0 is a tangent to the circle.
Solution
Consider the figure below

From the figure above radius can obtained by shortest distance formula
ax  by  c x  4y  5
d  from 0,0 
a2  b2 12  4 2
5
d r
17
From x 2  y 2  r 2
 The equation of the circleis x 2  y 2  25
17

Example 45
Find the equation of the circle whose centre is 1,1 and line x  4 y  5  0 is a
tangent to the circle.
Solution
Consider the figure below

128
From the figure above radius of the circle can be determined by shortest
Ax  By  C
distance formular, d 
A2  B 2
1  4(1)  5
and a, b   1,1
10
r 
12  4 2 17
From x  a    y  b   r 2
2 2

 The equation of the circleis 17 x 2  17 y 2  34x  34 y  66  0

4.11.2 POINT(S) OF INTERSECTIONS OF A LINE AND A CIRCLE


At point of intersection a line and the circle share same coordinates as shown
below,

fig 4. 18

129
Let d be shortest distance from the centre to the line;
(i) If d  r a line touches the circle (i.e. a line is a tangent) or b 2  4ac
(ii) If d  r a line cuts a circle or b 2  4ac
(iii)If d  r a line neither touch nor cuts the circle or b 2  4ac
4.11.3 TANGENTS
Tangent is a straight line which touches the circle at only one point.

fig 4. 19

Condition for a line to be tangent to the circle


There are two method used to show a certain line to be a tangent to the circle;
(a) By using shortest distance formula, if the shortest distance from centre to
the line is the same as radius of the circle that line is tangent to the circle,
i.e. d  r.
(b) Condition for perfect square, substitute a line into the circle and then form
the quadratic equation in term of x or in term of y and then check if the
discriminant is zero i.e. b 2  4ac .

Example 46
Show that line 5 x  12 y  4  0 touches the circle x 2  y 2  6 x  4 y  12  0 .
Solution

130
If a line touches the circle r  d which mean that radius (r ) of a circleis equal to the shortest
distance (d ) from the centre to a line.
Given circle x 2  y 2  6 x  4 y  12  0
g  3, f  2 and c  12
r  g 2  f 2  c  (3) 2  2 2  12  1.......... .......... .......... .......... .......... ........( i )
5 x  12 y  4
d from centre 3,2
5 2  122
5(3)  12(2)  4  13
d    1  1.......... .......... .......... .......... .......... .........( ii)
5 2  122 13
Since r  d  1
 The given line is a tangent t o the given circle

Example 47
Show that line x  y  2  0 touches the circle x 2  y 2  2 x  2 y  0 .
Solution
If a line touches the circle r  d which mean that radius (r ) of a circleis equal to the shortest
distance (d ) from the centre to a line.
Given circle x 2  y 2  2 x  2 y  0
g  1, f  1 and c  0
r  g 2  f 2  c  (1) 2  (1) 2  2.......... .......... .......... .......... .......... ......( i )

x y2
d from centre 1,1
1  (1)
2 2

1 1  2 2
d   2 .......... .......... .......... .......... .......... .......... .......... ..(ii)
12  (1) 2 2
Since r  d  2
 The given line is a tangent t o the given circle

Example 48
Show that line x  y  7  0 touches the circle x 2  y 2  2 x  4 y  3  0 .
Solution
If a line touches the circle r  d which mean that radius (r ) of a circleis equal to the shortest
distance (d ) from the centre to a line.

131
Given circle x 2  y 2  2 x  4 y  3  0
g  1, f  2 and c  3
r  g 2  f 2  c  12  (2) 2  (3)  2 2 .......... .......... .......... .......... .......... (i )

x y7
d from centre  1,2
12  (1) 2

1 2  7 4
d   2 2 .......... .......... .......... .......... .......... .......... .......... ..(ii)
12  (1) 2 2
Since r  d  2 2
 The given line is a tangent t o the given circle

Alternative
We need to show if line y  x  7 is tangent t o x 2  y 2  2 x  4 y  3  0
Substitute a line into circle and then apply condition for perfect square
x 2   x  7   2 x  4 x  7   3  0
2

x 2  6 x  9  0 compare with ax 2  bx  c  0
a  1, b  6, c  9
b 2  36 and 4ac  36
Since b 2  4ac condition for tangency
 The given line is a tangent t o the given circle

Alternative

132
We need to show if line x  y  7 is tangent t o x 2  y 2  2 x  4 y  3  0
Substitute a line into circle and then apply condition for perfect square
 y  7 2  y 2  2 y  7   4 y  3  0
y 2  8 y  16  0 compare with ay 2  by  c  0
a  1, b  8, c  16
b 2  64 and 4ac  64
Since b 2  4ac condition for tangency
 The given line is a tangent t o the given circle

EQUATIONS OF TANGENTS
Equations of tangents are equations which represent lines which touches the
circle at only one point or at point contact (tangency).

There are two main forms of equations of tangents


(a) Equation of a tangent to a circle at a given point
(b) Equation of a tangent to a circle from external point

(a) Equation of a tangent to a circle at a given point

(i) Equation of tangent at point x1 , y1  to the circle with centre at origin
Consider a tangent to the circle x  y  r at the given point x1 , y1  as shown
2 2 2

below

fig 4. 20

133
Radius of a circle is perpendicular to the tangent

Slope of radius m1  


y1
.......... .......... .......... .......... .......... .......... .......... ......( i )
x1
1 x
Slope of a tangent is obtained from m1m2  1, m2     1 .......... .(ii)
m1 y1

Now the equation of tangent is given as


x y  y1
 1 
y1 x  x1
 x1  x  x1   y1  y  y1 
 x1 x  x1  y1 y  y1
2 2

x1 x  y1 y  x1  y1 but x1  y1  r 2
2 2 2 2

x1 x  y1 y  r 2
The equation of tangent is x1 x  y1 y  r 2

Alternative derivation
Consider a tangent to the circle x  y  r at the given point x1 , y1 
2 2 2

The slope of tangent is the same as the slope of circle at point of contact.
x2  y2  r 2
dy
2x  2 y 0
dx
dy x

dx y

At tangenc y point  x1 , y1 , slope m   


x1
y1

134
Now the equation of tangent is given as
x y  y1
 1 
y1 x  x1
 x1  x  x1   y1  y  y1 
 x1 x  x1  y1 y  y1
2 2

x1 x  y1 y  x1  y1 but x1  y1  r 2
2 2 2 2

x1 x  y1 y  r 2
The equation of tangent is x1 x  y1 y  r 2

Example 49
Find the equation of tangent to the circle x 2  y 2  25 at point 3,4
Solution
Consider the figure below

Slope of normal line m1  4


3

From m1m2  1, m2   34


Equation of tangent w ith m2   34 and P3,4
From y  mx  x0   y0
y   34 x  3  4
 Tangent line is 3x  4 y  25  0

Alternative
dy dy x
From x 2  y 2  25, 2 x  2 y  0, 
dx dx y
At point P3,4 the slope is m   34
From y  mx  x0   y0
y   34 x  3  4
 Tangent line is 3 x  4 y  25  0

135
(ii) Equation of tangent at point x1 , y1  to the circle with centre at Ca, b
Consider a tangent to the circle x  a    y  b  r at the given point
2 2 2

x1 , y1  as shown below

fig 4. 21

Radius of a circleis perpendicular to the tamgent


y b
Slope of radius m1   1 .......... .......... .......... .......... .......... .......... .......... .......... .....( i )
x1  a
1 x a
Slope of a tangent is obtained from m1m2  1, m2   1 .......... .......... .......( ii)
m1 y1  b
Now the equation of tangent is given as
x  a y  y1
 1 
y1  b x  x1
  x1  a  x  x1    y1  b  y  y1 
 
 x1 x  x1  ax  ax1  y1 y  y1  by  by1
2 2

 x1 x  x1  ax  ax1  y1 y  y1  by  by1
2 2

x1 x  y1 y  ax  by  ax1  by1  x1  y1 but x1  y1  r 2


2 2 2 2

x1  a x   y1  b  y  ax1  by1  r 2

The equation of tangent is x1  a x   y1  by  ax1  by1  r 2

Note:
From x1  a x   y1  b  y  ax1  by1  r 2
But a   g , b   f
x1  g x   y1  f y  gx1  fy1  r 2
The equation of tangent is x1  g x   y1  f y  gx1  fy1  r 2

136
Alternative Derivation
Consider  x  a    y  b   r 2
2 2

2 x  a   2 y  b   0
dy
dx
dy xa

dx y b
xa
At  x1 , y1 , slopem   
y b
Now the equation of tangent is given as
x  a y  y1
 1 
y1  b x  x1
 x1  a x  x1    y1  b  y  y1 
 
 x1 x  x1  ax  ax1  y1 y  y1  by  by1
2 2

 x1 x  x1  ax  ax1  y1 y  y1  by  by1
2 2

x1 x  y1 y  ax  by  ax1  by1  x1  y1 but x1  y1  r 2


2 2 2 2

x1  a x   y1  b y  ax1  by1  r 2


Equation of tangent x1  a x   y1  b  y  ax1  by1  r 2

Example 50
Find the equation of tangent to the circle x 2  y 2  2 x  4 y  20  0 at point
4,6 .
Solution

Consider the figure below

137
Slope of normal line m1  4
3

From m1m2  1, m2   34


Equation of tangent w ith m2   34 and P4,6
From y  mx  x0   y0
y   34 x  4  6
 Tangent line is 3x  4 y  36  0

Alternative
dy dy dy x 1
From x 2  y 2  2 x  4 y  20  0, 2 x  2 y  2  4  0, 
dx dx dx y2
At point P4,6 the slope is m   34
From y  mx  x0   y0
y   34 x  4  6
 Tangent line is 3x  4 y  36  0

Alternative
dy dy dy x 1
From x 2  y 2  2 x  4 y  20  0, 2 x  2 y 24  0, 
dx dx dx y2
At point P4,6 the slope is m   34
y
From m 
x
3 y6
 
4 x4
 Tangent line is 3x  4 y  36  0

138
(b) Equation of a tangent to the circle from external point
Consider the tangent to the circle from external point Px1 , y1  as shown below

fig 4. 22

Let m be the slope of the tangent, then the equation of the tangent is given as
y  y1
m
x  x1
y  y1  mx  x1 
mx  x1   y  y1  0
mx  y  mx1  y1  0
The shortest distance d  from the centre to the tangent is equal to the radius r  of the circle,i.e. d  r 
Apply shortest distance formula in order to determine the slope(s)of the tangent(s)

Example 51
Find the equation of the tangents from the origin to the circle
x 2  y 2  4x  2 y  4  0 .
Solution
Consider the figure below

139
From x 2  y 2  4 x  2 y  4  0
g  2, f  1, c  4
Centre   g , f   2,1
Radius r   g 2  f 2  c   22   12  4  1
ax  by  c
By using shortest distance formula d 
a2  b2
mx  y
d r from 2,1
m 2  (1) 2
2m  1
1
m2  1
m 2  1  2m  1 square both sides

m 2  1  2m  1
2

3m 2  4m  0
m3m  4   0
m  0, m  4
3

But the required equation is y  mx


y  0, y  43 x
 The equations of tangents are y  0, 4 x  3 y  0

Example 52
Find the equation of the tangents from 2,11 to the circle x 2  y 2  25 .
Solution
Consider the figure below

140
From x 2  y 2  25.......... .......... .......... .......... .......... .......... ......( i )
Centre  C 0,0
Radius r   5
y  11
Determine the general equation of tangent, m 
x2
mx  y  2m  11  0.......... .......... .......... .......... .......... .......... .......( ii)
ax  by  c
By using shortest distance formula d 
a2  b2
mx  y  2m  11
d r from 0,0
m 2  (1) 2
 2m  11
5
m2  1
5 m 2  1   2m  11 square both sides

 
25 m 2  1   2m  11
2

25m 2  25  4m 2  44m  121


21m 2  44m  96  0
m  43 , m   247
But the required equation is mx  y  2m  11  0
 The equations of tangents are 4 x  3 y  25  0 and 24x  7 y  125  0

LENGTH OF A TANGENT FROM EXTERNAL POINT


Consider the tangent from external point Px1 , y1  to the circle
x 2  y 2  2 gx  2 fy  c  0

fig 4. 23

141
From the figure above the length of the tangent PB  L can be found by pythagoras thereom
where by radius r  of a circleis known and CB is length to be determined by distance formula,
2 2 2
CP  PB  CB
2
r 2  L2  CB
L2  CB  r 2 but r 2  g 2  f 2  c and CB   x1  g    y1  f   x1  y1  2 gx1  2 fy1  g 2  f 2
2 2 2 2 2 2


L2  x1  y1  2 gx1  2 fy1  g 2  f 2  g 2  f 2  c
2 2

L2  x1  y1  2 gx1  2 fy1  c
2 2

L  x1  y1  2 gx1  2 fy1  c
2 2

The length of the tangent from external point L  x1  y1  2 gx1  2 fy1  c


2 2

Note: Shortest distance from external point Px1 , y1  is given by


AB  BC  AC where AC  r

Example 53
Find the length of tangent from 6,1 to the circle x 2  y 2  4
Solution
Consider the figure below

2 2 2
CP  PB  CB
2
r 2  L2  CB
4  L2  37
L2  33
 The length of tangent from 6,1 to x 2  y 2  4 is 33 unit

142
Example 54
Find the length of tangent from  10,2 to the circle x 2  y 2  4 x  6 y  4  0
Solution
Consider the figure below

2 2 2
CP  PB  CB
2
r 2  L2  CB
r 2  g 2  f 2  c  (2) 2  32  4  9
CB   10  2  2  3  169
2 2 2

9  L2  169
L2  160
 The length of tangent from  10,2 to x 2  y 2  4 x  6 y  4  0 is 160 unit

4.11.4 INTERSECTION OF TWO CIRCLES


There are three categories of circles intersections which are;
(a) Internally intersection
(b) Externally intersection
(c) Orthogonally intersection

(a) Internally intersection


Consider the intersection of the two circles below where by 'd ' is a distance
between centres,

d  r1  r2

fig 4. 24

143
(b) Externally intersection
Consider the intersection of the two circles below where by 'd ' is a distance
between centres,

d  r1  r2

fig 4. 25

(c) Orthogonally intersection


Orthogonal circles are circles which intersect at right angle

fig 4. 26

Condition for orthogonal circles


By applying pythagoras theorem
r1  r2  d 2 .......... .......... .......... .......... .......... .......... .......... .......... .......... .....( i )
2 2

But r1  g1  f1  c1 , r2  g 2  f 2  c2 and d 2   g1  g 2    f1  f 2 
2 2 2 2 2 2 2 2

Substitute the above values into equation (i)


g 1
2 2
  2 2

 f1  c1  g 2  f 2  c2   g1  g 2    f1  f 2 
2 2

g1  g 2  f1  f 2  c1  c2  g1  2 g1 g 2  g 2  f1  2 f1 f 2  f 2
2 2 2 2 2 2 2 2

 c1  c2  2 g1 g 2  2 f1 f 2
2 g1 g 2  2 f1 f 2  c1  c2 .......... .......... .......... .......... .......... .......... .......... ........( ii)

Condition for orthogonal circles 2 g1 g 2  2 f1 f 2  c1  c2 where


c1 and c2 are constants .

144
Example 55
Show that the circles x 2  y 2  8 x  6 y  21  0 and x 2  y 2  2 y  15  0 are
orthogonal.
Solution
Given x 2  y 2  8 x  6 y  21  0.......... .......... .......... ........( i )
g1  4, f1  3 and c1  21
x 2  y 2  2 y  15  0.......... .......... .......... .......... .......... ....( ii)
g 2  0, f 2  1 and c2  15
Condition for orthogonal is 2 g1 g 2  2 f1 f 2  c1  c2
2 g1 g 2  2 f1 f 2  2(4)(0)  2(3)(1)  6
c1  c2  21  (15)  6
Since 2 g1 g 2  2 f1 f 2  c1  c2
 The two given circlesare orthogonal

Alternative
Given x 2  y 2  8 x  6 y  21  0.......... .......... .......... ........( i )
g1  4, f1  3 and c1  21
r1  g1  f1  c1  4
2 2 2

Centre  4,3
x 2  y 2  2 y  15  0.......... .......... .......... .......... .......... ....( ii)
g 2  0, f 2  1 and c2  15
r2  g 2  f 2  c2  16
2 2 2

Centre  0,1
Condition for orthogonal circlesis r1  r2  d 2
2 2

r1  r2  4  16  20
2 2

d 2  4 2  (3  1) 2  20
Since r1  r2  d 2
2 2

 The two given circlesare orthogonal

`
145
Example 56
Show that the circles x 2  y 2  8 x  6 y  23  0 and x 2  y 2  2 x  5 y  16  0 are
orthogonal.
Solution
Given x 2  y 2  8 x  6 y  23  0.......... .......... .......... .......... .....( i)
g1  4, f1  3 and c1  23
x 2  y 2  2 x  5 y  16  0.......... .......... .......... .......... .......... ....( ii)
g 2  1, f 2   52 and c2  16
Condition for orthogonal is 2 g1 g 2  2 f1 f 2  c1  c2
2 g1 g 2  2 f1 f 2  7
c1  c2  23  16  7
Since 2 g1 g 2  2 f1 f 2  c1  c2
 The two given circlesare orthogonal

Example 57
A circle through the origin is orthogonal to both circles x 2  y 2  4 x  8 y  0 and
x 2  y 2  2 x  6 y  1  0 find the equation of the circle.
Solution
Let the required equation of a circle which pass through t he origin be x 2  y 2  2 gx  2 fy  0.........( i )
If x 2  y 2  2 gx  2 fy  0 is orthogonal to x 2  y 2  4 x  8 y  0
2 gg1  2 ff1  c  c1
 4g  8 f  0
g  2 f  0.......... .......... .......... .......... .......... .......... .......... .......... .......... .(ii)
If x 2  y 2  2 gx  2 fy  0 is orthogonal to x 2  y 2  2 x  6 y  1  0
2 gg 2  2 ff 2  c  c2
2 g  6 f  1.......... .......... .......... .......... .......... .......... .......... .......... .......... .(iii)
Solve equations (ii) and (iii) simulteneously
g  2 f  0

2 g  6 f  1
g  15 , f  101
Substitute g and f into equation (i)
x 2  y 2  2 15 x  2101  y  0
 The equation is 5 x 2  5 y 2  2 x  y  0

146
Example 58
If the circles x 2  y 2  2 y  8  0 and x 2  y 2  24x  hy  0 cut orthogonally,
determine the value of h .
Solution
If the circles x 2  y 2  2 y  8  0 and x 2  y 2  24x  hy  0 are orthogonal;
i.e. 2 g1 g 2  2 f1 f 2  c1  c2
2(0)(12)  2(1) h2   8
h  8

4.11.5 CONCENTRIC CIRCLES


Concentric circles are circles which have the same centre as shown below;

or
fig 4. 27

Example 59
Find the equation of the circle which passes through the centre of the circle
x 2  y 2  8 x  10 y  7  0 and is concentric with the circle x 2  y 2  4 x  6 y  4.5  0 .
Solution
The centre of x 2  y 2  8 x  10 y  7  0 is  4,5
The centre of x 2  y 2  4 x  6 y  4.5  0 is 2,3
Concetric circlesare cicles which have the samecentre.
The required circle pass through  4,5 and its centre is 2,3

147
Determine radius by distance formula r
r 2  2  4  3  5  102
2 2

x  a 2   y  b 2  r 2
x  22   y  32  102
x  22   y  32  100
x 2  y 2  4 x  6 y  87  0
 The equation of a circleis x  2   y  3  100 or x 2  y 2  4 x  6 y  87  0
2 2

4.11.6 CIRCLE THROUGH THE POINT OF INTERSECTION OF THE


GIVEN TWO CIRCLES
Consider the figure below

fig 4. 28

The equation of the circle passing through the intersection of two circles is
 
given by x 2  y 2  2 g1 x  2 f1 y  c1  k x 2  y 2  2 g 2 x  2 f 2 y  c2  0
i.e. c1  kc2  0 where k is the constant to be found.

Example 60
Find the equation of a circle which passes through the origin and cuts on the
intersection of the circles x 2  y 2  6 x  8  0 and x 2  y 2  2 x  2  0
Solution
x 2  y 2  6 x  8  k x 2  y 2  2 x  2  0
The required line passes through t he origin 0,0 i.e. point 0,0 satisfy the equation;
x 2  y 2  6 x  8  k x 2  y 2  2 x  2  0
8  2k  0
k 4
x 2  y 2  6 x  8  4x 2  y 2  2 x  2  0
 5 x 2  5 y 2  14x  0

148
4.11.7 RADICAL AXIS (COMMON CHORD) OF TWO CIRCLES
Radical axis or common chord is a line which join two points of intersection of
the two intersecting circles.
Consider the circles x 2  y 2  2 g1 x  2 f1 y  c1  0 and x 2  y 2  2 g 2 x  2 f 2 y  c2  0
which intersect and make a common chord as shown below;

fig 4. 29

The two circlesshare the same coordinates at points of intersection which mean that;
x 2  y 2  2 g1 x  2 f1 y  c1  x 2  y 2  2 g 2 x  2 f 2 y  c2
 x 2  y 2  2 g1 x  2 f1 y  c1  0
 2
 x  y  2 g 2 x  2 f 2 y  c2  0
2

2 g1  2 g 2 x  2 f1  2 f 2 y  c1  c2   0
The equation of radical axis or common chord is 2g1  2g2 x  2 f1  2 f 2 y  c1  c2   0

Example 61
Find the equation of the common chord between the circles given by the
equations x 2  y 2  2 x  2 y  7  0 and x 2  y 2  x  y  1  0
Solution
Consider the circles x 2  y 2  2 x  2 y  7  0 and x 2  y 2  x  y  1  0
 x 2  y 2  2 x  2 y  7  0
 2
 x  y 2  x  y  1  0
3x  y  6  0
 The equation of common chord is 3x  y  6  0

Example 62
Find the equation of the common chord between the circles given by the
equations x 2  y 2  x  y  23  0 and x 2  y 2  x  y  1  0
Solution

149
Consider the circles x 2  y 2  x  y  23  0 and x 2  y 2  x  y  1  0
 x 2  y 2  x  y  23  0
 2
 x  y 2  x  y  1  0
2 x  2 y  22  0
 The equation of common chord is x  y  11  0

GENERAL EXAMPLES
Example 63
Find the values of c such that the line x  y  c  0 shall be a tangent to a
circle x 2  y 2  4 x  2  0 , for each values of c find the coordinates of the
point of contact.
Solution
From line x  y  c  0, y  c  x
Substitute y  c  x into x 2  y 2  4 x  2  0
x 2  c  x   4 x  2  0
2

 
2 x 2  2c  4x  c 2  2  0

If a line is a tangent t o a circleshould satisfy perfect square, i.e. b 2  4ac 
2c  4  8c 2  2
2

c  22  2c 2  2
c 2  4c  0
 c  0, c  4
Hence,
Determine the points of contact for each value of c
When c  0, 2 x 2  4 x  2  0
x 2  2 x  1  0, x  1
When x  1, y  1 from y  c  x 
When c  4, 2 x 2  12x  18  0
x 2  6 x  9  0, x  3
When x  3, y  1 from y  c  x 
 The points of contacts are 1,1 and 3,1 for the values of c  0 and c  4 respectively

150
Example 64
If ' ' p and ' q ' are the lengths of the perpendicular from the origin to the lines
x cos  y sin   k cos 2 and x sec  y cos ec  k respectively, prove

that p 2  4q 2  k 2 .
Solution
Consider the figure below;

ax  by  c
From d 
a2  b2
x cos  y sin  k cos 2
p from 0,0
cos2   sin 2 
p  k cos 2
p 2  k 2 cos2 2 .......... .......... .......... .......... .......... ....( i )
x sec  y cosec  k
q from 0,0
sec2   cosec 2
q  k sin cos
2q  k sin 2
4q 2  k 2 sin 2 2 .......... .......... .......... .......... .......... ....( ii)
Add equations (i) and (ii)
p 2  4q 2  k 2 cos2 2  k 2 sin 2 2

p 2  4q 2  k 2 cos2 2  sin 2 2 
 p 2  4q 2  k 2

151
Example 65
Find the coordinate of B such that the straight line 2 y  x  16  0 is a
perpendicular bisector of the line joining point A(3,4) and B.
Solution
Consider the figure below;

Given that x  2 y  16.......... .......... .......... .......... .......... .......... .......... .........( i )
Slope of line x  2 y  16  0 is m  2
But line x  2 y  16  0 is perpendicular to line AB, thereforeslope of line AB is 12 .

and A 3,4


1
Equation of line AB with m 
2
1 y4
From 
2 x3
2 x  y  2.......... .......... .......... .......... .......... .......... .......... .......... .......... .........( ii)
Solve equations (i) and (ii) simultaneously to obtain the midpoint of line AB
 x  2 y  16

2 x  y  2
x  4, y  6
 4,6 is a midpoint of line AB
By using midpoint formular, determine the coordinates of point B
 x  x y  y2 
From, x, y    1 2 , 1 
 2 2 

 4,6   x1  3 , y1  4 
 2 2 
 The coordinate of point Bx1 , y1  is  5,8

152
Example 66
Find the equation of the circle which passes through point A(4,3) and touches
the line x  2 y  7  0 at the point B (3,2) .
Solution
Consider the sketch below;

The slope of line x  2 y  7  0 is m   12


Since line x  2 y  7  0 is perpendicular to a line BC
Slope of line BC is 2 from, m1m2  1
Equation of line BC
b2
2
a 3
2a  b  4.......... .......... .......... .......... .......... .......... .......( i )
Also from distance formula, BC  AC  r
r 2  a  3  b  2  a  4  b  3
2 2 2 2

a    
 6a  9  b 2  4b  4  a 2  8a  16  b 2  6b  9
2
  
2a  10b  12.......... .......... .......... .......... .......... .......... ..(ii)
Solve equations (i) and (ii) simultaneously
2 a  b  4

2a  10b  12
a  149 , b   89 .......... .......... .......... .......... .......... .......... (iii)
r 2  a  3  b  2  845
2 2
81 .......... .......... .......... .......... ....( iv)

153
From x  a    y  b   r 2
2 2

x  149 2   y  89 2  845
81

9 x  14  9 y  8  845
2 2

 9 x 2  9 y 2  28x  16 y  65  0

Example 67
Find a pair of parallel lines from the equation 4 x 2  4 xy  y 2  6 x  3 y  4  0 .
Then, find the distance between them.
Solution
4 x 2  4 xy  y 2  6 x  3 y  4  0

4 x 2  4 y  6x  y 2  3 y  4  0 
 4 y  6  4 y  62  4(4)y 2  3 y  4
x
2(4)
 22 y  3  2 25  2 y  3  25
x 
2(4) 4
4 x  2 y  3  5
 Pair of parallel lines 2 x  y  4 and 2 x  y  1

The distance between lines 2 x  y  4  0 and 2 x  y  1  0


c1  c2  4 1 5
d    5
a 2  b2 2 2  12 5
 The distance between lines is 5 units

154
Example 68
Find the equations of two orthogonal lines through (3,2) which make an angle
of 45 with the line y  2 x  1 .
Solution
Given that y  2 x  1, m1  2, make   45 with the required lines
m2  m1
tan  
1  m1m2
m2
tan 45 
1  2m
m2
1
1  2m
1  2m  m  2
m  3
Equation of a line which has m  3 and pass through 3,2
From y  mx  x0   y0
y  3 x  3  2
3 x  y  11  0.......... .......... .......... .......... .......... .......... ........( i )
Since the required lines are orthogonal to each other   90  
From m1m2  1
 3m  1
1
m
3
and pass through 3,2
1
Equation of a line which has m 
3
From y  mx  x0   y0

y
1
x  3  2
3
x  3 y  3  0.......... .......... .......... .......... .......... .......... .......... (ii)
 The equations are 3x  y  11  0 and x  3 y  3  0

155
Alternative
Given that y  2 x  1, m1  2, make an angle   45 with the required lines
m2  m1
From tan  
1  m1m2
m2
tan 45 
1  2m
m2
1
1  2m
1  2m  m  2
m  3
Equation of a line which has m  3 and pass through 3,2
From y  mx  x0   y0
y  3 x  3  2
3x  y  11  0.......... .......... .......... .......... .......... .......... ........( i )
m  m1
Also from tan   2
1  m1m2
m2
tan  45  
1  2m
m2
1 
1  2m
 1  2m  m  2
1
3m  1, m 
3
and pass through 3,2
1
Equation of a line which has m 
3
From y  mx  x0   y0

y
1
x  3  2
3
x  3 y  3  0.......... .......... .......... .......... .......... .......... .......... (ii)
 The equations are 3 x  y  11  0 and x  3 y  3  0

156
Example 69
Find the equations of two lines through (3,2) which make an angle of 45
with the line y  2 x  1 .
Solution
Given that y  2 x  1, m1  2, make an angle   45 with the required lines
m  m1
From tan   2
1  m1m2
m2
tan 45 
1  2m
m2
1
1  2m
1  2m  m  2
m  3
Equation of a line which has m  3 and pass through 3,2 
From y  m x  x0   y0
y  3 x  3  2
3x  y  11  0.......... .......... .......... .......... .......... .......... ........( i )
m  m1
Also from tan   2
1  m1m2
m2

tan  45   1  2m
m2
1 
1  2m
 1  2m  m  2
1
3m  1, m 
3
and pass through 3,2
1
Equation of a line which has m 
3
From y  mx  x0   y0

y
1
x  3  2
3
x  3 y  3  0.......... .......... .......... .......... .......... .......... .......... (ii)
 The equations are 3 x  y  11  0 and x  3 y  3  0

157
Example 70
Find the equations of two lines through (3,2) which make an angle of 45
with the line 2 x  y  1  0 and find the angle between them.
Solution
Equation of a line which has m  3 and pass through 3,2
From y  m x  x0   y0
y  3 x  3  2
3 x  y  11  0.......... .......... .......... .......... .......... .......... ........( i )
m  m1
Also from tan   2
1  m1m2
m2

tan  45   1  2m
m2
1 
1  2m
 1  2m  m  2
1
3m  1, m 
3
and pass through 3,2
1
Equation of a line which has m 
3
From y  m x  x0   y0

y
1
x  3  2
3
x  3 y  3  0.......... .......... .......... .......... .......... .......... .......... (ii)
 The equations are 3 x  y  11  0 and x  3 y  3  0
The angle between lines 3x  y  11  0 and x  3 y  3  0
 m2  m1   1 3 1  3 
10
  tan 1    tan 1  3   tan    90
 1  m1m2   1 1  0
 The angle between lines is 90

158
Example 71
Find the equation of the circle whose centre lie on the point of intersection of
the lines x  2 y  3  0 and 2 x  3 y  4  0 and radius of 6 units.
Solution
Determine the point of intersection to obtain the centre of a circle.
x  2 y  3  0

2 x  3 y  4  0
x, y   1,2
Centre  a, b   1,2 and r  6
x  a 2   y  b 2  r 2
x  12   y  22  6 2
 x 2  y 2  2 x  4 y  31  0

Example 72
Find the equation of the circle whose centre is at the point 2,1 and which
passes through the point 4,3 .
Solution
Consider the sketch below;

From the figure above, a, b   2,1


r 2  4  2   3  1
2 2

r 2  20
From x  a    y  b   r 2
2 2

  x  2   y  1  20
2 2

159
Example 73
Find the equation of the circle whose centre is at the point k ,2k  and which
passes through the points 0,7 and 4,3 .
Solution
Consider the sketch below;

From the figure above, a, b   k ,2k 


r 2  k   2k  7 
2 2

r 2  5k 2  28k  49.......... .......... .......... .(i )


r 2  k  4   2k  3
2 2

r 2  5k 2  20k  25.......... .......... .......... .(ii)


Equate equations (i) and (ii)
5k 2  28k  49  5k 2  20k  25
8k  24
k 3
a, b   3,6
r 2  10
From  x  a    y  b   r 2
2 2

  x  3   y  6   10
2 2

Example 74
Find the equation of the circle concentric with the circle
x 2  y 2  4 x  6 y  11  0 and passing through the point 5,4 .
Solution

160
Given that circle x 2  y 2  4 x  6 y  11  0 is concetricwith the required circle,mean that it has the
same centre with the required circle;
i.e. a, b    2,3
The required circle pass through 5,4
r 2  5  2  4  3  98
2 2

From, x  a    y  b   r 2
2 2

x  22   y  32  98

Consider the sketch below;

 The equation of the circleis x  2   y  3  98


2 2

Example 75
Find the equation of the circle with centre C 1,2 and orthogonal to the circle
x 2  y 2  6x  2 y 1  0 .
Solution
Let the required circle be x 2  y 2  2 gx  2 fy  c  0
Centre of the circleis a, b    g , f    1,2
g , f   1,2.......... .......... .......... .......... .......... .......... .......... .......... .......... ........( i)
Also given that x 2  y 2  6 x  2 y  1  0 is orthogonal with x 2  y 2  2 gx  2 fy  c  0
From 2 g1 g 2  2 f1 f 2  c1  c2
2(3) g  2 f  c  1 but g , f   1,2
2(3)  2(2)  c  1
c  9.......... .......... .......... .......... .......... .......... .......... .......... .......... .......... .......... .(ii)
Substitute g  1, f  2, c  9 into x 2  y 2  2 gx  2 fy  c  0
 x 2  y 2  2x  4 y  9  0

161
Example 76
Find the equation of the circle which is orthogonal to x 2  y 2  2 x  3  0 and
passes through the origin and the point 1,1 .
Solution
Let the required equation be x 2  y 2  2 gx  2 fy  c  0
At origin 0,0
c  0.......... .......... .......... .......... .......... .......... .......... .......... .......... ..(i )
At origin 1,1
g  f  1.......... .......... .......... .......... .......... .......... .......... .......... ....( ii)
Condition for orthogonal circles
2 gg1  2 ff1  c  c1 but g1  1, f1  0, c1  3 from x 2  y 2  2 x  3  0
 2 g  (3)
3
g   .......... .......... .......... .......... .......... .......... .......... .......... .........( iii)
2
Substitute equation (iii) into equation (ii)
1
f 
2
Substitute the values of g , f and c into x 2  y 2  2 gx  2 fy  c  0
 x 2  y 2  3x  y  0

Example 77
The two circles x 2  y 2  2 y  8  0 and x 2  y 2  24x  ky  4  0 cut orthogonally.
Find the value of k .
Solution
x 2  y 2  2 y  8  0.......... .......... .......... .....( i )
x 2  y 2  24x  ky  4  0.......... .......... .......( ii)
2 g1 g 2  2 f1 f 2  c1  c2
k
 2   8  (4)
2
k  4

162
Example 78
Find the tangent of the acute angle between the pair of lines whose equation is
3x 2  10xy  7 y 2  0

Solution
3x 2  10xy  7 y 2  0
10 y  100 y 2  84 y 2
x
6
5y  2y
x
3
3 3
y  x, m1 
7 7
y  x, m2  1
m2  m1 1  73 2
From tan   
1  m1m2 1  73 5
2
 tan of angle is
5

Example 79
Find the equation of a circle with 0,1 and 2,3 as end points of its diameter.
Solution
Consider the figure below;

From perpendicularity condition m1m2  1


 y  1  y  3 
    1
 x  0  x  2 
 y  1 y  3   xx  2
 x2  y 2  2x  4 y  3  0

163
Alternatively

Determine the centre a, b  and radius r and then


substitute into x  a    y  b   r 2
2 2

a, b    x1  x2 , y1  y2    0  2 , 1  3   1,2.......... .......... (i)


 2 2   2 2 
r x2  x1 2   y2  y1 2 1  02  2  12  2.......... ...(ii)

Substitute equations (i) and (ii) into x  a    y  b   r 2
2 2

x  12   y  22   2 
2

 x2  y 2  2x  4 y  3  0

Example 80
Find the distance between parallel lines x  2 y  5  0 and 2 x  4 y  12  0
Solution
Consider pair of equations
x  2 y  5  0

2 x  4 y  12  0
x  2 y  5  0

x  2 y  6  0
c1  c2  5  (6) 5
From d    units
a2  b2 12  2 2 5
5
 The distance between parallel is units
5

164
Alternative 1
Consider pair of equations
x  2 y  5  0

2 x  4 y  12  0
c1 c2 5  12  5 12 5
From d       
a1  b1
2 2
a2  b2
2 2
12  2 2 22  42 5 2 5 5

5
 The distance between parallel is units
5

Alternative 2
Consider pair of equations
x  2 y  5  0

2 x  4 y  12  0
x  intercept of x  2 y  5  0 is 5,0, then find distance from a point to a
line 2 x  4 y  12  0 and viceversa is true;
ax  by  c
From shortest distance formula d 
a2  b2
2 x  4 y  12
d from 5,0
22  42
2(5)  4(0)  12 2 1
d  
20 2 5 5
5
 The distance between parallel is units
5

Alternative 3
Consider pair of equations
x  2 y  5  0

2 x  4 y  12  0
x  intercept of 2 x  4 y  12  0 is 6,0, then find distance from a point to a
line x  2 y  5  0 and viceversa is true;

165
ax  by  c
From shortest distance formula d 
a 2  b2
x  2y  5
d from 6,0
12  2 2
6  2(0)  5 1 1
d  
5 5 5
5
 The distance between parallel is units
5

Alternative 4
Consider pair of equations
x  2 y  5  0

2 x  4 y  12  0
y  intercept of 2 x  4 y  12  0 is 0,3, then find distance from a point to a
line x  2 y  5  0 and viceversa is true;
ax  by  c
From shortest distance formula d 
a2  b2
x  2y  5
d from 0,3
12  2 2
0  2(3)  5 1 1
d  
5 5 5
5
 The distance between parallel is units
5

Example 81
Find the equation of a tangent to a circle x 2  y 2  25 which is parallel to the
line 2 x  y  4  0
Solution

166
The tangent line y  mx  c is parallel to 2 x  y  4  0
i.e. m  2
y  2 x  c .......... .......... .......... .......... .......... .......... ......(i)
The line y  2 x  c is tangent t o the circle x 2  y 2  25
x 2  2 x  c   25
2


5 x 2  4cx  c 2  25  0 
Condition for tangency b 2  4ac
4c2  45c 2  25
c  5 5.......... .......... .......... .......... .......... .......... .......... (ii)
 The equation of tangent is y  2 x  5 5

Exercise 4.2
1. Find the equation of a circle with centre 2,3 and which touches the x -
axis.
2. Prove that circles x 2  y 2  6 x  2 y  0 and x 2  y 2  3x  y  0 touch to
each other.
3. Show that the circles x 2  y 2  16x  12 y  75  0 and
5 x 2  5 y 2  32x  24 y  75  0 touch to each other and find the equation
of the common tangent.
4. Show that the line 3 x  2 y  13  0 is a tangent to the circle
x 2  y 2  13  0 but not to x 2  y 2  4 x  8 y  4  0 .
5. If line y  mx  c is a tangent to circle x 2  y 2  r 2 , show that
c 2  r 2 1  m2 . 
6. Show that the line y  mx  c will be tangent to circle x 2  y 2  r 2 , if
c 2  r 2 1  m2 . 

APPLICATIONS OF COORDINATE GEOMETRY

Coordinate geometry is one of the most important and exciting ideas of


mathematics. In particular it is central to the mathematics students meet at
school. It provides a connection between algebra and geometry through
graphs of lines and curves. The coordinate geometry is an important branch
167
of mathematics. It mainly helps us to locate the points in a plane. Its uses
are spread in all fields like trigonometry, calculus, dimensional geometry
etc.

 Digital World of Mobiles, Computers: The Coordinate Geometry is used in


each and every process of Digitronics. Whenever you open a PDF file or
Text file, where the images and texts are modified with the use of
coordinate geometry. Consider a PDF file with images, words, different
shapes, they’re placed in a 2D coordinate plane of X and Y axes. Distances,
Slopes, Trigonometry are also applicable here.

 Describing position of an object: The Coordinate Geometry is used to


find and describe the position of an object from it’s initial position, called
Origin, where, all the coordinate axes intersect. Say, a book is placed above
10 m from the ground, 20 m from the place the observer is sitting. The
width of room is 30 m. So, using this information, it’s easy to find the
coordinates and distance from observer and the object, i.e., book.
 Location of Aeroplanes or other Areal transports: We have all seen the
planes and it’s all depended on Coordinate Geometry that the plane reaches
it’s destination correctly. The RADAR technology is combined with
Coordinate Geometry to get the correct position and make it reach exact
destination.
 Map Projections: All of us use Apple or Google Maps every day to
navigate and commute. We see the graph like lines before loading the
maps. So, easily we can tell Coordinate Geometry is used. We also have
2D and 3D maps, all accurate.
 Latitude and Longitudes: Latitudes and Longitudes are completely
based on Coordinate Geometry.
 Global Positioning System (GPS).

REVISION EXERCISE 4
1. If ' ' p and ' q ' are the lengths of the perpendicular from the origin to the
lines x cos  y sin   k cos 2 and x sec  y cos ec  k respectively,
prove that p 2  4q 2  k 2 .
2. Find the equations of the lines, which pass through the point (4,5) and
make equal angles with the lines 5 x  12 y  6  0 and 3 x  4 y  7  0 .
3. The line 3 x  4 y  8  0 meets the y-axis at point A, the point C has the
coordinate ( 2,1) . The line through C perpendicular to the line 3 x  4 y  8
meet at B. Calculate the area of triangle ABC.

168
4. Find the coordinate of B such that the straight line 2 y  x  16  0 is a
perpendicular bisector of the line joining point A(3,4) and B.
5. Find the equation of the circle which passes through point A and touches
the line at the point B where A(4,3) and B (3,2) and the line
l : x  2y  7
6. Find the equations of tangent from the origin to the circle whose equation is
x 2  y 2  5 x  5 y  10  0
7. Find the equation of the circle through the origin and through the point of
intersection of the circles x 2  y 2  2 x  4 y  4  0
and x 2  y 2  8 x  4 y  6  0 .
8. Show that the circles x 2  y 2  4 x  5 y  6  0 and
x 2  y 2  5 x  4 y  9  0 intersect internally.
9. Find the distance between lines;
a) 15x  8 y  34  0 and 15x  8 y  31  0
b) nx  y   p  0 and nx  ny  r  0

10. Find the tangent of the acute angle between the pair of lines whose
equation is 3x 2  10xy  7 y 2  0
11. Find a pair of parallel lines from the
equation 4 x 2  4 xy  y 2  6 x  3 y  4  0 . Then, find the distance between
them.
12. Find the acute angle of the equation 3x 2  3xy  6 y 2  x  7 y  2  0 .
13. Sketch the diagram for the locus of point which move such that it covers a
distance ' a' units from the curve x 2  y 2  2 x  4 y  20 where a  5 .
14. Show that the common chord of two circles x 2  y 2  2 g1 x  2 f1 y  c1  0
and x 2  y 2  2 g 2 x  2 f 2 y  c2  0 is perpendicular to the line joining
their centers.

15. If y  m1 x  c1 and y  m2 x  c2 , show that the angle (  ) between these


 m2  m1 
lines when intersecting is given by   tan 1   .
 1  m1 2 
m
16. The slope of two lines l1 and l2 are m1 and m2 respectively. If the straight
m2  1
line enclose an angle of 45 then prove that m1 
m2  1
17. Find the equations of two orthogonal lines through (3,2) which make an
angle of 45 with the line y  1  2 x .
18. Find the angle between lines 3 x  4 y  12  0 and x  5
169
19. At what angle are lines whose equation are, ax  by  c  0 and
a  bx  a  by  d  0 inclined to each other?
20. If a  b , show that the curve ax2  by 2  1 and xy  3 intersect at right
angle.
21. A line parallel to the y  axis meets the y  x at P and the line
2

y  x  2 at Q , find the locus of the mid-point of PQ .


22. What is the equation of the circle whose centre lies on the line
x  2 y  2  0 and touches the positive axis.
23. The lines 3 y  4 x , 4 y  3 x and y  8 are tangents to the circle. Find the
equation of the circle?
24. Prove that the circles x 2  y 2  2 x  2 y  23  0 and
x 2  y 2  10x  7 y  31  0 touch to each other. Find the point of contact.
25. Find the equation to the circle through the origin and through the point of
intersection of the circles x 2  y 2  2 x  4 y  4  0 and
x 2  y 2  8x  4 y  6  0
x 2  y 2  4x  y  3  0 , x  y  x  y 1  0
2 2
26. Show that the circles and
x  y  14x  15 y  7  0 are co-axial.
2 2

27. Find the equation of the circle which passes through the origin and cuts
both circles x 2  y 2  8 y  12  0 and x 2  y 2  4 x  6 y  3  0 orthogonally.
28. If a point P is moving such that the lengths of tangents drawn from P to the
circles x 2  y 2  4 x  6 y  12  0 and x 2  y 2  6 x  18 y  26  0 are in
the ratio 2 : 3
29. Prove that the straight line joining any two of the four points
am1 , a / m1 , am2 , a / m2 , am3 , a / m3 , am4 , a / m4  is perpendicular to the
straight line joining the other two if m1m2 m3 m4  1 .
a
30. A triangle is formed by the three straight lines y  m1 x  ,
m1
a a
y  m2 x  , y  m3 x  . Prove that its orthocenter always lies on the
m2 m3
line x  a  0 .
31. Find the equations to the two straight lines which are parallel to the line
4 x  3 y  1  0 and at a distance 2 from it, and also the areas of the triangles
which these two lines respectively make with the coordinate axes.
32. Find the equation to a line at right angle to the line bx  ay  ab through the
point where it meets the x-axis.

170
33. Two perpendicular lines are drawn through the origin so as to form an
isosceles right –angled triangle with the line lx  my  n  0 . Show that
their equations are (l  m) x  (l  m) y  0 and (m  l ) x  (m  l ) y  0 .
34. Show that the angle between the lines 6 x 2  xy  y 2  0 is 45
35. Find the equations to the bisectors of the angle between lines
3x 2  4 xy  5 y 2  0
36. Express in a single equation the pair of perpendicular straight lines through
the origin, one of which is the line ax  by  0 .
37. Prove that all points on the line 11x  3 y  11  0 are equidistant from the
lines 12x  5 y  12  0 and 3 x  4 y  3  0
38. Find the equation to a line perpendicular to the line 3 x  4 y  5  0 which
passes through the point of intersection of the two lines 3 x  y  1  0 and
x y 3  0
39. Find the equation to the circle which passes through the points (0,0) , (3,1)
and (3,9) .
40. Find the equation to the diameter of the circle x 2  y 2  6 x  2 y  15  0
which, when produced , passes through the point (8,2)
41. Find the equation to the circle whose centre lies on the line 3 x  y  7  0
and which passes through the points (1,1) and ( 2,1) .
42. If O is the origin and P , Q are the intersections of the circle
x 2  y 2  4 x  2 y  20 and the straight line x  7 y  20  0 , show that OP
and OQ are perpendicular. Find the equation to the circle through O , P
and Q .
43. If y  mx is a tangent to the circle x 2  y 2  2 gx  2 fy  c  0 , show that
g  mf 2  c(1  m 2 )
44. The point a, b  is the middle point of a chord of the circle x 2  y 2  r 2 .
Show that the equation to the chord is ax  by  a 2  b 2 .
45. Find the equations to the circle which passes through the point 1,2 and
through the points of intersection of the circles x 2  y 2  2 x  3 y  7  0 and
x 2  y 2  3x  2 y  1  0
46. Find the equations to the circles passing through the points of intersection
of the circles x 2  y 2  18x  2 y  8  0 , x 2  y 2  26x  6 y  24  0 and
touching the straight line y  10
47. Show that the circle x 2  y 2  2ax  2ay  a 2  0 touches both the axes.
Find the radii of the two circles which pass through the point 16,2 and
touches both the coordinate axes.

171
48. Prove that the equation to the circle whose centre lies in the first quadrant,
which touches the x-axis and which passes through the points A(0,6) ,
B (0,24) is x 2  y 2  24x  30 y  144  0 . Find also the equation to the
other chord through the origin whose length is equal to that of the chord
AB from the centre.
49. A circle of radius 5 has its centre in the positive quadrant, touches the x-
axis and intercepts a chord of length 6 on the y-axis .Show that its equation
is x 2  y 2  8 x  10 y  16  0 . If y  mx is a tangent from the origin apart
from the x-axis, find m .
50. Find the equations to the two circles each of which touch the three circles
x 2  y 2  4a 2 , x 2  y 2  2ax  0 and x 2  y 2  2ax  0
51. Show that the circles x 2  y 2  4 x  2 y  11  0 and
x 2  y 2  4 x  8 y  11  0 intersect at right angles and find the length of
their common chord.
52. Show that the circle x 2  y 2  2 x  8 y  1  0 touches the x-axis and that the
point ( h, k ) of contact of the other tangent from the point (3,0) must
satisfy the condition h  2k  1
53. Find the equation of the straight line which is such that the x  axis bisects
the angle between it and the straight line 2 x  5 y  18
54. Find the values of ‘a’ and ‘b’ if the straight lines ax  5 y  7 and
4 x  by  5 intersect at point 2,1 . If the lines meet the x-axis at ‘A’ and
‘B’, find the length of AB.
55. The equation (1  m) x 2  2 y 2  (m  3) x  12 y  6  0 represent a circle. Find
the centre, radius and area of this circle.
56. Find the equation of two tangents from the origin to the circle
x 2  y 2  4x  2 y  4  0 .
57. Find the equation of two tangents from the point 2,11 to the
circle x 2  y 2  25 .
58. Prove that the line 2 x  3 y  27  0 is a tangent to the circle
x 2  y 2  8x  4 y  7  0 .
59. Show that the tangents drawn at the points 12,5 and 5,12 to the circle
are x 2  y 2  169 are perpendicular to each other.
60. Find the points of intersection of the line y  2 x  1 and the circle
x 2  y 2  2 y  4  0 . What is the normal at one of the points of
intersection?
61. Find the points of intersection of the line y  x  3 and the circle
x 2  y 2  2 x  2 y  1  0 . What are the tangents at the points of
intersection? Where do the tangents intersect?
172
62. Find the area of the triangle made by points of contact of tangents drawn
from the point A(16,10) to the circle ( x  5) 2  ( y  2) 2  16 , and the centre
O of the triangle.
63. Show that the circles with equations x 2  y 2  6 x  4 y  2  0 and
x 2  y 2  8 x  2 y  22  0 are orthogonal.
64. Find the angle at which the circles x 2  y 2  16 and
x 2  y 2  2 x  4 y  0 intersect each other. Find the equation of the
common chord and the points of intersection.
65. Find the equations of tangents to the circle x 2  y 2  2 x  4 y  20  0
through the point P (8,1) .
66. A tangent is drawn from the point (  a ,0) to a variable circle, centre (a ,0) .
What is the locus of the point of contact?
67. Find the possible value of k given that the point ( 4, k ) is the same distance
from 9 x  8 y  1  0 as ( 2,5) is from y  12 x  2 .
68. If y 2  16  ( x  1) 2 is an equation of a circle, verify whether its radius is
cos  cos2   15 where  is an angle made by the radius and the polar
axis.
69. The tangent to the circle x 2  y 2  2 x  6 y  5  0 at the point (3,4) meets
the x-axis at M . Find the distance of M from the centre of the circle.
70. Find the equations of the tangents to the circle x 2  y 2  6 x  4 y  5  0 at
the points where it the x  axis .
71. The tangent to the circle x 2  y 2  4 x  6 y  77  0 at the point (5,6) meets
the axes at A and B . Find the coordinates of A and B hence deduce the
area of triangle AOB .
72. Find the coordinates of the point where the common chord of the circles
x 2  y 2  4 x  8 y  5  0 and x 2  y 2  2 x  4 y  5  0 meets the line joining
their centres.
73. The two curves y  2 x 2  3 and y  x 2  5 x  3 intersect at two points, one
of which is in the fourth quadrant. Find the tangent of the acute angle
between these curves at this point.
74. Find the equation of the circle which passes through the centre of the circle
x 2  y 2  8x  10y  7  0 and is concentric with the circle
2 x 2  2 y 2  8x  12y  9  0
75. A variable line passes through the point (2,1) and meets the coordinate
axes at A and B . Find the equation of the locus of the midpoint of AB .
76. A circle which passes through the origin, cuts off intercepts of length
4 and 6 units on the positive x and y axes respectively. Find the equation

173
of the circle and equations of tangents to the circle at points other than
origin where it cuts the axes.
77. The perpendicular line from point A 1,2 to the straight line
3 x  2 y  14 intersects the line at point B . If the perpendicular is extended
1
to C in such a way that AB  BC determine the coordinate of C .
2
78. Find in what ratio the point P4b  2a,9c  a  divides the line joining the
points Aa  b,3c  5a  and B5b  3a,11c  3a  if P lies between the points
A and B .
79. Write down the equation of the line which makes an angle of 150 with
the x  axis and that cuts the y  axis at a distance of 3 units below the
origin.
80. Find the equation of the circle that passes through the point 0,1, 0,4 and
2,5 hence determine the equations of the tangents to this circle through
the origin.
81. Find the equation of the straight line which passes through the intersection
of the lines 3 x  2 y  4  0 and x  y  2  0 and forms the triangle with the
axes whose area is 8 square units.
82. A line forms a triangle in the first quadrant with the coordinate axes. If the
area of a triangle formed is approximately 94 square units and the
perpendicular line drawn from the origin to the line makes an angle 60 
with x  axis , find the equation of the line.
83. Find the perpendicular distance from the point P which divides AB
externally in the ratio 2 :1 to the line 12x  5 y  10  0 . Given that A and B
are  2,5 and 4,2 respectively.
84. Find the equations to the circles passing through the points of intersection
of the circles x 2  y 2  18x  2 y  8  0 , x 2  y 2  26x  6 y  24  0 and
touching the straight line y  10 .
85. Find the equation of the circle which touches circles
x 2  y 2  6 x  2 y  9  0 , x 2  y 2  2 x  12 y  33  0 and
x 2  y 2  12x  2 y  33  0 .
86. The midpoint of a line joining the points A3,0 and A5,6 is M . A point
Ct ,4t  is such that CM is perpendicular to AB . Calculate the value of t
and the area of a triangle ABC .
87. Line AB is divided internally in the ratio 2 : 1 by the point 1,1 and
externally in the ratio 5 : 2 by the point 4,7 . Find the coordinates of A and B .

174
Chapter Five
FUNCTIONS
Introduction
Functions is the relationship between x and y such that x is independent
variable and y is the dependent variable.
Function is denoted by f x   y

Examples of functions
 f x   x  1
 f x   x 2  6 x  9
 f x   5 x 3  3x 2  x  4
 f x   e x
 f  x   Inx
 f  x   log x
 f  x   cos x
x2
 f x  
x 1

5.1 IMPORTANT TERMS USED IN FUNCTIONS


(i)Intercepts of functions are points in which the curves cuts the axes.
 x intercept occur when the value of y is zero.
 y intercept occur when the value of x is zero.

(ii) Table of values


Table of values is the table which represents the values of the
functions.

(iii)Domain and range


Domain are the valued values of x of the given functions.
Range are the valued values of y of the given functions.

175
5.2 GRAPHS OF FUNCTIONS
(a) Graphs of polynomial functions

(i) Linear functions


Linear functions are functions of the form f ( x)  ax  b where
a, b   and a  0

Example 1
Draw a graph of f x   2 x  1
Solution
By using table of values
x  2 1 0 1 2
y 3 1 1 3 5

Graph of f x   2 x  1

Alternative
By using intercepts of y  2 x  1
x intercept  12 ,0
y intercept 0,1

176
(ii) Quadratic functions
Quadratic functions are functions of the form ax 2  bx  c  0 where
a, b, c   and a  0

Example 2
Draw the graph of f x   x
2

Solution
By using table of values
x 2 1 0 1 2
y 4 1 0 1 4

Graph of f x   x
2

Example 3
Draw the graph of f x    x 2
Solution
By using table of values
x 2 1 0 1 2
y 4 1 0 1 4

Graph of f x   x 2

177
(iii)Cubic functions
Cubic functions are functions of the form ax3  bx 2  cx  d  0 where
a, b, c, d   and a  0

Example 4
Draw the graph of f x   x 3
Solution
By using table of values
x 2 1 0 1 2
y 8 1 0 1 8

Graph of f x   x
3

178
(iv) Quartic functions
Quartic functions are functions of the form ax 4  bx3  cx 2  dx  e  0 where
a, b, c, d , e   and a  0

Example 5
Draw the graph of ax 4  bx3  cx 2  dx  e  0
Solution

(b) Graphs of exponential functions


Exponential functions are functions which are in the form of e ax or a x where
a   and a  0

Example 6
Draw the graph of e x
Solution

179
Example 7
Draw the graph of e  x
Solution

Example 8
Draw the graphs of e x and e  x on the same xy plane.
Solution

180
(c) Graphs of logarithmic functions

Example 9
Draw the graph of Inx
Solution

(d) Graphs of composite functions


These are functions in which the output of one function become the input of the
next function.
This mean that the range of the first function become the domain of the next
function.

Notation of composite functions


Let f x  be the first function and g x  be the second function. The composite
function can be written as fogx  , f g x  , or fog .

Properties of composite functions


(i) Composite functions are not commutative;
i.e. fogx   gof x 
(ii) Composite functions is associative;
i.e. fogohx    fogohx 

181
Example 10
Given that f  x   x 2  1 and g x  x  1 , find;
(a) fog (x )
(b) gof (x )
(c) fof
(d) fog(3)
Solution
Given that f  x   x 2  1, g  x   x  1
(a ) fog x    
2
x 1 1  x  2
(b) gof  x   x 2  1  1  x 2  2
 
(c) fof  x   x 2  1  1  x 4  2 x 2  2
2

(d ) fog3
From fog x   x  2
fog3  3  2  5
 fog3  5

Example 11
Given that f  x   19  x 2 and g  x   x 2  3 , solve for x if gof x   0 .
Solution
Given that f x   19  x 2 and g  x   x 2  3


gof x   19  x 2   3  19  x
2
2
 3  16  x 2
gof x   16  x 2 but gof  x   0
16  x 2  0
 x  4

182
(e) Graphs of rational functions
a
Rational functions are functions which are in form of where 'a' is a
b
numerator function and 'b' is a denominator function where b  0 .

Asymptotes of rational function


These are limiting points or values of a graphs where a curve of a graph cannot
pass.

Types of Asymptotes
(a) Vertical asymptotes
(b) Horizontal asymptotes
(c) Slant/Oblique asymptotes

(a) Vertical Asymptotes (V.A)


Vertical Asymptotes are values of x  b where a curve cannot cross or pass.
Vertical asymptotes are obtained from denominator functions. Vertical
Asymptotes occurs when denominator is zero.

Example 12
2x  5
Determine the vertical asymptote of the function y  .
x 1
Solution
2x  5
y
x 1
Vertical asymptote occurs when denominator is zero,
x 1  0
x 1
 Vertical asymptote is x  1

183
Example 13
x3
Determine the vertical asymptote of the function y  .
x2  4
Solution
x3
y 2
x 4
Vertical asymptote occurs when denominator is zero,
x2  4  0
x2  4
x  2
 Vertical asymptote are x  2 and x  2

Example 14
6x  7
Determine the vertical asymptote of the function y  .
x  x6
2

Solution
6x  7
y 2
x  x6
Vertical asymptote occurs when denominator is zero,
x  3x  2  0
x  3, x  2
 Vertical asymptote are x  3 and x  2

Example 15
2x  8
Determine the vertical asymptote of the function y  .
3x  1
Solution
2x  8
y
3x  1
Vertical asymptote occurs when denominator is zero,
3x  1  0
1
x
3
1
 Vertical asymptote is x 
3

184
(b) Horizontal Asymptotes (H.A)
Horizontal Asymptotes are values of y where a curve cannot pass.

Note:
(i) If the degree of numerator is less than the degree of denominator,
horizontal asymptote is y  0
(ii) If the degree of numerator is equal to the degree of denominator,
horizontal asymptote is given as ratio of “leading coefficient of numerator
to the leading coefficient of denominator ”
leading coefficient of numerator
i.e. y 
leading coefficient of denominator
(iii) If the degree of numerator is greater than the degree of denominator no
horizontal asymptote
Example 16
x3
Determine the horizontal asymptote of the function y  .
x2  7
Solution
since the degree of numerator is less than that of denominator,
 Then the horizontal asymptote is y  0

Example 17
x2  3
Determine the horizontal asymptote of the function y  2 .
x 2
Solution
since the degree of numerator and denominator are equal, then
leading coefficient numerator 1
y  1
leading coefficient denominator 1
 Then the horizontal asymptote is y  1

Example 18
x2  8
Determine the horizontal asymptote of the function y  2 .
2x  1
Solution
since the degree of numerator and denominator are equal, then
leading coefficient numerator 1
y 
leading coefficient denominator 2
1
 Then the horizontal asymptote is y 
2

185
Example 19
6x2  1
Determine the horizontal asymptote of the function y  2 .
2x  9
Solution
Since the degree of numerator and denominator are equal, then
leading coefficient numerator 6
y  3
leading coefficient denominator 2
 The horizontal asymptote is 3

(c) Slant/oblique Asymptotes


Slant/oblique asymptotes occurs when the degree of numerator is greater than
degree of denominator.
Slant asymptote is obtained by long division of numerator with denominator
Example 20
x2  1
Determine the oblique asymptote of the function y  .
x 1
Solution
x2  1
Given that y 
x 1
x 1
x 1 x 1
2

x2  x
 x 1
 x 1
2
x2  1
  x  1 
2
y
x 1 x 1
 Oblique asymptote is y  x  1

Example 21
3x 2  2
Determine the oblique asymptote of the function y  .
x 1
Solution

186
3x 2  2
Given that y 
x 1
3x  3
x  1 3x  2
2

3x 2  3x
3x  2
3x  3
5
3x 2  2
 3 x  3 
5
y
x 1 x 1
 Oblique asymptote is y  3 x  3

Tips for drawing graphs of rational functions


(i) Find intercepts
(ii) Find asymptotes
(iii)Draw the graph
(iv) State domain and range

Example 22
2x  5
Sketch the graph of the function y  hence determine the domain and
x 1
range.
Solution
2x  5
Given that y 
x 1
 x - intercept occur when y  0
2x  5
0
x 1
2x  5  0
x  2.5
 y - intercept occur when x  0
5
y  5
1
Vertical asymptote occur when x  1  0
x 1
2
Horizontal asymptote , y   2
1
y2
187
Sketch

Domain  x : x  , x  2.5
Range  y : y  , y  2
or
Domain  x : x   except x  2.5
Range  y : y   except y  2
or
Domain  x : all real numbers except x  2.5
Range  y : all real numbers except except y  2
or
Domain  x : x  2.5 and x  2.5
Range  y : y  2 and y  2

Example 23
x3  2x
Sketch the graph of the function y 
 
2 x2  5
hence determine the domain and

range.
Solution

188
x3  2x
Given that y 

2 x2  5
 x - intercept occur when y  0
x3  2x
0

2 x2  5 
x3  2x  0
 
x x2  2  0
x  0, x   2
 y - intercept occur when x  0
y0
 
Vertical asymptote occur when 2 x 2  5  0
x2  5  0
x 5
No horizontal asymptote
Slant asymptote is y  12 x

Sketch


Domain  x : x  , x   5 
Range  y : y  

189
Example 24
x 2  3x  2
Sketch the graph of the function y  hence determine the domain
x2  4
and range.
Solution
x 2  3x  2
Given that y 
x2  4
 x - intercept occur when y  0
x 2  3x  2
0
x2  4
x 2  3x  2  0
x  056, x  3.56
 y - intercept occur when x  0
2 1
y 
4 2
Vertical asymptote occur when x 2  4  0
x  2
Horizontal asymptote , y  1
Sketch

Domain  x : x  , x  2
Range  y : y  

190
Example 25
x2  4
Sketch and state domain and range of the graph if y  2
x 1
Solution
x2  4
Given that y 
x2 1
 x - intercept occur when y  0
x2  4
0 2
x 1
x 40
2

x  2
 y - intercept occur when x  0,
y4
Vertical asymptote occur when x 2  1  0
x  1
Horizontal asymptote , y  1
Sketch

Domain  x : x  , x  1
Range  y : y  1 and y  4

191
Example 26
x
Sketch and state domain and range of the graph if y 
x 9
2

Solution
x
Given that y 
x2  9
 x - intercept occur when y  0
x
0 2
x 9
x0
 y - intercept occur when x  0,
y0
Vertical asymptote occur when x 2  9  0
x  3
Horizontal asymptote , y  0
Sketch

Domain  x : x  , x  3
Range  y : y  

192
Example 27
3x
Sketch and state domain and range of the graph if y 
x 2 1
Solution
3x
Given that y 
x2 1
 x - intercept occur when y  0
3x
0 2
x 1
x0
 y - intercept occur when x  0,
y0
Vertical asymptote occur when x 2  1  0
x  1
Horizontal asymptote , y  0
Sketch

Domain  x : x  , x  1
Range  y : y  

193
Example 28
1
Sketch and state domain and range of the graph if y 
2 x
Solution
1
Given that y 
2 x
 x - intercept occur when y  0
1
0
2 x
0 1
 y - intercept occur when x  0,
y  0.5
Vertical asymptote occur when 2  x  0
x2
Horizontal asymptote , y  0
Sketch

Domain  x : x  , x  2
Range  y : y  , y  0

194
Example 29
x2  x  6
Sketch and state domain and range of the graph if y 
x 2 1
Solution
x2  x  6
Given that y 
x 2 1
 x - intercept occur when y  0
x2  x  6
0
x 2 1
x2  x  6  0
x  2, x  3
 y - intercept occur when x  0,
y6
Vertical asymptote occur when x 2  1  0
x  1
Horizontal asymptote , y  1
Sketch

Domain  x : x  , x  1
Range  y : y  1 and y  6

195
Example 30
2x 2
Sketch and state domain and range of the graph if y 
x  1x  2
Solution
2x 2
Given that y 
x  1x  2
 x - intercept occur when y  0
2x 2
0
x  1x  2
x0
 y - intercept occur when x  0,
y0
Vertical asymptote occur when x  1x  2  0
x  1, x  2
Horizontal asymptote , y  2
Sketch

Domain  x : x  , x  1 and x  2
Range  y : y  0, and y  2

196
Example 31
x 1
Sketch and state domain and range of the graph if y 
x2  4
Solution
x 1
Given that y 
x2  4
 x - intercept occur when y  0
x 1
0 2
x 4
x 1
 y - intercept occur when x  0,
y  0.25
Vertical asymptote occur when x 2  4  0
x  2
Horizontal asymptote , y  0
Sketch

Domain  x : x  , x  2
Range  y : y  

197
GENERAL EXAMPLES

Example 32
If f ( x)  x 2  2 x  2 , find two functions g (x) for which fog( x)  x 2  4 x  5 .
Solution
Given that fog x   x 2  4 x  5.......... .......... .......... .......... ...( i )
f x   x 2  2 x  2.......... .......... .......... .......... .......... .......... .....( ii)
From fog x   f g  x   g  x   2 g  x   2  x 2  4 x  5
2

g x 2  2 g x   x 2  4 x  3  0
g x  

 2  4  4 x2  4x  3 
2

g x   1  1  x 2  4 x  3 
g x   1  x 2  4 x  4
g x   1  x  22
g x   1   x  2 
 g  x   x  3 and g  x   1  x

Alternative
Given that f  x   x 2  2 x  2.......... .......... .......... .......... ...( i )
fog x   x 2  4 x  5.......... .......... .......... .......... .......... .......( ii)
Since fog x  is quadratic equation then g  x  is linear
Let g  x   ax  b
fog x   ax  b   2ax  b   2
2

x 2  4 x  5  a 2 x 2  2ab  2a x  b 2  2b  2  
Equate then solve
1  a 2

 4  2ab  2a
5  b 2  2b  2

a  1, b  3, b  1
When a  1 , b  3
When a  1 , b  1
 g  x   x  3 and g  x    x  1

198
Example 33
Find the composite function fogx  and gof x  given that
f  3,6, 5,7, 9,0 and g  2,3, 4,5, 6,7
Solution
f g 2  f 3  6, 2,6
f g 4  f 5  7, 4,7 
f g 6  f 7  ,
 fogx   2,6, 4,7 
 gof x   3,7 

Example 34
Find the composite function gof x  given that f   2,1, 0,3, 4,5 and
g  1,1, 3,3, 7,9
Solution
gof  x    2,1, 0,3

Example 35
Given the functions f and g so that,
f 2  7, f 3  1, f 7  3 and g 1  3, g 3  7, g 7  2 . Find;
(i) gof 2
(ii) fog1
(iii) gofog7
Solution
(i ) gof 2  g  f 2  g 7  2
 gof 2  2

(ii) fog1  f g 1  f 3  1


 fog1  1

(iii) gofog7   g  f g (7)   g  f 2  g 7  2


 gofog7   2

199
REVISION EXERCISE 5
1. Sketch the graph of f ( x)  4
( x  1)( x  3)

2. Sketch the graph of f ( x)  x2  x  2


2

x  2x  3
3. Sketch the graph of f ( x)  3( x  3)
( x  1)( x  2)

4. Sketch the graph of f ( x)  x  3


2

x 1

5. Sketch the graph of f ( x)  4 x  3 x2  9 x  9


3 2

x 9
1
6. Sketch the graph of f ( x)  1  and identify its domain and range.
x2
7. The function f ( x)  x  x 2  9 x  9 has two real values to which a
3 2

x 9
function does not exist. Find these two values, define f ( x ) at these points
so as the function to be continuous.
8. Given that f ( x)  4 x 2  2 x  1 determine the range of values taken by
2

x  x 1
f ( x ) for real values of x.
9. Given that f ( x)  x 4  2 x 3  x 2  2 x , find the value of ' x' where the
curve f ( x ) cuts the x  axis .
10. Given f ( x)  x 2 and g ( x)  2 x  5 , solve the following equations,
(a) fog ( x)  9
(b) gog( x )  21
1
11. Given that f ( x)  3
x2
(a) Find the vertical and horizontal asymptote
(b) Sketch the graph of f ( x )
(c) Identify if the function is odd or even.
12. Sketch the curve 2 y  3 x  4 and identify its domain and range.
x( x  1)
13. If f ( x)  x 2  1 and g ( x)  x , find;
(a) gof (x )
(b) fog (x )

200
(c) Sketch the graph of gof (x ) and fog (x )
 4, x  1

14. Given f ( x)   x 2  1, 1  x  2
5, x2

(a) Sketch the graph of f ( x )
(b) State the domain and range of f ( x )
15. The function f and g are defined by f ( x)  x 2  3 and g ( x)  2 x  1
(a) Find fog (x )
(b) Find range of fog (x )
16. The function h and g are defined by h( x )  x 2 , 0  x  4 , g ( x )  x  3
where x   , sketch the graph of goh(x ) and state domain and range of
the function.
x 2  4x  3
17. Given f ( x) 
x2
(a) Sketch the graph of f ( x )
(b) Find its domain and range of f ( x )
(c) Find f (3)  f (5)
f ( 2.5)

18. Find the range of the function f : x  x  1 whose domain is given by x  3


a
19. The function f : x   b is such that f ( 2)  2 and f ( 1)  1
x
(a) Find the values of a and b
(b) Sketch the graph of f
x 2  2x  5
20. Find the horizontal asymptotes of the function
x2
1
21. The function f and g are defined as follows f ( x)  e  x and g ( x )  .
1 x
Find the ranges of f , g and gof
22. State the domain of; (a) f  x   4 x 2  9 and (b) f x   In1  x  .
23. Given that fogx   4 x 2  18x  20 and f x   x 2  3x  2 , find g x  .

201
Chapter Six
ALGEBRA
6.1 INDICES AND LOGARITHMS
Laws of indices and logarithms
(a ) logc  logc  logc
ab a b

a
(b) logc b  logc  logc
a b

an
 n logc
a
(c) logc
log a
(d ) logb 
a

log b
(e) loga  1
a

1
( f ) logb 
a
b
loga

Proof

 logc  logc
ab a b
(a) logc
Let m  logc and n  logc
a b

Change into exponential form


a  c m .......... .......... .......... .......... (i )
b  c n .......... .......... .......... .......... (ii)
Multiply equation (i) and equation (ii)
ab  c m  c n , ab  c m  n .......... .....( iii)
Change equation (iii) into logarithms form
m  n  logc but m  logc and n  logc
ab a b

 logc  logc
ab a b
logc

202
 logc  logc
ab a b
(b) logc

Let m  logc and n  logc


a b

Change into exponential form


a  c m .......... .......... .......... .......... (i )
b  c n .......... .......... .......... .......... (ii)
Divide equation (i) and equation (ii)
a cm a
 n,  c mn .......... .....( iii)
b c b
Change equation (iii) into logarithms form
a
m  n  logc b but m  logc and n  logc
a b

a
logc b  logc  logc
a b

n
(c) logc a  n logc a
Let m  logc
a

Express in exponential form a  c m


Raise to power n the above equation a n  c mn
Apply log to base c both sides
an c mn
logc  logc
an
 mn logc but logc  1
c c
logc
an
 nm but m  logc
a
logc
an
 n logc
a
logc

203
log a
(d) logb 
a

logb
Let m  logb
a

Express in exponential form a  b m


Apply log both sides
log a  log b m
log a  m log b
log a
m but m  logb
a

log b
log a
logb 
a

log b

(e) loga  1
a

log a
From logb  let a  b
a

log b
log a
loga  1
a

log a

loga  1
a

1
(f) logb 
a
b
loga
Let m  logb
a

Express in exponential form a  b m


Apply log to base ' a ' both sides
bm
loga  loga
a

1  m loga
b

1
m but m  logb
a
b
loga

1
logb 
a
b
loga

204
Example 1
If a  logb , b  logc and c  loga , prove that abc  1.
c a b

Solution
Take abc  logb  logc  loga
c a b

log c log a log b


abc   
log b log c log a
 abc  1

Example 2
3 x 5 x 5 x 3 x b
If a b  a b prove that log a  x log  .
a
Solution
a 3 x b 5 x  a 5  x b 3 x
b 5 x a 5 x

b 3 x a 3 x
b 2 x  a 2 2 x
b x  a1 x
b x  aa x
x
bx  b 
a x  
a a
Apply log both sides
x
b
log a  log 
a
b
 log a  x log 
a

Example 3
Prove that logb loga  1
a b

Solution
Consider L.H.S
log a logb log a logb
logb  loga    1
a b

logb log a log a logb


 logb loga  1
a b

205
Example 4
Solve log x  2 3 x  4 x 14 2
2

Solution
log x  2 
3 x 2
 4 x 14 2

Express into exponential form


x  22  3x 2  4 x  14
x 2  4 x  4  3 x 2  4 x  14
2 x 2  18
x  3

Example 5
Solve log10 32 log10
 1 x 
  0.
Solution
log10
3 2 log 10
1 x 
0
Express in exponential form

100  3  2 log10
1 x 

1 x 
1  3  2 log10
1 x 
 2  2 log10
1 x 
 1  log10
1  x  101
1
x  1
10
9
x  
10

206
Example 6
If x y  y 2 x and y 2  x 3 . Find the possible values of x and y .
Solution
Consider x y  y 2 x
Apply In both sides
yInx  2 xIny.......... .......... .......... .......... ...( i )
Also consider y 2  x 3
2 Iny  3Inx
3Inx
Iny  .......... .......... .......... .......... ......( ii)
2
Substitute equation (ii) into equation (i)
 3Inx 
yInx  2 x 
 2 
y  3x
Substitute y  3 x into y 2  x 3
3x 2  x 3
x3  9x 2  0
x 2 x  9  0
x  0, x  9
When x  0, y  3 x  0
When x  9, y  3 x  27
  x, y   0,0,  x, y   9,27

Example 7
y
a mx
If a     k m where a  1 ; show that y 
x
.
 
k m  x
Solution

207
y
a
Consider a    x

k
a
xIna  yIn 
k
xIna  y Ina  Ink 
xIna  yIna  yInk
x  y Ina   yInk.......... .......... .......... (i)
Also consider a x  k m
xIna  mInk
xIna
 Ink .......... .......... .......... .......... ..(ii)
m
Substitute equation (ii) into equation (i)

x  y Ina   y xIna 


 m 
xy
x y 
m
mx  my   xy
mx  my  xy
mx  m  x  y
mx
y 
mx

Exercise 6.1
1. Solve the equation log x  log5 giving the answer to three decimal places.
2x

1 1 1
2. Prove that   1
loga (abc) logb (abc) logc (abc)
1 1
3. Given that a 2  b 2  7 ab , prove that log (a  b)  (log a  log b) .
3 2

x y
2

  log x  log y  show that   7 .


1 2 x y
4. If log
 3  2 y x

208
6.2 PARTIAL FRACTIONS
Partial Fraction is the decomposition of rational function into simple parts.

FUNDAMENTAL THEOREM OF PARTIAL FRACTIONS


(a) The fraction should be in proper fraction form
(b) Linear factors none of which are repeated,
f ( x) A B Z
    
ax  bcx  d    ux  v  ax  b cx  d  ux  v 
i.e.

(c) Linear factors which are repeated,


f ( x) A B W Z
i.e.        
ax  bn ax  b1 ax  b2 ax  bn1 ax  bn
(d) Reducible Quadratic equation. If the quadratic equation is reducible
factorize the equation then apply theorem (b) above.
f ( x) f ( x) A B
  
ax 2  bx  c dx  f gx  h  dx  f  gx  h 
i.e.

(e) Irreducible Quadratic equations are equations which can’t be factorized,


f x  Ax  B Cx  D
 
i.e.
ax2  bx  c dx2  gx  h ax2  bx  c dx2  gx  h
(f) Repeated irreducible Quadratic factors,
f x  Ax  B Cx  D Kx  T
i.e.     

ax2  bx  c
n
 
ax2  bx  c
1
 
ax2  bx  c
2
 
ax2  bx  c 
n

(g) Linear and reducible quadratic factors,


f ( x) f ( x) A B C
   
i.e.
 
ax  b cx  dx  k ax  bdx  f gx  h ax  b dx  f  gx  h
2

(h) Linear and irreducible quadratic factors,


f ( x) A Bx  C
  2
i.e.
  
ax  b cx  dx  k ax  b cx  dx  k
2

(i) Linear factor and linear with repeated factors,
f ( x) A B C W Z
i.e.       
mx  nax  b mx  n ax  b ax  b
n 1 2
ax  b ax  bn
n1

209
(j) If the given fraction is improper fraction divide first to make it proper
fraction.
f ( x) h( x ) h( x )
i.e.  k ( x)  consider then partialize.
g ( x) g ( x) g ( x)

Note:
If the degree of numerator is greater or equal to the degree of denominator
divide by long division to make it proper fraction.
x3 x2 x3  2x
eg 2 , , etc
( x  9) x 2  5 x  6 ( x  3)( x  4)

Example 8
Express the following as partial fractions
3x  4 x5
(a) (b)
( x  1)( x  2) ( x  2)( x  1) 2

Solution

3x  4
a  
A

B
x  1x  2 x  1 x  2
3x  4 A x  2  B x  1

x  1x  2 x  1x  2
3 x  4  A x  2  B x  1
3 x  4   A  B x  2 A  B
Equate
A B  3
 2 A  B  4
A  73 , B  2
3

3x  4 7 2
  
x  1x  2 3x  1 3x  2

210
x5 A B C
(b)   
( x  2)( x  1) 2
( x  2) ( x  1) ( x  1) 2
1

x5 A( x  1) 2  B ( x  1)( x  2)  C ( x  2)

( x  1) 2 ( x  2) ( x  1) 2 ( x  2)
x  5  A( x  1) 2  B ( x  1)( x  2)  C ( x  2)
x  5  A( x 2  2 x  1)  B( x 2  x  2)  C ( x  2)
Equate coefficients and constant of both sides
Coefficient of x 2 ; 0  A  B.......... .......... ..(i )
Coefficient of x; 1  2 A  B  C.......... .....( ii)
Constant; 5  A  2 B  2C.......... ....( iii)
Solve equations (i), (ii) and (iii) simultaneously
7 7 4
A  , B   and C  
9 9 3
7 7 4
 
x5
 9 1 9  3
( x  1) ( x  2) ( x  1) ( x  1)
2 2
( x  2)
x5 7 7 4
   
( x  1) ( x  2) 9( x  1) 9( x  1)
2 2
3( x  2)

Example 9
2
Express in partial fractions.
x 1
2

Solution
2 2

x  1  x  1 x  1
2

2 A B
 
x  1x  1 x  1 x  1
2  A x  1  B x  1
A  1, B  1
2 1 1
 
x  1x  1 x  1 x  1
2 1 1
 2  
x 1 x 1 x 1

211
Example 10
2x
Express in partial fractions.
x2 1
Solution
2x 2x

x  1  x  1 x  1
2

2x A B
 
x  1x  1 x  1 x  1
2 x  A x  1  B  x  1
A  1, B  1
2x 1 1
 
x  1x  1 x  1 x  1
2x 1 1
 2  
x 1 x 1 x 1

Example 11
x2
Express 2 in partial fractions.
x 1
Solution
x2
x2 1
x2 1
By long division  1  2 .......... .......... ........( i )
x 1
2
x 1
1
Express in partial fractions
x 1
2

1 1 A B
  
x  1  x  1 x  1 x  1 x  1
2

1  A x  1  B x  1
1 1
A ,B
2 2
1 1 1
 2  2
x  1x  1 x  1 x  1
1 1 1
  .......... .......... .......... .........( ii)
x  1 2 x  1 2 x  1
2

Substitute equation (ii) into equation (i)


x2 1 1
  1 
x 1
2
2 x  1 2 x  1

212
Example 12
x3
Express 2 in partial fractions.
x 1
Solution
x3
x 2 1
x3 x
By long division 2  x  2 .......... .......... ........( i )
x 1 x 1
x
Express 2 in partial fractions
x 1
x x A B
  
x  1  x  1 x  1 x  1 x  1
2

x  A x  1  B x  1
1 1
A ,B
2 2
x 1 1
 2  2
x  1x  1 x  1 x  1
x 1 1
  .......... .......... .......... .........( ii)
x  1 2 x  1 2 x  1
2

Substitute equation (ii) into equation (i)


x3 1 1
 2  x 
x 1 2 x  1 2 x  1

213
Example 13
1
Express in partial fractions.
x 1
3

Solution
1 1

x  1  x  1x 2  x  1
3
.......... .......... .......... ........( i )

1 A Bx  C
 
x  1x 2  x  1 x  1 x 2  x  1
 
1  A x 2  x  1   x  1Bx  C 
1  Ax 2
 x  1  Bx  C  B x  C
2

Equate
0  A  B.......... .......... .......... .......... .......... .......... ....( ii)

0  A  B  C.......... .......... .......... .......... .......... ........( iii)
1  A  C.......... .......... .......... .......... .......... .......... ....( iv)

From equation (iv) C  A  1 substitute into equation (iii)
0  A  B  A 1
2 A  B  1.......... .......... .......... .......... .......... .......... ......( v)
Solve equations (ii) and (v) simultaneously
A  B  0

2 A  B  1
1 1 2
A  , B   , C  A 1  
3 3 3
1 1 x2
 3  
x  1 3 x  1 3 x  x  1
2
 

214
Example 14
1
Express in partial fractions.
x 1
3

Solution
1 1

x 3  1  x  1x 2  x  1
.......... .......... .......... ........( i )

1 A Bx  C
 
x  1x 2  x  1 x  1 x 2  x  1
 
1  A x 2  x  1   x  1Bx  C 
1  Ax 2
 x  1  Bx  B  C x  C
2

Equate
0  A  B.......... .......... .......... .......... .......... .......... ....( ii)

0   A  B  C.......... .......... .......... .......... .......... ........( iii)
1  A  C.......... .......... .......... .......... .......... .......... ....( iv)

From equation (iv) C  1  A substitute into equation (iii)
0   A  B 1 A
2 A  B  1.......... .......... .......... .......... .......... .......... ......( v)
Solve equations (ii) and (v) simultaneously
A  B  0

2 A  B  1
1 1 2
A  , B   , C  1 A 
3 3 3
1 1 x2
 3  
x  1 3x  1 3x  x  1
2

215
Example 15
1
Express in partial fractions.
x  4x
3

Solution
1 1 1
 
 
x 3  4 x x x 2  4 xx  2 x  2
1 A B C
  
xx  2x  2  x x  2 x  2
1  Ax  2 x  2  Bx x  2  Cx x  2
A   14 , B  18 , C  18
1 1 1 1
   
xx  2x  2  8x  2 8x  2 4 x

Example 16
1
Express in partial fractions.
x  x2
3

Solution
1 1

x 3  x 2 x 2  x  1
1 A B C
  
x 2  x  1 x x 2 x  1
1  Ax  x  1  B x  1  Cx 2
 
A x 2  x  B x  1  Cx 2  1
A  C  0

Equate  A  B  0
B  1

A  1, B  1, C  1
1 1 1 1
 3    2
x x 2
x 1 x x

216
Example 17
4x  3
Express in in partial fraction.
x  12
Solution
4x  3 A B
 
x  1 x  1 x  12
2

4 x  3  Ax  1  B
4  A

 3   A  B
A  4, B  1
4x  3 4 1
  
x  1 x  1 x  12
2

Example 18
1
Express in partial fraction.
x  x6
2

Solution
1 1

x 2  x  6  x  3 x  2 
1 A B
 
x  3x  2 x  3 x  2
1  A x  2   B x  3
1 1
A ,B
5 5
1 1 1
 2  
x  x  6 5 x  2  5 x  3

217
Example 19
4 x 2  5x  4
Express in partial fraction.
 x  2 2

Solution
4 x 2  5x  4
Given that .......... .......... .........( i )
x  22
Let u  x  2, x  u  2.......... .......... .........( ii)
Substitute equation (ii) into equation (i)
4u  2  5u  2  4
2

u2
4u 2  21u  22 21 22
2
 4   2 but u  x  2
u u u
4 x  5x  4
2
21 22
  4 
x  22
x  2 x  22

Example 20
8x 2  x 1
Express in partial fraction.
x  12
Solution
8x 2  x  1
Given that .......... .......... .........( i )
x  12
Let u  x  1, x  u  1.......... .......... .........( ii)
Substitute equation (ii) into equation (i)
8u  1  u  2  1
2

u2
8u 2  15u  7 15 7
 8   but u  x  1
u2 u u2
8x 2  x  1 15 7
  8  
x  12 x  1 x  12

218
Example 21
x2  4
Express in partial fraction.
x  13

Solution
x2  4
Given that .......... .......... .......... ......( i )
x  13
Let u  x  1, x  u  1.......... .......... .........( ii)

Substitute equation (ii) into equation (i)


u  12  4  u 2  2u  5  1  2  5 but u  x  1
u3 u3 u u2 u3
x2  4 1 2 5
   
x  1 x  1 x  1 x  13
3 2

Exercise 6.2
2
1. Express into partial fractions.
1  x2
x 4  6 x3  5x 2  7 x
2. Express into partial fractions.
x3
x2  x  2
3. Express into partial fractions.
x  110

6.3 SERIES
Series is a sequence in which terms are separated by addition sign, for example
(i ) 1  2  3  4  5    
(ii) 3  5  7  9
(iii) 1  2  4  8  16    

(a) Finite and infinite series


Finite series is a series which has countable number of terms, for example
(i) 1  2  3  4  5  6
(ii) 1  3  5  7  9

Infinite series is a series in which the number of terms are uncountable, for
example

219
(i ) 1  2  3  4  5    
(ii) 1  3  5  7  9    
(iii) 1  2  4  8    

(b) Sigma notation


Sigma is letter which represent summation of terms in a given series.
Sigma is represented by a letter 

(c) Sum of the first n squares and cubic numbers


(i) Sum of natural number
Consider a series of natural numbers below
1 2  3  4  5  6    n
In sigma notation the above series can be written as
n
1 2  3  4  5  6    n   r
r 1

By using sum of terms of Arithmetic Progression AP 

S n  2 A1  (n  1)d 
n
2
n
r  2  (n  1) 
n

r 1 2
n

 r  2 n  1
n
r 1

 r  2 n  1
n
r 1

(ii) Sum of square of natural number


Consider the series below
12  2 2  32  4 2  52      n 2
In sigma notation the above series can be written as
n
12  2 2  32  4 2  52      n 2   r 2
r 1

220
n
Now consider  r  1
r 1
3
 r3

when r  1, 23  13
when r  2, 33  23
when r  3, 43  33
 
 
 
when r  n  2, n  13  n  23
when r  n  1, n 3  n  1
3

when r  n, n  13  n3
 r  1 
n
 r 3  n  1  13
3 3

r 1

 r  3r 2  3r  1  r 3   n  1  12
n
3 2
3

r 1

 3r  3r  1  n  1  1
n
2 3

r 1
n n n

 3r 2   3r  1  n  1  1
r 1 r 1 r 1
3

221
n

r 2

n
n  12n  1
r 1 6

Example 19

Find the sum of the following series

(a) 3  5  7      2n  1
Solution
3  5  7      2n  1
Express a series in sigma notation
n
3  5  7      2n  1   2r  1
r 1
n n n

 2r  1  2 r  1
r 1 r 1 r 1
n
n 
 2r  1  2 2 (n  1)   n
r 1
n

 2r  1  n(n  1)  n  n
r 1
2
 2n
n

 2r  1  nn  2
r 1

 3  5  7      2n  1  nn  2

(b) 
2  10  17      n 2  1 
222
Solution
2  10  17      n 2  1  
Express a series in sigma notation

   
n
2  10  17      n 2  1   r 2  1
r 1

 r   
n n n
2
 1   r 2  1 
n
n  12n  1  n  n 2n 2  3n  7
r 1 r 1 r 1 6 6

 r   
n
n
2
1  2n 2  3n  7
r 1 6

 2  10  17      n 2  1    n
6

2n 2  3n  7 

Example 20

Find the sum of the first n terms of the following series

(a) 2  3  3 4  4  5  5  6    
(b) 1 3  2  4  3 5  4  6    
Solution

(a) 2  3  3 4  4  5  5  6    
Separate terms of the above series and termine the general term
2  3  4  5      (n  1)
3  4  5  6      (n  2)
Combine the above series
2  3  3  4  4  5  5  6      (n  1)(n  2)
Represent a serie in sigma notation
n
2  3  3  4  4  5  5  6      (n  1)(n  2)   r  1r  2 
r 1

 r  1r  2   r 
n n n n n
2
 3r  2   r 2  3 r   2 
n
n  12n  1  3 n(n  1)  2n
r 1 r 1 r 1 r 1 r 1 6 2
n
nn  12n  1  9n(n  1)  12n n
 r  1r  2 
r 1 6
 n  12n  1  9(n  1)  12
6

 r  1r  2  6 2n   
n
n n 2
2
 12n  22  n  6n  11
r 1 3

 
n
  r  1r  2  
n 2
n  6n  11
r 1 3

223
(b) 1 3  2  4  3 5  4  6    
Separate terms of the above series and termine the general term
1 2  3  4    n
3  4  5  6      (n  2)
Combine the above series
1 3  2  4  3  5  4  6      n(n  2)
Represent a serie in sigma notation
n
1 3  2  4  3  5  4  6      n(n  2)   r r  2 
r 1

 r r  2   r 
n n n n
2
 2 r   r 2  2 r 
n
n  12n  1  n(n  1)
r 1 r 1 r 1 r 1 6
n
nn  12n  1  6n(n  1) n
 r r  2 
r 1 6
 n  12n  7 
6
n
  r r  2   n  12n  7 
n
r 1 6

(iii)Sum of cubic of natural number


Consider the series below
13  23  33  43  53      n 3
In sigma notation the above series can be written as
n
13  23  33  43  53      n 3   r 3
r 1

224
225
4 r 3  n  1n3  3n 2  3n  1  2n 2  n  2n  1
n

r 1

 
n
4 r 3  n  1 n 3  n 2
r 1
n
4 r 3  n 2 n  1n  1  n 2 n  1
2

r 1

n 2 n  1
2
n 
n 2

 r  3
  n  1
r 1 4 2 

2
n
n 
 r   n  1
3

r 1 2 

Example 21
Evaluate the following series
n n
(a)  r  32
r 1
(b)  r  1
r 1
3

Solution
n
(a)  r  3
r 1
2

 r  32   r 2  6r  9   r 2  6 r   9
n n n n n

r 1 r 1 r 1 r 1 r 1

n  12n  1  6 n n  1  9n


n

 r  3
n

2

r 1 6 2 
n

 r  3 n  12n  1  3nn  1  9n
n

2

r 1 6
n

 r  3 n  12n  1  3n 2  6n  n n  12n  1  18n  36


n

2

r 1 6 6

 
n
  r  3 
n
2n 2  15n  37
2

r 1 6

226
n

 r  1
3
(b)
r 1

 r  1   r 3  3r 2  3r  1   r 3   3r 2   3r  1
n n n n n n
3

r 1 r 1 r 1 r 1 r 1 r 1

n  12  3 n n  12n  1  3 n n  1  n


n 2

 r  1
n

3

r 1 4 6  2 
n
n2
 r  1  n  12  n n  12n  1  3n n  1  n
3

r 1 4 2 2

r  13  n n n  12  1 n  12n  1  3 n  1  1


n


r 1 4 2 2 

 
n

 r  13  n nn  12  2n  12n  1  6n  1  4


r 1 4

  r  1  n 3  6n 2  13n  12
n
3 n
r 1 4

Example 22
Find the sum of the following series
(a) 1  2  3  4      (2n  1)
(b) 12  2 2  32  4 2      (n  1) 2

Solution
(a) 1  2  3  4      (2n  1)
Include the general term to the above series
1  2  3  4      n      (2n  1)
Express a seriesin sigma notation
2 n 1
1  2  3  4      n      (2n  1)   r let N  2n  1
r 1
N
2n  1
 r  2 N  1  2 2n  1  1  n(2n  1)
N
r 1
2 n 1

 r  n(2n  1)
r 1

1  2  3  4      (2n  1)  n(2n  1)

227
(b) 12  2 2  32  4 2      (n  1) 2
Include the general term to the above series
12  2 2  32  4 2      (n  1) 2  n 2
Express in sigma notation
n 1
12  2 2  32  4 2      (n  1) 2  n 2   r 2 let N  n  1
r 1

N  12 N  1  n  1 n  1  12(n  1)  1
N
N
r
r 1
2

6 6
n 1

r 2

n
n  12n  1
r 1 6

12  2 2  32  4 2      (n  1) 2 
n
n  12n  1
6

Example 23
n 1 1 1 1
Find the sum of the first terms of a series        and
2  3 3 4 4  5 5 6
hence evaluate sum to infinite of a series
Solution
Determine the general term of a series
Consider 2  3  4  5      (n  1) and 3  4  5  6      (n  2)

228
Include the general term to the given series
1 1 1 1 1
     
2  3 3 4 4  5 5 6 (n  1)(n  2)
Express in sigma notation
1 1 1 1 1 n
 1 
         
2  3 3 4 4  5 5 6 (n  1)(n  2) r 1  (r  1)(r  2) 
1 A B
Partialize  
(r  1)(r  2) (r  1) (r  2)
1  A(r  2)  B (r  1)
Equate coefficient of like terms
2 A  B  1.......... .......... .......... .......... .....( i )
A  B  0.......... .......... .......... .......... ......( ii)
Solve the simulteneous equations
A  1 a B  1
1 1 1
Then  
(r  1)(r  2) (r  1) (r  2)
n
 1  n  1 1 
  (r  1)(r  2)     r  1  r  2 

r 1   r 1  
1 1
when r  1, 
2 3
1 1
when r  2, 
3 4
1 1
when r  3, 
4 5
 
 
 
1 1
when r  n  2, 
n 1 n
1 1
when r  n  1, 
n n 1
1 1
when r  n, 
n 1 n  2
n
 1  1 1 
 1  1
      and   (r  1)(r  2)   2
r 1  ( r  1)(r  2)  2 n2 r 1  

229
Example 24
n
1 n
Prove that  3k  23k  1  3n  1
k 1
Solution
n
1 n
We need to prove that  3k  23k  1  3n  1
k 1

1
Express in partial fraction
3k  23k  1
1 A B
 
3k  23k  1 3k  2 3k  1
1  A3k  1  B3k  2
1 1
A  and B  
3 3
1 1 1
 
3k  23k  1 33k  2 33k  1
n n
1 1 1
Therefore,   
k 1 3k  2 3k  1 k 1 33k  2  33k  1
n
1 1
Consider  33k  2  33k  1
k 1

1 1
When k  1, 
3 1 3  4
1 1
When k  2, 
3 4 3 7
1 1
When k  3, 
3  7 3 10



1 1
When k  n  2, 
33n  8 33n  5
1 1
When k  n  1, 
33n  5 33n  2
1 1
When k  n, 
33n  2 33n  1

230
n
1 1 1 1
Therefore    
k 1 33k  2  33k  1 3 1 33n  1
n
1 1 1
 3k  23k  1  3  33n  1
k 1
n
1 n
 
k 1 3k  2 3k  1 3n  1

Example 25
2
r r  1r  2
Express the expression in partial fractions, hence evaluate
n 
2 2
 and  .
r 1 r r  1r  2  r 1 r r  1r  2 
Solution
2 A B C
Express   
r r  1r  2  r r  1 r  2 
2  Ar  1r  2   Br r  2   Cr r  1
A  1, B  2 and C  1
2 1 2 1
   
r r  1r  2  r r  1 r  2 
Hence evaluate
n
2 n
1 2 1 

r 1 r r  1r  2 
   
r 1  r
 
r  1 r  2 
n
1 2 1 
Consider   r  r  1  r  2 
r 1  
1 2 1
When r  1,  
1 2 3
1 2 1
When r  2,  
2 3 4
1 2 1
When r  3,  
3 4 5
1 2 1
When r  4,  
4 5 6



1 2 1
When r  n  3,  
n  3 n  2 n 1
231
1 2 1
When r  n  2,  
n  2 n 1 n
1 2 1
When r  n  1,  
n  1 n n  1
1 2 1
When r  n,  
n n 1 n  2
n
1 2 1  1 2 1 1 2 1
Therefore,           
r 1  r r  1 r  2  1 2 2 n  1 n  1 n  2
n
2 1 1
 r r  1r  2  2  n  1n  2 .......... .......... .......... .......... .......... .(i)
r 1
n
2 nn  3
 
r 1 r r  1r  2  2n  1n  2

From equation (i) substitute n  



2 1 1
 r r  1r  2  2    1  2
r 1

2 1 1 1
 r r  1r  2  2  
r 1
but

0

2 1
 
r 1 r r  1r  2  2

Example 26
n 
2r 1 1 2r 2r
Given that 2 2
 2
r r  1 r r  1
2 , find 
r 1 r 2
r  12
and  r r  1
r 1
2 2

Solution

232
1 1
When r  n  1,  2
n  1 n
2

1 1
When r  n, 
n 2 n  12
 1n
1  1 1
Therefore,   
 r 2 r  12   12  n  12
r 1 


n
2r 1

r 1 r r  1
2 2
 1
n  12
.......... .......... .......... ........( i )

n
2r nn  2 
 
r 1 r r  1 n  12
2 2

From equation (i) substitute n  



2r 1
 r r  1
r 1
2 2
 1
  12

2r 1 1
 r r  1
r 1
2 2
 1

but

0


2r
 1
r 1 r 2
r  12

233
Example 27
1
nn  2 
Express in partial fractions and deduce that

1 1 1 1 3 2n  3
      
nn  2 4 2n  1n  2
and hence find
1 3 2  4 3  5
20
1

n1 nn  2

Solution
1 A B
Consider  
nn  2 n n  2
1  An  2  Bn
A  12 and B   12
1 1 1
  
nn  2 2n 2n  2

We need to deduce that


1 1 1 1 3 2n  3
      
1 3 2  4 3  5 nn  2 4 2n  1n  2
Consider the L.H.S and express in sigma notation
n
1 1 1 1 1
     
1 3 2  4 3  5 nn  2 n 1 nn  2

234
235
Example 28
2
nn  2 
Express in partial fractions and deduce that

1 1 1 1 3 2n  3
      
nn  2 4 2n  1n  2
and hence find
1 3 2  4 3  5
100
1

n1 nn  2

Solution
2 A B
Consider  
nn  2 n n  2
2  An  2  Bn
A  1 and B  1
2 1 1
  
nn  2 n n  2

236
n
1 n3n  5
 nn  2  2n  1n  2 .......... .......... .......... .......... ........( ii)
n 1

Substitute equation (ii) into equation (i)


n
1 1 n 1 1  1  n3n  5  n3n  5

n 1 nn  2 
       
2 n1  n n  2  2  2n  1n  2  4n  1n  2
But n  20
20
1 203(20)  5 325
  
n 1 nn  2  420  120  2 462

237
Example 29
5 8 11 3n  2
    
nn  1n  2
Find the sum of the series
1 2  3 2  3  4 3  4  5

3r  2
hence find  r r  1r  2
r 1
Solution
Express the given series into sigma notation
5 8 11 3n  2 n
3r  2
     
1 2  3 2  3  4 3  4  5 nn  1n  2 r 1 r r  1r  2 
n
3r  2
Consider  r r  1r  2 .......... .......... .......... .......... .......... .......... ..(i)
r 1

3r  2
Express into partial fraction
r r  1r  2
3r  2 A B C
  
r r  1r  2 r r  1 r  2
3r  2  Ar  1r  2  Br r  2  Cr r  1
A  1, B  1 and C  2
3r  2 1 1 2
   .......... .......... .......... .......... .......... .....( ii)
r r  1r  2 r r  1 r  2
Substitute equation (ii) into equation (i)
n
3r  2 n
1 1 2 

r 1 r r  1r  2 
 
r 1
 
 r r  1

r  2


1 1 2
When r  1,  
1 2 3
1 1 2
When r  2,  
2 3 4
1 1 2
When r  3,  
3 4 5



1 1 2
When r  n  3,  
n  3 n  2 n 1

238
1 1 2
When r  n  2,  
n  2 n 1 n
1 1 2
When r  n  1,  
n 1 n n 1
1 1 2
When r  n,  
n n 1 n  2
n
1 1 2  1 1 1 2 1 2
  
r 1  r
      
r 1 r  2  1 2 2 n 1 n 1 n  2
n
3r  2 2n  3
 r r  1r  2  2  n  1n  2
r 1
n
3r  2 2n 2  4n  1
  2
r 1 r r  1r  2  n  3n  2
n
3r  2 2n 2  4n  1
From,   2 divide by n 2 to each term on R.HS
r 1 r r  1r  2  n  3n  2
n
3r  2 2  4 n  1n2

r 1 r r  1 r  2 

1  3  2
n 2
when n  
n

3r  2 2  4   1 2

r 1 r r  1r  2 

1  3  2 2

3r  2
 2
r 1 r r  1r  2 

Example 30
r2
Given that G (r )  find the expression for G (r )  G (r  1) and hence
r r  1
n
 r4  3 n3
show that   rr  1r  2  2  n  1n  2
r 1  

Solution

239
r2
Given that G (r ) 
r r  1
r 1 2 r 3
G (r  1)  
r  1r  1  1 r  1r  2
Now, G (r )  G (r  1) 
r2

r 3

r  2  r r  3 
2
r4
r r  1 r  1r  2 r r  1r  2  r r  1r  2
r4
 G (r )  G (r  1) 
r r  1r  2
n
r4 3 n3
Hence we need to show that   
r 1 r r  1r  2  2 n  1n  2
n
r4 r4 r2 r 3
Consider  r r  1r  2 but r r  1r  2  G(r )  G(r  1)  r r  1  r  1r  2
r 1
n
r4 n
 r2 r 3 
     
  
r 1    

    

r 1 r r 1 r 2 r r 1 r 1 r 2 
3 4
When r  1, 
1 2 2  3
4 5
When r  2, 
2  3 3 4
5 6
When r  3, 
3 4 4  5



n n 1
When r  n  2, 
n  2n  1 n  1n
n 1 n2
When r  n  1, 
n  1n nn  1
n2 n3
When r  n, 
nn  1 n  1n  2
n
 r2 r 3  3 n3
 
 r r  1 r  1r  2   1 2   n  1n  2
r 1 


n
r4 3 n3
  
r 1 r r  1r  2  2 n  1n  2

240
Exercise 6.3
1. Find the sum of the following series
(a) 1  3  5      (2n  1)
(b) 2  4  6      2n
(c) 2  9  28      (n 3  1)
(d) 2  12  36      (n 3  n 2 )

2. Find the sum of the first n terms of the following series


(a) 2  3  4    
(b) 13  2 3  33    
(c) 1 2  2  3  3 4    
(d) 1 3  2  4  3 5    

3. Find the sum of the following series


(a) 3  6  11      (n 2  2)
(b) 22  42  62      2n
2

(c) 12  32  5 2      (2n  1) 2

4. Evaluate the following

 r  3r 2 
n
3
(a)
r 1

 5r 
4
(b)
3
 6r 2
r 1
n
5. By using standard result for r
r 1
3
evaluate the following

 r 
10 20 10
(a) r
r 1
3
(b) r
r 4
3
(c)
r 5
3
1

1
6. Express
4r  1 2r  3
2
in partial fraction and show that
n
1 1 1
 4r
r 1
2
 
 12r  3 12 42n  12n  3
hence find the sum to infinity.

241
n
1
7. Find the sum of  2r 12r  12r  3
r 1
and hence show that the sum to

1
infinity is .
12
n
2
8. Find the sum to infinity if the series  4r
r 1
2
1
converges.

2
9. Express in partial fractions and deduce that
nn  1n  2
1 1 1 1 1 1
       and hence
1 2  3 2  3  4 3  4  5 nn  1n  2 4 2n  1n  2
1
show that the sum to infinity is .
4

6.4 PROOF MATHEMATICAL INDUCTION


Mathematical induction “If the basis step and inductive steps are proven (or
proved) to be true, then the statement is true for all natural numbers”

This induction method use three inductive steps to prove proposition


(a) Prove if it is true for the first at least two numbers
(b) Assume it is true for n  k
(c) Prove if it is true for n  k  1 and then make conclusion.

242
Example 31
n

 r  2 n  1
n
By using mathematical induction, prove that
r 1
Solution
n

 r  2 n  1
n
Given proposition
r 1

Step1 : We need to prove if it is true for n  1,2


1
When n  1, r 
1
1  1
r 1 2
1  1 it is true for n  1.......... .......... .......... .......( i )
2

 r  2 2  1
2
When n  2,
r 1

2
2  1
1 2 
2
3  3 it is true for n  2.......... .......... .......... .......( ii)
Step 2 : Assume it is true for n  k
k

 r  2 k  1.......... .......... .......... .......... .......... .......... .......... (iii)


k
r 1

Step 3 : We need to prove if it is true for n  k  1


k 1
k 1
r  k  1  1
r 1 2
Consider L.H.S and then prove that is equal to R.H.S
k 1 k k

 r   r  k  1 but  r  k  1 from equation (iii)


k
r 1 r 1 r 1 2
k 1

 r  2 k  1  k  1
k
r 1
k 1
k k  1  2k  1 k  1k  2  k  1k  1  1
r 
r 1 2

2

2
k 1
k  1 k  1  1 but n  k  1

r 1
r
2
n
  r  n  1 The proposition is true for all posivive integers
n
r 1 2

243
Example 32
n
By using mathematical induction, prove that r 2

n
n  12n  1
r 1 6
Solution
n
Given proposition
n
n  12n  1
r 2

r 1 6
Step1 : We need to prove if it is true for n  1,2
1
When n  1, r 2 1
 1  12  1
r 1 6
1  1 it is true for n  1.......... .......... .......... .......... .........( i )
2
When n  2,
2
2  12(2)  1
r 2

r 1 6

12  2 2  2  12(2)  1
2
6
5  5 it is true for n  2.......... .......... .......... .......... ........( ii)
Step 2 : Assume it is true for n  k
k

r
k
2
k  12k  1.......... .......... .......... .......... .......... .......... .......... (iii)

r 1 6
Step 3 : We need to prove if it is true for n  k  1
k 1
k 1
r 2
k  1  12(k  1)  1

r 1 6
Consider L.H.S and then prove that is equal to R.H.S
k 1 k k

 r  r  k  1 k  12k  1 from equation (iii)


k
but  r 2 
2 2 2

r 1 r 1 r 1 6
k 1

r 2

k
k  12k  1  k  12
r 1 6
k 1
k k  12k  1  6k  1
2


r 1
r  2

6
k 1
k  1 k 2k  1  6k  1
r
r 1
2

6
k 1
k  1 k 2k  1  6k  1  k  1 2k 2  7k  6  k  1 k  22k  3
r
r 1
2

6 6 6
k 1
k  1 k  1  12k  1  1
r
r 1
2

6
but n  k  1
n
 r 2 
n
n  12n  1 The proposition is true for all posivive integers
r 1 6

244
Example 33
2
n
n 
By using mathematical induction, prove that  r   n  1
3

r 1 2 
Solution
2
n  n
Given proposition  r   n  1 3

r 1 2 
Step1 : We need to prove if it is true for n  1,2
2
1 1

When n  1,  r   1  1 3

r 1 2 
1  1 it is true for n  1.......... .......... .......... .......... ....( i )
2
2  2
When n  2,  r   2  1 3

r 1 2 
2
2 
1  2   2  1
3 3

2 
9  9 it is true for n  2.......... .......... .......... .......... .(ii)
Step 2 : Assume it is true for n  k
2
k
k 
 r   k  1 .......... .......... .......... .......... .......... .......... .......... (iii)
3

r 1 2 
Step 3 : We need to prove if it is true for n  k  1
 k 1
k 1 2

 r  3
k  1  1
r 1  2 
Consider L.H.S and then prove that is equal to R.H.S
k 1 2
k k
k 
r   r  k  1 but  r   k  1 from equation (iii)
3 3 3 3

r 1 r 1 r 1 2 
k 1 2
k 
 r   k  1  k  1
3 3

r 1 2 
2 k 
k 1 2

 r  k  1  2  k  1
3

r 1 2 
2  k  4k  4  2 k  2 
k 1 2 2
1
k  1k  2
2

 r 3
 k  1 
 
  k  1    
r 1  4   2  2 
 k  1
k 1 2

 r  3
k  1  1 but n  k  1
r 1  2 
2
n
n 
  r   n  1
3
The proposition is true for all posivive integers
r 1 2 

245
Example 34

 
n
By using mathematical induction, prove that 2
r 1
r
 2 2n 1

Solution

 
n
Given proposition 2
r 1
r
 2 2n 1

Step1 : We need to prove if it is true for n  1,2

 
1
When n  1, 2
r 1
r
 2 21  1

2  2 it is true for n  1.......... .......... .......... .........( i )

 
2
When n  2, 2
r 1
r
 2 22 1

21  2 2 2
 22  1
6  6 it is true for n  2.......... .......... .......... .......( ii)
Step 2 : Assume it is true for n  k

 
k

2
r 1
r
 2 2 k  1 .......... .......... .......... .......... .......... .......... .......... (iii)

Step 3 : We need to prove if it is true for n  k  1

 
k 1

2
r 1
r
 2 2 k 1  1

Consider L.H.S and then prove that is equal to R.H.S

 
k 1 k k

2  2
r 1
r

r 1
r
2 k 1
but  2 r  2 2 k  1 from equation (iii)
r 1

     
k 1

2
r 1
r
 2 2 k  1  2 k 1  2 k 1  2  2 k 1  2 2 k 1  2  2 2 k 1  1

 
k 1

2
r 1
r
2 2 k 1  1 but n  k  1

  The proposition is true for all posivive integers


n
  2r  2 2n 1
r 1

246
Example 35
n
1 1
By using mathematical induction, prove that 2
r 1
r
 1
2n
Solution
n
1 1
Given proposition
r 1
r
2n
2  1

Step1 : We need to prove if it is true for n  1,2


1
1 1
When n  1, 2
r 1
r
 1
21
1
2  1
2 it is true for n  1.......... .......... .......... .......... (i )
2
1 1
When n  2,
r 1
2
22 r
 1

1 1 1
1
 2  1 2
2 2 2
3 3
 it is true for n  2.......... .......... .......... .......( ii)
4 4
Step 2 : Assume it is true for n  k
k
1 1
2
r 1
r
2k
 1
.......... .......... .......... .......... .......... .......... .......... ..(iii)

Step 3 : We need to prove if it is true for n  k  1


k 1
1 1
2
r 1
 1
2 k 1
r

Consider L.H.S and then prove that is equal to R.H.S


k 1 k k
1 1 1 1 1

r 1 2
r
  r  k 1 but  r  1  k from equation (iii)
r 1 2 2 r 1 2 2
k 1
1 1 1 2 k 1  2  1 2 k 1  1 1

r 1 2
r
 1  
2 k 2 k 1

2 k 1

2 k 1
 1 
2 k 1
k 1
1 1
2
r 1
r
 1
2 k 1
but n  k  1
n

1
r
1
 1 n The proposition is true for all posivive integers
r 1 2 2

247
Example 36
n
By using mathematical induction, prove that 2
r 1
r 1
 2n 1

Solution
n
Given proposition 2
r 1
r 1
 2n  1

Step1 : We need to prove if it is true for n  1,2


1
When n  1, 2
r 1
r 1
 21  1

1  1 it is true for n  1.......... .......... .......... .......... (i )


2
When n  2, 2
r 1
r 1
 22  1

211  221  22  1
3  3 it is true for n  2.......... .......... .......... .......( ii)
Step 2 : Assume it is true for n  k
k

2
r 1
r 1
 2 k  1.......... .......... .......... .......... .......... .......... .......... (iii)

Step 3 : We need to prove if it is true for n  k  1


k 1

2
r 1
r 1
 2 k 1  1

Consider L.H.S and then prove that is equal to R.H.S


k 1 k k

 2 r 1   2 r 1  2 k 11 but  2 r 1  2 k  1 from equation (iii)


r 1 r 1 r 1

 
k 1

2
r 1
r 1
 2 k  1  2 k  2 2 k  1  2 k 1  1
k 1

2
r 1
r 1
 2 k 1  1 but n  k  1
n
  2 r 1  2 n  1 The proposition is true for all posivive integers
r 1

248
Example 37
n

 3r  1  2 3n  1
n
By using mathematical induction, prove that
r 1
Solution
n

 3r  1  2 3n  1
n
Given proposition
r 1

Step1 : We need to prove if it is true for n  1,2


1

 3r  1  2 3  1
1
When n  1,
r 1

2  2 it is true for n  1.......... .......... .......... .......... ........( i )


2

 3r  1  2 3n  1
n
When n  2,
r 1

2
3(2)  1
25 
2
7  7 it is true for n  2.......... .......... .......... .......... .......... ...( ii)
Step 2 : Assume it is true for n  k
k

 3r  1  2 3k  1.......... .......... .......... .......... .......... .......... .......... ........( iii)
k
r 1

Step 3 : We need to prove if it is true for n  k  1


k 1
k 1
 3r  1  3k  1  1
r 1 2
Consider L.H.S and then prove that is equal to R.H.S
k 1 k k

 3r  1   3r  1  3(k  1)  1 but  3r  1  3k  1 from equation (iii)


k
r 1 r 1 r 1 2
k 1

 3r  1  2 3k  1  3(k  1)  1


k
r 1
k 1
k 3k  1  6(k  1)  2
 3r  1 
r 1 2
k 1
3k 2  7k  4 1
3r  1   k  13k  4  k  13k  1  1
1

r 1 2 2 2
k 1
k  1 3k  1  1
 3r  1 
r 1 2
but n  k  1
n
  3r  1  3n  1 The proposition is true for all posivive integers
n
r 1 2

249
Example 38
n
1 n
By using mathematical induction, prove that  r r  1  n  1
r 1

Solution
n
1 n
Given proposition  r r  1  n  1
r 1

Step1 : We need to prove if it is true for n  1,2


1
1 1
When n  1,  r r  1  1  1
r 1
1
2  1
2 it is true for n  1.......... .......... .......... .......... .......... .......... ...( i )
2
1 n
When n  2,  r r  1  n  1
r 1

1 1 2
 
1(1  1) 2(2  1) 2  1
2 2
 it is true for n  2.......... .......... .......... .......... .......... .......... (ii)
3 3
Step 2 : Assume it is true for n  k
k
1 k
 r r  1  k  1.......... .......... .......... .......... .......... .......... .......... .......... .......... (iii)
r 1

Step 3 : We need to prove if it is true for n  k  1


k 1
1 k 1
 r r  1  k  1  1
r 1

Consider L.H.S and then prove that is equal to R.H.S


k 1 k k
1 1 1 1 k

r 1 r r  1

r 1 r r  1

k  1k  1  1
but 
r 1 r r  1

k 1
from equation (iii)

k 1
k k  2  1 k 2  2k  1 k  1  k  1 2
1 k 1

r 1 r r  1
    
k  1 k  1k  2 k  1k  2 k  1k  2 k  1k  2 k  2
k 1
1 k 1 k 1
 r r  1  k  2  k  1  1
r 1
but n  k  1
n

1

n
The proposition is true for all posivive integers
r 1 r r  1 n 1

250
Example 39
By using mathematical induction, prove that
1 1 1 3 2n  3
     
1 3 2  4 nn  2  4 2n  1n  2 
Solution
1 1 1 3 2n  3
Given       .......... .......... .......... .......... .....(i)
1 3 2  4 nn  2 4 2n  1n  2
n
1 1 1 1
Express in sigma notation      .......... .......... .......( ii)
1 3 2  4 nn  2 r 1 r r  2
Substitute equation (ii) into equation (i)
n
1 3 2n  3
 r r  2  4  2n  1n  2 .......... .......... .......... .......... .......... .......... .......... .........( iii)
r 1

Step1 : We need to prove if it is true for n  1,2


1
1 3 23
When n  1,  r r  2  4  21  11  2
r 1
1
3  13 it is true for n  1
2
1 3 2(2)  3
When n  2,  r r  2  4  22  12  2
r 1

1 1 3 2(2)  3
  
1(1  2) 2(2  2) 4 22  12  2
11 11
 it is true for n  2
24 24
Step 2 : Assume it is true for n  k
k
1 3 2k  3
 r r  2  4  2k  1k  2 .......... .......... .......... .......... .......... .......... .......... .......... .......... (iv)
r 1

Step 3 : We need to prove if it is true for n  k  1


k 1
1 3 2k  5
 r r  2  4  2k  2k  3
r 1

251
Example 40
By using mathematical induction, prove that 9 n  1 is divisible by 8 for all
positive integers;
Solution
Given proposition 9 n  1  8m where m is positive integer
Step1 : We need to prove if it is true for n  1,2
When n  1, 91  1  8m
8  8m it is true for n  1.......... .......... .......... .......... (i )
When n  2, 9 2  1  8m
80  8m it is true for n  2.......... .......... .......... .......( ii)
Step 2 : Assume it is true for n  k
9 k  1  8m
9 k  8m  1.......... .......... .......... .......... .......... .......... .......... .........( iii)
Step 3 : We need to prove if it is true for n  k  1
9 k 1  1  8m
Consider L.H.S and then show that is divisible by 8
 
9 k 1  1  9 9 k  1 but 9 k  8m  1 from equation (iii)
9 k 1  1  98m  1  1
9 k 1  1  72m  8
9 k 1  1  89m  1
 9 n  1 is divisible by 8

Example 41
3 5 2n  1 1
By using mathematical induction, prove that     2  1 
n n  1
2
4 36 n2
Solution
252
3 5 2n  1 1
Given     2  1  .......... .......... .......... .......... .......... .......... .........( i)
n n  1
2
4 36 n2
3 5 2n  1 n
2r  1
Express in sigma notation     2
4 36
 
n n  1 r  2 r 2 r  1
2 2
.......... .......... ........( ii)

Substitute equation (ii) into equation (i)


n
2r  1 1
 r r  1
r 2
2 2
 1
n2
.......... .......... .......... .......... .......... .......... .......... .......... .......... .....( iii)

Step1 : We need to prove if it is true for n  2,3


2
2r  1 1
When n  2,  r r  1
r 2
2 2
 1
22
3
4  3
4 it is true for n  2
3
2r  1 1
When n  3,  r r  1
r 2
2 2
 1
32
8 8
 it is true for n  3
9 9
Step 2 : Assume it is true for n  k
k
2r  1 1
 r r  1
r 2
2 2
 1
k2
.......... .......... .......... .......... .......... .......... .......... .......... .......... (iv)

Step 3 : We need to prove if it is true for n  k  1


k 1
2r  1 1
 r r  1
r 2
2 2
 1
k  12
Consider L.H.S and then prove that is equal to R.H.S
k 1
2r  1 k
2r  1 2k  1 k
2r  1 1

r  2 r r  1
2 2
 
r  2 r r  1
2 2

k  12 k 2
but  r r  1
r 2
2 2
 1
k2
k 1
2r  1 1 2k  1 1 2k  1   k  12  2k  1 1

r  2 r r  1
 1 2 
k k  1 k
 1  2  2 2
 1    1
 k k  1 k   k  1 k k  12
2 2 2 2 2 2

k 1
2r  1 1
 r r  1
r 2
2 2
 1
k  12
but n  k  1

n
2r  1

1
 1 2 The proposition is true for all posivive integers
r  2 r r  1
22
n

253
Example 42
By using mathematical induction, prove that n 3  3n 2  2n is divisible by 3 for
all positive integers;
Solution
Given proposition n 3  3n 2  2n  3m where m is positive integer
Step1 : We need to prove if it is true for n  1,2
When n  1, 13  3(1) 2  2(1)  3m
6  3m it is true for n  1.......... .......... .......... .......... .........( i )
When n  2, 2 3  32  22  3m
2

24  3m it is true for n  2.......... .......... .......... .......... ......( ii)


Step 2 : Assume it is true for n  k
k 3  3k 2  2k  3m
k 3  3m  3k 2  2k .......... .......... .......... .......... .......... .......... .......... .........( iii)
Step 3 : We need to prove if it is true for n  k  1
k  13  3k  12  2k  1  3m
Consider L.H.S and then show that is divisible by 3
k  13  3k  12  2k  1  k 3  6k 2  11k  6 but k 3  3m  3k 2  2k
k  13  3k  12  2k  1  3m  3k 2  2k  6k 2  11k  6
k  13  3k  12  2k  1  3m  3k 2  9k  6
k  13  3k  12  2k  1  3m  k 2  3k  2 its divible by 3
 n 3  3n 2  2n is divisible by 3

254
Example 43
By using mathematical induction, prove that x 2n  y 2n is divisible by x2  y2
for all positive integers;
Solution

Given proposition x 2 n  y 2 n  x 2  y 2 m where m is positive integer 
Step1 : We need to prove if it is true for n  1,2
 
When n  1, x 2  y 2  x 2  y 2 m it is true for n  1.......... .......... .......... .(i )
When n  2, 
x 2( 2)  y 2( 2)  x 2  y 2 m 
x    
 y 2 x 2  y 2  x 2  y 2 m it is true for n  2.......... .......( ii)
2

Step 2 : Assume it is true for n  k



x 2k  y 2k  x 2  y 2 m 
 
x 2 k  x 2  y 2 m  y 2 k .......... .......... .......... .......... .......... .......... .......... .......( iii)
Step 3 : We need to prove if it is true for n  k  1

x 2k 1  y 2k 1  x 2  y 2 m 
Consider L.H.S and then show that is divisible by x 2  y 2
    
x 2 k 1  y 2k 1  x 2 k x 2  y 2 k y 2 but x 2 k  x 2  y 2 m  y 2 k from equation (iii) 
x 2 k 1  y 2k 1 
 x  y m  y x   y y
2 2 2k 2 2k 2

x 2 k 1  y 2k 1 
 x  y x m  y x   y y
2 2 2 2k 2 2k 2

x 2 k 1  y 2k 1 
 x  y x m  x  y y
2 2 2 2 2 2k

x 2 k 1  y 2k 1 
 x  y mx  y 
2 2 2 2k

 x 2 n  y 2 n is divisible by x 2  y 2

255
Example 44
By using mathematical induction, prove that 2 n  2  32 n 1 is divisible by 7 for all
positive integers;
Solution
Given proposition 2 n 2  32 n1  7 m where m is positive integer
Step1 : We need to prove if it is true for n  1,2
When n  1, 2 n 2  32 n1  7m
35  7m it is true for n  1.......... .......... .......... .......... .........( i )
When n  2, 2 2 2  32 ( 2 )1  7 m
259  7m it is true for n  2.......... .......... .......... .......... .....( ii)
Step 2 : Assume it is true for n  k
2 k  2  32 k 1  7m
7m  32 k 1
2  k
.......... .......... .......... .......... .......... .......... .......... .......... ...( iii)
4
Step 3 : We need to prove if it is true for n  k  1
2 k 3  32 k 3  7 m
Consider L.H.S and then show that is divisible by 8
7m  32 k 1
 
2 k  3  3 2 k  3  2 3 2 k  33 3 2 k   but 2 k 
4
from equation (iii)

 7m  32 k 1 
2 k 3
3 2 k 3
 8  
  27 32 k
 4 
2 k 3  32 k 3    
 2 7m  32 k 1  27 32 k
2 k 3  32 k 3  27m  33   273 
2k 2k

2 k 3  32 k 3  14m  63   273 


2k 2k

2 k 3  32 k 3  14m  213  2k

2 k 3  32 k 3  72m  33 2k

 2 n 2  32 n1 is divisible by 7

256
Example 45
By using mathematical induction prove that cos  i sin n  cosn  i sin n
where i   1 .
Solution
Given proposition  cos  i sin   cos n  i sin n
n

Step1 : We need to prove if it is true for n  1,2


When n  1,  cos  i sin 1  cos  i sin it is true for n  1.......... .......... .......... (i)
When n  2,  cos  i sin   cos 2  i sin 2
2

Consider L.H.S  cos  i sin   cos2   2i sin cos  i 2 sin 2  but i 2  1


2

 cos  i sin 2  cos2   2i sin cos  sin 2 


 cos  i sin 2  cos2   sin 2    i2 sin cos 
 cos  i sin 2  cos2  i sin 2 .......... .......... .......... .......... ........( ii)
Step 2 : Assume it is true for n  k
 cos  i sin k  cosk  i sin k .......... .......... .......... .......... .......... .......... .......... ...(iii)
Step 3 : We need to prove if it is true for n  k  1
 cos  i sin k 1  cosk  1  i sink  1
Consider L.H.S and then prove if it is equal to R.H.S
 cos  i sin k 1   cos  i sin k  cos  i sin 1.......... .......... .......... .......... ...(iv)
Substitute equations (i) and (iii) into equation (iv)
 cos  i sin k 1  cos k  i sin k cos  i sin 
 cos  i sin k 1  cos k cos  sin k sin   isin k cos  cosk sin 
 cos  i sin k 1  cosk     i sink   
 cos  i sin k 1  cosk  1  i sink  1 but n  k  1
  cos  i sin   cos n  i sin n
n

257
Example 46
Prove by mathematical induction for all positive integers that 6 n  8 n is multiple
of 7 if n is odd.
Solution
Given proposition 6 n  8n  7m where m is positive integer
Step1 : We need to prove if it is true for n  1,3 Take odd numbers 
When n  1, 6 n  8n  7m
14  7m it is true for n  1.......... .......... .......... .......... .........( i )
When n  3, 6 n  8n  7 m
728  7 m it is true for n  3.......... .......... .......... .......... .....( ii)
Step 2 : Assume it is true for n  k
6 k  8k  7 m
6 k  7m  8k .......... .......... .......... .......... .......... .......... .......... .......... .........( iii)
Step 3 : We need to prove if it is true for n  k  2
6 k  2  8 k  2  7m
Consider L.H.S and then show that is divisible by 7
    but 6
6 k 2  8k 2  6 2 6 k  82 8k k
 7m  8 k from equation (iii)
6 k 2  8k 2  6 7m  8   8 8 
2 k 2 k

 2 n 2  32 n1 is divisible by 7

258
Example 47
By using mathematical induction prove that
d n
dx
 
x  nx n 1 .

Solution
Given proposition
d n
dx
 
x  nxn 1

Step1 : We need to prove if it is true for n  1,2

When n  1,
d 1
dx
 
x  1x11

1  1 it is true for n  1.......... .......... .......... .......... ...( i )

When n  2,
d 2
dx
 
x  2 x 21

2 x  2 x it is true for n  2.......... .......... .......... .......( ii)


Step 2 : Assume it is true for n  k
d k
dx
 
x  kxk 1.......... .......... .......... .......... .......... .......... .......... .....( iii)

Step 3 : We need to prove if it is true for n  k  1

 
d k 1
dx
x  k  1x k

Consider L.H.S and then prove if it is equal to R.H.S

dx
x 
d k 1

d
dx
 
xxk  x k
d
dx
x   x d x k
dx
 

dx
 
d k 1
x  xk  x
d k
dx
x   but
d k
dx
 
x  kxk 1 from equation (iii)

 
d k 1
dx
x  
 x k  x kxk 1  x k  kxk  1  k x k  k  1x k 11

d
dx
 
x k 1  k  1x ( k 1) 1 but n  k  1


d
dx
 
x n  nx n 1

Example 48
x n1
By using mathematical induction prove that  x dx 
n
.
n 1
Solution

259
x n 1
Given proposition  x dx  n

n 1
Step1 : We need to prove if it is true for n  1,2
x11
When n  1,  x dx  1

11
x2 x2
 it is true for n  1.......... .......... .......... .......... ...( i )
2 2
x 21
When n  2,  x dx 
2

2 1
x3 x3
 it is true for n  2.......... .......... .......... .........( ii)
3 3
Step 2 : Assume it is true for n  k
x k 1
 x dx  k  1.......... .......... .......... .......... .......... .......... .......... .........( iii)
k

Step 3 : We need to prove if it is true for n  k  1


x k 11
x
k 1
dx 
k  1  1
x k 2
 x dx  k  2
k 1

Consider L.H.S and then prove if it is equal to R.H.S

x
k 1

dx   xxk dx By using integration by part  udv  uv   vdu 
u  x, du  dx.......... .......... .......... .......... .......... .......... .......... ......( iv)
x k 1
dv   x dx k
but  x dx 
k
from equation (iii)
k 1
x k 1
v .......... .......... .......... .......... .......... .......... .......... .......... .....( v)
k 1
 x k 1  x k 1
 xx dx  x k  1    k  1 dx
k

 x k 1  1
 xx dx  x k  1   k  1  xx dx
k k

1  x k 1 
 xx dx  k  1  xx dx  x k  1 
k k

k 2  x k 1  x k  2
k 1 
xx dx  x
k
 
 k  1  k 1
260
 x k  2  k  1 
    
k
xx dx
 k  1  k  2 
x k 2 x k 11

k 1
x dx   but n  k  1
k  2 k 11
x n1
  x dx 
n

n 1

Example 49
By using mathematical induction, prove that
sin 2 n 
cos  cos 2  cos 4    cos 2   n n 1  
2 sin
Solution
sin 2 n 
Given that cos  cos 2  cos 4    cos 2   n n 1  
2 sin 
n
Express in the form  cos 2 r 1  n
sin 2 n   
r 1 2 sin 
Step1 : We need to prove if its true for n  1,2
1
sin 21  
When n  1 r 1
cos 2   1
r 1

2 sin 
sin 2 2 sin  cos
cos  
2 sin  2 sin 
cos  cos it is true for n  1.......... .......... .......... .......... .....( i )
2
sin 2 2   
When n  2
r 1
 cos 2r 1 
2 2 sin 
sin 4
cos cos 2 
4 sin 
cos cos 2  cos cos 2 it is true for n  2.......... .......... ......( ii)
Step2 : Assume it is true for n  k
k
 
sin 2 k 
 cos 2r 1 
r 1 2 k sin 
.......... .......... .......... .......... .......... .......... .......... .......... ...( iii)

261
Step3 : We need to prove if its true for n  k  1
k 1
sin 2 k 1 

r 1
cos 2   k 1
r 1

2 sin 
Consider L.H.S and prove if it is equal to the R.H.S
k 1 k k
 
sin 2 k 
 cos 2r 1   cos 2r 1 . cos 2k  but
r 1 r 1
 cos 2r 1 
r 1 2 k sin 
from equation (iii)
k 1
sin 2    k
   
sin 2  cos 2 k  2 sin 2 k  cos 2 k  sin 2 2 k 
k
  

r 1
cos 2   k
r 1

2 sin 
. cos 2  
k

2 k sin 

2 2 k sin 

 
2 2 k sin   
k 1
 
sin 22 k   sin2 k 1 

r 1
cos 2  
r 1

22 k sin  2 k 1 sin
but n  k  1
n
sin2 n  

r 1
cos 2   n
r 1

2 sin
sin2 n  
 cos  cos 2  cos 4    cos 2   n n 1

2 sin

Example 50
 n
Prove by mathematical induction that 1  x   1  nx        x r      x n
n

r
 n n!
where   nCr  and n is positive integer.
 
r n  r !r!
Solution

262
k 1
1  x k 1

k!
x  1  x  but 1  x   
r 1 1 1!
xr
r 0 k  r !r! r 0 1  r !r!
k 1
1  x k 1   k!
xr 
1!
xr
r 0 k  r !r! r 0 1  r !r!

1  x k 1

k 1
k  1! x r but n  k  1 its true for n  k  1
r 0 k  1  r !r!
n
1  x n

n!
xr
r 0 n  r !r!

n
 1  x   1  nx        x r      x n
n

r

263
Example 51
 cos x sin x 
Prove by mathematical induction that if A    then
  sin x cos x 
 cosnx sin nx 
An   
 sin nx cos nx
Solution
 cos nx sin nx   cos x sin x 
Consider An    but A   
 sin nx cos nx  sin x cos x 
n
 cos x sin x   cos nx sin nx 
 sin x cos x    sin nx cos nx
   
Step1 : We need to prove if it is true for n  1,2
1
 cos x sin x   cos x sin x 
When n  1,  sin x cos x    sin x cos x  it is true n  1
   
2
 cos x sin x   cos 2 x sin 2 x 
When n  2,  sin x cos x    sin 2 x cos 2 x 
   
Consider L.H.S show if it is equal to R.H.S
2
 cos x sin x   cos x sin x   cos x sin x  cos2 x  sin 2 x 2 sin x cos x 
 sin x cos x    sin x cos x   sin x cos x    2 
   2 sin x cos x cos x  sin x 
2
   
2
 cos x sin x   cos 2 x sin 2 x 
 sin x cos x    sin 2 x cos 2 x  its true for n  2
   
Step2 : Assume it is true for n  k
k
 cos x sin x   cos kx sin kx 
 sin x cos x    sin kx cos kx
   
Step3 : We need to prove if it is true for n  k  1
 cosk  1x sink  1x 
k 1
 cos x sin x 
 sin x cos x  
   sink  1x cosk  1x

264
Consider L.H.S show if it is equal to R.H.S
k 1 k 1 k
 cos x sin x   cos x sin x   cos x sin x   cos x sin x   cos kx sin kx 
 sin x cos x      but     sin kx cos kx
    sin x cos x    sin x cos x    sin x cos x   
k 1
 cos x sin x   cos kx sin kx   cos x sin x 
 sin x cos x    
   sin kx cos kx  sin x cos x 
k 1
 cos x sin x   cos kx cos x  sin kx sin x cos kx sin x  sin kx cos x 
 sin x cos x   
   sin kx cos x  cos kx sin x  sin kx sin x  cos kx cos x 
k 1
 cos x sin x   cos kx cos x  sin kx sin x sin kx cos x  cos kx sin x 
 sin x cos x  
   sin kx cos x  cos kx sin x  coskx cos x  sin kx sin x 
 cosk  1x sink  1x 
k 1
 cos x sin x 
 sin x cos x    its true for n  k  1
   sink  1x cosk  1x 
n
 cos x sin x   cos nx sin nx   cos x sin x 

 sin x cos x   sin nx cos nx but A   sin x cos x 
     
 cos nx sin nx 
 An   
 sin nx cos nx

Example 52
Prove by mathematical induction that
n (n  1)
sin sin
sin  sin 2  sin 3      sin n  2 2
 
sin 
2
Solution

265
2 (2  1)
2 sin sin
When n  2, r 1
sin r  2
 
2
sin 
2
3 
sin sin  
sin  sin 2  2 
 
sin 
2
     3 
2 sin  cos  sin  
 3     2 2 2 
2 sin  cos  
 2  2  
sin 
2
 3      3    
2 sin  cos   2 sin  cos  it is true for n  2
 2  2  2  2
Ste2 : Assume it is true for n  k
k (k  1)
k sin sin

r 1
sin r  2

 
2 .......... .......... .......... .......... .......... .......... .......... .......( i )
sin 
2
Step3 : We need to prove if it is true for n  k  1

sin
k  1 sin (k  1  1)
k 1


r 1
sin r  2
 
2
sin 
2
Consider L.H.S and then show if it is equal to R.H.S
k (k  1)
k 1 k k sin sin
 sin r   sin r  sink  1 but  sin r  2 2 from equattion (i)
r 1 r 1 r 1  
sin 
2

266
k (k  1) k (k  1)  
k 1 sin sin sin sin  sin  sink  1
2  sink  1  2
 2 2 2
sin r 
r 1    
sin  sin 
2 2
k (k  1)     k 1  k 1
k 1
sin sin  2 sin  sin  cos 
 2  2   2 
By double angle for sin

2 2
sin r 
r 1  
sin 
2
(k  1)  k     k 1 
sin sin  2 sin   cos  
k 1
2  2 2  2  
 sin r  By factor formula
r 1   
sin 
2
(k  1)  k k 2  k 
k 1
sin  sin  sin   sin 
2  2  2   2 

r 1
sin r 
 
sin 
2
(k  1) k 2 (k  1)  k 11
k 1
sin sin  sin sin 
 2   

2 2 2
sin r   but n  k  1
r 1    
sin  sin 
2 2
n  n 1
sin sin 
 2 
n
  sin r 
2
it is true for all positive integers
r 1  
sin 
2

267
Example 53
n 5 n 3 7n
Show that   is a natural number for n N
5 3 15
Solution
n5 n3 7n
Let    N we need to prove if the given expression is a natural number
5 3 15
for all natural numbers
Step1 : We need to prove if its true for n  1,2
15 13 7(1)
When n  1,   N
5 3 15
1  N it is true for n  1
25 23 7(2)
When n  2,   N
5 3 15
10  N it is true for n  2
Step2 : Assume it is true for n  k
k 5 k 3 7k
  N
5 3 15
3k 5  5k 3  7k  15N
3k 5  15N  5k 3  7k .......... .......... .......... .......... .......... .......... .......... .......... (i )
Step1 : We need to prove if its true for n  k  1
k  15  k  13  7k  1  N
5 3 15
3k  1  5k  1  7k  1  15N
5 3

3k 5  15k 4  36k 3  48k 2  33k  9  15N .......... .......... .......... .......... .......... .(ii)
Substitute equation (i) into equation (ii)
15N  5k 3  7k  15k 4  36k 3  48k 2  33k  9  15N
15N  15k 4  31k 3  48k 2  26k  9  15N it is true
 n 5 n 3 7n 
      N for all n  N
 5 3 15 

268
Example 54
By using mathematical induction prove that
n1
 n log a  nn  1log x
1
 log ax
r 0
r

2
Solution
n 1
 n log a  nn  1 log x
1
Consider
r 0
 log ax r

2
Step1 : We need to prove if it is true for n  1,2
0
When n  1,  log ax r 1
 log a 1  1log x
r 0 2
log a  log a its true for n  1
1
 2 log a  (2)2  1 log x
1
When n  2,
r 0
 log ax r

2
log a  log ax  2 log a  log x
log a  log a  log x  2 log a  log x
2 log a  log x  2 log a  log x its true for n  2
Step2 : Assume if it is true for n  k
k 1
 k log a  k k  1 log x
1
 log ax
r 0
r

2
Step3 : We need to prove if it is true for n  k  1
k

 log ax  k  1log a  2 k k  1log x


r 1
r 0

Consider L.H.S and then show if it is equal to R.H.S


k k 1

 log axr   log axr  log axk


r 0 r 0
k
 k log a  k k  1 log x  log axk
1
 log ax
r 0
r

2
k
k k  1 log x  log a  log x k
1

r 0
log ax r
 k log a 
2
k

 log ax  k  1log a  2 k k  1log x  k log x


r 1
r 0
k
1 
 log ax  k  1log a   2 k k 1  k  log x
r 0
r

269
k

 log ax  k  1log a  2 k k  1log x


1
r
but n  k  1
r 0
n 1
  log ax r  n log a  nn  1log x its true.
1
r 0 2

Example 55
Use the principle mathematical induction to prove that 1  p n  1  np
Solution
Consider 1  p   1  np
n

Step1 : We need to prove if it is true for n  1,2


When n  1, 1  p 1  1  p 
1  p   1  p  it is true for n  1
When n  2, 1  p   1  2 p 
2

Consider L.H.S 1  p   1  2 p  p 2  1  2 p 
2

1  p 2  1  2 p  it is true for n  2
Step2 : Assume it is true for n  k
1  p k  1  kp.......... .......... .......... .......... .......... .......... ...(i)
Step3 : We need to prove if it is true for n  k  1
1  p k 1  1  (k  1) p
Consider L.H.S and then show if it is greater than R.H.S
1  p k 1  1  p k 1  p 1 but 1  p k  1  kp from equation (i)
1  p k 1  1  kp1  p 
1  p k 1  1  p  kp  kp 2 
1  p k 1  1  k  1 p  kp 2   1  (k  1) p
1  p k 1  1  (k  1) p
 1  p   1  np its true for all positive integer
n

270
Example 56
By using mathematical induction prove that 3 n  n
Solution
Consider 3n  n
Step1 : We need to prove if it is true for n  1,2
When n  1, 31  1
3  1 it is true for n  1
When n  2, 32  2
9  2 it is true for n  2
Step2 : Assume it is true for n  k
3k  k .......... .......... .......... .......... .......... .......... ...(i )
Step3 : We need to prove if it is true for n  k  1
3k 1  k  1
Consider L.H.S and then show if it is greater than R.H.S
3k 1  3k 31 but 3k  k from equation (i)
3k 1  3k but 3k  k  1
3k 1  k  1
 3n  n its true for all positive integer

Example 57
1  r n1
Given that r  r  r  r      r  where r  1 and n  0 prove by
0 1 2 3 n

1 r
mathematical induction.
Solution

271
272
Exercise 6.4
1. By using mathematical induction, prove that
1 r n 
a  ar  ar      ar
2 n 1
 a 
 1 r 
2. By using mathematical induction, prove that

2  8    n2 1   1
6
nn  12n  5

1 1 1 1
     1
nn  1
3. By using mathematical induction, prove that
2 6 n
4. For n N prove by mathematical induction that
  n 1    n 
cos     sin  
  2   2 
cos  cos     cos  n  1       cos  n  1  
 
sin 
2
5. By using mathematical induction, prove that
1 1 1 1 n
     
1 3 3  5 5  7 2n  12n  1 2n  1
6. Prove by mathematical induction that the sum of the first n odd natural
numbers is n 2 .
7. By using mathematical induction, prove that
1 1 1 1 1
   
1 2  3 2  3  4 nn  1n  2 4 2n  1n  2
8. By using mathematical induction, prove that a 2 n  b 2 n is divisible by
(a  b)(a  b) for all positive integers;
9. By using mathematical induction, prove that

5  55  555      555
 
5
81

 5  10n1  9n  10 
n digits

10. By using mathematical induction, prove that n(n 2  5) is divisible by 3 for


all positive integers
11. By using mathematical induction, prove that
1  13   1  54   1  79       1  2nn1   n  12
2

12. By using mathematical induction, prove that 8 n  3 n is divisible by 5 for


every positive integers

273
13. By using mathematical induction, prove that

1 3  2  3  3  3      n  3
2 3 n

2n  1  3
n1

4
14. By using mathematical induction, prove that 3 2 n  7 is divisible by 8 for all
positive integers
15. By using mathematical induction, prove that 34 n 2  24 3n1  is divisible by
17 for all positive integers
16. Prove by induction that 3  2 2  32  2 3  33  2 4  3n  2 n1 
5

12 n

6 1

17. By using mathematical induction, prove that 4 n1  5 2 n1 is divisible by


21 for all positive integers
18. By using mathematical induction, prove that
7  77  777      777
 
7 n1

 81 10  9n  10
 7  
ndigits

19. By using mathematical induction, prove that x 2 n  y 2 n is divisible by x  y


20. Prove by induction that
1  sec 2 1  sec 4 1  sec8     1  sec 2 n    tan 2 n  cot
21. Prove by induction that the number of all subsets of a set containing n
distinct elements is 2 n .
22. Prove by mathematical induction S n 
n
2 A1  n  1d 
2

6.5 ROOTS OF A POLYNOMIAL FUNCTIONS


Roots of the equation are the values of the variable which satisfies the given
equation.
Consider the quadratic equation ax 2  bx  c  0 by using quadratic general
 b  b 2  4ac
formula x 
2a
By using the discriminant from the above formula we can determine the nature
of the roots as shown below,
(i) If b 2  4ac  0 the roots are real and distinct
(ii) If b 2  4ac  0 the roots are real and equal
(iii)If b 2  4ac  0 the roots are complex numbers
Note: If b 2  4ac  0 condition for real roots

274
Roots of equations are categorized into two types which are
(a) Roots of Quadratic equations
(b) Roots of Cubic equations

6.5.1 ROOTS OF QUADRATIC EQUATIONS


Consider the quadratic equation ax 2  bx  c  0 , the roots of this equation can
 b  b 2  4ac
be evaluated by using quadratic general formula x  ,
2a
Let the roots of this equation be  and  which implies that
 b  b 2  4ac  b  b 2  4ac
 and  
2a 2a
 b  b 2  4ac  b  b 2  4ac  b
Sum of roots       ……….. (i)
2a 2a a

b
Sum of roots   
a

  b  b 2  4ac   b  b 2  4ac  c
Product of roots       …….. (ii)
 2 a  2 a  a
  

Product of roots c
 
a

Alternative determination of sum and product of roots of a quadratic equation.


b c
Consider the quadratic equation ax 2  bx  c  0 or x 2  x   0.......... ...( i )
a a
Let the roots of this equation be  and  which implies that x   and x  
then form a quadratic equation form the above roots

275
x    x    0.......... .......... .......... .......... .......... .......... ...(ii)
x    x    0.......... .......... .......... .......... .......... .......... ..(iii)
Multiply equation (i) and equation (ii)
x   x     0
x 2  (   ) x   .......... .......... .......... .......... .......... .......... .....( iv)
Equate equation (i) and equation (iv)
c
Sum of roots   
b , Product of roots  
a a

Generally the standard form of a quadratic equation which involves roots is


given as x 2     x    0 or
x 2  sum of rootsx  product of roots  0

SOME USEFUL IDENTITIES


(i) a  b  a  b a  b 
2 2

(ii) a  b  a  b  2ab or a  b  a  b  2ab


2 2 2 2 2 2

3 3

(iii) a  b  a  b  a  ab  b
2
 2

(iv) a  b
3 3
 a  b a 2
 ab  b  2

Example 58
If the roots of the equation 3 x 2  5 x  1  0 are  and  . Find the value of
(a)  2   2 
(b)     
2 2

(c)  3   3
2 2
(d) 
 
Solution

276
The roots of 3x 2  5 x  1  0 are  and 
a  3, b  5 and c  1
b 5
Sum of roots       
a 3
c 1
Product of roots    
a 3

(a)  2    2       
5
9
5
 2    2 
9

2
5  1  16
(b)                  3     3  
2 2 2 2 2

 3 3 9
16
 2     2 
9


(c)  3   3       2     2   
5 16 80
3 9 27

80
 3   3 
27

2 2 3 3 80
80
(d )    27

   1
3 9

Example 59
Given the roots of 4 x 2  8 x  1  0 are  and  . Find the value of
1 1
(a) 
2 2
(b)    2
(c)  3    3

277
1 1
(d) 
 2   2
Solution
The roots of 4 x 2  8 x  1  0 are  and 
a  4, b  8 and c  1
8
Sum of roots        2
4
c 1
Product of roots    
a 4
1  2   2    2  2  2  2 14 
1
(a) 2  2  2 2    10
     2  14 2
1 1
   10
 2
2
1
(b)       2  2   2   2   2  2       4   2  4   3
2 2 2

4
      3
2

   
1  1  7
(c) 3    3    2   2        2   2  2  
2

4
2

 4  8
7
 3    3 
8

1 1       2
(d )  2 2 2   32
    
2
 2  14 2
1 1
 2  2  32
  

Example 60
The roots of the equation 2 x 2  4 x  1  0 are   2 and   2 . Find the
equation whose roots are  and  .
Solution

278
The roots of 2 x 2  4 x  1  0 are   2 and β  2
a  2, b  4 and c  1

Sum of roots    2    2      4  
b
a
4
  4
2
    6.......... .......... .......... .......... .......... .......... .........( i )

Product of roots    2  2    2     4 


1
2
  2     
7
2
  26  
7
2
17
  .......... .......... .......... .......... .......... .......... .......... ..(ii)
2
From general form of quadratic equation
x 2  sum of rootsx  product of roots  0
17
x 2  6x  0
2
 The equation whose roots are  and  is 2 x 2  12x  17  0

Example 61
The roots of the equation x  6 x  q  0 differ by one. Find the possible
2

value of q .
Solution

279
Example 62
If the roots of the equation x 2  2 px  q  0 differ by two. Show that
p2  1  q .
Solution
Consider x 2  2 px  q  0
    2 p.......... .......... .......... .......... .......... .......... ...(i )
  q.......... .......... .......... .......... .......... .......... .......... .(ii)
But     2 which mean   2   .......... .......... ......( iii)
Substitute equation (iii) into equation (i)
2      2 p
   p  1.......... .......... .......... .......... .......... .......... .....( iv)
Substitute equation (iv) into equation (iii)
  2   p  1  1  p
   p  1.......... .......... .......... .......... .......... .......... .....( v)
Substitute equations (iv) and (v) into equation (ii)
 p  1 p  1  q
p 2 1  q
 p2  1 q

Example 63
If the roots of the equation ax 2  bx  c  0 is twice the other, prove
that 2b 2  9ac .
Solution

280
Consider ax 2  bx  c  0
b
     .......... .......... .......... .......... .......... .......... .....( i)
a
c
  .......... .......... .......... .......... .......... .......... .......... .(ii)
a
But   2 .......... .......... .......... .......... .......... .......... ...(iii)
Substitute equation (iii) into equation (i)
b
2    
a
b
   .......... .......... .......... .......... .......... .......... .......( iv)
3a
Substitute equation (iv) into equation (iii)
 b  2b
  2  
 3a  3a
2b
   .......... .......... .......... .......... .......... .......... .......( v)
3a
Substitute equations (iv) and (v) into equation (ii)
 2b  b  c
     
 3a  3a  a
2b 2 c

9a 2 a
 2b 2  9ac

Example 64
If the sum of squares of the roots of the equation ax 2  bx  c  0 is one.
Prove that b 2  a 2  2ac .
Solution
Consider ax 2  bx  c  0
b
     .......... .......... .......... .......... .......... .......... .....( i)
a
c
  .......... .......... .......... .......... .......... .......... .......... .(ii)
a
But  2   2  1 .......... .......... .......... .......... .......... .......... ...(iii)
Square equation (i)
2
 b
    2
  
 a
281
b2
 2  2   2 
a2
b2
 2   2  2 
a2
c b
2
1  2   2
a a
a 2  2ac  b 2
 b 2  a 2  2ac

Example 65
If the roots of 3 x 2  kx  12  0 are equal. Find the possible value of k
Solution
Consider 3 x 2  kx  12  0
k
     .......... .......... .......... .......... .......... .......... .....( i )
3
12
   4.......... .......... .......... .......... .......... .......... ....( ii)
3
But    .......... .......... .......... .......... .......... .......... ....( iii)
Substitute equation (iii) into equation (i)
k
  
3
k
   .......... .......... .......... .......... .......... .......... .......... .(iv)
6
Substitute equation (iv) into equation (iii)
k
   .......... .......... .......... .......... .......... .......... .......... .(v)
6
Substitute equations (iv) and (v) into equation (ii)
 k  k 
      4
 6  6 
k2
 4, k 2  144
36
 k  12

282
Example 66
Find the relationship between p and q if the roots of the equation
px 2  qx  1  0 are equal.
Solution

 q  q  1
     
 2 p  2 p  p
q2 1
2

4p p
q2
1
4p
q2  4 p

Example 67
If  and  are roots of the equation 2 x 2  3 x  4  0 without solving the
equation, evaluate  
Solution
283
Given that  and  are roots of 2 x 2  3 x  4  0
3
    .......... .......... .......... .......... .......... .......... .......... ..(i )
2
4
   2.......... .......... .......... .......... .......... .......... .......... (ii)
2
Required,   
Consider    2
 2    
  
2
   2   
        2  .......... .......... .......... .......... .......( iii)
Substitute equations (i) and (ii) into equation (iii)
3
    2 2
2

6.5.2 ROOTS OF CUBIC EQUATIONS


Consider a cubic equation ax3  bx 2  cx  d  0 which implies that
b 2 c d
x3  x  x   0 ……………………………………..…. (i)
a a a
Let  ,  and  be roots of a cubic equation above, if  ,  and  are roots of
equation meaning that x   , x   and x   also in factor form these
roots can be expressed as x    0, x    0 and x    0
Now it’s possible to form cubic equation from the above factors as shown
below
x   x   x     0
x   x 2  x  x     0
x 3  x 2  x 2  x  x 2  x  x    0
x 3       x 2       x    0.......... .......... .(ii)
Equate equation (i) and equation (ii)
b
     
Sum of roots a

c
Sum of product of roots in pair      
a

284
Product of roots d
  
a

General equation of roots is given by


x 3       x 2       x    0 or
x 3  sum of rootsx 2  sum of product of rootsx  product of roots  0

USEFUL IDENTITIES
(i) a  b  c  a  b  c  2ab  ac  bc
2 2 2 2

3 3

(ii) a  b  c  a  b  c  a  b  c  ab  ac  bc  3abc
3 2 2 2

Example 68

The equation 3x 3  6 x 2  4 x  7  0 has roots  ,  and  . Find the equation


whose roots are 1 , 1 and 1
  

Solution
Consider 3 x 3  6 x 2  4 x  7  0
a  3, b  6, c  4 and d  7
b
        2.......... .......... .......... .......... .......... .......... .......... .......... .......... .(i )
a
c 4
       .......... .......... .......... .......... .......... .......... .......... .......... .....( ii)
a 3
d 7
     .......... .......... .......... .......... .......... .......... .......... .......... .......... ......( iii)
a 3
Sum of roots,sum of product of roots and product of roots of the required equation
1 1 1      4 3 4
     .......... .......... .......... .......... .......... .......... ........( iv)
    7
3 7
1 1 1     2 6
     .......... .......... .......... .......... .......... .......... ........( v)
    7
3 7
1 1 3
   .......... .......... .......... .......... .......... .......... .......... .......... .......... .......( vi)
  7
3 7
From, x 3  sum of rootsx 2  sum of product of rootsx  product of roots  0
4 6 3
x3  x 2  x   0
7 7 7
7x  4x  6x  3  0
3 2

285
Example 69
If the roots of the equation x 3  9 x 2  3 x  39  0 are  ,  and  show that the
equation whose roots are   3 ,   3, and   3 is x 3  24x  84  0 .
Solution
Given that  ,  and  are roots of x 3  9 x 2  3 x  39  0
      9.......... .......... .......... .......... .......... .......... .......... .......... .......... .....( i )
      3.......... .......... .......... .......... .......... .......... .......... .......... .........( ii)
  39.......... .......... .......... .......... .......... .......... .......... .......... .......... .......... .(iii)
  3,   3 and   3 are roots of the required equation
sum of roots    3    3    3        9  9  9  0.......... .......( iv)
sum of product of roots   3  3    3  3    3  3
sum of product         6       27  3  6(9)  27  24........( v)
Product of roots
  3  3  3    3       9       27
  3  3  3  39  3(3)  9(9)  27  84.......... .......... .......... .......... .......( vi)
From, x 3  sum of rootsx 2  sum of product of rootsx  product of roots  0
 x 3  24x  84  0

Example 70
Given the equation 2 x 3  3 x 2  13x  7  0 has roots  ,  and  . Find the
equations whose roots are
(a)  2 ,  2 and  2 (b) 1 , 1 and 1
  
Solution

286
Given that  ,  and  are roots of 2 x 3  3 x 2  13x  7  0
3
       .......... .......... .......... .......... .......... .......... .......... .......... .......... .(i )
2
13
       .......... .......... .......... .......... .......... .......... .......... .......... ...(ii)
2
7
  .......... .......... .......... .......... .......... .......... .......... .......... .......... .......... .(iii)
2
(a )  ,  2 and  2 are roots of the required equation
2

sum of roots   2   2   2         2       .......... .....( iv)


2 61
4
sum of product of roots                 
2 2 2 2 2 2 2 2 2

                 2 2    2   2 


2 2 2 2

2
 13   7  3  127
        2            2    
2
.......... .....( v)
 2  2  2  4
Product of roots
2
7
          .......... .......... .......... .......... .......... .......... ......( vi)
2 2 2 2 49
2 4
From, x 3  sum of rootsx 2  sum of product of rootsx  product of roots  0
61 127 49
x3  x 2  x 0
4 4 4
 4 x 3  61x 2  127x  49  0

1 1 1
(b) , and are roots of the required equation
  
1      13 2 13
1 1
sum of roots       7 .......... .......... .......... ........( iv)
    7
2

1 1 1       3 2 3
sum of product of roots       7 .......... .......... ...( v)
    7
2

1 1
Product of roots    2 7 .......... .......... .......... .......... .......... .......... .......... ...( vi)
 7 2
From, x 3  sum of rootsx 2  sum of product of rootsx  product of roots  0
x 3  13 7 x 2  3 7 x  2 7  0
 7 x 3  13x 2  3x  2  0

287
Example 71
If  ,  and  are roots of the equation x 3  x 2  4 x  7  0 . Find the equation
whose roots are    ,    and   
Solution
Given that  ,  and  are roots of x 3  x 2  4 x  7  0
      1.......... .......... .......... .......... .......... .......... .......... .......... .......... ......( i )
      4.......... .......... .......... .......... .......... .......... .......... .......... .........( ii)
  7.......... .......... .......... .......... .......... .......... .......... .......... .......... .......... ..(iii)
But    ,    and    are roots of the required equation
sum of roots                 2       2(1)  2.......... ........( iv)
sum of product of roots                       
 
  2   2   2  3     
        2       3     
2

sum of product of roots                12  4  5.......... ......( v)


2

Product of roots
            2   2   2   2    3   3   3 .......... .......... ....( vi)
But  3   3   3         2   2   2         3 .......... .(vii)
Substitute equation (vii) into equation (vi)
   
  2   2   2            2   2   2        but       1 
    2  2
2
    
2
  2   2        
 5          7  4  11.......... .......... .......... .......... .......... .......... .....( viii)
From, x 3  sum of rootsx 2  sum of product of rootsx  product of roots  0
 x 3  2 x 2  5 x  11  0

EQUATIONS WITH COMMON ROOTS


Equations with common roots are two or more equations which have common
values of the variable.

Example 72
If x 2  bx  2  0 and x 2  qx  4  0 have common roots, find the relation
between b and q .
Solution

288

 x  bx  2  0.......... .......... .......... ....( i )
2

 2

 x  qx  4  0.......... .......... .......... ....( ii)
6
x .......... .......... .......... .......... .......( ii)
bq
Substitute equation (iii) into equation (i)
2
 6   6 
   b   2  0
 b  q   b  q 
 2b  q  bq  18  0
2 2

6.6 REMAINDER THEOREM


Introduction
Remainder is a number or expression left over after dividing two numbers or
functions.
If f ( x ) is divided by x  a and f (a )  k then the remainder of f ( x ) is k .
Methods used to determine remainder of a polynomial
(a) By long division
(b) By remainder theorem
(c) By synthetic division

Example 73
By using long division, remainder theorem and synthetic division, find the
remainder when 2 x 3  3 x 2  x  4 is divided by x  1
Solution
By long division
2x 2  x  2
x  1 2 x 3  3x 2  x  4
2x3  2x 2
x2  x  4
x2  x
 2x  4
 2x  2
6

The remainder is 6

289
By remainder theorem
Take x  1  0  x  1 and then substitute x  1 into 2 x 3  3 x 2  x  4 .
2 x 3  3 x 2  x  4 = 213  312  1  4  6
The remainder is 6

By synthetic division
Given expression 2 x 3  3 x 2  x  4
Divisor x  1  0  x  1
-1 2 3 -1 4

-2 -1 2

2 1 -2 6
The remainder is 6

Example 74
Find the value of k if x  1 is a factor of x 3  4 x 2  kx  6 . Find the other factors
of the expression
Solution
Let f ( x)  x 3  4 x 2  kx  6
If x  1 is a factor then f ( x)  0,
Now x  1  0, x  1
f (1)  (1) 3  4(1) 2  k (1)  6
f (1)  1  k but f (1)  0
0  1 k
 The value of k  1

Example 75
When a polynomial px2  qx  r is divided by x  1, x  1 and x  2 has a
remainder 2, 8 and 2 respectively. Evaluate the values of p, q and r .
Solution

290
Let f ( x)  px 2  qx  r
Consider the following
x  1  0, x  1 the remainder is f (1)  2
x  1  0, x  1 the remainder is f (1)  8
x  2  0, x  2 the remainder is f (2)  2
f (1)  p (1) 2  q (1)  r
2  p  q  r.......... .......... .......... .......... .......... ..(i )
f (1)  p (1) 2  q (1)  r
8  p  q  r.......... .......... .......... .......... .......... ..(ii)
f (2)  p(2) 2  q (2)  r
2  4 p  2q  r.......... .......... .......... .......... ........( iii)
Solve equations (i), (ii) and (iii) simulteneously
p q  r  2

p  q  r  8
4 p  2 q  r  2

 p  3, q  3 and r  8

Example 76
Find the remainder when x 5  2 x 3  x  12 is divided by x 3  1
Solution
Let f ( x)  x 5  2 x 3  x  12
Take x 3  1  0, x  1
f (1)  (1) 5  2(1) 3  (1)  12  12
 The remainder is 12

Example 77
Find the remainder when x 4  3 x 3  5 x 2  12x  6 is divided by x 2  x  2
Solution

291
By long division
x2  2x  5
x 2  x  2 x 4  3 x 3  5 x 2  12 x  6
x 4  x3  2x 2
2 x 3  3 x 2  12 x  6
2x3  2x 2  4x
 5x 2  8x  6
 5 x 2  5 x  10
 3x  4
 The remainder is  3 x  4

Example 78
Find the value of 'a' provided that x 2 ( x  1)  a(2 x  a) gives the remainder 4
when divided by x  1
Solution
Let f ( x)  x 2  x  1  a2 x  a 
Take x  1  0, x  1, f (1)  4
f (1)  12 1  1  a2  a   4
2  a2  a   4
a2  a  2  0
 The possible value of a is 2 or  1

Example 79
If x 2  1 has the factor for the polynomial x 3  ax 2  bx  c and 12 is a remainder
when a polynomial is divided by x  2 , determine the values of a, b and c
hence find the value of a 2  bc
Solution

292
Example 80
The polynomial y 5  4 y 2  xy  z gives a remainder of 3 y  2 when divided
by y 2  1 . Evaluate x and z
Solution
Ley f ( y )  y 5  4 y 2  xy  z
Take y 2  1  0, y  1,
f ( y )  y 5  4 y 2  xy  z  3 y  2
f (1)  15  4(1) 2  x(1)  z  3(1)  2
x  z  0.......... .......... .......... .......... .......... ....( i )
f (1)   1  4(1) 2  x(1)  z  3(1)  2
5

x  z  4.......... .......... .......... .......... .......... .....( ii)


Solve equation (i) and (ii) simulteneously
x  z  0

x  z  4
 x  2 and z  2

293
Example 81
Show that ( x  a ) is a factor of ( x  b) 5  (b  a ) 5 .
Solution
Let f ( x)  x  b   b  a 
5 5

If x  a is a factor, x  a and we need to show that f (a)  0


f (a)  a  b   b  a 
5 5

f (a)  a  b    a  b 
5 5

f (a)  a  b   a  b 
5 5

f (a)  0
 x  a is a factor of x  b   b  a 
5 5

Example 82
Show that a  b  c is a factor of a 3  b 3  c 3  3abc
Solution
Let f ( x)  a 3  b 3  c 3  3abc
If a  b  c is a foctor, a  b  c  0, a  b  c  we ned to show that f (a)  0
f (a)  b  c   b 3  c 3  3bc b  c 
3

f (a)  b  c   b 3  c 3  3b 2 c  3bc 2


3


f (a)  b  c   b 3  3b 2 c  3bc 2  c 3
3

f (a)  b  c   b  c   0
3 3

 a  b  c is a foctor of a 3  b 3  c 3  3abc

Example 83
When the expression x 3  kx 2  2 is divided by x  2 , the remainder is 1 less
than when divided by x  1 . Find k
Solution

294
Let f ( x)  x 3  kx 2  2
When f ( x) is divided by x  2, the remainder is r  1
When f ( x) is divided by x  1, the remainder is r
From data above x  2, x  1
f (2)  (2) 3  k (2) 2  2  r  1
4k  11  r.......... .......... .......... .......... .......... .......... ....( i )
f (1)  (1) 3  k (1) 2  2  r
k  1  r.......... .......... .......... .......... .......... .......... ........( ii)
Substitute equation (ii) into equation (i)
4k  11  k  1
3k  10
 k  10 3

Example 84
When the expression x 3  ax 2  2 x  1 is divided by x  2 , the remainder is
three times as greater as when the expression is divided by x  1, find 'a'
Solution
Let f ( x)  x 3  ax2  2 x  1
When f ( x) is divided by x  2, the remainder is 3r
When f ( x) is divided by x  1, the remainder is r
From data above x  2, x  1
f (2)  (2)3  a(2) 2  2(2)  1  3r
4a  13  3r.......... .......... .......... .......... .......... .......... ....( i )
f (1)  (1) 3  a(1) 2  2(1)  1  r
a  4  r.......... .......... .......... .......... .......... .......... ........( ii)
Substitute equation (ii) into equation (i)
4a  13  3a  4
4a  13  3a  12
 a  1

295
Example 85
Using synthetic division, find the value of c , given that the polynomial
P( x)  x 3  cx 2  2cx  4 is divisible by x  1.
Solution
Consider the expression x 3  cx 2  2cx  4
Divisor x 1  0  x  1
1 1 c -2c 4

1 c+1  c 1
1 c+1  c 1 c5

If x  1 is factor of P( x) the remainder should be equal to zero


c5 0
 The value of c  5

Example 86
When a polynomial is dived by x  2 the remainder is 11 and when the
polynomial is divided by x  1 the remainder is  1 . Determine the remainder
when the polynomial is divided by x  2x  1 .
Solution
We need to find the remainder when P( x) is divided by x  2x  1
The remainder should be linear , ax  b  r
Let f ( x)  ax  b  r
From the data above x  2, x  1
f (2)  a(2)  b  11
2a  b  11.......... .......... .......... .......... .......... .......... .......... .......... ....( i )
f (1)  a(1)  b  1
a  b  1.......... .......... .......... .......... .......... .......... .......... .......... ....( ii)
Solve simulteneously equations (i) and (ii)
2a  b  11

a  b  1
a  4, b  3
Remainder r   ax  b  4 x  3
 Remainder is 4 x  3

296
Example 87
When a polynomial is dived by x  1 the remainder is  5 and when the
polynomial is divided by x  1 the remainder is 15 . Determine the remainder
when the polynomial is divided by x 2  1 .
Solution
We need to find the remainder when P ( x) is divided by x 2  1   x  1 x  1
The remainder should be linear , ax  b  r
Let f ( x)  ax  b  r
From the data above x  1, x  1
f (1)  a (1)  b  5
a  b  5.......... .......... .......... .......... .......... .......... .......... .......... ....( i )
f (1)  a (1)  b  15
a  b  15.......... .......... .......... .......... .......... .......... .......... .......... ....( ii)
Solve simulteneously equations (i) and (ii)
a  b  5

a  b  15
a  5, b  10
Remainder r   ax  b  5 x  10
 Remainder is 5 x  10

Example 88
If the polynomial p (x) is divided by x  1, x and x  1 the remainder are 1, 2 and
3 respectively. Find the remainder when p (x) is divided by x( x 2  1) .
Solution

297
Example 89
If x  2 is a factor of a polynomial and when the polynomial is divided by x  1
the remainder is 9 . Determine the remainder when the polynomial is divided by
x  2x 1 .
Solution
We need to find the remainder when P( x) is divided by x  2x  1
The remainder should be linear , ax  b  r
Let f ( x)  ax  b  r
From the data above x  2, x  1
f (2)  a(2)  b  0
 2a  b  0.......... .......... .......... .......... .......... .......... .......... .......... ....( i )
f (1)  a(1)  b  9
a  b  9.......... .......... .......... .......... .......... .......... .......... .......... .......( ii)
Solve simulteneously equations (i) and (ii)
 2 a  b  0

 ab 9
a  3, b  6
Remainder r   ax  b  3x  6
 Remainder is 3x  6

298
Example 90
If x  2 and x  3 are factors of a polynomial, find the remainder when the
polynomial is divided by x  2x  3 .
Solution
We need to find the remainder when P( x) is divided by x  2x  3
The remainder should be linear , ax  b  r
Let f ( x)  ax  b  r
From the data above x  2, x  3
f (2)  a(2)  b  0
2a  b  0.......... .......... .......... .......... .......... .......... .......... .......... ....( i )
f (3)  a(3)  b  0
3a  b  0.......... .......... .......... .......... .......... .......... .......... .......... ....( ii)
Solve simulteneously equations (i) and (ii)
2 a  b  0

3a  b  0
a  0, b  0
Remainder r   ax  b  0
 Remainder is 0

6.7 INEQUALITIES
Inequalities equations are almost equal to normal equations instead of an equal
sign is replaced by inequalities signs.

Inequalities signs
(i) < stand for less than
(ii) > stand for greater than
(iii)  stand for less than or equal to
(iv)  stand for greater than or equal to

RULES FOR INEQUALITIES OPERATIONS


(i) Addition and subtraction in either side of the inequality is possible
(ii) Multiplication by a negative number reverse the inequality sign
(iii)Division of negative number reverse the inequality sign

NOTE: Operation of inequalities problems one side must be made equal to zero

299
Inequalities problems are categorized into three forms which are Quadratic
functions inequalities, Rational functions inequalities and Absolute value
inequalities.

(a) Quadratic functions inequalities


Quadratic functions inequalities are inequalities of the
forms ax 2  bx  c  0 , ax 2  bx  c  0 , ax 2  bx  c  0 and ax 2  bx  c  0

Example 91
Solve x 2  5 x  6  0
Solution
Consider x 2  5 x  6  0
x  2x  3  0
Boundaries x  2, x  3
By using number line

 x  2 and x  3

Alternatively
By using table
x2 2 x3 x3
x2  ve  ve  ve
x 3  ve  ve  ve
x  2x  3  ve  ve  ve

 x  2 and x  3

Example 92
Solve 4 x  x 2  5  0
Solution
Consider 4 x  x 2  5  0
x2  4x  5  0
x  1x  5  0
Boundaries x  1, x  5

300
By using number line

 x  1 and x  5
Alternatively
By using table
x  1 1  x  5 x5
x 1  ve  ve  ve
x 5  ve  ve  ve
x  1x  5  ve  ve  ve
 x  1 and x  5

Example 93
Solve 4 x  3x  1  2
Solution
Consider 4 x  3 x  1  2
4x2  x  5  0
x  14 x  5  0
Boundaries x   54 , x  1
By using number line

 x   54 and x  1

Alternatively
By using table
x   54  54  x  1 x 1
x 1  ve  ve  ve
4x  5  ve  ve  ve
x  14 x  5  ve  ve  ve

 x   54 and x  1

301
Example 94
Solve 5x  7x  3  16x
Solution
Consider 5 x  7 x  3  16x
5 x 2  38x  21  0
x  7 5 x  3  0
Boundaries x  53 , x  7
By using number line

 53  x  7

Alternatively
By using table
x  53 3
x7 x7
5
5x  3  ve  ve  ve
x 7  ve  ve  ve
5x  3x  7  ve  ve  ve

 53  x  7

(b) Rational functions inequalities


ax  b
Rational functions inequalities are inequalities of the forms 0,
cx  d
ax  b ax  b ax  b
 0,  0 and 0
cx  d cx  d cx  d

Example 95
x2
Solve 0
x 1
Solution
x2
Consider 0
x 1
Boundaries x  2, x  1
302
x  1 1  x  2 x2
x2  ve  ve  ve
x 1  ve  ve  ve
x2  ve  ve  ve
x 1

 1  x  2

Example 96
x2
Solve 0
x 1
Solution
x2
Consider 0
x 1
Boundaries x  2, x  1
x  1 1  x  2 x2
x2  ve  ve  ve
x 1  ve  ve  ve
x2  ve  ve  ve
x 1

 x  1 and x  2

Example 97
x3
Solve 2
x 1
Solution
x3
Consider 2
x 1
x3
20
x 1
 x5
0
x 1
Boundaries x  5, x  1
303
x 1 1 x  5 x5
 x5  ve  ve  ve
x 1  ve  ve  ve
 x5  ve  ve  ve
x 1

1  x  5

Example 98
x3
Solve 2
x 1
Solution
x3
Consider 2
x 1
x3
20
x 1
 x5
0
x 1
Boundaries x  5, x  1
x 1 1 x  5 x5
 x5  ve  ve  ve

x 1  ve  ve  ve
 x5  ve  ve  ve
x 1

 x  1 and x  5

Example 99
x  1x  2  0
x  1x  3
Solve

Solution

304
Consider
x  1x  2  0
x  1x  3
Boundaries x  2, x  1, x  1, x  3
x  2  2  x  1 1  x  1 1  x  2 x2
x 1  ve  ve  ve  ve  ve
x2  ve  ve  ve  ve  ve
x  1x  2  ve  ve  ve  ve  ve
x 1  ve  ve  ve  ve  ve
x 3  ve  ve  ve  ve  ve
x  1x  3  ve  ve  ve  ve  ve
 x  1 x  2   ve  ve  ve  ve  ve
 x  1 x  3

 2  x  1 and 1  x  2

Example 100
Solve x  1x  2  0
x  1x  3
Solution
Consider
x  1x  2  0
x  1x  3
Boundaries x  2, x  1, x  1, x  3
x  2  2  x  1 1  x  1 1  x  2 x2
x 1  ve  ve  ve  ve  ve
x2  ve  ve  ve  ve  ve
x  1x  2  ve  ve  ve  ve  ve
x 1  ve  ve  ve  ve  ve
x 3  ve  ve  ve  ve  ve
x  1x  3  ve  ve  ve  ve  ve
 x  1 x  2   ve  ve  ve  ve  ve
 x  1 x  3

 x  2,  1  x  1 and x  2

305
Example 101
Solve 2 x2  7 x  4  2
2

3 x  14 x  11
Solution
2x2  7x  4
Consider 20
3 x 2  14x  11
 4 x 2  21x  26
0
3 x 2  14x  11
4 x 2  21x  26
0
3 x 2  14x  11
4 x  13x  2  0
3x  11x  1
Boundaries x  1, x  2, x  134 , x  113
x 1 1 x  2 2  x  134 13
 x  113 x  113
4
4x  13  ve  ve  ve  ve  ve
x2  ve  ve  ve  ve  ve
4 x  13x  2  ve  ve  ve  ve  ve
3x 11  ve  ve  ve  ve  ve
x 1  ve  ve  ve  ve  ve
3x  11x  1  ve  ve  ve  ve  ve
4 x  13x  2  ve  ve  ve  ve  ve
3x  11x  1
1  x  2 and 13
4  x  113

Example 102
Solve 2 x2  7 x  4  2
2

3 x  14x  11
Solution
2x2  7x  4
Consider 20
3x 2  14x  11
 4 x 2  21x  26
0
3 x 2  14x  11
4 x 2  21x  26
0
3 x 2  14x  11

306
4 x  13x  2  0
3x  11x  1
Boundaries x  1, x  2, x  134 , x  113
x 1 1 x  2 2  x  134 13
 x  113 x  113
4
4x  13  ve  ve  ve  ve  ve
x2  ve  ve  ve  ve  ve
4 x  13x  2  ve  ve  ve  ve  ve
3x 11  ve  ve  ve  ve  ve
x 1  ve  ve  ve  ve  ve
3x  11x  1  ve  ve  ve  ve  ve
4 x  13x  2  ve  ve  ve  ve  ve
3x  11x  1

 x  1, 2  x  134 and x  113

(c) Absolute value inequalities


Absolute value inequalities are inequalities of the forms f ( x)  0 ,
f ( x)  0 f ( x)  0 and f ( x)  0

Example 103
Solve x  3  5
Solution
Consider x  3  5
  x  3  5
When ( ve)
 x  3  5
x8
When (-ve)
  x  3  5
x  3  5
x  2

307
 x  2 and x  8

Example 104
Solve 5  x  1  7
Solution
From 5  x  1  7
Consider 5  x  1 and x  1  7
5  x 1
  x  1  5
When ( ve)
x  1  5
x4
When ( ve)
  x  1  5
x  1  5
x  6
x  4 and x  6
Also consider x  1  7
  x  1  7
When ( ve)
x  1  7
x6
When ( ve)
  x  1  7
x  1  7
x  8
x  6 and x  8

 8  x  6 and 4  x  6

308
6.8 MATRICES
Introduction
Matrix is the branch of mathematics which deal with the arrangement of
numerical information in rows and columns.

Column
This is the vertical arrangement of numerical information

Row
This is the horizontal arrangement of numerical information

Matrix is always named by capital letter and its members are named
(represented) by small letters.
a b c
 f 
Example A  d e
 g h k 

Order of Matrix
The order of matrix is determined by counting first the number of rows
followed by the number of columns.
a b c 
 
Example B  d e f  the order of this matrix is 3  3
 g h k 
Generally the order of matrix is represented by
m n where m is the number of rows and n is the number of columns

Types of matrices
Matrices are categorized into many types as explained below

(i) Row matrix is a matrix which consist of one row,


eg A  a b c

a 
 
(a) Column matrix is a matrix which consist of one column, eg B  b 
 c 

309
(b) Square matrix is a matrix which has the same number of rows and
a b c 
a b   
columns, eg C    , D  d e f  etc.
c d   g h k 

(c) Diagonal matrix is a square matrix which has elements on diagonal


 a 0 0
 
and zero elsewhere, eg, D  0 b 0 where a, b, c   .
0 0 c 

(d) Identity matrix is a square matrix in which all elements in leading


diagonal is one and zero elsewhere,
1 0 0
1 0
eg, A    , B  0 1 0
0 1 0 0 1

(e) Zero matrix is a matrix whose elements are all zero,


0 0 0 
0 0
eg, A    , B  0 0 0
0 0 0 0 0

(f) Triangular matrices are square matrices in which most non-zero


elements form a shape of triangle, here we have upper and lower
a b c   a 0 0
   
triangles eg, A  0 e f  , B  d e 0
0 0 k   g h k 

(g) Singular matrix is a matrix whose determinant is zero.

(h) Non-singular matrix is a matrix whose determinant is non zero.

310
(A) DETERMINANT OF 3 3 MATRIX
   
Determinant of 3 3 matrix is evaluated by using the pattern    .
 
   
A learner should choose either row pattern or column pattern to apply on
evaluation of determinant of the matrix.
 a11 b12 c13 
 
Consider the general matrix A  a21 b22 c23  the determinant of matrix
a31 b32 c33 
A is given by
b22 c23 a c a b
A   a11  b12 21 23  c13 21 22
b32 c33 a31 c33 a31 b32

Example 105
1 4  3 
 
If A  1  10 7  find A .
1  2 1 
 
Solution
1 4 3
 10 7 1 7 1  10
A  1  10 7  1 4  (3) 4
2 1 1 1 1 2
1 2 1
A 4

(B) CRAMER’S RULE


Cramer’s Rule is rule used to solve system of equations by using determinant
concepts.
This is sometimes referred to as Determinant method.
Consider the system of equations below
a11 x  b12 y  c13 z  d1
a21 x  b22 y  c23 z  d 2
a31 x  b32 y  c33 z  d 3

311
Write the system of equations in matrix form
 a11 b12 c13   x   d1 
a b c   y   d 
 21 22 23    2 
a31 b32 c33   z   d 3 
Determine the determinant of the matrix
a11 b12 c13
b c a c a b
  a21 b22 c23   a11 22 23  b12 21 23  c13 21 22
b32 c33 a31 c33 a31 b32
a31 b32 c33
 d1 
 
Replace the first column of a matrix by d 2  then determine the determinant
 d 3 
d1 b12 c13
b c d c d b
 x  d 2 b22 c23   d1 22 23  b12 2 23  c13 2 22
b32 c33 d 3 c33 d 3 b32
d 3 b32 c33
 d1 
 
Replace the second column of a matrix by d 2  then determine the determinant
 d 3 
a11 d1 c13
d c a c a d
 y  a21 d 2 c23   a11 2 23  d1 21 23  c13 21 2
d 3 c33 a31 c33 a31 d 3
a31 d 3 c33
 d1 
 
Replace the third column of a matrix by d 2  then determine the determinant
 d 3 
a11 b12 d1
b d a d a b
 z  a21 b22 d 2   a11 22 2  b12 21 2  d1 21 22
b32 d 3 a31 d 3 a31 b32
a31 b32 d 3

Finally determine the values of x, y and z


x y 
 x , y and z  z
  

312
Example 106
By using Cramer’s rule solve the following system of equations;
x  4 y  3z  3
x  10 y  7 z  13
x  2y  z  3
Solution
Express the system of equations in matrix form
1 4  3  x    3 
    
1  10 7  y    13 
1  2 1  z   3 
    
Determine , x, y and z
1 4 3
 10 7 1 7 1  10
  1  10 7  1 4  (3) 4
2 1 1 1 1 2
1 2 1
3 4 3
 10 7 13 7 13  10
x  13  10 7  3 4  (3) 8
2 1 3 1 3 2
3 2 1
1 3 3
13 7 1 7 1 13
y  1 13 7  1  (3)  (3) 4
3 1 1 1 1 3
1 3 1
1 4 3
 10 13 1 13 1  10
z  1  10 13  1 4  (3)  12
2 3 1 3 1 2
1 2 3
Then determine x, y and z
x 8
x  2
 4
y 4
y  1
 4
z 12
z  3
 4
 x  2, y  1 and z  3

313
(C) INVERSE OF 3 3 MATRIX
The inverse of a matrix is a matrix in which the product of a matrix and its
inverse is equal to the identity matrix. This is sometimes referred to as Matrix
method.
Let A be the given matrix, the inverse of this matrix is denoted by A 1 . The
product of AA 1  I or A1 A  I where I is identity matrix

Procedures used to determine the inverse of a matrix


(i) Determine the determinant of a matrix
(ii) Determine the cofactor of a matrix
(iii)Determine the adjoint of a matrix (Adjoint is a transpose of cofactor)
1
A 1  Adj A where Adj- stands for Adjoint
A

Cofactor of a matrix
Cofactor of a matrix is determined by computing the inverse of the minors of a
   
 
matrix by apply the whole pattern   
   
If A is a matrix, the cofactor of A is denoted by Ac
 b22 c23 a21 c23 a21 b22 
    
 b32 c33 a31 c33 a31 b32 
 b c a c a b 
Ac    12 13  11 13  11 12 
 b32 c33 a31 c33 a31 b32 
 b12 c13 a c a b 
  11 13  11 12 
 b22 c23 a21 c23 a21 b22 

Transpose of a matrix
Transpose of a matrix is the exchange of rows and columns
If A is a matrix, the transpose of A is denoted by AT
 a11 b12 c13  a11 a21 a31 
   
If A  a21 b22 c23  then A  b12 b22 b32 
T

a31 b32 c33  c13 c23 c33 

314
(D) SOLVING SYSTEM OF EQUATIONS BY INVERSE METHOD
Inverse method is one of the method used to solve system of equations by pre-
multiplying the inverse of the matrix to the given matrix form.

Procedures
(a) Express the system of equations in matrix form
(b) Determine the determinant of the matrix
(c) Determine the inverse of the matrix
(d) Pre-multiply the inverse of a matrix both sides
(e) Evaluate the values of variables of the system of equations

Example 107
By using matrix method solve the system of equations;
x  y  2z  4
x  2 y  3z  2
3x  y  2 z  4
Solution
 1  1 2  x   4 
    
 1 2 3  y    2 .......... .......... .......... .......... .........( i )
 3 1 2  z   4 
    
1 1 2 
 
Let A   1 2 3 
3 1 2
 
Determine the determinant of matrix A i.e. A 
1 1 2
2 3 1 3 1 2
A  1 2 3 1  (1) 2  16
1 2 3 2 3 1
3 1 2
Determine the cofactor of matrix A i.e. Ac  
 2 3 1 3 1 2 
   
 1 2 3 2 3 1 
7  5
 1  1
2 1 2 1 1   
A  
c
     4  4  4
 1 2 3 2 3 1  
 7  1 3 
 1 2 1 2 1 1  
    
 2 3 1 3 1 2 

315
Determine the Adjoint of matrix A
Adj A  Transpose of Ac
 1 4  7
 
Adj A   7  4  1 
5  4 3 
 

Determine the Inverse of matrix A i.e. A1 
 1 4  7
1 1  
A1   Adj A   7  4 1 
A  16  
5  4 3 
Pre - multiply by A1 into equation (i)
 1 4  7  1  1 2  x   1 4  7  4 
1     1   
 7  4  1  1 2 3  y    7  4  1  2 
 16   3 1 2  z   16   5  4 3  4 
  5  4 3      
 1 0 0  x    16 
   1  
 0 1 0  y    16 
 0 0 1  z   16   16 
    
 x  1 
   
  y     1
z  1 
   

Example 108
Determine the condition for  for the system below to have a unique solution.
x yz 9
2 x  5 y  z  52
2x  y  z  0
Solution

316
Express the system of equation in matrix form
 1 1 1  x   9 
    
 2 5   y    52 
 2 1  1 z   0 
    
1 1 1 
 
Let A   2 5  
 2 1  1
 
If matrix A has unique solution A  0
1 1 1
A  2 5  0
2 1 1
1 1 1
5  2  2 5
2 5     0
1 1 2 1 2 1
2 1 1
  11  0
  11
 For the system of equations to have unique solution   11

6.9 BINOMIAL THEOREM

Introduction
Pascal Triangle is the triangle which gives the coefficient of terms when
a  bn is expanded.

In writing the expansion of a  b , the following should be noted


n

(i) There should be n  1 terms


(ii) The coefficients are taken from Pascal triangle
(iii)Reading from either end of the expansion, the coefficients are the same

317
BUILD UP OF PASCAL TRIANGLE
The important concepts or formula used to build up Pascal triangle is
combination formula,
n!
i.e. C r 
n

n  r !r!
Development of Pascal Triangle

Pascal Triangle

318
Example 109
By using Pascal Triangle expand the following;
(a) a  b
4

(b) x  2
3

(c) x  3
5

(d) 2x  1
3

Solution
(a) a  b  a 4  4a 3b  6a 2b 2  4ab3  b 4
4

(b) x  2  x 3  6x 2  12x  8
3

(c) x  3  x 5 15x 4  90x 3  270x 2  405x  243


5

(d ) 2 x  1  8 x 3  12x 2  6 x  1
3

What is Binomial Expansion?


Binomial Expansion is an expansion of two terms under the same exponent.
The expansion of a  b is called binomial expansion because it has two terms
n

under the same exponent.


The binomial expansion deals with the expansion of a expression a  b raised
to power 'n' where 'n' is positive integer.
Hence Binomial Theorem state that
a  b n  nC0 a n0b 0  nC1a n1b1  nC2 a n2b 2  nC3 a n3b 3     nCn a nn b n
Express the R.H.S into sigma notation
n
a  b n   nCr a nr b r
r 0

n
Binomial Theorem a  bn   nCr a nr br
r 0

319
Example 110
By using Binomial theorem expand the following;
(i) x  24
(ii) 3x 1
5


(iii) x 2  1  3

Solution
(i ) x  2
4

n
a  b    nCr a nr b r
r 0
4
x  2 4
  4 C r x 4 r 2 r
r 0

x  2  C0 x 40 2 0  4C1 x 3 21  4C2 x 2 2 2  4C3 x1 23  4C4 x 0 2 4


4 4

 x  2  x 4  8 x 3  24x 2  32x  16
4

(ii) 3x  1
5

n
a  b   nCr a nr b r
r 0
5
3x  1   5Cr 3x 5r  1r
5

r 0

3x  1  C0 3x 5  10  5C1 3x 4  11  5C2 3x 3  12  5C3 3x 2  13  5C4 3x 1  14  5C5 3x 0  15
5 5

 3x  1  243x 5  405x 4  270x 3  90x 2  15x  1


5


(iii) x 2  1 
3

n
a  b    nCr a nr b r
r 0

x     
3 3
 1   3C r x 2 1   3C r x 2
2 3 3 r r 3 r

r 0 r 0

x 2
 3
 1  3C 0 x    C x   C x   C x 
2 3 3
1
2 2 3
2
2 1 3
3
2 0

  3
 x 2  1  x 6  3x 4  3x 2  1

320
Example 111
Find the value of 'a' if the 7 th and 8 th terms of the expansion 2  m20 are
equal.
Solution
Consider 2  m 
20

n
From a  b    n C r a nr b r
n

r 0
20
Then 2  m    20C r 2 20r m r and G.T  20C r 2 20r m r
20

r 0

The 7 term occurs when r  6 and 8th term occurs when r  7


th

20
C6 2 206 m 6  20C7 2 207 m 7
20! 14 6 20! 13 7
2 m  2 m
14!6! 13!7!
20!
1413!6!
 
2 213 m 6 
20! 13 7
13!76!
2 m

m  1

THE EXPANSION OF 1  x WHEN 'n' IS POSITIVE INTEGER


n

Consider the expansion of 1  x in ascending power of


n
x where 'n' is
n
positive integer. From Binomial theorem a  b    Cr a nr b r we can
n n

r 0

reduced the theorem into simple form as follows


n
1  x  n
  n C r 1nr x r but 1nr  1
r 0
n
1  x n   nCr x r
r 0

n
1  x n   nCr x r
r 0

321
Example 112
Expand the following;
(a) 1  x 
5

(b) 1  x
4

(c) 1  2 x 
3

Solution
(a ) 1  x 
5

n
1  x n
  nCr x r
r 0
5
1  x 5
  5C r x r
r 0

1  x   C0 x 0  5C1 x1  5C2 x 2  5C3 x 3  5C4 x 4  5C5 x 5


5 5

 1  x   1  5 x  10x 2  10x 3  5 x 4  x 5
5

(b) 1  x 
4

n
1  x n   nCr x r
r 0
4
1  x  4
  4Cr x r
r 0

1  x 4  4C0 x 0  4C1 x1  4C2 x 2  4C3 x 3  4C4 x 4


 1  x   1  4 x  6 x 2  4 x 3  x 4
5

(c) 1  2 x 
3

n
1  2 x n   nCr 2 x r
r 0
3
1  2 x    3Cr 2 x r
3

r 0

1  2 x 3  3C0 2 x 0  3C1 2 x 1  3C2 2 x 2  3C3 2 x 3


 1  2 x   1  6 x  12x 2  8 x 3
3

322
Example 113
10

Obtain the first four terms of the expansion 1   in ascending powers of x ,
x
 2
hence find the value of 1.005 correct to four decimal places.
10

Solution
10
 x
Consider 1  
 2
10 r
 x 10
 x
By using Binomial theorem 1     10C r  
 2 r 0 2
10 0 1 2 3
 x  10  x  10  x  10  x  10  x 
1    C0    C1    C 2    C3  
 2 2 2 2 2
10
 x 45
 1    1  5 x  x 2  15x 3
 2 4
Hence find, 1.005
10

1.00510  1  0.00510
x
Determine x by equating  0.005, x  0.01
2
1.00510  1  0.00510  1  50.01  0.01  150.01
45 2 3

4
 1.005  1.0511
10

THE EXPANSION OF 1  x WHEN 'n' IS NOT POSITIVE INTEGER


n

If 'n' is not a positive integer the expression above can be expanded to give an
infinity series of terms in ascending powers of x provided that 1  x  1.
n
1  x 
n
  nCr x r
r 0

1  x n  nC0 x 0  nC1 x1  nC2 x 2  nC3 x 3    


1  x n  1  nx  nn  1 x 2  nn  1n  2 x 3    
2! 3!
nn  1 2 nn  1n  2  3
1  x n  1  nx  x  x  
2! 3!

where 'n' is not positive integer.

323
Example 114
Expand the following in ascending power of x as far as the term in x 3
(a) 1  x 
2

(b) 4  x
1

(c) 1  x 2
1

Solution
(a) 1  x 
2

nn  1 2 nn  1n  2 3


From 1  x   1  nx  x  x  
n

2! 3!
1  x 2  1   2x   2 2  1 x 2   2 2  1 2  2 x 3
2! 3!
 1  x   1  2 x  3x  4 x
2 2 3

(b) 4  x 
1

nn  1 2 nn  1n  2 3


From 1  x   1  nx  x  x  
n

2! 3!
  x   1 1  1  x   1 1  1 1  2  x  
1 2 3
1  x
4  x   4 1    4 1   1  
1 1 
     
 4   4  2!  4  3! 4 
1
 
4  x   4 1  x   1 1  x  x  x 
2 3
1 1

 4 4  4 16 64 
1 x x2 x3
 4  x     
1

4 16 64 256

(c) 1  x  2
1

nn  1 2 nn  1n  2  3


From 1  x   1  nx  x  x  
n

2! 3!
    1 
1  x 12  1  12 x  2 2  1 x 2  2 2  1 2  2 x 3
1 1 1 1

2! 3!
 1  x  2  1  12 x  18 x 2  161 x 3
1

324
Example 115
Prove that if x is very small that its cube and higher power can be neglected
1 x x2
 1  x  , by taking x  1 , prove that 5 is approximately to 181 .
1 x 2 9 81
Solution
1 x 1  x 1  x  2
1

 1  x  2 1  x  2 .......... .......... .(i )


1
 
1
Consider
1 x 1  x 1  x  2
1

1  x n  1  nx  nn  1 x 2    
2!
1  x  2  1  1 2 x  2  2  1 x 2
1
1 1

2!
1  x  2  1  1 2 x  18 x .......... .......... .......... .......... .......... .......... .........( ii)
1
2

1  x  12  1  1 2 x   2  2  1 x 2
1 1

2!
1  x   1  1 2 x  3 8 x 2 .......... .......... .......... .......... .......... .......... .........( iii)
 12

Substitute equations (ii) and (iii) into equation (i)


1 x
1 x
 
 1  1 2 x  18 x 2 1  1 2 x  3 8 x 2  1  x  
x2
2
1 x x2
  1 x 
1 x 2

1 181
Take x  and then prove that 5 
9 81
1 1 x x2
Substitute the value of x  into the series  1 x 
9 1 x 2
1  19 1 
 1  19  9
2

1  19 2
5 181

4 162
1 181
5
2 162
181
5 2
162
181
 5
81

325
Example 116
If x is very small such that its fourth and higher powers are neglected, show
that 4 1  x  4 1  x  m  px2 and find the value of ' m' and ' p ' . Putting x  1
16

show that 4
17  4 15  3.9985
Solution
Consider 4 1  x  4 1  x  1  x  4  1  x  4 .......... .......... .......... .....( i )
1 1

1  x n  1  nx  nn  1 x 2  nn  1n  2 x 3    


2! 3!
1  x  4  1  1 4 x  4  4  1 x 2  4  4  1 4  2 x 3
1 1 1 1 1 1

2! 3!
1  x 14  1  1 x  3 x 2  7 x 3 .......... .......... .......... .......... .......... ....( ii)
4 32 128
1  x 14  1  1 x  3 x 2  7 x 3 .......... .......... .......... .......... .......... ....( iii)
4 32 128
Substitute equations (ii) and (iii) into equation (i)
 1 3 7 3  1 3 7 3
4
1  x  4 1  x  1  x  x 2  x   1  x  x 2  x 
 4 32 128   4 32 128 
3
4
1  x  4 1  x  2  x 2 .......... .......... .......... .......... .......... .......... ....( iv)
16
Equate equation (iv) with 4 1  x  4 1  x  m  px 2
3
 The value of m  2 and p 
16
Hence,
3 2
Put x  116 into 4 1  x  4 1  x  2  x
16
4 1  116  4 1  116  2   16 
3 1 2
16
3
4 17
16 4 15
16  2
163
1 4
2 
17  4 15  2  3
16
3

 3 
4
17  4 15  2 2  3   3.9985
 16 
 4 17  4 15  3.9985

326
Example 117
1
Write down the first three terms of the binomial expansion 3 1  , hence by
x
using the substitution x  1000 find the value of 3 37 to six decimal places.
Solution
1
1  1 3
Consider 1   1  
3
x  x
nn  1 2
From 1  x   1  nx  x  
n

2!
 13  1 1 2
1
1  1 3
 1    1  1 3 1 x  
1
3 1 3
x
x  x 2!
1 1
3 1  1  13 x  2
x 9x
Hence
1 1
3 1  1 1
31000  
91000
2
1000
999 1 1
3  1 
1000 3000 910002
33 1 1
37  1  
10 3000 910002
10  1 1 
3
37  1     3.332222
3  3000 910002 
 3 37  3.332222

Example 118
Show that the first three terms in the expansion 1  8x  in ascending powers
1
4

x 1  5x
of are the same as the first three terms in the expansion of . Use the
1  3x
corresponding approximation to write 1.16 4 as a rational fraction in its lowest
1

terms.
Solution

327
First consider 1  8 x  4
1

nn  1 2
From 1  x   1  nx  x  
n

2!
1  8 x  1
4
  1 8 x 2
1 1
 1  14 8 x   4 4
2!
1  8 x  1
4
 1  2 x  6 x 2 .......... .......... .......... .......... .......... .......( i )
1  5x
Also consider
1  3x
1  5x
1  3x

 1  5 x 1  3 x   1  5 x  1  3 x  9 x 2  1  2 x  6 x 2
1

1  5x
 1  2 x  6 x 2 .......... .......... .......... .......... .......... .......... ..(ii)
1  3x
1  5x
 1  8 x  4   1  2x  6x 2
1

1  3x
Hence
1.16  1  0.16 4 by comparing 8 x  0.16 determine x  0.02
1
4
1

 1.16 4  1  20.02  60.02 


1 2 1297
1250

THE EXPANSION OF 1  x WHEN x  1 AND 'n' IS NOT POSITIVE


n

INTEGER
If 'n' is not a positive integer the expression above can be expanded to give an
1
infinity series of terms in ascending powers of provided that x  1 .
x
n r
 1 n
1
1  x  n
 x 1    x n  n C r  
n

 x r 0  x
 0 1
1 n 1 n 1 n 1
2 3

1  x n  x  C0    C1    C 2    C3      
n n

  x  x  x  x 
  1  nn  1  1  nn  1n  3  1  
1 2 3

1  x  n
 x 1  n  
n
        
  x 2!  x  3!  x 
  1  nn  1  1  nn  1n  3  1  
1 2 3

1  x  n
 x 1  n  
n
        
  x 2!  x  3!  x 
where x  1 and 'n' is not positive integer.
328
Example 119
1
Expand the following in ascending power of x as far as the term in
x4
(a) 1  x 2
(b) 2  x1

Solution
(a) 1  x 
2

  1  nn  1  1  nn  1n  3  1  


1 2 3

1  x   x 1  n  
n n
        
  x 2!  x  3!  x 
  1  (2) 2  1  1  (2) 2  1 2  3  1  
2 1 2 3
 1
1  x 
2 2 2 
 x 1    x 1  (2)  
      
 x   x  2!  x 3!  x 
1  2  1   1  
2 3

1  x 
2
 2 1   3   5 
x  x  x   x  

1  x 2  1 2 3 5
  
x2 x3 x4 x5
 1  x   2  3  4
2 1 2 3
x x x

(b) 2  x 
1

  1  nn  1  1  nn  1n  3  1  


1 2 3

1  x   x 1  n  
n n
        
  x 2!  x  3!  x 
  2  (1) 1  1  2  (1) 1  1 1  3  2  
1 1 2 3
1  2
2  x   x 1    x 1  (1)  
1 1 
     
 x   x  2!  x  3 !  x 
1  2  2  4  2  
2 3

2  x   1        
1

x x  x 3 x 

 2  x    2  3  4
1 1 2 4 32
x x x 3x

329
Example 120
Expand x  2 as a series of descending power of x as far as the third term,
1
2

use the expansion to find the value of 2 by substituting x  100 .


Solution
1
 2
2

Consider  x  2   x 1  
1 1
2 2

 x
nn  1 2
From 1  x   1  nx  x  
n

2!
x  2 2  x 12 1  1 2 2 x   2  2  1 2 x 2 
1 1 1

 2! 
1  1 
x  2 1 1 1
 x 2 1   2   x 2  1  3
1 1
2

 x 2x  x 2 2x 2

 x  2 2  x 
1 1

1

x 2 x
3
 
Hence,
Evaluate 2 when x  100

100  212  100  1  1 3


100 2 100  
1 1
98  10  
10 2103
1 1
7 2  10  
10 2103
1 1 1 
2  10     1.4142
7 10 2103 
 2  1.4142

330
CONSTANT (ABSOLUTE) TERM OF BINOMIAL EXPANSION
This is a term independent of x or is the term which containing no variable x .
When your computing for constant term equate x raise its power and x 0

Example 121
4

What is the independent term of x in the expansion of  x  


1
 x
Solution
4
 1
x 
 x
From general term TG  nCr a n  r b r
r
4 r 1 4 4r r 4 4 2 r
TG  Cr x
4
   Cr x x  Cr x
 x
TG 4Cr x42r
Find the value of r where the constant term occurs
Equate x 42 r  x 0
4  2r  0, r  2
The constant term T0  occurs when r  2
T0  4C 2 x 42( 2)  4C 2  6
 The constant term is 6

COEFFICIENT OF TERMS OF BINOMIAL EXPANSION


n r r
Consider the general term of the series TG  Cr a
n
b

Example 122
Find the term containing x 3 from the following expansion.
(a) x  2
7

(b) 2 x  1
10

(c) 3  x 
4

Solution

331
(a)  x  2
7

From general term TG  n C r a nr b r


TG  7 C r x 7r 2 r
Find the value of r where the term containing x 3 occurs
Equate x 7r  x 3
7  r  3, r  4
The term containing x 3 Tx3  occurs when r  4
Tx3  7 C 4 x 74 2 4  7 C 4 x 3 2 4  560x 3
 The term containing x 3 is 560x 3

(b) 2 x  1
10

From general term TG  n C r a nr b r


TG 10C r 2 x  1 10C r 2 x 
10 r r 10 r
10C r 210r x10r
Find the value of r where the term containing x 3 occurs
Equate x10r  x 3
10  r  3, r  7
The term containing x 3 Tx3  occurs when r  7
Tx3 10C7 2107 x107 10C7 2 3 x 3  x 3
 The term containing x 3 is 960x 3

(c) 3  x 
4

From general term TG  nCr a n  r b r


TG  4Cr 34 r  x 
r

Find the value of r where the term containing x 3 occurs


Equate x r  x 3
r 3
The term containing x 3 Tx3  occurs when r  3
 
Tx3  4C3 343  x   4C3 3 x 3  12x 3
3

 The term containing x 3 is  12x 3

332
MIDDLE TERM OF THE BINOMIAL EXPANSION
n
Consider the Binomial expansion below a  b n   nCr a nr b r
r 0
n r
Now, consider the general term TG  Cr a b n r

n
When 'n' is even the middle term occur when r 
2
n 1
When 'n' is odd there are two middle terms which occurs when r 
2
Example 123
Find the middle term of the following expressions
(a) 1  x 
8

(b) 2 x  5
4

Solution
(a) 1  x 
8

From general term TG  n Cr a nr b r


TG  8Cr 18r x r  8Cr x r
Find the value of r where the middle term occurs,
n 8
The middle term occurs when r    4
2 2
The middle term Tm  occurs when r  4
Tm  8C4 x 4  70x 4
 The middle term is 70x 4

(b) 2 x  5
4

From general term TG  n C r a nr b r


TG  4C r 2 x 
4 r
5 r  4 C r 2 4 r 5 r x 4 r
Find the value of r where the middle term occurs,
n 4
The middle term occurs when r    2
2 2
The middle term Tm  occurs when r  2
Tm  4C 2 2 2 5 2 x 2  600x 2
 The middle term is 600x 2

333
Example 124
Find the middle term(s) of the following expressions
(a) 2  x 
5

(b) xy  1
7

Solution
(a) 2  x 
5

From general term TG  n C r a nr b r


TG  5C r 2 5r x r
Find the value of r where the middle term occurs,
n 1 5 1
The middle term occurs when r    3 or 2
2 2
The middle term Tm  occurs when r  3
Tm  5C3 2 2 x 3  40x 3
Also the middle term Tm  occurs when r  2
Tm  5C 2 2 3 x 2  80x 3
 The middle term are 40x 3 and 80x 3

(b) xy  1
7

From general term TG  nCr a n r b r


TG  7Cr  xy 1r  7Cr xy 
7r 7r

Find the value of r where the middle term occurs,


n 1 7 1
The middle term occurs when r    4 or 3
2 2
The middle term Tm  occurs when r  4
Tm  7C4 xy   7C4  xy  35xy 
74 3 3

Also the middle term Tm  occurs when r  3


Tm  7 C3 xy  7 C3 xy  35xy
7 3 4 4

 The middle term are 35xy and 35xy


4 4

334
Example 125
10
If the constant term in the expansion of  p  is 405 , find the value of p
 x 
 x2 
Solution
10
 p
consider  x  2 
 x 
From general term TG  n C r a nr b r

 x
r
 p  10  10 r   5r 
  2   C r  x  2   p  x  C r x  2   p 
10 r   r  2 r 10  5 
TG  C r
10 r

 x 
 5r 
 5 
The constant term T0  occurs when x  2 
 x0 , r  2
T0 10C 2  p   405
2

45 p 2  405
 The possible values of p  3

Exercise 6.5

1. Find the first five terms in the expansion of


1  x 3 in ascending power
1 x
of x .
x 1 1
2. If is so large that 3
and higher power of can be neglected, show that
x x
x 1 1 3
  2
x  1 x x
2

3
3. By using binomial series find a linear approximation for
1  2 x 2  x 
4. Find the first four terms of the expansion 1 2 x . By taking x  1100 find
102 correct to four decimal places.
5. If the term containing x 2 and higher power of x can be neglected. Show
 21  x 
2
that

1  x  1  x 2 
6. Use the expansion 1 150  2 to find the value of 2 to 4 significant
1

figures.
7. Expand 1  5x  3 as far as the term in x 3 by putting x  1 20 , find the value
1

of 3 6 to 3 significant figures.
335
8. Expand 2  x  in ascending power of x by taking the first three terms,
5

hence find the value 2.0015 correct to five decimal places.


9. Use binomial theorem to find the value of 1.0110 correct to three decimal
places.
10. By using binomial series for 1  x 4 evaluate an approximate value of
1

4
630 correct to 3 significant figures.
11. Expand 1  8x  2 in ascending power of x up to the term x 3 . Hence use the
1

result to find an approximate value of 3 to four decimal places.

General Examples

Example 126
The roots of the equation x 3  px2  qx  30  0 are in the ratio 2 : 3 : 5 , find the
values of ‘p’ and ‘q’.
Solution
Let  ,  and  be roots of the equation
 :  :  2 :3:5
  2k
  3k
  5k where k is a common factor
      2k  3k  5k  10k   p.......... .......... .......... .......... ......( i)
      2k 3k   2k 5k   3k 5k   31k 2  q.......... ......( ii)
  2k 3k 5k   30k 3  30.......... .......... .......... .......... .......... (iii)
Consider 30k 3  30
k 3  1
k  1.......... .......... .......... .......... .......... .......... .......... .......... .......... .(iv)
Substitute equations (iv) into equations (i) and (ii)
p  10
q  31
 The values p  10 and q  31

336
Example 127
The roots of the equation x 3  mx 2  nx  48  0 are in the ratio 1: 2 : 3 , find the
values of 'm' and 'n' .
Solution
Let  ,  and  be roots of the equation
 :  :   1: 2 : 3
 k
  2k
  3k where k is a common factor
      k  2k  3k  6k  m.......... .......... .......... .......... ......( i)
      k 2k   k 3k   2k 3k   11k 2  n.......... ......( ii)
  k 2k 3k   6k 3  48.......... .......... .......... .......... .......... (iii)
Consider 6k 3  48
k 3  8
k  2.......... .......... .......... .......... .......... .......... .......... .......... .......... .(iv)
Substitute equations (iv) into equations (i) and (ii)
m  12, n  44
 The values m  12 and n  44

Example 128
One of the zeroes of the polynomial function
f  x   x 4  2  h x 3  2h  5x 2  5h  6x  6h is obtained whenh  1. Find
the values of the constant p, q and r when f x   1  2 x  x 2  px 2  qx  r  .
Solution

337
Example 129
If m and n are roots of equation x 2  3 x  2  0 , find the equation whose roots
are m  n and n  m .
Solution
x 2  3x  2  0
Sum of roots; m  n  3.......... .......... .......... .......... .......... ......( i )
Product of roots; mn  2.......... .......... .......... .......... .......... ....( ii)
Sum and product of roots of the required equations
m  n   n  m   0.......... .......... .......... .......... .......... .......... .....( iii)
m  n n  m   m  n 2  m 2  n 2  2mn 
m  n n  m   m  n 2  4mn  (3) 2  4(2)  17.......... .(iv)
x 2  sum of rootsx  product of roots  0
 x 2  17  0

Example 130
Find the set of values of p for which f ( x)  x 2  3 px  p is greater than zero
for all values of x .
Solution
Given that f  x   x 2  3 px  p

For f x   0 this occur when f x  has complex roots i.e. b 2  4ac 
From b 2  4ac
3 p 2  4 p
9 p2  4 p  0
p9 p  4  0
Boundaries p  0, p  4
9

 The set of values is p  0

338
REVISION EXERCISE 6
1. If the quadratic equation ax 2  2bx  c  0 and ax 2  2cx  b  0
(where b  a ) have common roots, show that a  4b  4c  0 .
2. Prove that if the equations x 2  ax  1  0 and x 2  x  b  0 have
common roots, then b  12  a  11  ab .
3. If the equation x 2  ax  b  0 and cx 2  2ax  3b  0 , have a common
5a 2 (c  2)
root and a, b are non-zero then show that b  .
c  32
4. Find the relation between ‘q’ and ‘r’ so that x 3  3 px 2  qx  r shall be
a perfect cube for all values of x .
5. Given that x  1 and x  1 are factors of the expression x 3  ax 2  bx  c
and has the remainder of 12 when divided by x  2 , find
a  b c  c  a b .
abc
6. The roots of ax 2  bx  c  0 differ by 3, show that b 2  9a 2  4ac .
7. The roots of the equation x 3  px2  qx  30  0 are in the ratio 2 : 3 : 5 ,
find the values of ‘p’ and ‘q’.
8. Use synthetic division to find the value of q given that the polynomial
p ( x)  x 3  qx 2  2qx  4 is divided by x  1 .
9. When the expression x 5  4 x 2  kx  s is divided by x 2  1 , the
remainder is 2x  3 . Find the values of k and s .
1 1 1
10. Prove that   1
loga (abc) logb (abc) logc (abc)
3x 2  1
11. Express in partial fraction.
x  13
8x 2  x 1
12. Express in partial fraction.
x  15
4 x 2  5x  4
13. Express in partial fraction.
x  28
14. Prove the following laws of logarithms
 logc  logc
ab a b
(a) logc
 logc  logc
ab a b
(b) logc
log a
(c) logb 
a

logb
(d) loga  1
a

339
1
(e) logb 
a
b
loga
n
(f) logb a  n logb a

1 1
15. Given that a 2  b 2  7 ab , prove that log (a  b)  (log a  log b)
3 2
16. If a  logb , b  logc and c  loga , prove that abc  1.
c a b

b
log a  x log  .
3 x 5x 5 x 3x
17. If a b  a b prove that a
18. Prove the following;
7
(a) loga a a a 
8
(b) logb a loga b  1
 2b b 2 
( a b )  1  
 2 logc  log
a  a a2 
(c) 2 logc c 

loga  logb
x x
(d) logab  x

loga  logb
x x

19. Solve the following equations;


(a) x  log10 (1  2 x )  x log10 5  log10 6
(b) log2 x  8 log x 2  3 2

(c) log10 3  2 log10 (1  x)   0


(d) log( 2 x ) (3x  4 x  14)  2
2

(e) log2 sin x  log2 cos x  log2 (1tan x)  log2 (1tan x)  1

20. Prove by using mathematical induction that;


(a) cos  i sin n  cos n  i sin n where i   1
(b) 6 n  8 n is a multiple of 7 for all positive integer if n is odd.
d n
(c) ( x )  nx n 1
dx
x n1
(d)  x dx 
n

n 1
(e) 1  pn  1  np
(f) n! n 2 for n  4
(g) 2 n  n 2 for n  4
(h) 2 n  n where n N
340
(i) 3n  (n  1)! for n  4
(j) abn  a nb n
n
1 1
(k) 
k 1 k
2
 2 
n
for n  Z 

(l) The number of all subsets of a set containing n distinct elements


is 2 n .

21. By using mathematical induction, prove that for all positive integers;
(a) 9 n  1 is divisible by 8
(b) x 2 n  y 2 n is divisible by x 2  y 2
(c) n 3  3n 2  n is divisible by 3
(d) n(n 2  5) is divisible by 3
(e) 2 n  2  32 n 1 is divisible by 7
(f) a 2 n  b 2 n is divisible by (a  b)(a  b)

22. Show that ;


n
n(n  1)
(a) r 
r 1 2
n
n(n  1)(2n  1)
(b) r
r 1
2

6
 n(n  1) 
n 2

(c)  r   3

r 1  2 
1
23. Find the sum of the series x  2 x 2  3x 3  ...  nx n , if x  and n  14 ,
2
show that the sum of the series lies between 1.999 and 2 .
n 24
24. Using the standard results for  r 3 evaluate
r 1
r
r 16
3

x2
25. If x is real find the set of possible values of the function
x 1
26. Find the set of values of p for which f ( x)  x  3 px  p is greater
2

than zero for all values of x .


27 If  and  are the roots of the quadratic equation ax 2  bx  c  0 ,
show that ;
b
(a) The sum of roots     
a

341
c
(b) The product of roots  
a
28 If  ,  and  are the roots of the cubic equation
ax 3  bx 2  cx  d  0 , show that;
b
(a) The sum of roots       
a
c
(b) The sum of product of roots      
a
d
(c) The product of roots   
a
29 The roots of the equation 3 x  5 x  1  0 are  and  , find the value
2

of ;
(a)  2    2
(b)  2     2
(c)  3   3
2 2
(d) 
 
30 Given the roots of the equation 4 x 2  8 x  1  0 are  and  , find
the value of
1 1
(a) 2  2
 
(b)    2
(c)  3     3
1 1
(d) 
 2  2
31 The roots of the equation 2 x 2  4 x  1  0 are   2 and   2 , find
the equation whose roots are  and  .
32 The roots of the equation x 2  6 x  q  0 differ by one, find the
possible values of q .
33 If the roots of the equation x 2  2 px  q  0 differ by 2 , show that
p2  q  1
34 If the roots of the equation ax 2  bx  c  0 is twice the other, prove
that 2b 2  9ac
35 If the sum of the square of the roots of the equation ax 2  bx  c  0 is
one, prove that b 2  2ac  a 2
36 If the roots of 3 x 2  kx  12  0 are equal, find k .
37 The roots of x 2  px  ( p  1)  0 are equal, find p .

342
38 Find the relation between p and q . If the roots of this equation
px 2  qx  1  0 are equal.
39 Prove that the equation (k  2) x 2  2 x  k  0 has real roots for
whatever the value of k
40 The equation 3 x 3  6 x 2  4 x  7  0 has roots  ,  and  . Find the
equation whose roots are 1 , 1 , 1 .
  
41 If the roots of the equation x 3  9 x 2  3 x  39  0 are  ,  and  .
Show that the equation whose roots are   3 ,   3 and   3 , is
x 3  24 x  84  0 .
42 Write down the cubic equation given that       4 ,
 2   2   2  66 and  3   3   3  280 .
43 If the roots of the equation ax3  bx 2  cx  d  0 are  , 2 and 3 .
Show that 11b 2  36ac .
44 Find the value of k if ( x  1) is a factor of x 3  4 x 2  kx  6 , find also
the other factors of the expression.
45 Find the values of a and b if ( x  1) and ( x  2) are both factors of
x 3  ax 2  bx  4 .
46 What value of k will make x 3  2 x 2  kx  6 vanish when x  1 , for
what other values of x does the expression vanish.
47 Given that x  2 y is a factor of x 3  6 x 2 y  11xy 2  6 y 3 factorize the
expression completely.
48 Show that ( x  a ) is a factor of ( x  b) 7  (b  a) 7 .
49 Show that a  b  c is a factor of a 3  b 3  c 3  3abc .
50 When ax 2  bx  c is divided by x  1 the remainder is 8 , when divided
by x  1 the remainder is  6 and when divided by x  2 the remainder
is  4 , find the value of a , b and c .
51 The remainder when a polynomial is divided by x  2 is 3 and the
remainder when it is divided by x  1 is 6 . If the remainder when
f ( x ) is divided by x  2x  1 is px  q , find the values of p and
q.
52 When the expression x 3  kx 2  2 is divided by x  2 , the remainder is
1 less than when divided by x  1 . Find k
53 When the expression x 3  ax 2  2 x  1 is divided by x  2 , the
remainder is three times as greater as when the expression is divided by
x  1, find 'a'
54 Use synthetic division to find the quotient and remainder when;
(a) 4 x 3  3 x 2  x  7 is divided by x  2

343
(b) 2 x 3  3 x 2  5 is divided by x  3
(c) x 3  4 x 2  25x  28 is divided by x  1
55 Using synthetic division, find the value of a c , given that the
polynomial P( x)  x 3  cx 2  2cx  4 is divisible by x  1.
56 Given that the equation 18x 3  3x 2  88x  80  0 has repeated roots,
solve the equation.
1 1 1
57 Prove that a b c  (b  c)(c  a)(a  b)
bc ac ab
1 1 1
58 Prove that a b c  (a  b)(b  c)(c  a)
a2 b2 c2
a b bc c a
59 Prove that b  c c  a a  b  0
c a a b bc
a  b  2c a b
b  c  2a  2a  b  c 
3
60 Prove that c b
c a a  c  2b
61 If the polynomial p (x) is divided by x  1, x and x  1 the remainder are
1, 2 and 3 respectively. Find the remainder when p (x ) is divided by
x ( x 2  1) .
62 If 4 x 3  kx 2  px  2 is divisible by x 2  32 . Prove that kp  8
63 Find the condition for the simultaneous equations below to have no
x  5 y  az  2
solutions; 2 x  y  3z  1
7 x  8 y  8z  k
64 Determine the value of 'a' so that the following system in unknowns
x, y and z has;
(a) No solution
(b) More than one solution
(c) A unique solution
65 Which condition must be placed on a, b and c so that the following
system in unknowns x, y and z has a solution

344
x  2 y  3z  a
2 x  6 y  11z  b
x  2 y  7z  c
66 If logc  x , logb  y and loga  z , prove that x  y  z  xyz  2 .
ab ac bc

67 If the equation 2 x 3  9 x 2  12x  p  0 has two equal roots, find the


possible values of p .
68 If x 2  1 is a factor of 3x 4  x 3  4 x 2  px  q , find the values of p and
q.
69 If 4 x 3  mx 2  px  2 is divisible by x 2  k 2 , prove that mp  8
70 A polynomial expression P (x) when divided by ( x  1) leaves
remainder 3 and when divided by ( x  2) leaves remainder 1. Show that
when divided by ( x  1)( x  2) it leaves the remainder  2x  5
2 3
71 Solve the following inequality 
x 2 ( x  1)( x  2)
72 Show that if x is so small that x 3 and higher powers of x can be
3 1
(1  2 x)  4(1  x)
2 2
neglected, then  3  x  5x 2
1 x 2

2r  1 1 1 n
2r  1
2 , find  2
73 Given that 2  
r (r  1) 2
r (r  1)
2
r 1 r (r  1)
2

1 2 0
 
74 For M  0 p 0 prove by induction method, that
0 0 3
 2(1  p n ) 
1 0
 1 p 
M n  0 pn 0
0 0 3n 
 
 
75 Use laws of logarithm, show that 2 log n  b 
a
c
1
n log a 2  1n log c 2  log b 2 .
76 If sin and cos are roots of ax 2  bx  c  0 , show that
a 2  b 2  2ac  0

345
Chapter Seven
TRIGONOMETRY
Introduction
Trigonometry is the branch of mathematics which deals with measurement of
right angled triangle with their corresponding sides.

7.1 TRIGONOMETRIC RATIOS


The angles of the right angled triangles are determined by the ratios of the sides,
which are called trigonometric ratios.
Consider the sketch below with right angled triangle OAB.

fig 7. 1

From the above figure;


AB Opposite length
sin    sin  
OA Hypotenuse length
OB Adjacent length
cos    cos  
OA Hypotenuse length
AB Opposite length
tan    tan  
OB Adjacent length
sin
Also tan 
cos
Opp Adj Opp
Shortly, sin   , cos   , tan  
Hyp Hyp Adj

346
Other trigonometric functions (secant, cosecant and cotangent)

1 1 1
sec   , co sec   and cot 
cos  sin  tan

QUADRANT
Quadrant are position which divide xy  plane into four equal parts.

fig 7. 2

7.2 SPECIAL ANGLES


These are very useful angles used to solve or to prove some trigonometric
equations which needs the exact values of angles, some of these angles
according to ordinary level syllabus are 0  , 30 , 45 , 60 ,90  etc.

0 30  45  60  90 

sin 0 1 2 3 1
2
2 2

cos 1 3 2 1 0
2
2 2

tan 0 3 1 
3
3

347
Example 1
By using special angle evaluate cos75
Solution
 2  3   2  1 
 
cos75  cos 45  30  cos 45 cos30  sin 45 sin 30  
2
   
 2    2  2 
    
 2  3  1  6 2
cos75   



 2  2  4
6 2
 cos75 
4

Example 2
By using special angle evaluate sin15
Solution
 2  3   2  1 
 
sin15  sin 45  30  sin 45 cos30  cos 45 sin 30  
2
   
 2    2  2 
    
 2  3  1  6 2
sin15   



 2  2  4
6 2
 sin15 
4

Alternative
 3  2   1  2 
 
sin15  sin 60  45  sin 60 cos 45  cos60 sin 45  
2
   
 2    2  2 
    
 2  3  1  6 2
sin15   



 2  2  4
6 2
 sin15 
4

348
Example 3
By using special angle evaluate tan 105
Solution
tan 45  tan 60
tan 105  tan 45  60  
1  tan 45 tan 60
1  3  1  3  1  3  4  2 3
tan 105     
1  3  1  3  1  3 
  2

 tan 105   2  3 

Example 4
By using special angle evaluate cos 20
Solution
 
From cos60  cos3 20 let x  20
cos60  cos3x but cos3x  4 cos3 x  3 cos x
1
2  4 cos3 x  3 cos x
8 cos3 x  6 cos x  1  0
cos x  0.93969262 or  0.766044443
 The valid value of cos 20  0.93969262

ODD AND EVEN FUNCTIONS

A function is said to be odd function if and only if f (  x )  f ( x )

A function is said to be even function if and only if f ( x )  f ( x )

349
TRIGONOMETRICAL FUNCTIONS AS AN ODD AND EVEN
FUNCTIONS
Consider the figure below;

fig 7. 3

Consider OPN
y
sin  .......... .......... .......... .......... .......... .......... ..(i )
r
x
cos  .......... .......... .......... .......... .......... .......... .(ii)
r
y
tan  .......... .......... .......... .......... .......... .......... .(iii)
x
Also consider OMN
-y
sin(- )  .......... .......... .......... .......... .......... ........( iv)
r
x
cos(- )  .......... .......... .......... .......... .......... .......... .(v)
r

-y
tan(- )  .......... .......... .......... .......... .......... .........( vi)
x
Substituteequation (i) into equation (iv)

Sine is odd function sin(-)  sin

Substitute equation (ii) into equation (v)

Cosine is even function cos(- )  cos

350
Substitute equation (iii) into equation (vi)

Tangent is odd function tan(- )   tan

7.3 TRIGONOMETRIC IDENTITIES


Trigonometric identities are trigonometric expression connected together to
form useful relationship between basic trigonometric functions including their
reciprocals.

Derivation of trigonometric identities.


Consider the sketch below

fig 7. 4

From the triangle above determine cos , sin and tan


x
cosθ   x  r cosθ......... .......... .......... .......... ..(i)
r
y
sin θ   y  r sin θ......... .......... .......... .......... ...(ii)
r
y sin θ
tan θ   tan θ  .......... .......... ................ ..(iii)
x cosθ

351
Apply Pythagoras theorem
2 2 2
ON  NP  OP
x 2  y 2  r 2 .......... .......... .......... .......... .......... .......... .......... ....( iv)
Substituteequations(i) and (ii) into equaion (iv)
rcos 2  rsin 2  r 2
r 2 cos2   r 2 sin 2   r 2
r 2 cos2   sin 2    r 2

………………………………….(v)
cos2   sin 2   1

Divide by cos2  throughout equation (v)


cos2  sin 2  1
 
cos  cos  cos2 
2 2

1  tan 2   sec2 
……….…………………………… (vi)

Divide by sin 2  throughout equation (v)


cos2  sin 2  1
 
sin 2  sin 2  sin 2 

cot 2   1  cos ec 2 
……..………………………… (vii)

Example 5
Prove that tan   cot   sec cos ec
Solution
tan   cot  sec cosec
Consider L.H.S
sin cos sin 2   cos2 
tan   cot   
cos sin cos sin
1
tan   cot   sec cosec
cos sin
 tan   cot  sec cosec

352
Example 6
Prove that tan 2   sin 2   tan 2  sin 2 
Solution
tan 2   sin 2   tan 2  sin 2 
Consider L.H.S
sin 2  sin 2  sin 2   sin 2  cos2 
tan   sin  
2 2
 
cos2  1 cos2 
sin 2  1  cos2  
tan   sin  
2 2
 tan 2  sin 2 
cos 2

 tan 2   sin 2   tan 2  sin 2 

Exercise 7.1
Prove the following identities.
cos2  sin 3 
a)   1  sin cos
1  tan  sin  cos
3
b) sin 6 x  cos6 x  1  sin 2 2 x
4
c) tan A  tan B  tan C  tan A tan B tan C

d) sec4 A  sec2 A  tan 4 A  tan 2 A


e) sin 8   cos8   (sin 2   cos2  )(1  2 sin 2  cos2  )

1  sin
f)  sec  tan 
1  sin
sec  tan 
g)  1  2 sec tan   2 tan 2 
sec  tan 
sin 1  cos
h)   2 cos ec
1  cos sin
cos ec cos ec
i)   2 sec2 
cos ec  1 cos ec  1

353
7.4 ELIMINATION OF PARAMETER
Elimination of parameter is the process of removing the common term from two
or more equations by using simple trigonometric identities. Parameter is
common letter in the given set of equations.

Example 7
Eliminate  from the pair of equations x  5 tan  and y  cot
Solution
x  5 tan  .......... .......... .......... .......... .........( i )
y  cot .......... .......... .......... .......... .......... .(ii)
1
From y  cot 
tan 
1 1
y , tan   .......... .......... .......... ....( iii)
tan  y
Substitute equation (iii) into equation (i)
1
x  5 
 y
 xy  5

Example 8
Eliminate  from the pair of equations x  sin  cos and y  sin  cos
Solution
x  sin  cos .......... .......... .......... .......... .........( i)
y  sin  cos .......... .......... .......... .......... ........( ii)
Add equations (i) and (ii)
x  y  2 sin
x y
 sin .......... .......... .......... .......... .......... .(iii)
2
Subtract equations (i) and (ii)
x  y  2 cos
x y
 cos .......... .......... .......... .......... .......... .(iv)
2
Substitute equations (iii) and (iv) into cos2   sin 2   1
 x y  x y
2 2

    1
 2   2 
 x  y   x  y   4
2 2

354
Example 9
Eliminate  from the pair of equations x  sec  tan  and y  sec  tan 
Solution
x  sec  tan  .......... .......... .......... .......... .........( i)
y  sec  tan  .......... .......... .......... .......... ........( ii)
Add equations (i) and (ii)
x  y  2 sec
x y
 sec .......... .......... .......... .......... .......... .(iii)
2
Subtract equations (i) and (ii)
x  y  2 tan 
x y
 tan  .......... .......... .......... .......... .......... .(iv)
2
Substitute equations (iii) and (iv) into 1  tan 2   sec2 
 x y  x y
2 2

1    
 2   2 
 x  y   x  y   4
2 2

Example 10
Eliminate  from the pair of equations x  cos 2 and y  1  cos
Solution
x  cos 2 .......... .......... .......... .......... .......... ....( i )
y  1  cos .......... .......... .......... .......... .......... (ii)
From x  cos 2  2 cos2   1.......... .......... ...( iii)
Also from y  1  cos , cos  1  y.......... ....( iv)
Substitute equation (iv) into equation (iii)
x  21  y   1
2

2 y2  4 y  x 1  0

355
Exercise 7.2
1. Eliminate  from the following pair of equations.
(a) x  cos and y  2 sin
(b) x  a tan  and y  b cos
(c) x  2 sec and y  5 tan 
(d) x  sin and y  cos 2

2. Eliminate  from the following pair of equations.


(a) x  3 cos and y  2 tan 
(b) x  1  sin and y  2  cos 2
(c) x  tan  and y  tan 2
(d) x  a sin  b cos and y  b sin  a cos

3. Eliminate  from the following pair of equations.


(a) x  tan  and y  cos 2
(b) x  sin  tan  and y  sin  tan 
(c) x  m cot and y  n cos ec
(d) x  m tan   n sec and y  n tan   m sec

4. Eliminate  from the following pair of equations.


(a) x  p cos ec and y  q sec
(b) x  a sec and y  cos 2
(c) x  tan  and y  sin  cos
(d) x  cos and y  cosec  cot

356
7.5 COMPOUND ANGLE FORMULA
Compound angle formula are formula which describes the combination of two
or more angle(s) into simple form. We have three major compound angle
formulae which are compound angle formula for sine, compound angle formula
for cosine and compound angle formula for tangent.
Consider the sketch below on a unit circle.

fig 7. 5

Line ONR is parallel to line SMQ which implies that ROQ  OQM  A alternate angles 
Since OQP  90 which implies that MQP  OQP  OQM  90  A
Sum of angles of the triangle, PMQ  180
MQP  PMQ  MPQ  180
90 
 A  90  MPQ  180

 MPQ  A

(a) Compound angle for sine


ConsiderMPQ
MP
cos A  , MP  cos A sin B.......... ......( i)
sin B
Also consider ORQ
RQ
sinA  , RQ  sin A cos B.......... .......... ......( ii)
cosB

357
Also consider ONP
NP NM  MP
sin(A  B)    NM  MP
OP 1
sin(A  B)  MP  NM.......... .......... .......... .......... .. (iii)
Substitute equation (i) and equation (ii) into equation (iii)
sin(A  B)  sin A cos B  cos A sin B

Likewise
sin(A  B)  sin A cos B  cos A sin B

(b) Compound angle for cosine


Consider MPQ
MQ
sinA  , MQ  sin A sin B But MQ  NR
sin B
NR  sin A sin B.......... .......... .......... .......... .......... ..(i)
Also consider ORQ
OR
cosA  , OR  cos A cos B.......... .......... ......( ii)
cos B
Again consider ONP
ON ON
cos(A  B)    ON But ON  OR  NR
OP 1
cos(A  B)  OR  NR.......... .......... .......... .......... ..(iii)
Substitute equation (i) and equation (ii) into equation (iii)

cos(A  B)  cos A cos B  sin A sin B

Likewise
cos(A  B)  cos A cos B  sin A sin B

358
(c) Compound angle for tangent
sin(A  B) sin A cos B  cos A sin B
tan( A  B)  
cos(A  B) cos A cos B  sin A sin B
Divide the above equation by cos A cos B to each term

tan A  tan B
tan( A  B) 
1  tan A tan B

And also,

sin(A  B) sin A cos B  cos A sin B


tan( A  B)  
cos(A  B) cos A cos B  sin A sin B
Divide the above equation by cos A cos B to each term

tan A  tan B
tan( A  B) 
1  tan A tan B

Example 11
Prove that sin A  B   sin A  B   2 sin A cos B
Solution
sin A  B   sin A  B   2 sin A cos B
Consider L.H.S
sin A  B   sin A  B   sin A cos B  cos A sin B   sin A cos B  cos A sin B 
 sin A  B   sin A  B   2 sin A cos B

Example 12
Prove that sin A  B   sin A  B   2 cos Asin B
Solution
sin A  B   sin A  B   2 cos A sin B
Consider L.H.S
sin A  B   sin A  B   sin A cos B  cos A sin B   sin A cos B  cos A sin B 
 sin A  B   sin A  B   2 cos A sin B

359
Example 13
Prove that cos A  B  cos A  B  2 cos A cos B
Solution
cos A  B   cos A  B   2 cos A cos B
Consider L.H.S
cos A  B   cos A  B   cos A cos B  sin A sin B   cos A cos B  sin A sin B 
 cos A  B   cos A  B   2 cos A cos B

Example 14
Prove that cos A  B  cos A  B  2 sin Asin B
Solution
cos A  B   cos A  B   2 sin A sin B
Consider L.H.S
cos A  B   cos A  B   cos A cos B  sin A sin B   cos A cos B  sin A sin B 
 cos A  B   cos A  B   2 sin A sin B

Example 15
Prove that sin (A  B) sin (A  B)  sin 2 A  sin 2 B
Solution
sin A  B sin A  B   sin 2 A  sin 2 B
Consider L.H.S
sin A  B sin A  B   sin A cos B  cos A sin B sin A cos B  cos A sin B 
sin A  B sin A  B   sin 2 A cos2 B  cos2 A sin 2 B
  
sin A  B sin A  B   sin 2 A 1  sin 2 B  1  sin 2 A sin 2 B
sin A  B sin A  B   sin 2 A  sin 2 A sin 2 B  sin 2 B  sin 2 A sin 2 B
 sin A  B sin A  B   sin 2 A  sin 2 B

360
Example 16
1  tan 
Prove that tan( 45  ) 
1  tan 
Solution
1  tan 
tan( 45   ) 
1  tan 
Consider L.H.S
tan 45  tan 
tan( 45   )  but tan 45  1
1  tan 45 tan 

1  tan 
 tan( 45   ) 
1  tan 

Example 17
tan( 45  )  1  tan  
2

Prove that  
tan( 45  )  1  tan  
Solution
tan( 45   )  1  tan  
2

 
tan( 45   )  1  tan  
Consider L.H.S
tan 45  tan 
tan( 45   ) tan  but tan 45  1
 1  tan 45

tan( 45   ) tan 45  tan 


1  tan 45 tan 


1  tan 
tan( 45   )  1  tan   1  tan  
 1  tan     
tan( 45   ) 1  tan   1  tan   1  tan  
1  tan 
tan( 45   )  1  tan  
2

  
tan( 45   )  1  tan  

Example 18
cot A cot B  1
Prove that cot(A  B) 
cot B  cot A

361
Solution
cot A cot B  1
cot(A  B) 
cot A  cot B
Consider L.H.S
cos(A  B) cos A cos B  sin A sin B
cot(A  B)  
sin( A  B) sin A cos B  cos A sin B
Divide by sin A sin B to each term
cot A cot B  1
 cot(A  B) 
cot A  cot B

Example 19
π
If A  B  , prove that (1  tan A)(1  tan B)  2
4
Solution
π
From A  B  apply ' tan' both sides
4
π
tan  A  B   tan
4
tan A  tan B
1
1  tan A tan B
tan A  tan B  1  tan A tan B
tan A  tan B  tan A tan B  1
tan A  tan B1  tan A  1
Add by 1 both sides
1  tan A  tan B1  tan A  1  1
 1  tan A1  tan B   2

Exercise 7.3
1. Prove the following identities:
(a) cos(A  B) cos(A  B)  cos2 A  sin 2 B
cot A cot B  1
(b) cot(A  B) 
cot B  cot A
tan A  tan B  tan C  tan A tan B tan C
(c) tan( A  B  C) 
1  tan A tan B  tan B tan C  tan A tan C

2. Prove the following identities:


(a) sin (x  A)  cos(A  B)  cot(x  B) sin (A  B)
sin (x  B)

362
7.6 DOUBLE ANGLE FORMULA
Double angle formula are formula which express twice of the given angle into
half of it, basically we have three categories of double angle formula which are
double angle for sine, cosine and tangent, all double angle formula are derived
from compound angle addition formula as shown below;
(a) Double angle formula for sine
consider sin(A  B)  sin A cos B  cos A sin B
Let A  B
sin(A  A)  sin A cos A  cos A sin A
sin(2A)  sin A cos A  sin A cos A

sin 2A  2 sin A cos A

(b) Double angle formula for cosine


consider cos(A  B)  cos A cos B  sin A sin B
Let A  B
cos(A  A)  cos A cos A  sin A sin A

cos(2A)  cos2 A  sin 2 A

cos2 A  sin 2 A  1 make either cos2 A or sin 2 A the subject,


But
then substitute to the equation above

cos(2A)  2 cos2 A  1

and

cos(2A)  1  2 sin 2 A

(c) Double angle formula for tangent


tanA  tanB
consider tan(A  B) 
1 - tanAtanB
Let A  B
tanA  tanA
tan(A  A) 
1 - tanAtanA
2 tanA
tan( 2A) 
1 - tan 2 A

363
Example 20
Prove that sin 4x  4sin x cos x cos2x
Solution
sin 4 x  4 sin x cos x cos 2 x
Consider L.H.S
sin 4 x  2 sin 2 x cos 2 x
 sin 4 x  4 sin x cos x cos 2 x

Example 21
1  sin 2  
Prove that  tan 2    
1  sin 2 4 
Solution

2
1  sin 2  tan 4  tan  
    tan 4   2  tan 2 4   
1  sin 2  tan 4  tan  

1  sin 2
  tan 2 4   
1  sin 2

Example 22
Prove that cos 4θ  1  8 sin 2 θ cos 2 θ
Solution

364
cos 4  1  8 sin 2  cos2 
Consider L.H.S
cos 4  cos2 2  sin 2 2

cos 4  cos2   sin 2  2
 4 sin 2  cos2 
cos 4  cos4   sin 4   2 sin 2  cos2   4 sin 2  cos2 
cos 4  cos4   sin 4   6 sin 2  cos2  .......... .......... .......... .......... .(i )

From cos2   sin 2  2
 cos4   sin 4   2 sin 2  cos2 
cos4   sin 4   1  2 sin 2  cos2  .......... .......... .......... .......... ........( ii)
Substitute equation (ii) into equation (i)
cos 4  1  2 sin 2  cos2   6 sin 2  cos2 
 cos 4  1  8 sin 2  cos2 

Exercise 7.4
1. Prove that;
(a) cos 6 x  1  2 sin 2 3 x
2 tan 4 x
(b) tan 8 x 
1  tan 2 4 x
sin 8 x
(c)  8 cos x cos 2 x cos 4 x
sin x
2. Prove the following identities:
1  cos 2 1  tan 2 
(a)  tan  cos 2 
1  cos 2 (d) 1  tan 2 
(b) cot 2  cos ec2  cot  (e) tan   cos ec2  cot 2
(c) sin 2  2 tan 
1  tan 
2

3. Prove the following identities:


4 tan (1  tan 2 )
(a) tan 4 
1  6 tan 2   tan 4 
(b) 2 cos   2  2  2 cos 4
1  sin 2  cos 2
(c)  tan 
1  sin 2  cos 2
cos   sin  cos   sin 
(d) 2 tan 2θ  
cos   sin  cos   sin 

4. If tan 2   2 tan 2   1 prove that cos 2  sin 2   0 .

365
7.7 TRIPLE ANGLE FORMULA
Triple angle formula are formula which describes the treble of angle into single
angles as shown below;
(a) Triple angle formula for sine

Consider sin 3x  sin(2 x  x )  sin 2 x cos x  cos 2 x sin x


sin 3x  sin 2 x cos x  cos 2 x sin x , but sin2x  2sinxcosx and cos 2 x  1  2 sin 2 x
 
sin3x  2sinxcosx cos x  1  2 sin 2 x sin x
sin3x  2sinxcos2 x  sinx - 2sin 3 x
 
sin3x  2sinx 1 - sin 2 x  sinx - 2sin 3 x
sin3x  2sinx - 2sin 3 x  sinx - 2sin 3 x
sin3x  3sinx - 4sin3 x

(b) Triple angle formula for cosine


Consider cos 3x  cos(2 x  x )  cos 2 x cos x  sin 2 x sin x
cos 3x  cos 2 x cos x  sin 2 x sin x, but sin2x  2sinxcosx and cos 2x  2 cos2 x  1
 
cos3x  2 cos2 x  1 cos x  2sinxcosx sin x
cos 3x  2 cos3 x  cos x  2 cos x sin 2 x

cos 3x  2 cos3 x  cos x  2 cos x 1  cos2 x 
cos 3x  2 cos3 x  cos x  2 cos x  2 cos3 x
cos3x  4cos3 x - 3cosx

(c) Triple angle formula for tan


tan x  tan 2 x
consider tan3x  tan x  2x  
1  tan x tan 2 x
2 tan x
tan x 
1  tan 2 x  tan x 1  tan x   2 tan x
2
tan 3x 
 2 tan x  1  tan 2 x  tan x 2 tan x 
1  tan x  
 1  tan x 
2

tan x  tan 3 x  2 tan x 3 tan x  tan 3 x


tan 3x  
1  tan 2 x  2 tan 2 x 1  3 tan 2 x
3 tan x  tan 3 x
tan 3x 
1  3 tan 2 x

366
Example 23
Prove that sin   sin 3  cos   cos3  3
3 3

sin cos
Solution
Consider L.H.S

  

sin 3   sin 3 cos3   cos3 sin 3   3 sin  4 sin 3  cos3   4 cos3   3 cos 
sin cos sin cos
sin   sin 3 cos   cos3 3 sin  3 sin   3 cos   3 cos
3 3 3 3
  
sin cos sin cos
sin   sin 3 cos   cos3
3 3
  3  3 sin 2   3 cos2   3
sin cos
sin   sin 3 cos   cos3
 
3 3
  6  3 sin 2   cos2  but sin 2   cos2   1
sin cos
sin   sin 3 cos3   cos3
3
  3
sin cos

Example 24
1 1 1 1
If sin   n   , prove that sin 3   n  0
3

2 n 2 n3 
Solution
1 1
Consider sin 3  3 sin  4 sin 3  but sin   n  
2 n
3
3 1  1  1 
sin 3   n    4  n  
2 n  2  n 
3
3 1 1 1
sin 3   n     n  
2 n 2 n
3 3 1 3 3 3 1
sin 3  n   n  n  3
2 2n 2 2 2n 2n
1 1 
sin 3    n 3  3 
2 n 
1 1 
 sin 3   n 3  3   0
2 n 

367
Exercise 7.5
1. Prove that;
(a) sin 3θ  3 sin θ  4 sin3 θ
(b) cos 3θ  4 cos3 θ  3 cos θ
3 tan θ  tan 3 θ
(c) tan 3θ 
1  3 tan 2 θ

7.8 HALF ANGLE FORMULA


Half angle formula are formula which describes the trigonometric functions into
another trigonometric functions with half of the given angle, half angle formula
can be derived from double angle formula.
(a) Half angle formula for sine
Consider sin 2x  2 sin x cos x.......... ...( i)
Take half of each angle from equation (i)
 2x  x x
sin   2 sin  cos 
 2  2 2
x x
sin x  2 sin  cos 
2 2
In general half angle formula for sine is given by
 mx   mx 
sin mx  2 sin  cos  where m  
 2   2 
(b) Half angle formula for cosine
Consider cos 2x  cos2 x  sin 2 x.......... ...(i)
cos 2x  2 cos2 x  1.......... ........( ii)
cos 2x  1  2 sin 2 x.......... .........( iii)
Take half of each angle from equation (i), (ii) and (iii)

x x
cos x  cos2    sin 2  
2 2
x
cos x  2 cos2    1
2
x
cos x  1  2 sin 2  
2

368
In general half angle formula for cosine is given by
 mx  2  mx 
cos mx  cos2    sin  
 2   2 

(c) Half angle formula for tangent


2 tan x
Consider tan 2x  .......... ....( i)
1  tan 2 x
Take half of each angle from equation (i)

x
2 tan 
tan x  2
x
1  tan 2  
2
 mx 
In general half angle formula for tan is given by 2 tan  
tan mx   2 
 mx 
1  tan 2  
 2 

Example 25
sin  
Prove that  tan 
1  cos 2
Solution
sin  
 tan  
1  cos 2
Consider L.H.S
sin 2 sin2 cos2 

1  cos 1  2 cos2 2   1
sin 2 sin2 cos2 

1  cos 2 cos2 2 
sin  
  tan  
1  cos 2

369
Example 26
tan x  sec x  1 1  sin x
Prove that 
tan x  sec x  1 cos x
Solution
Consider L.H.S

tan x  sec x  1 sin x  1  cos x 2 sin 2x cos 2x   1  1  2 sin 2  2x 
 


tan x  sec x  1 sin x  1  cos x 2 sin 2x cos 2x   1  1  2 sin 2  2x  
tan x  sec x  1 cos 2x   sin 2x  cos 2x   sin 2x  cos 2x   sin 2x 
  
tan x  sec x  1 cos 2x   sin 2x  cos 2x   sin 2x  cos 2x   sin 2x 
tan x  sec x  1 1  2 sin 2x cos 2x 

tan x  sec x  1 cos2  2x   sin 2  2x 
tan x  sec x  1 1  sin x
 
tan x  sec x  1 cos x

Exercise 7.6
sin   
1. Prove that  cot 
1  cos 2
1  cos  
2. Prove that  tan 
sin 2
sin  
3. Prove that  tan 
1  cos 2
   
4. Prove that sin  2 sin  cos 
2 2
2  2 
5. Prove that cos  cos    sin  
2 2
2 
6. Prove that cos  2 cos    1
2
2 
7. Prove that cos  1  2 sin  
2

370
7.9 t-FORMULA
t-formula is any trigonometric formula expressed in term t where t is claimed to
 mx 
be tan of half of the given angle (i.e. t  tan  where m   ). We can
 2 
derive t-formula from any trigonometric formula like double angle formula, half
angle formula and etc.

(a) t-formula for sine


x x
consider sin x  2 sin  cos 
2 2
x x
2 sin  cos 
2 2 x x
sin x  but cos    sin    1
2 2

1 2 2
x x
2 sin  cos 
sin x  2  2  ………………...………………………...………(i)
x x
cos2    sin 2  
2 2
x
Divide by cos   to each term on right hand side (R.H.S) of the equation (i)
2

2
x
2 tan  
2 x
sin x  let t  tan 
x
1  tan 2  
2
2
2t
sin x 
1 t2

(b) t-formula for cosine


x x
consider cos x  cos    sin  
2 2

2 2
x x
cos2    sin 2  
2 2 x x
cos x  but cos    sin    1
2 2

1 2 2

371
x x
cos2    sin 2  
cos x  2  2  ………………………………………...…………(i)
x x
cos2    sin 2  
2 2
Divide by sin x  2t to each term on right hand side (R.H.S) of the equation (i)
1 t
2

x
1  tan 2  
2 x
cos x  let t  tan 
x
1  tan 2  
2
2
1 t2
cos x 
1 t2

(c) t-formula for tangent


x
2tan 
Consider tanx  2
x
1 - tan 2  
2
x
Let t  tan  
2

2t
tan x 
1 t2

 mx 
NB; t is equal to tan of half angle, i.e. t  tan  where m  
 2 

Example 27
5 sin  10t  
Show that  where t  tan  .
3 cos  4 sin  3  8t  3t 2 2
Solution

372
Consder L.H.S and divide by cos to each term
5 sin  5 tan 

3 cos  4 sin  3  4 tan 
 2 tan 2 
5 2  

5 sin  
  2 
1 tan
let tan 2  t
3 cos  4 sin   2 tan 2 
3  4 2  

 1  tan 2 
 2t 
5 
5 sin   1 t2 

3 cos  4 sin   2t 
3  4 2 
1 t 
5 sin  52t 

3 cos  4 sin  31  t 2  42t 
5 sin  10t
 
3 cos  4 sin  3  8t  3t 2

7.10 R-FORMULA
The expression of the form a cos mx  b sin mx where a , b   can be
expressed in either R sin(x  ) , R sin(x  ) , R cos(x  ) or R cos(x   ) ,
but for simplest if a and b are positive numbers use R sin(x  ) or
R cos(x   ) because when we expand the above compound angles they give
us positive terms as R sin(x  )  Rsin x cos   cos x sin  and
R cosx    R sin x cos   cos x sin  , also if
either a or b is negative number use R sin(x  ) or R cos(x  ) because when
we expand the above compound angles they give us one of the term is negative
as shown below R sin(x  )  R sin x cos   cos x sin  and
R cosx    R sin x cos   cos x sin  .

373
Example 28

Express sin x  3 cos x in the form of R sin(x  )

Solution

Equate sin x  3 cos x with R sinx   


sin x  3 cos x  R sinx   
sin x  3 cos x  Rsin x cos  cos x sin 
sin x  3 cos x  R sin x cos  R cos x sin
Equate coefficient of sin x and cos x
1  R cos .......... .......... .......... .......... .......... (i )
3  R sin .......... .......... .......... .......... .......( ii)
Divide equation (ii) by equation (i)
3 R sin

1 R cos
3  tan 
  tan 1 3
  60
Square equations (i) and (ii) then add
1  3 2
 R 2 cos2   R 2 sin 2 

4  R 2 cos2   sin 2  
R2  4
R  2 (Take the magnitude of R  2)
 sin x  3 cos x  2 sin x  60  

374
Example 29

Express 3 cos x  3 sin x in the form of R cos(x  )

Solution

Equate 3 cos x  3 sin x with R cos x   


3 cos x  3 sin x  R cos x   
3 cos x  3 sin x  Rcos x cos  sin x sin  
3 cos x  3 sin x  R cos x cos  R sin x sin 
Equate coefficient of cos x and sin x
3  R cos .......... .......... .......... .......... .......... (i )
3  R sin  .......... .......... .......... .......... .......( ii)
Divide equation (ii) by equation (i)
3 R sin 

3 R cos
3
 tan 
3
 3
  tan 1  
 3 
  30
Square equations (i) and (ii) then add
32   3 2
 R 2 cos2   R 2 sin 2 

12  R 2 cos2   sin 2  
R 2  12
R  2 3 (Take the magnitude of R  2 3 )
 3 cos x  3 sin x  2 3 cos x  30  

375
Example 30

Express 3 cos x  4 sin x in the form of R sin(x  )

Solution

Equate 3 cos x  4 sin x with R sin x   


3 cos x  4 sin x  R sin x   
3 cos x  4 sin x  Rsin x cos  cos x sin 
3 cos x  4 sin x  R sin x cos  R cos x sin
Equate coefficient of cos x and sin x
3  R sin .......... .......... .......... .......... .......... (i )
4  R cos .......... .......... .......... .......... .......( ii)
Divide equation (i) by equation (ii)
3 R sin

4 R cos
3
 tan 
4
3
  tan 1  
4
  36 52
Square equations (i) and (ii) then add
32  4 2  R 2 sin 2   R 2 cos2 

25  R 2 cos2   sin 2  
R 2  25
R  5 (Take the magnitude of R  5)

 3 cos x  4 sin x  5 sin x  36 52 

376
Example 31

Express 3 cos x  4 sin x in the form of R cos(x  )

Solution

Equate 3 cos x  4 sin x with R cos x   


3 cos x  4 sin x  R cos x   
3 cos x  4 sin x  Rcos x cos  sin x sin  
3 cos x  4 sin x  R cos x cos  R sin x sin 
Equate coefficient of cos x and sin x
3  R cos .......... .......... .......... .......... .......... ...( i )
4  R sin  .......... .......... .......... .......... .......... ....( ii)
Divide equation (ii) by equation (i)
4 R sin 

3 R cos
4
 tan 
3
4
  tan 1  
3
  53 7
Square equations (i) and (ii) then add
32  4 2  R 2 sin 2   R 2 cos2 

25  R 2 cos2   sin 2  
R 2  25
R  5 (Take the magnitude of R  5)
 3 cos x  4 sin x  5 cos x  53 7 
Example 32

Express 2 sin x  2 3 cos x in the form of Rf ( x   )

Hints

We can either express the given expression as R sin(x  ) or R sin(x  ) or


R cos(x   ) or R cos(x   )

377
Alternative 1

Equate 2 sin x  2 3 cos x with R sin x   


2 sin x  2 3 cos x  R sin x   
2 sin x  2 3 cos x  Rsin x cos  cos x sin 
2 sin x  2 3 cos x  R sin x cos  R cos x sin 
Equate coefficient of sin x and cos x
2  R cos .......... .......... .......... .......... .......... (i )
 2 3  R sin .......... .......... .......... .......... .......( ii)
Divide equation (ii) by equation (i)
 2 3 R sin

2 R cos
 3  tan 

  tan 1  3  60 
  300
Square equations (i) and (ii) then add
 
2
2 2   2 3  R 2 sin 2   R 2 cos2 

16  R 2 cos2   sin 2  
R 2  16
R  4 (Take the magnitude of R  4)
  
 2 sin x  2 3 cos x  4 sin x  300 or 2 sin x  2 3 cos x  4 sin x  60 

Alternative 2

Equate 2 sin x  2 3 cos x with R sinx   


2 sin x  2 3 cos x  R sinx   
2 sin x  2 3 cos x  Rsin x cos  cos x sin  
2 sin x  2 3 cos x  R sin x cos  R cos x sin 

378
Equate coefficient of sin x and cos x
2  R cos .......... .......... .......... .......... .......... (i )
2 3  R sin .......... .......... .......... .......... .......( ii)
Divide equation (ii) by equation (i)
2 3 R sin

2 R cos
3  tan 
 
  tan 1 3  60
Square equations (i) and (ii) then add
  2
2 2  2 3  R 2 sin 2   R 2 cos2 

16  R 2 cos2   sin 2  
R 2  16
R  4 (Take the magnitude of R  4)
 2 sin x  2 3 cos x  4 sin x  60  
Alternative 3

Equate 2 sin x  2 3 cos x with R cos x   


2 sin x  2 3 cos x  R cos x   
2 sin x  2 3 cos x  R cos x cos  sin x sin  
2 sin x  2 3 cos x  R cos x cos  R sin x sin 
Equate coefficient of sin x and cos x
2   R sin  .......... .......... .......... .......... .......... ..(i )
 2 3  R cos .......... .......... .......... .......... .......( ii)
Divide equation (ii) by equation (i)
2  R sin 

2 3 R cos
1
tan  
3

379
 1 
  tan 1    210
 3
Square equations (i) and (ii) then add
 
2
2 2   2 3  R 2 sin 2   R 2 cos2 
16  R 2 cos2   sin 2  
R 2  16
R  4 (Take the magnitude of R  4)
 2 sin x  2 3 cos x  4 cosx  210 

Alternative 4

Equate 2 sin x  2 3 cos x with R cos x   


2 sin x  2 3 cos x  R cos x   
2 sin x  2 3 cos x  Rcos x cos  sin x sin  
2 sin x  2 3 cos x  R cos x cos  R sin x sin
Equate coefficient of sin x and cos x
2  R sin  .......... .......... .......... .......... .......... ..(i )
 2 3  R cos .......... .......... .......... .......... .......( ii)
Divide equation (ii) by equation (i)
2 R sin 

 2 3 R cos
1
tan   
3
1 
  tan 1     150
 3
Square equations (i) and (ii) then add
 
2
2 2   2 3  R 2 sin 2   R 2 cos2 

16  R 2 cos2   sin 2  
R 2  16
R  4 (Take the magnitude of R  4)
 2 sin x  2 3 cos x  4 cos x  150  
380
NB:All the above alternatives represent the same thing in different format, but
if we simplify the above form to its simplest form give the same expressions as
follows;

(a) 2 sin x  2 3 cos x  4 sinx  300   4sin x cos300  cos x sin 300   2 sin x  2 3 cos x
(b) 2 sin x  2 3 cos x  4 sinx  60   4sin x cos 60  cos x sin 60   2 sin x  2 3 cos x
(c) 2 sin x  2 3 cos x  4 cosx  150   4cos x cos150  sin x sin150   2 sin x  2 3 cos x
(d) 2 sin x  2 3 cos x  4 cosx  210   4cos x cos 210  sin x sin 210   2 sin x  2 3 cos x

Example 33

Express  3 sin x  3 cos x in the form of R sin(x  ) and R cos(x  )

Solution

381

12  R 2 cos2   sin 2  
R 2  12
R  2 3 (Take R  2 3 )

 3 sin x  3 cos x  2 3 sin x  210 
Again consider - 3sinx - 3 cos x equal to Rcos(x - )
- 3sinx - 3 cos x  R cos(x  )
- 3sinx - 3 cos x  R cos x cos   sin x sin  
- 3sinx - 3 cos x  R cos x cos   R sin x sin 


12  R 2 cos2   sin 2  
R 2  12
R  2 3 (Take R  2 3 )

 3 sin x  3 cos x  2 3 sin x  240 
382
Example 34

Express 2 cos3x  5 sin 3x in the form of R cos(3x  )

Solution

383
Example 35

Express a cos mx  b sin mx as a single trigonometric function where


a , b, m   .

Solution
Use either of the following R-form,

384
Example 36

Express 3 sin 2 x  3 sin x cos x  4 cos2 x in the form of a  b cos(2x  ) .


Solution

385
Maximum and Minimum values by using R-form.

Maximum value is the highest value or amplitude of the function or curve.

Minimum value is the lowest value or amplitude of the function or curve.

Example 37
Find the maximum and minimum values 5sin x 12cos x
Solution
5 sin x  12 cos x  R sin x   
5 sin x  12 cos x  Rsin x cos  cos x sin  
5 sin x  12 cos x  R sin x cos  R cos x sin 
Equate
5  R cos .......... .......... .......... .......... .......... ..(i )
12  R sin  .......... .......... .......... .......... .......... ..(ii)
Square equations (i) and (ii) then add
5 2  (12) 2  R 2 cos2   R 2 sin 2 

169  R 2 cos2   sin 2  
R 2  169
R  13
 The maximum value is 13 and the minimum value is  13

Example 38

Find the maximum and minimum values 3sin x  4 cos x


Solution

386
Square equations (i) and (ii) then add
32  4 2  R 2 cos2   R 2 sin 2 

25  R 2 cos2   sin 2  
R 2  25
R  5
 The maximum value is 5 and the minimum value is  5

Example 39
Find the maximum, minimum and their corresponding angles values of
3 sin x  cos x
Solution
3 sin x  cos x  R sinx   
3 sin x  cos x  Rsin x cos  cos x sin  
3 sin x  cos x  R sin x cos  R cos x sin 
Equate
3  R cos .......... .......... .......... .......... .......... ......( i )
1  R sin  .......... .......... .......... .......... .......... .......... (ii)
Square equations (i) and (ii) then add
 3 12 2
 R 2 cos2   R 2 sin 2 
4  R 2 cos2   sin 2  
R2  4
R  2
Divide equation (ii) by equation (i)
1 sin 

3 cos

387
Example 40
Find the maximum and minimum value of 3sin x  4 cos x  7
Solution
3 sin x  4 cos x  7
Express 3 sin x  4 cos x  R sinx   
3 sin x  4 cos x  Rsin x cos  cos x sin  
3 sin x  4 cos x  R sin x cos  R cos x sin 
Equate
3  R cos .......... .......... .......... .......... .......... .......( i )
4  R sin  .......... .......... .......... .......... .......... .......( ii)
Square equations (i) and (ii) then add
32  4 2  R 2 cos2   R 2 sin 2 

25  R 2 cos2   sin 2  
R 2  25
R  5
3 sin x  4 cos x  5 sin x   
Then 7  3 sin x  4 cos x  7  5 sinx   
At maximum sin x     1
7  3 sin x  4 cos x  7  5  12
At minimum sin x     1
7  3 sin x  4 cos x  7  5  2
 The maximum value is 12 and the minimum value is 2

388
Example 41
1
Find the maximum and minimum of
5 cos x  12 sin x  10
Solution
1 1
Consider 
5 cos x  12 sin x  10 R cosx     10
Express 5 cos x  12 sin x  R cos x   
5 cos x  12 sin x  Rcos x cos  sin x sin 
5 cos x  12 sin x  R cos x cos  R sin sx sin
Equate
5  R cos .......... .......... .......... .......... .......... ..(i )
12  R sin .......... .......... .......... .......... .......... ..(ii)
Square equations (i) and (ii) then add
5 2  122  R 2 cos2   R 2 sin 2 

169  R 2 cos2   sin 2  
R 2  169
R  13
5 cos x  12 sin x  13cos x   
1 1
Therefore 
5 cos x  12 sin x  10 13cos x     10
At maximum cosx     1
1 1 1
 
5 cos x  12sin x  10 13  10 3
At minimum cos x     1
1 1 1
 
5 cos x  12sin x  10  13  10 23
1 1
 The maximum value is and the minimum value is 
3 23

389
Example 42
Find the maximum, minimum and their corresponding angles values of
3 sin x  3 cos x
Solution
3 sin x  3 cos x  R sin x   
3 sin x  3 cos x  Rsin x cos  cos x sin 
3 sin x  3 cos x  R sin x cos  R cos x sin
Equate
3  R cos .......... .......... .......... .......... .......... ..(i )
3  R sin .......... .......... .......... .......... .......... ..(ii)
Square equations (i) and (ii) then add
 3  3
2 2
 R 2 cos2   R 2 sin 2 

12  R 2 cos2   sin 2  
R 2  12
R  2 3
Divide equation (ii) by equation (i)
3 sin

3 cos
3  3 
tan   ,   tan 1    60
3  3
3 sin x  3 cos x  2 3 sin x  60  
At maximum value sin x  60  1  
x  60  sin 1 1
x  60  90
x  150
At minimum value sin x  60  1  
x  60  sin 1  1
x  60  90
x  30
 The maximum value is 2 3 at 150 and the minimum value is  2 3 at  30

390
Exercise 7.7
1. Find the maximum and minimum values of the following:
(a) 3 sin x  4 cos x
(b) 5 sin x  12 cos x
(c) 7 cos x  sin x
(d) cos x  3 sin x
(e)  3 sin x  4 cos x

2. Find the maximum, minimum and their corresponding angles values of


the following:
(a) 3 sin x  4 cos x
(b) 3 cos x  sin x
(c) 3 sin x  3 cos x
(d)  3 cos x  4 sin x
(e)  2 sin x  5 cos x

3. Find the maximum, minimum values and their corresponding angles, for
0   x  360
(a) 3 sin x  cos x
(b) 3 cos x  3 sin x

4. Find the maximum and minimum values of the following:


(a) 3 sin x  4 cos x  7
(b) 5 cos x  12 sin x  10
(c) 5 cos x  12sin x  13
(d) 6  3 cos x  4 sin x

391
7.11 FACTOR FORMULAE
These are formula used to express two or more terms of basic trigonometric
functions into factors, most especially sine and cosine. Factor formula are
derived from compound angle formula as shown below;
Consider the compound angles of sin(A  B) and sin(A  B)
sin(A  B)  sin A cos B  cos A sin B.......... .......... .......... ......( i)
sin(A  B)  sin A cos B  cos A sin B.......... .......... .......... ......( ii)

add aquation (i) and equation (ii)


sin(A  B)  sin(A  B)  2 sin A cos B.......... .......... .......... ....( iii)

subtract equation (i) and equation (ii)


sin(A  B)  sin(A  B)  2 cos A sin B.......... .......... .......... ....( iv)

Let A  B  p.......... .......... .......... .......... .......... .......... .......... ( v)


A  B  q.......... .......... .......... .......... .......... .......... .......... ( vi )
add aquation ( v) and equation (vi)
2A  p  q
pq
A .......... .......... .......... .......... .......... .......... ........( vii )
2
subtract aquation ( v) and equation (vi)
2B  p  q
pq
B .......... .......... .......... .......... .......... .......... ........( vii )
2
substitute equation (v), (vi), (vii) and (viii) into equation (iii)

pq  pq
sin p  sin q  2 sin  cos 
 2   2 
substitute equation (v), (vi), (vii) and (viii) into equation (iv)

pq  pq
sin p  sin q  2 cos  sin 
 2   2 

392
Also consider the compound angles of cos(A  B) and cos(A  B)
cos(A  B)  cos A cos B  sin A sin B.......... .......... .......... ......( i)
cos(A  B)  cos A cos B  sin A sin B.......... .......... .......... ......( ii)

add aquation (i) and equation (ii)


cos(A  B)  cos(A  B)  2 cos A cos B.......... .......... .......... ....( iii)

subtract equation (i) and equation (ii)


cos(A  B)  cos(A  B)  2 sin A sin B.......... .......... .......... ....( iv)

Let A  B  p.......... .......... .......... .......... .......... .......... .......... ( v)


A  B  q.......... .......... .......... .......... .......... .......... .......... ( vi )
add aquation ( v) and equation (vi)
2A  p  q
pq
A .......... .......... .......... .......... .......... .......... ........( vii )
2
subtract aquation ( v) and equation (vi)
2B  p  q
pq
B .......... .......... .......... .......... .......... .......... ........( vii )
2

pq  pq
sin p  sin q  2 sin  cos 
 2   2 

pq  pq
cos p  cos q  2 cos  cos 
 2   2 

substitute equation (v), (vi), (vii) and (viii) into equation (iv)

pq  pq
cos p  cosq  2 sin  sin 
 2   2 
NOTE
If the terms are in factor form its possible to express in addition or subtraction form (i.e.
reverse of factor formula). We use equations (iii) and equations (iv) above,

(a) 2 sin A cos B  sin(A  B)  sin(A  B)


(b) 2 cos A sin B  sin(A  B)  sin(A  B)
(c) 2 cos A cos B  cos(A  B)  cos(A  B)
(d)  2 sin A sin B  cos(A  B)  cos(A  B)

393
Example 43
Express sin8x  sin 2x in factor form
Solution
 pq  pq
sin p  sin q  2 sin  cos 
 2   2 
 8x  2 x   8x  2x 
sin 8 x  sin 2 x  2 sin  cos 
 2   2 
 sin 8 x  sin 2 x  2 sin 5 x cos3x

Example 44
Express sin 6x  sin 4x in factor form
Solution
 pq  pq
sin p  sin q  2 cos  sin 
 2   2 
 6x  4x   6x  4x 
sin 6 x  sin 4 x  2 cos  sin 
 2   2 
 sin 6 x  sin 4 x  2 cos5 x sin x

Example 45
Express cos7 x  cos3x in factor form
Solution
 pq  pq
cos p  cos q  2 cos  cos 
 2   2 
 7 x  3x   7 x  3x 
cos7 x  cos3x  2 cos  cos 
 2   2 
 cos7 x  cos3x  2 cos5 x cos 2 x

Example 46
Express cos4x  cos2x in factor form
Solution

394
 pq  pq
cos p  cos q  2 sin  sin 
 2   2 
 4x  2x   4x  2x 
cos 4 x  cos 2 x  2 sin  sin 
 2   2 
 cos 4 x  cos 2 x  2 sin 3 x sin x

Example 47
Express sin 2 x  sin 40 in factor form
Solution
 pq  pq
sin p  sin q  2 sin  cos 
 2   2 
 2 x  40   2 x  40 
sin 2 x  sin 40  2 sin  cos 
 2   2 
  
 sin 2 x  sin 40  2 sin x  20 cos x  20 

Example 48
Express 1 2 sin xcos x in factor form
Solution
 pq  pq
sin p  sin q  2 sin  cos 
 2   2 
 90  2 x   90  2 x 
1  2 sin x cos x  sin 90  sin 2 x  2 sin

 cos 
 2   2 
  
1  2 sin x cos x  2 sin 45  x cos 45  x 
Example 49
Express as the sum or difference of two sine 2 sin 5x cos2x
Solution
From 2 sin A cos B  sin A  B   sin A  B 
2 sin 5 x cos 2 x  sin5 x  2 x   sin5 x  2 x 
 2 sin 5 x cos 2 x  sin 7 x  sin 3 x

395
Example 50
Express as the sum or difference of two sine 2 cos6x sin 2x
Solution
From 2 sin A cos B  sin A  B   sin A  B 
2 cos6 x sin 2 x  sin6 x  2 x   sin6 x  2 x 
 2 cos6 x sin 2 x  sin 8 x  sin 4 x

Example 51
Express as the sum or difference of two sine sin 3x cos2x
Solution
From 2 sin A cos B  sin A  B   sin A  B 

sin 3 x cos 2 x 
1
2 sin 3x cos 2 x   1 sin3x  2 x   sin3x  2 x 
2 2
 sin 3 x cos 2 x  sin 5 x  sin x 
1
2

Example 52
Express as the sum or difference of two sine 2 sin 2x cos 3x 
Solution
From, 2 sin A cos B  sin A  B   sin A  B 
2 sin 2x cos 3x   sin 2x  3x   sin 2x  3x 
 2 sin 2x cos 3x   sin 56 x   sin 16 x 

Exercise 7.8
1. Express the following in factors:
(a) sin 6x  sin 4x
(b) cos3x  cos x
(c) sin x  sin 9x
(d) sin 4x  sin 2x
(e) cos 7x  cos3x
2. Express the following in factors:
5x 3x
(a) sin  sin
2 2
(b) cos8x  cos12x
(c) cos11x  cos5x
(d) sin 4 x  cos 60
(e) cos(5x  30 )  cos(x  20 )

396
3. Express the following in factors:
(a) sin 8A  sin 2B
(b) sin 2x  cos 2x
3
(c)  cos 4x
2
(d) 1  2 sin  cos 

4. Express as the sum or difference of two sine or two cosine:


(a) 2 sin 3x cos x
(b) 2 sin 4x cos 2x
(c) 2 cos5x cos 4x
(d)  2 sin 7x sin 3x
(e) 2 sin x cos8x
5. Express as the sum or difference of two sine or two cosine:
(a) sin 6x cos 4x
(b) sin x cos5x
(c) cos5x cos10x
(d)  sin 9x sin 2x
(e) 2 sin 3x sin 4x
6. Express as the sum or difference of two sine or two cosine:
(a)  2 sin 3x cos 2x
(b)  cos5x cos 7x
(c) sin x sin 4x
(d) 2 cos 2x
7. Express as the sum or difference of two sine or two cosine:
(a) 2 sin(A  B) cos(A  B)
(b) cos(A  B) sin(A  B)
(c) 2 sin(x  y) sin(x  y)
(d) cos A  B  cos A  B 
 2   2 
(e) 2 cos M  P  cos M  P 
 4   4 
8. Factorize completely the trigonometric expression
cos  cos3  cos5  cos7

397
Example 53
cos 9  cos 5
Prove that   sin 2 sec10
sin 17  sin 3
Solution
Consider L.H.S
cos9  cos5  2 sin 7 sin 2

sin17  sin 3 2 cos10 sin 7
cos9  cos5  sin 2

sin17  sin 3 cos10
cos9  cos5
   sin 2 sec10
sin17  sin 3

Example 54
cos A  cos B  A B 
Prove that  cot 
sin A  sin B  2 
Solution
Consider L.H.S
 A B   A B 
2 cos  cos 
cos A  cos B  2   2 

sin A  sin B  A B   A B 
2 cos  sin 
 2   2 
cos A  cos B  A B 
  cot 
sin A  sin B  2 

Example 55
sin   sin 3  sin 5  sin 7
Prove that  tan 4
cos   cos 3  cos 5  cos 7
Solution

398
Consider L.H.S
sin  sin 3  sin 5  sin 7

sin 7  sin   sin 5  sin 3 
cos  cos3  cos5  cos7 cos7  cos   cos5  cos3 
sin  sin 3  sin 5  sin 7 2 sin 4 cos3  2 sin 4 cos

cos  cos3  cos5  cos7 2 cos 4 cos3  2 cos 4 cos
sin  sin 3  sin 5  sin 7 sin 4 2 cos3  2 cos 

cos  cos3  cos5  cos7 cos 4 2 cos3  2 cos 
sin  sin 3  sin 5  sin 7
  tan 4
cos  cos3  cos5  cos7

Example 56
sin 6 sin 3  sin 2 sin 
Prove that  tan 5
cos 6 sin 3  cos 2 sin 
Solution
Consider L.H.S
sin 6 sin 3  sin 2 sin  12 cos9  cos3   12 cos3  cos 
 1
cos6 sin 3  cos 2 sin 2 sin 9  sin 3   2 sin 3  sin 
1

sin 6 sin 3  sin 2 sin  cos9  cos  2 sin 5 sin 4


 
cos6 sin 3  cos 2 sin sin 9  sin 2 cos5 sin 4
sin 6 sin 3  sin 2 sin
  tan 5
cos6 sin 3  cos 2 sin

Example 57
Prove that sin 5  2 sin 3  sin  2 sin cos 4  cos 2 
Solution
Consider L.H.S
sin 5  2 sin 3  sin  sin 5  sin  2 sin 3

399
Exercise 7.9
1. Prove the following identities;
(a) sin x  sin 3x  sin 5x  sin 7x  4 cos x cos 2x cos 4x
(b) cos9x  cos 7x  cos5x  cos3x  4 cos x cos 2x cos 6x
3  3
(c) cos 
    cos

     2 sin 
 4   4 
 
(d) cos     cos     2 cos
4  4 

2. Prove the following identities;


(a) sin A  sin B  tan A  B 
cos A  cos B  2 
(b) cos A  cos B   tan A  B 
sin A  sin B  2 
(c) sin A  sin B  tan A  B  cot A  B 
sin A  sin B  2   2 
(d) sin A  sin B  cot A  B  tan A  B 
sin A  sin B  2   2 

3. Prove the following identities;


(a) sin 6  2 sin 4  sin 2  4 cos2  sin 4
sin 3  sin 5  sin 7  sin 9
(b)  tan 6
cos 3  cos 5  cos 7  cos 9
sin 11 sin   sin 7 sin 3
(c)  tan 8
cos11 sin   cos 7 sin 3
tan 5  tan 3
(d)  4 cos 2 cos 4
tan 5  tan 3

4. Prove the following identities;


(a) sin 3x  sin 2x  sin x  4 sin x cos x  cos 3x 
2  2 
 2   4 
(b) cos  cos    cos    0
 3   3 
400
   2 
(c) sin   sin     sin   0
 3  3 
(d) cos 5  2 cos 3  cos   sin sin 2  sin 4
3 1
(e) cos 20 cos 40  sin 5 sin 25 
   

4
sin 70  cos 40 1
(f) 
cos50  sin 20
 
3

7.12 ANGLE OF THE TRIANGLE


Triangle is a figure with three sides and three corresponding angles. Let the
angles of the triangles be A, B and C, the sum of angles of the triangle is equal
to 180
Consider the sketch below;

fig 7. 6

From the figure above, A  B  C  180


Relationship between these angles
A  B  180  C.......... .......... .......... .......... .......... .......... ...( i)
Apply sin both sides to equation (i)
sin(A  B)  sin(180 - C)  sin180 cos C  cos180 sin C
sin(A  B)  sinC...... .......... .......... .......... .......... .......... ......(ii)

Apply cos both sides to equation (i)


cos(A  B)  cos(180  C)  cos180 cos C  sin 180 sin C
cos(A  B)  - cosC................ .......... .......... .......... .......... ..(iii)

401
Divide by 2 both sides of equation (i)
A  B 180  C

2 2
AB C
 90   .......... .......... .......... .......... .......... .......... ..(iv)
2 2
Apply sin both sides to equation (iv)
 AB   C C C
sin   sin 90    sin 90 cos  cos 90 sin
 

 2   2 2 2

 AB C
sin   cos .......... .......... .......... .......... .......... ........( v)
 2  2

Divide by 2 both sides of equation (i)


A  B 180  C

2 2
AB C
 90   .......... .......... .......... .......... .......... .......... ..(iv)
2 2
Apply sin both sides to equation (iv)
 AB   C C C
sin   sin 90    sin 90 cos  cos 90 sin
 

 2   2 2 2

 A B C
sin   cos .......... .......... .......... .......... .......... .......... ( v)
 2  2

Apply cos both sides to equation (iv)


 A B   C C C
cos   cos 90    cos 90 cos  sin 90 sin
 

 2   2 2 2

 A B C
cos   sin .......... .......... .......... .......... .......... .......... .( vi )
 2  2

402
Example 58
If A, B and C are the angles of a triangle, prove
tan A  tan B  tan C  tan A tan B tan C
Solution
tan A  tan B  tan C  tan A tan B tan C
Consider sum of angles of a triangle A  B  C  
tan  A  B  C   tan 
tan A  tan B  C 
0
1  tan A tan B  C 
tan A  tan B  C   0
tan B  tan C
tan A  0
1  tan B tan C
tan A1  tan B tan C   tan B  tan C  0
tan A  tan A tan B tan C  tan B  tan C  0
 tan A  tan B  tan C  tan A tan B tan C

Example 59
If A, B and C are the angles of a triangle, prove
sin 2A  sin 2B  sin 2C  4 sin A sin B sin C
Solution
sin 2 A  sin 2 B  sin 2C  4 sin A sin B sin C
Consider L.H.S
sin 2 A  sin 2 B  sin 2C  2 sin A  B cos A  B   2 sin C cosC.......... .......( i )
From A  B  C   , A  B    C
sin A  B   sin  C 
sin A  B   sin C.......... .......... .......... .......... .......... .......... .......... .......... ...( ii)
Substitute equation (ii) into equation (i)
sin 2 A  sin 2 B  sin 2C  2 sin C cos A  B   2 sin C cosC
sin 2 A  sin 2 B  sin 2C  2 sin C cos A  B   cosC .......... .......... .......... ..(iii)
Also apply cos into A  B    C
cos A  B   cos  C 
cos A  B    cosC.......... .......... .......... .......... .......... .......... .......... ........( iv)
Substitute equation (iv) into equation (iii)
sin 2 A  sin 2 B  sin 2C  2 sin C cos A  B   cos A  B 
sin 2 A  sin 2 B  sin 2C  2 sin C  2 sin A sin B 
 sin 2 A  sin 2 B  sin 2C  4 sin A sin B sin C

403
Example 60
If A, B and C are the angles of a triangle, prove
 A  B  C 
sin A  sin B  sin C  4 cos  cos  cos 
2 2 2
Solution
 A  B  C 
sin A  sin B  sin C  4 cos  cos  cos 
2 2 2
Consider L.H.S
 A B   A B  C  C 
sin A  sin B  sin C  2 sin  cos   2 sin  cos .......... .......( i )
 2   2  2 2
A B  C
From A  B  C   , 
2 2
 A B   C 
sin   sin  
 2  2 2
 A B  C 
sin   cos .......... .......... .......... .......... .......... .......... .......... .......... ...(ii)
 2  2
Substitute equation (ii) into equation (i)
C   A B  C  C 
sin A  sin B  sin C  2 cos  cos   2 sin  cos 
2  2  2 2
 C   A  B   C 
sin A  sin B  sin C  2 cos  cos   sin .......... .......... .......... ......( iii)
 2   2   2 
A B  C
Also apply cos into 
2 2
 A B   C 
cos   cos  
 2  2 2
 A B  C 
cos   sin .......... .......... .......... .......... .......... .......... .......... .......... .......( iv)
 2  2

404
Substitute equation (iv) into equation (iii)
 C   A  B   A  B 
sin A  sin B  sin C  2 cos  cos   cos 
 2   2   2 
 C   A    B 
sin A  sin B  sin C  2 cos  2 cos  cos 
 2   2   2 
 A  B  C 
 sin A  sin B  sin C  4 cos  cos  cos 
2 2 2

Exercise 7.10
1. If A, B and C are the angles of a triangle, prove the following identities:
(a) cos 2A  cos 2B  cos 2C  1  4 cos A cos B cos C
A B C
(b) cos A  cos B  cos C  1  4 sin sin sin
2 2 2

2. If A, B and C are the angles of a triangle, prove the following identities:


A B C
(a) sin A  sin B  sin C  4 sin sin cos
2 2 2
(b) sin 2A  sin 2B  sin 2C  4 cos A cos B sin C
(c) sin 2A  sin 2B  sin 2C  4 cos A sin B cos C
(d) cos 2A  cos 2B  cos 2C  1  4 sin A sin B cos C
(e) cos 2A  cos 2B  cos 2C  1  4 sin A cos B sin C

3. If A, B and C are the angles of a triangle, prove the following identities:


(a) cos 2A  cos 2B  cos 2C  1  4 cos A sin B sin C
sin A  sin B  sin C A B
(b)  cot cot
sin A  sin B  sin C 2 2
(c) tan 2A  tan 2B  tan 2C  tan 2A tan 2B tan 2C
(d) cot B cot C  cot A cot C  cot A cot B  1


4. If A  B  C  , prove that sin 2A  sin 2B  sin 2C  4 cos A cos B cos C .
2

405
7.13 TRIGONOMETRIC EQUATIONS
These are equations connecting trigonometric functions including their
reciprocals.
There are three ways of solving trigonometric equations, either by;
(a) Expressing the given equations into quadratic trigonometric equations or
into polynomial equations.
(b) By factorization method.
(c) By t-formula, i.e. equations of the form
 mx 
a cos(mx )  b sin(mx)  c, let t  tan 
 2 
GENERAL ANGLE FORMULA
1. Sine general angle formula,
  n  (1) n  where   principle angle and n  0,1,2...

2. Cosine general angle formula,

  2n   where   principle angle and n  0,1,2...


3. Tangent general angle formula,
  n   where   principle angle and n  0,1,2...

Example 61
Solve 2 sin 2   3 sin   1  0 for 0     360
Solution
2 sin 2   3 sin  1  0
sin  1 or sin  0.5
Consider sin  1
  sin 1 1  90 ,   90 
From   n   1  wher e n  0,1,2,...
n

  180 n   1n 90


  90
Also consider sin  0.5
  sin 1 0.5  30 ,   30 
From   n   1  wher e n  0,1,2,...
n

  180 n   1n 30


  30 ,150
 30 ,90 ,150

406
Example 62
Find the general solution of 2 tan 2   sec   1
Solution
2 tan 2   sec  1
 
2 sec2   1  sec  1
2 sec2   sec  3  0
sec  1 or sec   32
cos  1 or cos   23
Consider cos  1
  cos1 1  0  ,   0  
From   2n   wher e n  0,1,2,...
  2n
Also consider cos   23
  cos1  23   131 48 ,   131 48
From   2n   wher e n  0,1,2,...
  2n  131 48
  2n and   2n  131 48

Example 63
Solve cos 2x  sin x  0 for  500  x  2500
Solution
cos 2 x  sin x  0
1  2 sin 2 x  sin x  0
2 sin 2 x  sin x  1  0

407
Example 64
Find the general solution of cos x  sin x  2
Solution
cos x  sin x  2 .......... .......... .......... .......... .......... .......... (i )
1 t 2 2t
But cos x  and sin x 
1 t 2 1 t 2
1 t 2 2t
  2
1 t 2 1 t 2

1  t 2  2t  2 1  t 2 
1  t 2  2t  2  2t 2
 
2  1 t 2  2t  2  1  0

t
2  4  4 2 1   2 1  2

1  x
but t  tan  
2 2  1 
2 2 1 
2 1 2
 x 1
tan  
2 2 1
 1 
x
 tan 1    22.5   22.5  
2  2 1
From   n  
x
 n  22.5
2
 x  2n  45

408
Example 65
Solve sin 3  sin  0 for 0     360
Solution
sin 3  sin  0
3 sin  4 sin 3   sin  0
2 sin  4 sin 3   0

sin 1  2 sin 2   0 
sin  0 or sin   2
2

Consider sin  0
  sin 1 0  0  ,   0  
From   n   1  wher e n  0,1,2,...
n

  180 n
  0  ,180
Also consider sin  2
2

  sin 1 2
2

 45 ,   45 
From   n   1  wher e n  0,1,2,...
n

  180 n   1n 45


  45 ,135
Also consider sin   2
2

  sin 1     45 ,   45 


2
2  

From   n   1  wher e n  0,1,2,...


n

  180 n   1n  45 


  225 ,315
  0  ,45 ,135 ,180 ,225 ,315

Example 66
Solve sin 4  sin 2  cos  0 for 0     300
Solution

409
sin 4  sin 2  cos  0
2 sin 3 cos  cos  0
cos 2 sin 3  1  0
cos  0, sin 3  1
2

Consider cos  0
  cos1 0  90   90 
From   2n  
  360 n  90
  90 ,270

Also consider sin 3  1


2

3  sin 1 12  30   30  

Also from x  n   1 
n

3  180 n   1 30
n

  60 n   1n 10


  10 ,50 ,130 ,170 250 ,290

  10 ,50 ,90 ,130 ,170 250 ,270 ,290

Example 67
sin   sin 5
Solve  3
cos  cos 5
Solution
sin   sin 5 sin 5  sin  2 sin 3 cos 2
   tan 3  3
cos  cos 5 cos 5  cos 2 cos 3 cos 2
tan 3  3
3  tan 1 3

3  60   60 
From, x  n  
3  180 n  60
  60 n  20 where n  0,1,2...
  20 ,80 ,140 ,200 ,260

410
Exercise 7.11
1. Solve the following equations for 0     360
(a) 2 cos2   cos   1  0
(b) 2 tan 2   6  0
(c) 2 cos3   cos  0
(d) tan x  sec x  3

2. Solve the following equations for 0     360


(a) 8 sin   cosec  4
(b) 4 sin 2   12 cos2   0
(c) 1  5 sec   2 sec2   tan 2 
(d) 3  sec   cos   (2  sin ) tan 
(e) 5 sin 2   sin 2  2

3. Find the general solutions of the following equations:


(a) sin   3 cos   1
(b) cos 2x  cos x  0
(c) cos 2x  sin x  0
(d) 2 sin x  cosecx  0

4. Find the general solutions of the following equations:


(a) sin 2x  cos x  0
(b) sin 2x  sin x  0
(c) 3 tan x  cos x  0
(d) tan 2 x  2 cos2 x  4

5. Find the general solutions of the following equations:


(a) cos 2x  5 sin x  3
(b) sin 2x  cot x  0
(c) 2 cos 2 x  cos 4 x  1
(d) tan 4x  tan x  0
(e) tan 4x  tan 2x  0

6. By using t-formula, find the general solutions of the following equations:


(a) sec x  tan x  3
(b) cos 4x  2 sin 4x  1
(c) sin 2x  3 cos 2x  0
 
(d) 3 cos(x  45 )  sin( x  45 )
(e) 2 tan x  cot x  1  0

411
7. Find the general solutions of the following equations:
(a) 3 sin 2 x  cos2 x  cos x  0
(b) 2 tan x sin x  3
(c) 4 sin 2 x  12 sin 2 x  35 cos2 x  0
(d) 3 sin 4 x  2 sin 2 x  1  0
(e) cos3  cos 2  cos  0

8. Find the general solutions of the following equations:


(a) sin 4  sin 3  sin  sin 2
(b) sin 4  sin 3  sin 2  0
(c) tan 2 x  (1  3) tan x  3  0
(d) tan x  tan 2x  tan 3x  tan x tan 2x tan 3x

9. By using R-formula, find the general solutions of the following equations:


(a) sin x  3 cos x  1
(b) 3 sin x  3 cos x  2
(c) 4 cos x  3 sin x  5
(d) 2 cos x  2 sin x  0

10. Find the general solutions of the following equations:


sin   sin 3  sin 5
(a) 1
cos  cos 3  cos 5
(b) sin   sin 3  sin 5  sin 7  3
cos  cos 3  cos 5  cos 7 3
(c) sin 6 sin 3  sin 2 sin   1
cos 6 sin 3  cos 2 sin  3
(d) tan 3   3 tan   0

11. Find the general solutions of the following equations:


(a) 4 cos  3 sec  tan 
(b) cot  tan   2 cos ec
(c) sec 2 2  tan 2  1
(d) sin tan   1  tan   sin

412
7.14 INVERSE OF TRIGONOMETRIC FUNCTIONS
The inverse of trigonometric functions are occasionally also called arc
functions or anti-trigonometric functions.
Notation;
(a) sine inverse  sin 1 x  arcsin x
(b) cosine inverse  cos1 x  arccos x
(c) tangent inverse  tan 1 x  arctan x
(d ) cotangent inverse  cot 1 x  arc cot x
(e) sec inverse  sec1 x  arc sec x
( f ) cosecinverse  cosec 1 x  arccosecx

Relationship basic trigonometric inverse and reciprocal trigonometric


inverse.
1
(a ) sec-1 x  cos-1  
x
1
( b) cosec-1 x  sin -1  
x
1
(c) cot -1 x  tan -1  
x

Proof;
(a ) consider sec-1 x
1
Let p  sec-1 x  secp  x  x
cosp
1 1
 cos p  p  cos 1   But p  sec-1 x
x x
1
sec-1 x  cos 1   Proved
x

( b) consider sec-1 x
1
Let p  cosec
cosec x1 x cosecp  x 
-1
x
sinp
1 1
 sin p  p  sin 1   But p  cosec-1 x
x x
1
cosec-1 x  sin 1   Proved
x

413
(c) consider cot -1 x
1
Let p  cot -1 x  cotp  x  x
tanp
1 1
 tan p  p  tan 1   But p  cot -1 x
x x
1
cot -1 x  tan 1   Proved
x

NB: Relationship between trigonometric functions and its trigonometric inverse


functions.
(a ) sin 1 (sin x )  x (a ) sin(sin 1 x )  x
(b) cos1 (cos x )  x (b) cos(cos1 x )  x
(b) tan 1 (tan x )  x (b) tan(tan 1 x )  x
AND
(c) sec1 (sec x )  x (c) sec(sec1 x )  x
(d) cos ec 1 (cos ecx)  x (d) cos ec(cos ec 1 x )  x
(e) cot 1 (cot x )  x (e) cot(cot1 x )  x

Example 68
Find x if tan 1 2 x  1  tan 1 2 x  1  tan 1  18 
Solution
tan 1 2 x  1  tan 1 2 x  1  tan 1  18 
Let tan 1 2 x  1  A, tan A  2 x  1
tan 1 2 x  1  B, tan B  2 x  1

tan 1  18   C ,
1
tan C 
8
Now, A  B  C
tan  A  B   tan C
tan A  tan B
 tan C
1  tan A tan B
2 x  1  2 x  1  1
1  2 x  12 x  1 8
2 1
2

4x 8
x2  4
x  2
 The valid value of x is 2

414
Example 69
 x
If tan 1 3  cot 1 x  find the value of
2
Solution

tan 1 3  cot1 x 
2
1
Let A  tan 3 , tan A  3
1
B  cot x, cot B  x, tan B  1
x


Now; A  B 
2
 
tan  A  B   tan  
2
tan A  tan B 1

1  tan A tan B 0
1  tan A tan B  0
1
1  3   0
 x
1
3   1
 x
x  3

Example 70

If tan 1 x  tan 1 y  ; xy  1 , find the value of x  y  xy .
4
Solution

tan 1 x  tan 1 y 
4
1
Let tan x  A, tan A  x
tan 1 y  B, tan B  y

Now, A  B 
4

tan  A  B   tan
4
tan A  tan B
1
1  tan A tan B
tan A  tan B  1  tan A tan B
x  y  1  xy
 x  y  xy  1

415
Example 71
 1 x 
Solve 2 tan 1    tan x
1

1 x 
Solution
1 x 
2 tan 1  1
  tan x.......... .......... .......... .......... .(i )
1 x 
1 x  1 x
Let tan 1    A, tan A 
1 x  1 x
tan 1 x  B, tan B  x
Apply tan both sides 2 A  B
tan 2 A  tan B
2 tan A
 tan B
1  tan 2 A
1 x 
2 
1 x   x
 1 x 
2

1  
1 x 
1 x    1 x 2 
2   x 1    
 1  x    1  x  

21  x 1  x   x 1  x   1  x 
2 2

 
2 1  x 2  x4 x 
2  2x 2  4x 2
3x 2  1
1 1 3
x2  , x   
3 3 3
3
 The valid value of x is
3

Example 72
416
Find the principle value of tan 1 sin   
 
 2  
Solution
    
tan 1 sin    tan 1  1  
  2  4
    
 tan 1 sin    
  2  4

Example 73
Prove that tan 1 x  1 cos1  1  x 
2 1 x 
Solution
1 1 x 
tan 1 x  cos1  .......... .......... .......... .......... .......... .......... .......... .(i )
2 1 x 
1 x 
2 tan 1 x  cos1  .......... .......... .......... .......... .......... .......... .......... .(ii)
1 x 
Let P  2 tan 1 x
1 x
Also let A  tan 1 x , tan A  x , hence cos A  , sin A 
1 x 1 x
P  2A
Apply cos both sides
cos P  cos 2 A
cos P  cos2 A  sin 2 A
1 x
cos P  
1 x 1 x
1 x
cos P 
1 x
 1 x 
P  cos1   but P  2 tan
1
x
1 x 
1 x 
2 tan 1 x  cos1  
1 x 
1 1 x 
 tan 1 x  cos1  
2 1 x 

Exercise 7.12
417
 1 
1. Given that sin sin 1    cos1 x   1 , find the value of x
 5 
1  
2. Find the value of cot   2 cot 1 3 
2 
1  a  1  a  b 
3. Find the value of tan    tan  
b a  b
4. Solve costan x   sin cot
1  1 3 

 4
5. Solve for x if, 2 tan 1 (cos x )  tan 1 (2 cos ecx)
 
6. Prove that cot 1  1  sin x  1  sin x   x
 1  sin x  1  sin x  2
 
 
7. Prove that 2 tan 1  1   sec1  5 2   2 tan 1  1   
5   8 4
 7 
 1 x  1 x   1
8. Prove that tan 1     cos1 x
 1  x  1  x  4 2

1  x  2 1  x  2  
9. If tan    tan    , find the value of x
 x  4  x  4 4
10. Prove that cot 1 7  cot 1 8  cot 1 18  cot 1 3
1 3 4  7
11. Show that tan sin 1  
2 4 3
x 
12. Solve for if, tan 1 3x  tan 1 2 x 
4
11 1 1 1 1 
13. Prove that tan    tan    tan   
2 5 8 4
1  cos x   x
14. Prove that tan   
 1  sin x  4 2
x  xy
15. Find the value of tan 1    tan 1  
 
y  1  xy 
16. Solve for x if, cos2 sin 1 x   , x  0
1
9
 2x  1  1  x 
2
17. Solve for x if, tan  2 
 cot 
1
  
1 x   2x 

418
1 1 1  x 
18. Prove that tan x cos1  
2 1  x 
 2x  1  3x  x 
3
19. Prove that tan 1
x  tan 1
2 
 tan  
1  x   1  3x
2


20. Prove that tan 1    tan 1    tan 1    tan 1   
1 1 1 1
 3 5 7 8 4

21. Solve for x if , tan 1  1  x     tan 1 x


1  x  4
22. Prove that cos1 x  cos1 y  cos1 xy   1  x 1  y 
2 2

1  3
23. Prove that 2 sin    sin  
1 1

2 
 2 
24. Prove that tan 1  3   tan 1  1   tan 1  13   55.3
4  3 9

7.15 RADIANS AND SMALL ANGLES


Angle is the amount of revolution undergone by a revolving line in a plane.
Angles are measured in degree or radian.
Radian is the angle subtended at the centre of the circle by an arc whose length
is equal to the radius. When an angle is expressed in radians, the word ‘radian’
is omitted. Thus   180 is the short form of writing  radians  180
Converting Degrees into Radians and Radians into Degrees
The length of an arc of a given circle is proportional to the angle it subtends at
the centre. But an angle of 360 is subtended by an arc of length 2r ,
therefore an angle of  is subtended by an arc of length ' L' .
Consider the figure below;

fig 7. 7

419
2r  360
L  
Cross multiplication
360 L  2r
180 L  r
L  L
 
But  s (radians)
r 180 r

Formula used convert degrees to radians s
180

180 s
Formula used convert radians to degrees 

Approximation of Small Angles

Approximation of small angle is very important in simplification of the basic


trigonometric functions when the limits of the angle is approaching to zero and
also this concept is used in derivative of trigonometric functions most
especially in first principle.

Approximation of sine, cosine and tangent of angle, if angle is approaching to


zero.

(a) Approximation of sin 


Consider the series of sin 
 3  5  7  9 11
sin          .......... .
3! 5! 7! 9! 11!
When  is very small we truncate the serie by ignoring the higher powers

sin   

420
(b) Approximation of cos 
   
cos  cos2    sin 2  
2 2
 
cos  1  2 sin 2  
2
 
2

cos  1  2 
2
1
cos  1   2
2
1
cos   1   2
2

(c) Approximation of tan 


Consider the sketch below with right angled triangle OPN.

fig 7. 8

PN PN
From the above figure tan    .......... .......... ........( i)
ON r
As  tending to zero points P and M approaching to N, which implies that
PN  MN

421
Substitute PN  MN into equation (i)
MN
tan  .......... .......... .......... .......... .......... .......... .......... .....( ii)
r
But from arc length MN  r.......... .......... .......... .......... .........( iii)
Substitute equation (iii) into equation (ii)
r
tan 
r
tan  

Example 74
1  sin
Find the expression involving  for an approximation of for
5  3 tan   4 cos
small value of  .
Solution
1  sin
5  3 tan   4 cos
If  is very small sin   , tan    and cos  1  12  2
1  sin 1

5  3 tan   4 cos 5  3  4 1  12  2  
1  sin 1

5  3 tan   4 cos 1  3  2 2
1  sin 1

5  3 tan   4 cos 1   2  1
1  sin 1
 
5  3 tan   4 cos 2  1

Example 75
1  cos 2
Approximate as  is very small.
 sin 
Solution
1  cos 2
Consider
 sin 

1  cos 2 1  1  12 2 

2

 sin  2
1  cos 2 2 2
 2
 sin  
1  cos 2
 2
 sin 
422
General Examples

Example 76
Express cos2  sin 2 in the form r cos2    . Hence or otherwise find the
general solution in radians for the equation sin 2  cos2 1
Solution
cos 2  sin 2  r cos2   .......... .......... .......... .......... ........( i )
cos 2  sin 2  r cos 2 cos  sin 2 sin  
cos 2  sin 2  r cos 2 cos  r sin 2 sin  .......... .......... .......( ii)
Equate
1  r cos .......... .......... .......... .......... .......... .......... .......... ......( iii)
1  r sin  .......... .......... .......... .......... .......... .......... .......... .......( iv)
Square equations (iii) and (iv) then add
2  r 2 cos2   sin 2  
r  2 .......... .......... .......... .......... .......... .......... .......... .......... .(v)
Divide equation (iv) by equation (iii)
1  tan

  tan 1 1  45 
4
 cos 2  sin 2  2 cos2  45 

Hence solve sin 2  cos 2  1


cos 2  sin 2  1 but cos 2  sin 2  2 cos2  45 
1  2 cos2  45 

cos2  45  
1
2

2  45   cos1  1 


 2
2  45   45   45 
  

From, x  2n  
2  45  2n  45

2  2n  45  45 but 45 
4

423
 
2  2n  
4 4
 
  n  
8 8
When ( )
 
  n    n
8 8
When ()
  
  n    n 
8 8 4

 General solutions are   n and   n 
4

Alternative (Otherwise)
cos 2  sin 2  1
By t - formula
1 t 2 2t
 0
1 t 2
1 t 2
1  t 2  2t  1  t 2
t2 t  0
t t  1  0
t  0, t  1 but tan   0
tan   0
  tan 1 0  0,   0
  n  
  n.......... .......... .......... .......... .......... .......... .......( i )
tan   0
  tan 1  1  45 ,   45 
  n  45

  n  .......... .......... .......... .......... .......... .......... ..(ii)
4

 General solutions are   n and   n 
4

424
Example 77
Find the general solution of tan 4  tan 2  0
Solution
tan 4  tan 2  0
2 tan 2
 tan 2  0
1  tan 2 2
2 tan 2  tan 2 1  tan 2 2   0
tan 3 2  3 tan 2  0

tan 2 tan 2 2  3  0 
either tan 2  0 or tan 2  3
Consider tan 2  0
2  tan 1 0  0
2  0,   0
From, x  n  
2  n
n
  .......... .......... .......... .......... .......... ...(i)
2
Also consider tan 2  3
2  tan 1 3

2  60   60 
From, x  n  
 
2  n  60 but  60  
 3
n  
   30 but  30  
2  6
n 
   .......... .......... .......... .......... .........( ii)
2 6
n n 
 The general solutions are   ,   
2 2 6

425
Example 78
1  sin 2
Express in term of tan 
1  sin 2
Solution
1  sin 2 cos2   2 sin cos  sin 2   cos  sin 
2

Consider    
1  sin 2 cos2   2 sin cos  sin 2   cos  sin 
1  sin 2 cos  sin
 Divide by cos to each term
1  sin 2 cos  sin
1  sin 2 1  tan 
 
1  sin 2 1  tan 

Alternative
1  sin 2 1  2 sin cos
Consider
1  sin 2

1  2 sin cos
Divide by cos  to each term
2

1  sin 2 sec2   2 tan  1  2 tan   tan 2   1  tan  


2

    
1  sin 2 sec2   2 tan  1  2 tan   tan 2   1  tan  
1  sin 2 1  tan 
 
1  sin 2 1  tan 

Example 79
1  sin
Express in term of tan 2 
1  sin
Solution
cos2 2   2 sin2 cos2   sin 2 2   cos2   sin2  
2
1  sin
Consider    
1  sin     
cos 2  2 sin 2 cos 2  sin 2
2   
 
2 
  cos 
2   sin  
 
2 

1  sin cos2   sin2 


 Divide by cos2  to each term
1  sin cos 2   sin 2 
 

1  sin 1  tan 2 


 
1  sin 1  tan 2 

426
Alternative
1  sin 1  2 sin2 cos2 
Consider
1  sin

1  2 sin2 cos2 
Divide by cos   to each term
2 
2

sec2 2   2 tan 2  1  2 tan 2   tan 2 2   1  tan 2  
2
1  sin
    
1  sin sec2 2   2 tan 2  1  2 tan 2   tan 2 2   1  tan  
 
2 

1  sin 1  tan 2 


 
1  sin 1  tan 2 

Example 80

Solve cos1 x  cos1 x 8   
2
Solution
 
cos1 x  cos1 x 8 

2
Let A  cos1 x, cos A  x, sin A  1  x 2
 
B  cos1 x 8 , cos B  x 8 , sin B  1  8 x 2

Now, A  B 
2
 
cos A  B   cos 
2
cos A cos B  sin A sin B  0
cos A cos B  sin A sin B
8x 2  1  x 1  8 x  Square both sides then simplify
2 2

9x 2 1  0
 x  13

Example 81
If  and  are two values of  which satisfy the
equation a cos 2  b sin 2  c , such that tan  and tan  are not equal, prove
that
2b
(a) tan   tan  
ac
ca c
(b) tan  tan   , where a, b and are fixed numbers.
ca
Solution
427
ca
 tan  tan   (Product of roots)
ca

Example 82
If sin 1 y  2 cos1 x , show that y 2  4 x 2 1  x 2  .
Solution
sin 1 y  2 cos1 x.......... .......... .......... .......... ........( i )
Let A  sin 1 y, sin A  y
B  cos1 x, cos B  x, sin B  1  x 2
A  2 B.......... .......... .......... .......... .......... .......... ...( ii)
Apply sin both sides
sin A  sin 2 B
sin A  2 sin B cos B
y  2x 1  x 2

 y 2  4x 2 1 x 2 

428
Example 83
If tan 1 a   and tan 1 b   where ab  1, find   
Solution
tan 1 a  A, tan A  a.......... .......... .......... .......... .......... ......( i )
tan 1 b  B, tan B  b.......... .......... .......... .......... .......... ......( ii)
Then,     tan 1 a  tan 1 b
   A B
Apply tan both sides
tan      tan  A  B 
tan A  tan B
tan     
1  tan A tan B
a b
tan      if ab  1
1  ab
a b a b
tan       
1 1 0
     tan 1 
      90

Example 84
If cos  cos   a and sin   sin   b , show that a 2  b 2  21  cos(   ) 
Solution
a  cos  cos  .......... .......... .......... .......... .......... .......... .......... .....( i )
b  sin  sin  .......... .......... .......... .......... .......... .......... .......... ......( ii)
Square equations (i) and (ii) then add
a 2  b 2  cos  cos    sin  sin  
2 2

a 2  b 2  cos2   sin 2    cos2   sin 2    2cos cos   sin  sin  


a 2  b 2  2  2cos   
 a 2  b 2  21  cos   

Example 85
If sec  cos  a and cos ec  sin  b , prove that a 2b 2 a 2 
 b2  3  1 .
Solution

429
Consider
sec  cos  a.......... .......... .......... .......... .......... ...(i )
cosec  sin  b.......... .......... .......... .......... .......... (ii)
Square equations (i) and (ii) then simplify
a 2  sec2   2  cos2  .......... .......... .......... .......... ...( iii)
b 2  cos ec 2  2  sin 2  .......... .......... .......... .......... .(iv)
Add equations (iii) and (iv) then simplify
1
a2  b2  3
cos  sin 2 
2

1
a2  b2  3  .......... .......... .......... .........( v)
cos2  sin 2 
Multiply equations (iii) and (iv) then simplify
2
a b 2

1  cos   1  sin  
2 2 2 2

cos2  sin 2 
2
a b 2

sin   cos  
2 2 2 2

cos2  sin 2 
a 2 b 2  cos2  sin 2  .......... .......... .......... .......... .......... (vi)
Multiply equations (v) and (vi)
 a 2 b 2 a 2  b 2  3  1

Example 86
If x  r sin cos , y  r sin sin  , and z  r cos , prove that x2  y2  z 2  r 2 .
Solution
x  r sin cos .......... .......... .......... .......... .......... ........( i )
y  r sin sin  .......... .......... .......... .......... .......... ........( ii)
z  r cos .......... .......... .......... .......... .......... .......... ....( iii)
Square equations (i), (ii) and (iii) then add

x 2  y 2  z 2  r 2 sin 2  cos2   sin 2  sin 2   cos2  
x2  y2  z 2  r 2 sin  cos   sin    cos  
2 2 2 2

x2  y2  z 2  r 2 sin   cos  
2 2

 x2  y2  z2  r 2

430
Example 87
If x sin3   y cos3   sin  cos and x sin   y cos  0 , prove that
x  y 1
2 2

Solution
Consider x sin   y cos  0
sin y

cos x
y
tan   .......... .......... .......... .......... .......... .......... .......... .......... .(i )
x
Also consider x sin3   y cos3   sin  cos
Divide by cos3  to each term
x tan 3   y  tan  sec
x tan 3   y  tan  1  tan 2  .......... .......... .......... .......... .......... ..(ii)
Substitute equation (i) into equation (ii)
3 2
 y y  y
x   y  1  
x x x
y3 y x2  y2
y
x2 x x2
y3  x2 y y
2
 2 x2  y 2
x x
y3  x2 y  y x2  y 2
x  y   x
2 2 2
 y2
x  y   1
2 2 1

x  y 
2 2
1
2

x2  y2  1
 x2  y2  1

431
Example 88
If cos1 x  cos1 y  cos1 z   , show that x 2  y 2  z 2  2 xyz  1 .
Solution
Consider cos1 x  cos1 y  cos1 z  
Let cos1 x  A, cos A  x, sin A  1  x 2
cos1 y  B, cos B  y, sin B  1  y 2
cos1 z  C , cosC  z , sin C  1  z 2
Therefore, A  B  C  
A B  C apply cos both sides
cos A  B   cos  C 
cos A cos B  sin A sin B   cosC
xy  1  x 1  y    z
2 2

xy  z  1  x 1  y 
2 2

xy  z   1  x 1  y 
2 2 2

x 2 y 2  2 xyz  z 2  1  y 2  x 2  x 2 y 2
 x 2  y 2  z 2  2 xyz  1

Example 89
1  2 tan 
Prove that if sin( x   )  2 cos(x   ) then cot x 
2  tan 
Solution
Consider sin( x   )  2 cos(x   )
sin x cos  cos x sin   2 cos x cos  2 sin x sin 
sin x cos  2 sin x sin   2 cos x cos  cos x sin 
sin xcos  2 sin    cos x2 cos  sin  
cos  2 sin   cos x
2 cos  sin  sin x
cot x 
cos  2 sin  divide by cos to each term
2 cos  sin 
 cot x 
1  2 tan  
2  tan  

432
Example 90
cot  2
Prove that if sin( x   )  2 cos(x   ) then cot x 
2 cot  1
Solution
Consider sin( x   )  2 cos(x   )
sin x cos  cos x sin   2 cos x cos  2 sin x sin 
sin x cos  2 sin x sin   2 cos x cos  cos x sin 
sin xcos  2 sin    cos x2 cos  sin  
cos  2 sin   cos x
2 cos  sin  sin x
cot x 
cos  2 sin  divide by sin to each term
2 cos  sin 
 cot x 
cot  2
2 cot  1

Example 91
2  tan 
Prove that if sin( x   )  2 cos(x   ) then tan x 
1  2 tan 
Solution
Consider sin( x   )  2 cos(x   )
sin x cos  cos x sin  2 cos x cos  2 sin x sin
sin x cos  2 sin x sin  2 cos x cos  cos x sin
sin xcos  2 sin   cos x2 cos  sin 
sin x 2 cos  sin 

cos x cos  2 sin 
2 cos  sin
tan x  divide by cos to each term
cos  2 sin
2  tan 
 tan x 
1  2 tan 

433
Example 92

Prove that sin 4 x 



4 tan x 1  tan 2 x 
(1  tan 2 x) 2
Solution
Consider L.H.S

sin 4 x  2 sin 2 x cos 2 x  4 sin x cos x cos 2 x  4 sin x cos x cos2 x  sin 2 x 
 4 sin x cos x  cos x  sin x 
2 2
sin 4 x     but 1  cos2 x  sin 2 x
 1  1 
 4 sin x cos x  cos x  sin x 
2 2
sin 4 x    
 cos x  sin x  cos x  sin x 
2 2 2 2

Divide by cos4 x to the numerator and denominator


 4 sin x cos x  cos2 x  sin 2 x 
  
sin 4 x    
2 2
cos x cos x
 cos2 x  sin 2 x  cos2 x  sin 2 x 
  
 cos2 x  cos2 x 

sin 4 x 

4 tan x 1  tan x
2

 
1  tan 2 x 1  tan 2 x 
 sin 4 x 

4 tan x 1  tan 2 x 
1  tan x  2 2

Example 93
Given that a sin  b cos  c , show that a cos  b sin   a 2  b 2  c 2 .
Solution
Given that a sin  b cos  c
a sin  b cos 2  c 2
a 2 sin 2   2ab sin cos  b 2 cos2   c 2
 
a 2 1  cos2   2ab sin cos  b 2 1  sin 2   c 2  
a 2  a 2 cos2   2ab sin cos  b 2  b 2 sin 2   c 2

a 2  b 2  a 2 cos2   2ab sin cos  b 2 sin 2   c 2 
a 2  b 2  a cos  b sin   c 2
2

a 2  b 2  c 2  a cos  b sin 
2

 a cos  b sin   a 2  b 2  c 2

434
Example 94
Express r (sin  b cos ) in the form R cos(   ) giving the values of R
and tan  , prove that the maximum values of the expression is r 1  b and
2

1
that occurs when tan  
b
Solution
r (sin  b cos )  R cos   
r sin   rb cos  R cos cos   R sin  sin 
Equate
r  R sin  .......... .......... .......... .......... .......... .......... .......... ......( i )
rb  R cos  .......... .......... .......... .......... .......... .......... .......... ....( ii)
Divide equation (i) by equation (ii)
r R sin 

rb R cos 
1
tan   .......... .......... .......... .......... .......... .......... .......... .........( iii)
b
Square equations (i) and (ii) then add
r 2  r 2 b 2  R 2 sin 2   R 2 cos2 
  
r 2 1  b 2  R 2 sin 2   cos2  
r2 1  b   R
2 2

R  r 1  b 2
1
 The maximum value is R  r 1  b 2 that occurs when tan  
b

Example 95
3
Show that 1  sin 2  (sin  cos ) 2 . Hence solve (sin  cos ) 2  for
4
0   θ  90
Solution

435
x  n   1 
n

2  180 n   1 14 28


n

  90 n   1n 7 14


  7 14 and 82 46

Example 96

Show that cot      tan      0


4  4 
Solution

436
Consider L.H.S
    1  
cot     tan       tan    
4  4  tan      4 
 
4 
 
1  tan tan   tan 
tan
   
cot     tan      4  4
4  4  tan   tan  1  tan  tan 
4 4
   
 cot     tan      0
4  4 

APPLICATIONS OF TRIGONOMETRY
Trigonometry, the branch of mathematics that describes the relationship
between the angles and lengths of triangles, helped early explorers plot the stars
and navigate the seas. Nowadays, trigonometry is found in everything from
architecture to zigzag scissors. While it may seem as if trigonometry is never
used outside of the classroom, you may be surprised to learn just how often
trigonometry and its applications are encountered in the real world.

Architecture and Engineering


Much of architecture and engineering relies on triangular supports. When an
engineer determines the length of cables, the height of support towers, and the
angle between the two when gauging weight loads and bridge strength,
trigonometry helps him to calculate the correct angles. It also allows builders to
correctly lay out a curved wall, figure the proper slope of a roof or the correct
height and rise of a stairway. You can also use trigonometry at home to
determine the height of a tree on your property without the need to climb
dozens of feet in the air, or find the square footage of a curved piece of land.

Music Theory and Production


Trigonometry plays a major role in musical theory and production. Sound
waves travel in a repeating wave pattern, which can be represented graphically
by sine and cosine functions. A single note can be modeled on a sine curve, and
a chord can be modeled with multiple sine curves used in conjunction with one
another. A graphical representation of music allows computers to create and
understand sounds. It also allows sound engineers to visualize sound waves so
that they can adjust volume, pitch and other elements to create the desired
sound effects. Trigonometry plays an important role in speaker placement as
well, since the angles of sound waves hitting the ears can influence the sound
quality.

437
Electrical Engineers
Modern power companies use alternating current to send electricity over long-
distance wires. In an alternating current, the electrical charge regularly reverses
direction to deliver power safely and reliably to homes and businesses.
Electrical engineers use trigonometry to model this flow and the change of
direction, with the sine function used to model voltage. Every time you flip on a
light switch or turn on the television, you’re benefiting from one of
trigonometry's many uses.
Manufacturing Industry
Trigonometry plays a major role in industry, where it allows manufacturers to
create everything from automobiles to zigzag scissors. Engineers rely on
trigonometric relationships to determine the sizes and angles of mechanical
parts used in machinery, tools and equipment. This math plays a major role in
automotive engineering, allowing car companies to size each part correctly and
ensure they work safely together. Trigonometry is also used by seamstresses
where determining the angle of darts or length of fabric needed to craft a certain
shape of skirt or shirt is accomplished using basic trigonometric relationships.

Flight engineering
Flight engineers have to take in account their speed, distance, and direction
along with the speed and direction of the wind. The wind plays an important
role in how and when a plane will arrive where ever needed this is solved using
vectors to create a triangle using trigonometry to solve. For example, if a plane
is travelling at 234 mph, 45 degrees N of E, and there is a wind blowing due
south at 20 mph. Trigonometry will help to solve for that third side of your
triangle which will lead the plane in the right direction, the plane will actually
travel with the force of wind added on to its course.

Trigonometry in physics
In physics, trigonometry is used to find the components of vectors, model the
mechanics of waves (both physical and electromagnetic) and oscillations, sum
the strength of fields, and use dot and cross products. Even in projectile motion
you have a lot of application of trigonometry.

Trigonometry in criminology
In criminology, trigonometry can help to calculate a projectile’s trajectory, to
estimate what might have caused a collision in a car accident or how did an
object fall down from somewhere, or in which angle was a bullet shot etc.
438
Trigonometry in marine biology
Marine biologists often use trigonometry to establish measurements. For
example, to find out how light levels at different depths affect the ability of
algae to photosynthesize. Trigonometry is used in finding the distance between
celestial bodies. Also, marine biologists utilize mathematical models to measure
and understand sea animals and their behaviour. Marine biologists may use
trigonometry to determine the size of wild animals from a distance.
Trigonometry in marine engineering
In marine engineering trigonometry is used to build and navigate marine
vessels. To be more specific trigonometry is used to design the Marine ramp,
which is a sloping surface to connect lower and higher level areas, it can be a
slope or even a staircase depending on its application.
Trigonometry used in navigation
Trigonometry is used to set directions such as the north south east west, it tells
you what direction to take with the compass to get on a straight direction. It is
used in navigation in order to pinpoint a location. It is also used to find the
distance of the shore from a point in the sea. It is also used to see the horizon.

REVISION EXERCISE 7
1  sin 2 in term of
1. Express tan 
1  sin 2
2. Solve the equations;
a) tan 1 1  x   tan 1 1  x   32.


b) cos1 x  cos1 x 8   
2
3. If  and  are two values of  which satisfy the equation
a cos 2  b sin 2  c , such that tan  and tan  are not equal, prove that
2b
a) tan   tan  
ac
439
ca c
b) tan  tan   , where a, b and are fixed numbers.
ca
4. If sin 1 y  2 cos1 x , show that 
y 2  4x2 1  x2 
5. If tan x  sin x  m and tan x  sin x  p , show that m  p  4 mp
2 2

3
6. If sin 3  p and sin 2    q , prove that p 2  16q 3  12q 2 .
4
7. If tan 1 a   and tan 1 b   where ab  1, find   
8. Express cos 3 as a cubic equation in cos and use the result to find all
roots of the equation 4 x 3  3 x  1  0 .
9. Find the general solution of
(a) 4 sin x cos x  2 sin x  2 cos x  1  0
1
(b) sin 3 x cos3 x  cos2 2 x  0
2
(c) 1  tan tan 3  0
10. Find the maximum and minimum values of the functions;
1
(a) f ( )  3 sin   4 cos (b) g ( ) 
3 sin   4 cos  7

 x  y
11. If cos    cos     , then 9 x 2
1 1
 12xy cos  4 y 2  k , find k .
2 3
12. Prove that ;

 
a) sin 2 sin 1 x  cos1 x  1  x 2

b) 4 tan 1    tan 1    
1 24
2  7 
1  x2
c) cos2 tan 1 x  
1  x2
d) sin 4 x  cos4 x  1  2 cos2 x
 x y
e) tan 1 x  tan 1 y  tan 1  
 1  xy 

440
f) sec A  cosecA1  tan A  cot A  tan Asec A  cot A cosecA
g) 1  sec x  tan x 1  cosecx  cot x   21  tan x  cot x  sec x  cosecx
13. If A, B, C are the angles of the triangles,
A B C
prove that cos A  cos B  cosC  1  4 sin sin sin
2 2 2

14. If 2 A  B  45 , show that tan B  1  2 tan A  tan 2 A


2

1  2 tan A  tan A
15. If sec  cos  a and cos ec  sin  b , prove that a 2b 2 a 2 
 b2  3  1 .

16. If x sin 3   y cos3   sin  cos and x sin   y cos  0 , prove that
x2  y2  1

17. Find the values of x in radians between 0 and 2 which satisfy the
equation 6 tan 2 x  4 sin 2 x  1 .
18. Solve the following simultaneous equations for the values of x and y
cos x  4 sin y  1
between 0  and 360 ,
4 sec x  3 cos ecy  5

tan   sin  x and tan   sin   y , prove that x 2  y 2   16xy .


2
19. If
20. If x  2 sin and 3 y  cos , show that x 2  36 y 2  4

21. If cos  cos   a and sin   sin   b , show that

a 2  b 2  21  cos(   ) 

22. If x cos  y sin   a and x sin   y cos  b ,prove that

bx  ay
(a) tan   (b) x 2  y 2  a 2  b 2
ax  by
23. Given that sin x  sin y  a and cos x  cos y  b , show that

x y a b
2 2
ab
a) cos  2
 (b) sin( x  y ) 
 2  4 a2  b2
24. If cos1 x  cos1 y  cos1 z   , show that x 2  y 2  z 2  2 xyz  1 .

441
cos4  sin 4 
25. If   1 . Prove that ;
cos2  sin 2 

(a) cos2   sin2  1


4 4

cos  sin 
(b) sin 4   sin 4   2 sin 2  sin 2 

26. If 2 tan 2  tan 2  tan 2   tan 2  tan 2   tan 2  tan 2   tan 2  tan 2   1 ,
prove that sin 2   sin 2   sin 2   1

27. If tan 2   1  m 2 , prove that sec  tan 3  cosec   2  m 2  


3/ 2

28. If 10sin 4   15 cos4   6 , find the value of 27 cos ec 6  8 sec6 


29. Show that sec2 x cos ec 2 x  4 for all those x which sin x cos x  0
30. If m  sin x  cos x and n  sec x  cosecx , prove that n(m  1)(m  1)  2m
31. If m  cot  cos and n  cot  cos , show that m 2  n 2  2
 16mn
32. If x 2  a cot  b cosec and y 2  b cot  a cos ec , prove that
x4  y 4  b2  a2
33. If a 3  cosec  sin and b 3  sec  cos , prove that a 2 b 2 (a 2  b 2 )  1
34. If x  r sin cos , y  r sin sin  and z  r cos , prove that
x2  y2  z 2  r 2 .

sin A cos A m 1 n2
35. If  m and  n , prove that tan A  
sin B cos B n m2 1
m2 1
36. If tan   n tan  and sin  m sin  , prove that cos   2 2

n 1
   
37. Find the value of sin A cosec
2
 A   cot2   A  cos A
2  2 
38. If p , q and r are given by ( p  r )(q  r )  r 2  1 , show that

1 1 1


tan 1    tan 1    tan 1  
 p q r
39. Express r (sin  b cos ) in the form R cos(   ) giving the values of
R and tan  , prove that the maximum values of the expression is

r 1  b 2 and that occurs when tan   1


b

442
40. Express 5 sin 2 x  3 sin x cos x  cos2 x in the form a  b cos(2 x   ) where
a,b,  are independent of x . Hence, otherwise, find the maximum and

minimum values of 5 sin 2 x  3 sin x cos x  cos2 x as x varies.


41. Find the value of x between 0  and 360 which satisfy the equation,
10 sin 2 x  10 sin x cos x  cos2 x  2

42. Solve for values of x and y between  90 and 180 inclusive ,

tan x  tan y  1

 1
cos x cos y 
 2
43. Express cos2 A16 cos4 A  24 cos2 A  9 as a single term of cosine using
cos3 A  4 cos3 A  3 cos A

44. Prove that cos2   cos2   2   cos2   4   3


 3   3  2

45. Prove that cot  tan  2 cot 2 ,


Hence, deduce that tan  2 tan 2  4 tan 4  8 cot8  cot
46. If  and  are two unequal values of  which satisfy the equation
a cos  b sin  c show that
 2b
(i) tan  tan 
2 2 ac
  ca
(ii) tan tan 
2 2 ca
1 1 b
(iii) sin (   ) sec (    ) 
2 2 c
47. (a) Write 2 cos x  5sin x in the form k cos(x  b) for k  0 , 0  x  2 .
(b) Use (a) to solve the equation 2 cos x  5sin x  2 for 0  x  
2ab
48. If sin x  sin y  a and cos x  cos y  b show that sin( x  y ) 
a2  b2
49. If n is even or odd show that,
 cos A  cos B   sin A  sin B  n A B 
n n

     2 cot  0
 sin A  sin B   cos A  cos B   2 
50. Prove that cos2 3 x  cos2 x   sin 4 x sin 2 x

443
sin16
51. Show that  16 cos cos 2 cos 4 cos8
sin
52. Solve 3 cos2 x  3 sin x cos x  2 sin 2 x  3 .
53. Find the general solution of the equation 2 cos2  2 sin cos  1
54. Find the solution of 3 cos2 x  3 sin x cos x  2 sin 2 x  1  0
55. Solve 2 sin x cos x  4 cos3 x  3 cos x  0 for 0  x  
tan x 1  sec x
56. Prove that   2 cos ecx
1  sec x tan x
57. Solve the equation 5 sin 2 x  4 cos2 x  4  tan 2 x giving all values of x in
the interval 0  to 360
58. Solve cot  tan  2 cosec in the interval 0  to 360
59. Given that 6 sin 2   4 cos2   5 tan  . If t  tan , show that the equation
reduces to 5t 3  6t 2  5t  4  0 . Hence solve the equation for
0     360 .
60. Solve 4 sin 2 x  12 sin 2 x  35 cos2 x  0 for 0     90
5 3
61. Prove that sin 6   cos6    cos 4 if cos 4  1  8 sin 4   8 sin 2  .
8 8
Hence solve the equation sin 6   cos6   sin 2 cos 2 for 0   θ  90
 A  1 x
62. If sec A  tan A  x , prove that tan  .
 2  1 x
cot  2
63. Prove that if sin( x   )  2 cos(x   ) then cot x 
2 cot  1
sin 5 A
64. Prove that  4 cos2 2 A  2 cos 2 A  1
sin A

65. Prove that sin 4 x 



4 tan x 1  tan 2 x 
(1  tan 2 x) 2
sin 4 A(1  cos 2 A)
66. Prove that  tan A
cos 2 A(1  cos 4 A)
67. Prove that sin 3  2 cos2 sin  sin
68. If sin(  45 )  3 cos(  45 )  0 , show that tan  1 hence find  if
0     360
   
69. Show that cot     tan     0
4  4 
3
70. Show that 1  sin 2  (sin  cos ) 2 . Hence solve (sin  cos ) 2  for
4
0   θ  90

444
sin x  2 sin 2 x  sin 3 x 2 x 
71. Given that y  , prove that y   tan   . Hence
sin x  2 sin 2 x  sin 3 x 2
find the exact value of tan 2 15 in the form p  q r where p, q and r are
integers, find the value of x between 0  and 360 for which
 x
2 y  sec2    0
2
 
72. Prove that cos 2  1  tan 2  , Hence deduce that tan    3  2 2
2
2

1  tan  8
tan 4   6 tan 2   1
73. Show that cos 4 
(1  tan 2  ) 2
  1  sin 2
74. Prove that tan 
2
   and hence prove that
4  1  sin 2
tan 22.5  2 1
75. Find the greatest and the least values of the expression below and values of
5
x where 0     360 for which they occur
6 sin x  3 cos x  4
76. Express 12cos x  5sin x in the form R cos(x   ) where R is constant and
 is acute angle. Hence state the maximum and minimum values of
3
20  12 cos x  5 sin x
77. Solve the equation 3 cos 2  sin 2  2 for any values of  between 0 
and 360
1
78. Determine the minimum and maximum values of , state
sin x  3 cos x  5
clearly values of x for which they occur. Hence solve sin x  3 cos x  0
for 0     360
sin A  sin 3 A  sin 5 A  sin 7 A
79. Prove that  tan 4 A
cos A  cos3 A  cos5 A  cos 7 A
80. If sin( y   )  2 cos(y   ) , prove that tan y  2  tan
1  2 tan 
81. If x  cos2 A  cos2B and y  sin 2 A  sin 2 B , show that
1 2
cos2 ( A  B )  (x  y 2 )
4
82. If x  b cos  a cos3 and y  b sin   a sin 3 , show that
x 2  y 2  (a  b) 2  4ab sin 2 

445
83. If x  sin  sin 2 and y  cos  cos 2 , show that
y 2  x 2  2 cos3 (1  cos )
84. Given that x  sin  cosec and y  cos  sec , prove that
x 2  y 2  1  4 cosec 2 2 .
85. If sin 2 A  sin 2 B  q and cos 2 A  cos 2 B   p , show that
q p2  q2
cos(A  B) 
p2  q2
yx
86. Given that tan(   )  x and tan(   )  y , prove that tan 2 
1  xy
87. Show that sin(C  2 D)  sin C  tan(C  D)
cos(C  2 D)  cosC
88. Show that sin(  240 )  sin  sin(120   )  0
89. Show that there is a positive value of x for which
tan 1 (2 x  1)  tan 1 (2 x  1)  tan 1 (2)
1  63  1  1  1  3 
90. Show that cos    2 tan    sin  
 65  5 5
91. Solve the equation tan 1 (2)  tan 1 ( x)  tan 1 (4)
1 1  2x 
92. Prove that 2 tan ( x)  sin  2 
 1  x 
1  1  1  1 
93. Find x if tan x   tan    2 tan  
1

 3 8
94. Without using tables or calculators, determine the value of
1 1 1
tan 1    tan 1    tan 1  
2 5 8
1  1  1  1 
95. Given that tan    tan    x , find the value of x in the range
2 5
0 x 
 1  1   2 
96. Without using table or calculators evaluate cos sin    sin  
1
  
2 2 
  
x from the equation 
97. Find 2 sin 1 ( x)  sin 1 ( x 2 ) 
2
98. Show without using tables or calculators that
1
2 tan 1 2  tan 1 3    tan 1  
 3

446
99. Solve the equation tan 1 x   tan 1 3x     
4
100. Show that sec1  5   sec1  5   sec1  25 
 3 4  24 
101. Prove that sin 2 A  sin 2 B  sin 2C  8 sin A  sin B  sin C 
sin A  sin B  sin C 2 2 2
cos2 t  3 cost  2 2  cost
102. Prove that 
sin 2 t 1  cost
sinx  y  a  b tan x a
103. Prove that  , then show that 
sinx  y  a  b tan y b
1 1
104. If sin y  2 cos x , show that y  4 x 1  x
2 2
 2

105. Solve for x given that tan cos1 x   sintan 1 2
106. If sin 1 x  sin 1 y  sin 1 z   , prove that

x 4  y 4  z 4  4x 2 y 2 z 2  2 x 2 y 2  y 2 z 2  z 2 x 2 .
tan 10  tan 20
107. By using special angles evaluate
1  tan 10 tan 20
108. If tan  2x   cosecx  sin x , show that tan  2x   2  5
2

109. Without using calculating devices show that 3 cosec20  sec 20 .
1 x2
110. Prove that cos2 tan 1 x  
1 x2
111. Prove that cos tan   33 tan   1  3 sec  10 sin

447
Chapter Eight

LINEAR PROGRAMMING
Introduction

In our daily life we face problems of maximization and minimization concepts


at which the best solution is done by the concepts of linear programming.

Linear programming is a study which deals with solving of problems of


maximization and minimization of a certain requirement, or linear
programming is defined as method which deals with optimization of linear
functions subjected to certain conditions.

Linear programming uses graphical techniques in solving problems under a


certain objectives function depending on the condition imposed.

Terms used in linear programming

Linear constraints are inequalities in terms of x and y used as boundaries in


optimization problems.

Non-negative constraints are inequalities which guide the requirements to be


greater than or equal to zero i.e x  0, y  0 .

Feasible region is the area bounded by linear constraints which satisfy the linear
constraints.

Objective function is an equation or expression in term of x and y used to find


the optimal value.

Optimal solution is the solution obtained from optimal point of a feasible region

Optimal value is the maximization or minimization value of linear


programming problem obtained from objective function.

Decision variable are letters which represent a certain value in linear


programming problems eg x and y .

448
BASIC PRINCIPLES OF LINEAR PROGRAMMING

(i) Obtain the set of linear inequalities (constraints) from the word problem
(ii) Obtain the objective function from the word problem
(iii)Represent the constraints in xy  plane and shade unrequired part/region
(iv) Obtain the corner points from the feasible region and substitute them into
objective function in order obtain the optimal point.
(v) Use the optimal point to obtain the optimal value and draw/make the
conclusion
NOTE

1. At least means minimization (minimization problem use  )


2. At most means maximization (maximization problem use  )

8.1 Graphical Solutions


Graphical method of linear programming is used to solve problems by finding
the highest or lowest point of intersection between the objective function line
and the feasible region on a graph.

Example 1
A company produces two types of ornaments A and B that requires gold and
silver. Each unit of type A requires 4 grams of silver and 8 grams of gold.
Type B requires 8 grams of silver and 4 grams of gold. The company has only
400 grams of silver and 640 grams of gold. Each unit of type A brings a profit
1000Tsh and each type B brings a profit of 1600Tsh . In which way should a
company produce so as to obtain the maximum profit?
Solution
Let x be ornaments of type A
y be ornaments of type B
A B Max. material
Silver 4 8 400
Gold 8 4 640
Profit 1000 1600

449
Objective function
f x, y   1000x  1600y
Constraints
4 x  8 y  400
8 x  4 y  640
x0
y0
Represent the above constraints graphically

f x, y   1000x  1600y


Corner points 1000x  1600y f x, y 

A80,0 100080  16000 80,000 Tsh

B70,10 100070  160010 102000Tsh


C 0,50 10000  160050 80,000 Tsh

 In order to maximize the profit the company should produce 70 ornaments


of type A and 10 ornaments of type B

450
Example 2
A manufacturing company make two models A and B of a product. Each piece
of model A requires 9 labor hours for fabricating and 1 hour of finishing while
each piece of model B requires 12 labor hours for fabricating and 3 hours of
finishing. For fabricating and finishing the maximum labor hours available are
180 and 30 per week respectively. The company makes a profit of 8000/  on
each piece of model A and 12000/  on each piece of model B . How many
pieces of model A and model B should be manufactured per week to realize
maximum profit? What is maximum profit per week?
Solution
Let x represent piece of model A
Let y represent piece of model B
Model A Model B Max. labor hours
Fabricating 9 12 180
Finishing 1 3 30

Profit 8000/  12000/ 

Constraints
(i ) 9 x  12 y  180
(ii) x  3 y  30
(iii) x  0
(iv) y  0

Objective function, f  x, y   8000x  12000y

Graphing
(i ) 9 x  12 y  180 intercepts 20,0  and 0,15
(ii) x  3 y  30 intercepts 30,0  and 0,10
(iii) x  0, y  0

451
Corner points f x, y   8000x  12000y Value
A0,10 80000  1200010 120000/ 
B12,6 800012  120006 168000/ 
C 20,0 800020  120000 160000/ 
D0,0 80000  120000 0/ 

 In order to realize maximum profit,12 pieces of model A and 6 pieces of model B


should be manufactured per week. The maximum profit per week is 168000/ 

452
Example 3
A calculator company produces scientific and graphic calculators. Long term
prediction indicates an expected demand of at least 100 scientific calculators
and 80 graphic calculators each day. Because of limitations on production
capacity, no more than 200 scientific and 170 graphic calculators can be made
daily. To satisfy a shipping contract, a total of at least 200 calculators must be
shipped each day. If each scientific calculator sold results in 2000/  loss and
each graphic calculator produces 5000/  profit. How many of each type
should be made daily to maximize net profit. What is the maximum profit per
day?
Solution
Let x represent scientificcalculator
Let y represent graphic calculator

Constraints
(i ) x  100
(ii) y  80
(iii) x  200
(iv) y  170
(v) x  y  200
Objective function, f  x, y   2000x  5000y
Graphing
x  y  200 intercepts 200,0 and 0,200

453
Corner points f x, y   2000x  5000y Value
A120,80  2000120  500080 160,000 / 

B100,100  2000100  5000100 300,000 / 

C 100,170  2000100  5000170 650,000 / 

D200,170  2000200  5000170 450,000 / 

E 200,80  2000200  500080 0/ 

 In order to maximize the profit,100 scientificcalculators and 170 graphic calculators


should be made daily. The maximum profit per day is 650,000 / 

Exercise 8.1
1. Monalisa Furniture Company manufactures two types of furniture chairs
and tables. The company has 100m 3 of woods and 90 hours of production
time per week. The company has limited storage capacity to the extent that
the total production cannot exceed 70 items per week and the number of
chairs cannot exceed the number of tables by more than 35 . A chair
requires 2m 3 of wood and 1 hour labour per chair. A table requires 1m 3 of
wood and 2 hours labour per table. The company makes the profit of
Tshs.80000 and Tshs.50000 for each chair and table respectively. How can
the company proceed to maximize profit?

2. Following an illness, a patient is required to take pills containing mineral


and vitamins. The contents and costs of two types of pills, Feelgood and
Getbetter, together with the partients daily requirement are shown in the
following table.
Minerals Vitamins Cost
Feelgood 80mg 4mg 3000
Getbetter 20mg 3mg 1500
Daily requirement 420mg 31mg

If the daily prescription contains x Feelgood pills and y Getbetter pills,


find the cheapest way of prescribing the pills and the cost.

454
3. A store sells two types of toys, A and B. The store owner pays 8 /  and
14 /  for each one unit of toy A and B respectively. One unit of toys A
yields a profit of 2 /  while a unit of toys B yields a profit of 3 /  . The
store owner estimates that no more than 2000 toys will be sold every month
and he does not plan to invest more than 20000/  in inventory of these
toys. How many units of each type of toys should be stocked in order to
maximize his monthly total profit?
4. A company produces two types of tables, T1 and T2. It takes 2 hours to
produce the parts of one unit of T1, 1 hour to assemble and 2 hours to
polish. It takes 4 hours to produce the parts of one unit of T2, 2.5 hour to
assemble and 1.5 hours to polish. Per month, 7000 hours are available for
producing the parts, 4000 hours for assembling the parts and 5500 hours
for polishing the tables. The profit per unit of T1 is 90/= and per unit of T2
is 110/=. How many of each type of tables should be produced in order to
maximize the total monthly profit?
5. A farmer plans to mix two types of food to make a mix of low cost feed for
the animals in his farm. A bag of food A costs 10/= and contains 40 units
of proteins, 20 units of minerals and 10 units of vitamins. A bag of food B
costs 12/= and contains 30 units of proteins, 20 units of minerals and 30
units of vitamins. How many bags of food A and B should the consumed
by the animals each day in order to meet the minimum daily requirements
of 150 units of proteins, 90 units of minerals and 60 units of vitamins at a
minimum cost?
6. John has 20,000/= to invest in three funds F1, F2 and F3. Fund F1 is offers
a return of 2% and has a low risk. Fund F2 offers a return of 4% and has a
medium risk. Fund F3 offers a return of 5% but has a high risk. To be on
the safe side, John invests no more than 3000/= in F3 and at least twice as
much as in F1 than in F2. Assuming that the rates hold till the end of the
year, what amounts should he invest in each fund in order to maximize the
year end return?

455
8.2 Transportation Problems
Transportation is a type of linear programming that deals with shipping of
product from multiple origins to multiple destinations.
Always the objects of transportation problem is to find a possible way of
transporting good to meet the demand of each destination that minimizes the
total transportation costs while satisfying the supply and demand conditions.

Example 4
A manufacturer of a certain product has two stores P1 and P2 . There are 80
units of his product stored in P1 and 70 units in P2 . Two customers Peter and
Haule orders 35 units and 60 units of the product respectively. The supply cost
from the stores to the respective customers is given below
To Peter Haule
From

P1 80/  120/ 
P2 100/  130/ 
How should each of them be supplied in order to minimize the cost?
Solution
Let x be units from P1 to Peter
y be units from P2 to Haule

456
Constraints
x  35.......... .......... .......... .......... .......... .......... .......... ..(i )
y  60.......... .......... .......... .......... .......... .......... .......... .(ii)
x  y  80.......... .......... .......... .......... .......... .......... ......( iii)
35  x   60  y   70
x  y  25.......... .......... .......... .......... .......... .......... .......( iv)
x  0, y  0.......... .......... .......... .......... .......... .......... .....( v)
Objective Function
f  x, y   80x  120 y  10035  x   13060  y 
f  x, y   11300 20x  10 y.......... .......... .......... .......... ...( vi)

f x, y   11300 20x  10 y


Corner points f x, y   11300 20x  10 y

A0,25 11050

B0,60 10700

C 20,60 10300

D35,45 10150

E 35,0 10600

F 25,0 10800

 35 units of P1 and no unit of P2 are supplied to Peter and 45 units of P1 and 15 units
of P2 are supplied to Haule to minimize the cost of 10150/ 

457
Example 5
A oil company has two deposits A and B with capacity of 7000 litres and
4000 litres respectively. The company has to supply oil to three petrol pumps
P, Q and R whose requirements are 4500 , 3000 and 3500 litres respectively.
The distance between the deposits and petrol pumps are given in the table
below;

From A B
To

P 7 3
Q 6 4
R 3 2

How should the delivery be scheduled in order to minimize the transportation


cost?
Solution
Let
x  Litres from deposit A to petrol pump P
y  Litres from deposit A to petrol pump Q
z  Litres from deposit A to petrol pump R

458
Constraints
x  4500.......... .......... .......... .......... .......... .......... .......... .......... .......( i )
y  3000.......... .......... .......... .......... .......... .......... .......... .......... .......( ii)
z  3500.......... .......... .......... .......... .......... .......... .......... .......... .......( iii)
x  0, y  0 and z  0.......... .......... .......... .......... .......... .......... .........( iv)
x  y  z  7000
z  7000  x  y  but z  3500
7000   x  y   3500
x  y  3500.......... .......... .......... .......... .......... .......... .......... .......... .......( iv)
7000   x  y   0
x  y  7000.......... .......... .......... .......... .......... .......... .......... .......... .......... (v)
Objective function
f x, y, z   7 x  6 y  3z  34500  x   43000  y   23500  z 
f x, y, z   4 x  2 y  z  32500
But z  7000  x  y 
f x, y   4 x  2 y  700  x  y   32500
f x, y   3x  y  39500.......... .......... .......... .......... .......... .......... .......... .......... .(vi)
Graph

459
Corner points f x, y   3x  y  39500

A500,3000 44000
B4000,3000 54500
C 4500,2500 55500
D4500,0 53000
E 3500,0 50000

The optimal value is 44000


So the transportation will be;
500 litres from A to P
3000 litres from A to Q
3500 litres from A to R
4000 litres from B to P
No litres from B to Q
No litres from B to R
 The minimum cost is 44000/ 

Exercise 8.2
1. A cement dealer has two deposits D1 and D2 holding 120 and 40 tons
respectively. He has two customers C1 and C2 who have ordered 80 and
50 tons of cement respectively. C1 is 20km from D1 and 40km from D2 .
C2 is 15km from D1 and 30km from D2 . Delivery costs are proportional to
the distance travelled. How should he supply his customers to minimize the
total transport cost?
2. A manufacturer has two warehouses at NYEGEZI which contains 25 units
of his products. He has another warehouse at BAGAMOYO which
contains 30 units. He has to supply to the shop in DODOMA and
SINGIDA with 20 and 15 units respectively. The transport costs per each
units are as follows;
DODOMA SINGIDA
NYEGEZI 45 /  36 / 
BAGAMOYO 42 /  30 / 

460
3. Peter has made 900tons and 600 tons of bricks at his houses H1 and
H 2 respectively. He plans to build new houses at three sites P,Q and R. He
expects to use 500tons of bricks at P, 600 tons of bricks at Q and 400 tons
of bricks at R. The transport cost per ton of bricks from each of his house to
each of three sites are shown below;
To P Q R
From
H1 600 /  300/  400 / 
H2 400 /  200 /  600 / 

(a) Determine the tons of bricks that he will transport to each site at
minimum cost.
(b) What will be the overall minimum cost?

APPLICATIONS OF LINEAR PROGRAMMING

(a) Food and Agriculture


Farmers apply linear programming techniques to their work. By determining
what crops they should grow, the quantity of it and how to use it efficiently,
farmers can increase their revenue.
In nutrition, linear programming provides a powerful tool to aid in planning for
dietary needs. In order to provide healthy, low-cost food baskets for needy
families, nutritionists can use linear programming. Constraints may include
dietary guidelines, nutrient guidance, and cultural acceptability. Mathematical
modeling provides assistance to calculate the foods needed to provide nutrition
at low cost, in order to prevent non-communicable disease. Unprocessed food
data and prices are needed for such calculations, all while respecting the
cultural aspects of the food types. The objective function is the total cost of the
food basket. Linear programming also allows time variations for the frequency
of making such food baskets.

(b) Applications in Engineering


Engineers also use linear programming to help solve design and manufacturing
problems. For example, in airfoil meshes, engineers seek aerodynamic shape
optimization. This allows for the reduction of the drag coefficient of the airfoil.
Constraints may include lift coefficient, relative maximum thickness, nose
radius and trailing edge angle. Shape optimization seeks to make a shock-free

461
airfoil with a feasible shape. Linear programming therefore provides engineers
with an essential tool in shape optimization.

(c) Transportation Optimization


Transportation systems rely upon linear programming for cost and time
efficiency. Bus and train routes must factor in scheduling, travel time and
passengers. Airlines use linear programming to optimize their profits according
to different seat prices and customer demand. Airlines also use linear
programming for pilot scheduling and routes. Optimization via linear
programming increases airlines' efficiency and decreases expenses.

(d) Efficient Manufacturing


Manufacturing requires transforming raw materials into products that maximize
company revenue. Each step of the manufacturing process must work
efficiently to reach that goal. For example, raw materials must pass through
various machines for certain amounts of time in an assembly line. To maximize
profit, a company can use a linear expression of how much raw material to use.
Constraints include the time spent on each machine. Any machines creating
bottlenecks must be addressed. The amount of products made may be affected,
in order to maximize profit based on the raw materials and the time needed.

(e) Energy Industry


Modern energy grid systems incorporate not only traditional electrical systems,
but also renewables such as wind and solar photovoltaic. In order to optimize
the electric load requirements, generators, transmission and distribution lines,
and storage must be taken into account. At the same time, costs must remain
sustainable for profits. Linear programming provides a method to optimize the
electric power system design. It allows for matching the electric load in the
shortest total distance between generation of the electricity and its demand over
time. Linear programming can be used to optimize load-matching or to
optimize cost, providing a valuable tool to the energy industry.

(f) Management

1. Production Management:

LP is applied for determining the optimal allocation of such resources as


materials, machines, manpower, etc. by a firm. It is used to determine the
optimal product- mix of the firm to maximize its revenue. It is also used for
product smoothing and assembly line balancing.
462
2. Personnel Management:

LP technique enables the personnel manager to solve problems relating to


recruitment, selection, training, and deployment of manpower to different
departments of the firm. It is also used to determine the minimum number
of employees required in various shifts to meet production schedule within
a time schedule.

3. Inventory Management:

A firm is faced with the problem of inventory management of raw


materials and finished products. The objective function in inventory
management is to minimize inventory cost and the constraints are space
and demand for the product. LP technique is used to solve this problem.

4. Marketing Management:

LP technique enables the marketing manager in analyzing the audience


coverage of advertising based on the available media, given the advertising
budget as the constraint. It also helps the sales executive of a firm in
finding the shortest route for his tour. With its use, the marketing manager
determines the optimal distribution schedule for transporting the product
from different warehouses to various market locations in such a manner
that the total transport cost is the minimum.

5. Financial Management:

The financial manager of a firm, mutual fund, insurance company, bank,


etc. uses the LP technique for the selection of investment portfolio of
shares, bonds, etc. so as to maximize return on investment.

6. Blending Problem:

LP technique is also applicable to blending problem when a final product is


produced by mixing a variety of raw materials. The blending problems
arise in animal feed, diet problems, petroleum products, chemical products,
etc. In all such cases, with raw materials and other inputs as constraints, the
objective function is to minimize the cost of final product.

463
Chapter Nine
DIFFERENTIATION
Introduction

Differentiation is the process of finding out the slope or gradient of continuous


functions.

dy
The slope of the function is denoted by or f ( x)
dx

SLOPE OF THE CURVE

Consider the curve below

fig 9. 1

change of y
Slope 
change of x
y  y  y
f ( x) 
x  x  x
y
f ( x) 
x
y
Slope of any curve is given by f ( x ) 
x

464
9.1 DERIVATIVE BY FIRST PRINCIPLE
Derivative by first principle is the process of computing slope of any function
by using definition where by small change of independent variable is
approaching to zero.
Consider the sketch below which represents the statements above

fig 9. 2

From the figure above as x  0, point A approches to point B and a curve (or arc AB ) tends
to line AB. The triangle ABC is obtained after magnification of small triangle which we can' t
see by normal eyes since AB  0 likewise CB  0.
change of y y
Slope  
change of x x
lim f ( x  x)  f ( x)
f ( x) 
x  0 x  x  x
lim f ( x  x)  f ( x)
f ( x) 
x  0 x
For simplicity let x  h
lim f ( x  h)  f ( x )
f ( x) 
h0 h

lim f ( x  x)  f ( x)
f ( x) 
x  0 x

lim f ( x  h)  f ( x)
f ( x) 
h0 h

465
Example 1
By using first principle differentiate f ( x)  x
Solution

f ( x)  x
f ( x  h)  x  h
lim f x  h   f x 
From f  x  
h0 h
lim x  h  x
f x  
h0 h
lim h
f x  
h0h
 f  x   1

Example 2
By using definition differentiate f ( x)  2 x
Solution
f ( x)  2 x
f ( x  h)  2 x  h 
lim f  x  h   f  x 
From f  x  
h0 h
lim 2 x  h   2 x
f  x  
h0 h
lim 2 x  2h  2 x
f  x  
h0 h
lim 2h
f  x  
h0 h
 f  x   2

466
Example 3
By using definition differentiate f ( x)  3x  5
Solution
f ( x)  3 x  5
f ( x  h)  3x  h   5
lim f  x  h   f  x 
From f  x  
h0 h
lim 3x  h   5  3x  5
f  x  
h0 h
lim 3x  3h  5  3 x  5
f  x  
h0 h
lim 3h
f x  
h0 h
 f  x   3

Example 4
By using first principle differentiate f ( x)  x 2
Solution
f ( x)  x 2
f ( x  h)   x  h 
2

lim f x  h   f  x 
From f x  
h0 h
lim  x  h 2  x 2
f x  
h0 h
lim x 2  2hx  h 2  x 2
f x  
h0 h
lim 2hx  h 2
f x  
h0 h
lim
f x   2 x  h 
h0
 f x   2 x

467
Example 5
By using first principle differentiate f ( x)  2 x 3
Solution
f ( x)  2 x 3
f ( x  h)  2 x  h 
3

lim f  x  h   f  x 
From f  x  
h0 h
lim 2 x  h 3  2 x 3
f  x  
h0 h
lim 2 x 3  6hx 2  6h 2 x  2h 3  2 x 3
f  x  
h0 h
lim 6hx 2  6h 2 x  2h 3
f  x  
h0 h

f  x  
lim
h0

6 x 2  6hx  2h 2 
 f  x   6 x 2

Example 6
By using definition differentiate f ( x)  x 2  x
Solution
f ( x)  x 2  x
f ( x  h)   x  h    x  h 
2

lim f  x  h   f x 
From f  x  
h0 h

f  x  
 
lim  x  h 2   x  h   x 2  x 
h0 h

f  x  
  
lim x 2  2hx  h 2  x  h  x 2  x 
h0 h
lim 2hx  h 2  h
f  x  
h0 h
lim
f  x   2 x  h  1
h0
 f x   2 x  1

468
Example 7
By using first principle differentiate f ( x)  x 2 x  1
Solution
f ( x)  x 2  x  1  x 3  x 2
f ( x  h)   x  h    x  h 
3 2

lim f  x  h   f  x 
From f  x  
h0 h

f  x  
 
lim  x  h 3   x  h 2  x 3  x 2 
h0 h

f  x  
  
lim x 3  3hx 2  3h 2 x  h 3  x 2  2hx  h 2  x 3  x 2 
h0 h
lim 3hx 2  3h 2 x  h 3  2hx  h 2
f  x  
h0 h

f  x  
lim
h0

3 x 2  3hx  h 2  2 x  h 
 f  x   3 x 2  2 x

Example 8
1
By using first principle differentiate f ( x) 
x
Solution
1
f ( x) 
x
1
f ( x  h) 
xh
lim f  x  h   f  x 
From f  x  
h0 h
1 1
lim x  h  x
f  x  
h0 h
lim x   x  h 
f  x  
h  0 hx x  h 
lim h
f  x  
h  0 hx x  h 
lim 1
f  x  
h  0 x  hx
2

 f  x    2
1
x

469
Example 9
1
By using first principle differentiate f ( x ) 
x2
Solution
1
f ( x)  2
x
1
f ( x  h) 
x  h 2
lim f  x  h   f  x 
From f  x  
h0 h
1 1

lim  x  h 2 x 2
f x  

h0 h
lim x 2  x  h 2
f  x  
h  0 hx2  x  h 2

f  x  

lim x 2  x 2  2 xh  h 2 
h0 hx2 x  h 
2

lim  2 xh  h 2
f  x  
h  0 hx2  x  h 2
lim  2 x  h
f x  
h  0 x 2 x  h 2
lim  2 x
f  x  
h  0 x4

 f x    3
2
x

Example 10
1
By using first principle differentiate f ( x) 
x2
Solution

470
1
f ( x) 
x2
1
f ( x  h) 
xh2
lim f  x  h   f  x 
From f  x  
h0 h
1 1

lim x  h  2 x  2
f  x  
h0 h
lim  x  2    x  h  2 
f  x  
h  0 h x  2  x  h  2 
lim h
f  x  
h  0 h x  2  x  h  2 
lim 1
f  x  
h  0  x  2  x  h  2 
1
f  x  
x  2x  2
 f  x   
1
x  22

Example 11
By using first principle differentiate f ( x)  x
Solution
f ( x)  x
f ( x  h)  x  h
lim f x  h   f x 
From f x  
h0 h
lim x  h  x
f x  
h0 h

471
Rationalize the numerator
 x  h  x  x  h  x 
lim
f  x   

 
 x  h  x 
h0 h  
lim xhx
f  x  
h0 h xh  x  
lim
f  x  
h
h0 h xh  x  
lim
f  x  
1
h0 xh  x

f  x  
1 1

x x 2 x
 f  x  
1
2 x

Example 12
By using first principle differentiate f ( x)  x  7
Solution
f x   x7
f x  h   x  h   7
From first Principle
lim f  x  h   f  x 
f  x  
h0 h
lim  x  h   7  x  7
f  x  
h0 h
Rationalize the numerator
lim  x  h   7  x7 x  h   7  x7 
f  x     
h  0  h x  h   7  x  7 

f  x  
lim  x  h   7  ( x  7)
h0  x  h   7  x  7 h
lim
f  x  
h 1

h0  
x  h   7  x  7 h 2 x  7
 f  x  
1
2 x7

472
Example 13
1
By using first principle differentiate f ( x) 
x
Solution
1
f ( x) 
x
1
f ( x  h) 
xh
lim f  x  h   f  x 
From f  x  
h0 h
1 1

lim x  h
f  x   x
h0 h
lim x  x  h
f  x  
h0h x xh  
Rationalize the numerator
lim  x  x  h  x  x  h 
f  x     
h0 
 h x x  h   x  x  h 


lim  x  x  h 

f  x    
h0 
  

h x x  h  x  x  h
 
lim  h 
f  x    
 
h  0 h x x  h  x  x  h 

 
lim  1 
f  x    
h0 
  
x  hx x  x  h 
2

 1 
f  x    
 
x x  x 
 
 f  x   
1
2x x

473
Example 14
By using first principle differentiate f ( x)  x n
Solution

Consider the powered function below


f ( x)  x n
f ( x  h)   x  h 
n

lim f ( x  h)  f ( x )
From first principle f ( x) 
h0 h
lim ( x  h) n  x n
f ( x)  .......... .......... .......... .......... ..(i )
h0 h
n
But x  h    x nr h r
n

r 0

nx n1h n(n  1) x n2 h 2


x  h   x 
n n
  .......... .......... ........( ii)
1! 2!
Substitute equation (ii) into equation (i)
 n nx n1h n(n  1) x n2 h 2 
 x    ...   x n
f ( x) 
lim  1! 2! 
h0 h
n(n  1) x n2 h 2
n 1
lim nx h   ...
f ( x)  2!
h0 h
lim n(n  1) x n2 h
f ( x)  n 1
nx   ...
h0 2!
f ( x)  nx n1

474
Example 15
By using definition differentiate f ( x)  e
x

Solution
f ( x)  e x
f ( x  h)  e x  h
lim f  x  h   f  x 
From f  x  
h0 h
lim e x  h  e x
f  x  
h0 h
lim e x e h  e x
f  x  
h0 h

f  x  
lim e x e h  1  h 2 h3
but e  1  h      
h

h0 h 2! 3!

 h 2 h3  

e 1  h 
x
       1
f x  
lim  2! 3!  
h0 h
 h 2 h3 
e  h x
     
f x  
lim  2! 3! 
h0 h
lim x  h h 2 
f x   e 1       
h  0  2! 3! 
 f x   e x

475
Example 16
x
By using definition differentiate f ( x)  e
Solution
f ( x)  e  x
f ( x  h)  e  x  h 
lim f x  h   f x 
From f x  
h0 h
lim e  x  h   e  x
f  x  
h0 h
lim e  x e h  e  x
f  x  
h0 h

f  x  

lim e  x e h  1  h 2 h3
but e h  1  h      
h0 h 2! 3!
 h 2 h3  
e 1  h         1
x

f  x  
lim  2! 3!  
h0 h
 h 2 h3 
e   h       
 x

f  x  
lim  2! 3! 
h0 h
lim  x  h h2 
f  x   e   1       
h0  2! 3! 
 f x   e x

476
Example 17
By using definition find the derivative of f ( x )  Inx
Solution
f ( x)  Inx
f ( x  h)  In  x  h 
lim f x  h   f x 
From f x  
h0 h
lim In  x  h   Inx
f x  
h0 h
 xh  h
In  In1  
f x  
lim  x   lim  x  let z  h
h0 h h0 h x
limIn 1  z  z2 z3 z4
f x   but In 1  z   z       
h0 h 2 3 4
z2 z3 z4
lim z  2  3  4    
f x  
h
but z 
h0 h x
h h2 h3 h4
lim x 2 x 2 3x 3 4 x 4    
  
f x  
h0 h
lim 1 h2 h3
f x  
h
    
h  0 x 2 x 2 3x 3 4 x 4

 f  x  
1
x

Example 18
By using definition find the derivative of f ( x)  Inx
2

Solution

477
lim 2 In 1  z  z2 z3 z4
f  x   but In 1  z   z       
h 0 h 2 3 4
 z2 z3 z4 
2 z        
f  x  
lim  2 3 4  but z 
h
h 0 h x
 h h2 h3 h4 
2  2  3  4     
f  x  
lim  x 2x 3x 4 x 
h 0 h
lim 1 h2 h3 
f  x  
h
2  2  3  4     

h  0  x 2x 3x 4 x 
 f x  
2
x

Exercise 9.1
1. Differentiate the following by using first principle;
(a) f x   Inx 3
(b) f x   e 2 x
(c) f x   e x  x
(d) f x  x  x
2. Differentiate the following by using first principle;
2x  x2
(a) f x  
x
x 1
(b) f  x  
x  x2
(c) f  x   x 2  1
(d) f x   x 2 x  1

478
9.2 POWER RULE
Power Rule is the principle used to differentiate functions raised to power n
Power rule is given by

d n
dx
x   nx n1

Proof
The derivation of Power Rule is obtained from Binomial Expansion, consider
n
the binomial expansion a  b  a
n nr
br
r 0

Consider the powered function below


f ( x)  x n
f ( x  h)   x  h 
n

lim f ( x  h)  f ( x )
From first principle f ( x) 
h0 h
lim ( x  h) n  x n
f ( x)  .......... .......... .......... .......... ..(i )
h0 h
n
But x  h    x nr h r
n

r 0

nx n1h n(n  1) x n2 h 2


x  h   x 
n n
  .......... .......... ........( ii)
1! 2!
Substitute equation (ii) into equation (i)
 n nx n1h n(n  1) x n2 h 2 
 x    ...   x n
f ( x) 
lim  1! 2! 
h0 h
n(n  1) x n2 h 2
n 1
lim nx h   ...
f ( x)  2!
h0 h
lim n(n  1) x n2 h
f ( x)  n 1
nx   ...
h0 2!
f ( x)  nx n1

Power Rule
d n
dx
 
x  nxn 1

479
Example 19
Differentiate the function f ( x)  x 5
Solution
f x   x 5
 f x   5x 4

Example 20
Find the derivative of f ( x)  3 x 4  x 2
Solution
y  3x 4  x 2
dy
  12 x 3  2 x
dx

Example 21
Differentiate the function y  x m
Solution
y  xm
dy
  mx m1
dx

Example 22
Find the derivative of f ( x)  m1 x m
Solution
y  m1 x m
dy
  x m1
dx

Example 23
Find the derivative of f ( x)  n11 x n
Solution
y  n11 x n
dy n n1
  x
dx n1

480
Example 24
Find the derivative of f ( x)  k  1x
k 1

Solution
y  k  1x k 1


dy
dx
 
 k 2 1 x k

Example 25
2 m1
Find the derivative of f ( x)  x
Solution
y  x 2 m1

 2m  1x 2 m2


dy

dx

9.3 PRODUCT RULE


Product Rule is the rule used to find the derivative of two or more functions
which are expressed in product form.
Consider two functions u and v expressed in product as f ( x)  uv
lim f ( x  x)  f ( x)
By using First Principle f ( x) 
x  0 x
f ( x)  uv
f ( x  x)  u  u v  v 
lim f ( x  x)  f ( x)
f ( x) 
x  0 x

f ( x) 
lim u  u v  v   uv
x  0 x
lim uv  uv  vu  uv  uv
f ( x) 
x  0 x
lim uv  vu  uv
f ( x)  but uv  0
x  0 x
lim uv  vu
f ( x) 
x  0 x
lim v u
f ( x)  u v
x  0 x x
dv du
f ( x)  u v
dx dx
Product Rule
d
uv  u dv  v du
dx dx dx

481
Example 26
Find the derivative of f ( x)  x  12 x  1
Solution
y   x  12 x  1
du
Let u  x  1, 1
dx
dv
v  2 x  1, 2
dx
From,
d
uv  v du  u dv
dx dx dx
 2 x  11   x  12
dy
dx
dy
  4x 1
dx

Example 27
Find the derivative of y  x n x 2 
Solution
 
y  xn x2
du
Let u  x n ,  nx n1
dx
dv
v  x2 ,  2x
dx
From,
d
uv  v du  u dv
dx dx dx
dy
dx
 
 x 2 nx n1  x n 2 x 

dy
 nx n1  2 x n1
dx
  n  2x n1
dy
dx

482
Example 28

Find the derivative of y  x m x n  1 
Solution

y  xm xn 1 
du
Let u  x m ,  mx m1
dx
dv
v  x n  1,  nx n1
dx
From,
d
uv  v du  u dv
dx dx dx
dy
dx
   
 x n  1 mx m1  x m nx n1 
dy
 mx m n1  mx m1  nx m n1
dx
  mx m1  m  n x m n1
dy
dx

Alternative

y  xm xn  1 
y  xm xn  xm
y  x m n  x m

 m  n x m n1  mx m1
dy

dx

9.4 QUOTIENT RULE


u
Quotient Rule of functions of the form of f ( x)  where u and v are functions
v
of independent variable.

483
u
Consider two functions u and v expressed in quotient as f ( x) 
v
lim f ( x  x)  f ( x)
By using First Principle f ( x) 
x  0 x
u
f ( x) 
v
u  u
f ( x  x) 
v  v
lim f ( x  x)  f ( x)
f ( x) 
x  0 x
u  u u
lim 
f ( x)  v   v v
x  0 x
lim vu  u   u v  v 
f ( x) 
x  0 vxv  v 
lim uv  vu  uv  uv
f ( x) 
x  0 vv  v x
lim vu  uv
f ( x) 
x  0 vv  v x
u v
lim v  u
f ( x)  x x
x  0 vv  v 
u du v dv
As x  0,  ,  and v  0
x dx x dx
du dv
v u
f ( x)  dx dx
vv 
du dv
v u
f ( x)  dx 2 dx
v

Quotient Rule du dv
v u
d u dx dx
 
dx  v  v 2

484
Example 29
x
Find the derivative of the function f ( x ) 
x 3
Solution
x
y
x 3
du
Let u  x, 1
dx
dv
v  x  3, 1
dx
du dv
u
d u
v
x  3  x
   dx 2 dx 
dx  v  v x  32
dy 3
 
dx x  32

Example 30

 f 
If f 3  4, g 3  2, f 3  3 and g 3  4 . Find the value of 
 g 
 
Solution

 f  gf   fg 
From quotient rule   
g g2

 f  gf   fg  2(3)  4(4) 14
      3.5
g g2 22 4

f
    3.5
g

485
9.5 CHAIN RULE
Chain Rule is a rule used to find the derivative of two or more functions in
which one function is within another function or functions raised to power n , i.e.

f u (x)  or u( x)n


The derivative of f u ( x)  can be obtained as follows,
d
 y   dy . du where y  f u ( x)
dx du dx
dy dy du
 .
dx du dx
dy dy du
Chain Rule  .
dx du dx

Example 31
Find the derivative of the function y  x  4x
2
  3

Solution
 
3
y  x 2  4 x .......... .......... .......... .......... ........( i )
du
Let u  x 2  4 x,  2 x  4.......... .......... .....( ii)
dx
Substitute equation (ii) into equation (i)
dy
y  u 3 ,  3u 2 .......... .......... .......... .......... ....( iii)
du
dy dy du
From,  
dx du dx
dy
dx
 
 3u 2  2 x  4 but u  x 2  4 x

dy
dx
   
 3 x 2  4 x 2 x  4  3 x 4  24x 3  48x 2 x 2  4 x
2

dy
 
  3 x 4  24x 3  48x 2 x 2  4 x
dx

486
Example 32
Find the derivative of the function y  x  510
Solution
y   x  5 .......... .......... .......... .......... .......... .....( i )
10

du
Let u  x  5,  1.......... .......... .......... ........( ii)
dx
Substitute equation (ii) into equation (i)
dy
y  u10 ,  10u 9 .......... .......... .......... .......... ....( iii)
du
dy dy du
From,  
dx du dx
dy
dx
 
 10u 9 1 but u  x  5

  10x  5
dy 9

dx

Example 33
Find the derivative of the function y  x  3
4

Solution
 1
y  x 4  3  x 4  3 2 .......... .......... .......... .......... ........( i )
du
Let u  x 4  3,  4 x 3 .......... .......... .......... .......... ....( ii)
dx
dy 1  12 1
y  u 2,  2u 
1
.......... .......... .......... .......... .(iii)
du 2 u
dy dy du
From,  
dx du dx
dy  1 

dx  2 u 
 
  4 x 3 but u  x 4  3

dy 2x3
 
dx x4  3

487
Example 34
Find the derivative of the function y  3 x  10
Solution
y  3 x  10   x  10 3 .......... .......... .......... .......... .........( i )
1

du
Let u  x  10,  1.......... .......... .......... .......... .........( ii)
dx
dy 1  2 3 1
y  u 3,  3u 
1

 
2
.......... .......... .......... ........( iii)
du 33 u
dy dy du
From,  
dx du dx
dy  1 
  1 but u  x  10
  
dx  3 3 u 2 

dy 1
 

dx 3 3 x  10 2 
Example 35
1
Find the derivative of the function y 
2 x  56
Solution
1
y .......... .......... .......... .......... .......... .......... ..(i )
2 x  56
du
Let u  2 x  5,  2.......... .......... .......... .......... .....( ii)
dx
1 dy
y  6  u 6 ,  6u 7 .......... .......... .......... .......... ..(iii)
u du
dy dy du
From,  
dx du dx
dy
dx
 
  6u 7  2  12u 7 but u  2 x  5

 122 x  5
dy 7

dx
dy  12
 
dx 2 x  57

488
Example 36
1
Find the derivative of the function y  5
8x 1
Solution
1
y5 .......... .......... .......... .......... .......... .......... ..(i )
8x 1
du
Let u  8 x  1,  8.......... .......... .......... .......... .....( ii)
dx
1 1 dy
y  5  5  u 2 ,   52 u  2 .......... .......... .......( iii)
5 7

u u 2
du
dy dy du
From,  
dx du dx
dy
dx
 7

  52 u  2  8  20u  2 but u  8 x  1
7

  208 x  1 2
dy 7

dx

9.6 DERIVATIVE OF TRIGONOMETRIC FUNCTIONS

(a) Derivative of sine (sin)


Consider a function f ( x)  sin x
lim f ( x  h)  f ( x )
From first principle f ( x) 
h0 h
f ( x)  sin x and f ( x  h)  sin( x  h)
lim sin( x  h)  sin x
f ( x)  (Expand the compound angle)
h0 h
lim sin x cosh cos x sinh  sin x
f ( x) 
h0 h
lim sin x  h cos x  sin x
f ( x) 
h0 h
lim sin x  h cos x  sin x
f ( x) 
h0 h
lim h cos x
f ( x) 
h0 h
f ( x)  cos x
d
sin x   cos x
dx

489
Alternative Derivation
Consider a function f ( x)  sin x
lim f ( x  h)  f ( x )
From first principle f ( x) 
h0 h
f ( x)  sin x and f ( x  h)  sin( x  h)
lim sin( x  h)  sin x
f ( x)  (Apply factor formula)
h0 h
h  h
2  cos x  
lim 2  2  h
f ( x)  but cos x    cos x
h0 h  2
lim h cos x
f ( x) 
h0 h
f ( x)  cos x
d
sin x   cos x
dx

Example 37
Find the derivative of f ( x)  sin 4 x
Solution
y  sin 4 x.......... .......... .......... .......... .......... .....( i )
du
Let u  4 x,  4.......... .......... .......... .........( ii)
dx
dy
y  sin u ,  cosu.......... .......... .......... .........( iii)
du
dy dy du
From chain rule,  
dx du dx
 cosu 4   4 cosu but u  4 x
dy
dx
dy
  4 cos 4 x
dx

490
Example 38
Find the derivative of f ( x)  sin x 3
Solution
y  sin x 3 .......... .......... .......... .......... .......... .....( i )
du
Let u  x 3 ,  3 x 2 .......... .......... .......... .........( ii)
dx
dy
y  sin u ,  cosu.......... .......... .......... .........( iii)
du
dy dy du
From chain rule,  
dx du dx
dy
dx
 
 cosu  3 x 2  4 cosu but u  x 3

dy
  3 x 2 cos x 3
dx

Example 39
Find the derivative of f ( x)  2 sin x
Solution
y  2 sin x .......... .......... .......... .......... .......... ......( i )
du 1
Let u  x ,  .......... .......... .......... ......( ii)
dx 2 x
dy
y  2 sin u,  2 cos u.......... .......... .......... .........( iii)
du
dy dy du
From chain rule,  
dx du dx
 1  cosu
 2 cosu 
dy
 but u  x
dx 2 x  x
dy cos x
 
dx x

491
Example 40
Find the derivative of f ( x)  sinx 4  2 x  3
Solution
 
y  sin x 4  2 x  3 .......... .......... .......... .......... .......... .......... .(i )
du
Let u  x 4  2 x  3,  4 x 3  2.......... .......... .......... .........( ii)
dx
dy
y  sin u,  cosu.......... .......... .......... .......... .......... ........( iii)
du
dy dy du
From chain rule,  
dx du dx
dy
dx
   
 cosu  4 x 3  2  4 x 3  2 cosu but u  x 4  2 x  3

dy
  
  4 x 3  2 cos x 4  2 x  3
dx

Example 41
Find the derivative of f ( x)  x sin x
2

Solution
y  x 2 sin x.......... .......... .......... .......... .......... .....( i )
du
Let u  x 2 ,  2 x.......... .......... .......... .........( ii)
dx
dv
v  sin x,  cos x.......... .......... .......... .......... .(iii)
dx
dy dv du
From product rule, u v
dx dx dx
 x 2 cos x  sin x 2 x 
dy
dx
dy
  x 2 cos x  2 x sin x
dx

492
Example 42
Find the derivative of f ( x)  x sin x
Solution
y x sin x .......... .......... .......... .......... .......... .......... ....( i )
du 1
Let u  x ,  .......... .......... .......... .......... ......( ii)
dx 2 x
dv 1
v  sin x ,  cos x .......... .......... .......... .........( iii)
dx 2 x
dy dv du
From product rule, u v
dx dx dx
dy
 x
 1 
cos x   sin x 
 1 
  
dx 2 x  2 x 
dy cos x sin x
  
dx 2 2 x

Example 43
sin x
Find the derivative of f ( x ) 
x
Solution
sin x
y .......... .......... .......... .......... .......... .......( i )
x
du
Let u  sin x,  cos x.......... .......... .......... ..(ii)
dx
dv
v  x,  1.......... .......... .......... .......... .......... .(iii)
dx
du dv
v u
dy
From Quotient rule,  dx 2 dx
dx v
dy x cos x  sin x
 
dx x2

Example 44
x2
Find the derivative of f ( x) 
sin x
Solution

493
x2
y .......... .......... .......... .......... .......... .........( i )
sin x
du
Let u  x 2 ,  2 x.......... .......... .......... .......... (ii)
dx
dv
v  sin x,  cos x.......... .......... .......... .......... ..(iii)
dx
du dv
v u
dy
From Quotient rule,  dx 2 dx
dx v
dy 2 x sin x  x cos x 2
 
dx sin 2 x

Example 45
By using first principle differentiate f ( x)  x sin x
Solution
f ( x)  x sin x
f ( x  h)  x  h sinx  h 
lim f x  h   f  x 
From f  x  
h0 h
lim x  h sinx  h   x sin x
f  x  
h0 h
lim x  h sin x cosh cos x sinh   x sin x
f  x  
h0 h
lim x  h sin x  h cos x   x sin x
f  x  
h0 h
lim x sin x  hx cos x  h sin x  h 2 cos x  x sin x
f  x  
h0 h
lim hx cos x  h sin x  h 2 cos x
f  x  
h0 h
lim
f  x   x cos x  sin x  h cos x
h0
 f  x   x cos x  sin x

494
Example 46
By using definition differentiate f ( x)  sinx  2
Solution
f ( x)  sin x  2 
f ( x  h)  sin x  2  h   sin x  2   h
lim f  x  h   f  x 
From f  x  
h0 h
lim sin x  2   h  sin x  2 
f  x  
h0 h
lim sin x  2 cosh cos x  2 sinh  sin x  2 
f  x  
h0 h
lim sin x  2   h cos x  2   sin x  2 
f  x  
h0 h
lim h cos x  2 
f  x  
h0 h
 f  x   cos x  2 

(b) Derivative of cosine (cos)


Consider a function f ( x)  cos x
lim f ( x  h)  f ( x )
From first principle f ( x) 
h0 h
f ( x)  cos x and f ( x  h)  cos(x  h)
lim cos(x  h)  cos x
f ( x)  (Expand the compound angle)
h0 h
lim cos x cosh sin x sinh  cos x
f ( x) 
h0 h
lim cos x  h sin x  cos x
f ( x) 
h0 h
lim  h sin x
f ( x) 
h0 h
f ( x)   sin x
d
cos x    sin x
dx

495
Alternative Derivation
Consider a function f ( x)  cos x
lim f ( x  h)  f ( x )
From first principle f ( x) 
h0 h
f ( x)  cos x and f ( x  h)  cos(x  h)
lim cos(x  h)  cos x
f ( x)  (By factor formula)
h0 h
 h h
 2 sin x   sin 
f ( x) 
lim  2 2
h0 h
h  h
 2  sin x  
lim 2  2  h
f ( x)  but sin x    sin x
h0 h  2
lim  h sin x
f ( x) 
h0 h
f ( x)   sin x

d
cos x    sin x
dx

Example 47
Find the derivative of the function f ( x)  cos2 x  9
Solution
y  cos2 x  9 .......... .......... .......... .......... .......... .....( i )
du
Let u  2 x  9,  2.......... .......... .......... .......... .(ii)
dx
dy
y  cosu ,   sin u.......... .......... .......... .......... ...( iii)
du
dy dy du
From chain rule,  
dx du dx
dy
 2 sin u but u  2 x  9
dx
  2 sin2 x  9 
dy
dx
496
Example 48
Find the derivative of the function f ( x)  5 cos x 5
Solution

y  5 cos x 5 .......... .......... .......... .......... .......... .......... (i )


du
Let u  x 5 ,  5 x 4 .......... .......... .......... .......... ....( ii)
dx
dy
y  5 cosu,  5 sin u.......... .......... .......... .......... (iii)
du
dy dy du
From chain rule  
dx du dx
dy
dx
 
  5 sin u  5 x 4  25x 4 sin x 5

dy
  25x 4 sin x 5
dx

Example 49
Find the derivative of f ( x)  x 2 cos x 3
Solution
y  x 2 cos x 3 .......... .......... .......... .......... .......... .......... .......( i )
du
Let u  x 2 ,  2 x.......... .......... .......... .......... .......... ...(ii)
dx
dv
v  cos x 3 ,  3x 2 sin x 3 .......... .......... .......... .......... .....( iii)
dx
dy dv du
From product rule, u v
dx dx dx
dy
dx
   
 x 2  3 x 2 sin x 3  cos x 3 2 x   3 x 4 sin x 3  2 x cos x 3

dy
  2 x cos x 3  3 x 4 sin x 3
dx

497
Example 50
Find the derivative of f ( x)  sin 2 x cos3x
Solution
y  sin 2 x cos3x.......... .......... .......... .......... .......... .......... ..(i )
du
Let u  sin 2 x,  2 cos 2 x.......... .......... .......... .......... .(ii)
dx
dv
v  cos3x,  3 sin 3x.......... .......... .......... .......... ......( iii)
dx
dy dv du
From product rule, u v
dx dx dx
 sin 2 x 3 sin 3 x   cos3 x2 cos 2 x 
dy
dx
dy
  2 cos3x cos 2 x  3 sin 3x sin 2 x
dx

Example 51
cos x
Find the derivative of f ( x ) 
3x  4
Solution
cos x
y .......... .......... .......... .......... .......... .......( i )
3x  4
du
Let u  cos x,   sin x.......... .......... .......... ..(ii)
dx
dv
v  3x  4,  3.......... .......... .......... .......... .....( iii)
dx
du dv
v u
dy
From Quotient rule,  dx 2 dx
dx v
dy 3x  4 sin x   3 cos x

dx 3x  42
dy 4  3x sin x  3 cos x
 
dx 3x  42

498
Example 52
By using definition find the slope of f ( x)  cos6 x
Solution
f ( x)  cos6 x
f ( x  h)  cos6 x  h   cos6 x  6h 
lim f  x  h   f  x 
From first principle f  x  
h0 h
lim cos6 x  6h   cos6 x
f  x  
h0 h
lim cos6 x cosh sin 6 x sin 6h  cos 6 x
f  x  
h0 h
lim cos6 x  6h sin 6 x  cos6 x
f  x  
h0 h
lim  6h sin 6 x
f  x  
h0 h
lim
f  x    6 sin 6 x 
h0
 f  x   6 sin 6 x

Example 53
Find the derivative of y  cosx 4
Solution
y  cos x  .......... .......... .......... .......... .......... .......... .......... .........( i)
4

du
Let u  cos x,   sin x.......... .......... .......... .......... .......... .......( ii)
dx
dy
y  u4,  4u 3 .......... .......... .......... .......... .......... .......... .......... (iii)
du
dy dy du
From chain rule ,  
dx du dx
 4u 3  sin x  but u  cos x
dy
dx
 4cos x  sin x
dy 3

dx
dy
  4 cos3 x sin x
dx

499
Example 54
Find the derivative of y  cos10 x
Solution
y  cos10 x  cos x  .......... .......... .......... .......... .......... .......... ......( i )
10

du
Let u  cos x,   sin x.......... .......... .......... .......... .......... .......( ii)
dx
dy
y  u 10 ,  10u 9 .......... .......... .......... .......... .......... .......... ........( iii)
du
dy dy du
From chain rule ,  
dx du dx
 10u 9  sin x  but u  cos x
dy
dx
 10cos x  sin x
dy 9

dx
dy
  10 cos9 x sin x
dx
Example 55
Find the derivative of y  cos7 x
Solution
y  cos 7 x  cos 7 x  2 .......... .......... .......... .......... .......... .......... .....( i )
1

du
Let u  cos 7 x,  7 sin 7 x.......... .......... .......... .......... .......... .....( ii)
dx
dy 1  12 1
y  u 2,  u 
1
.......... .......... .......... .......... .......... .........( iii)
du 2 2 u
dy dy du
From chain rule ,  
dx du dx
  7 sin 7 x  but u  cos7 x
dy 1

dx 2 u
dy  7 sin 7 x
 
dx 2 cos7 x

Example 56
Find the derivative of f ( x)  cosm x
Solution
y  cosm x
y  cos x 
m

 mcos x  sin x
dy m 1

dx
dy
  m cosm1 x sin x
dx
500
(c) Derivative of tangent (tan)
Consider a function f ( x)  tan x
lim f ( x  h)  f ( x )
From first principle f ( x) 
h0 h
f ( x)  tan x and f ( x  h)  tan( x  h)
lim tan( x  h)  tan x
f ( x)  (Expand the compound angle)
h0 h
tan x  tanh
lim  tan x
f ( x)  1  tan x tanh
h0 h
lim tan x  tanh  tan x1  tan x tanh 
f ( x) 
h0 h1  tan x tanh 
lim tan x  tanh  tan x  tan 2 x tanh
f ( x) 
h0 h1  tan x tanh 
lim tanh  tan 2 x tanh
f ( x)  but tanh  h
h  0 h1  tan x tanh 
lim h  h tan 2 x

f ( x) 
h  0 h1  h tan x 
lim 1  tan 2 x
f ( x) 
h  0 1  h tan x 
f ( x)  1  tan 2 x  sec2 x
d
tan x   sec2 x
dx

501
Alternative Derivation
Consider a function f ( x)  tan x
lim f ( x  h)  f ( x )
From first principle f ( x) 
h0 h
f ( x)  tan x and f ( x  h)  tan( x  h)
sin( x  h) sin x

lim cos(x  h) cos x
f ( x) 
h0 h
lim cos x sin( x  h)  sin x cos(x  h)
f ( x) 
h0 h cos x cos(x  h)
lim cos x sin x cosh cos2 x sinh  sin x cos x cosh sin 2 x sinh
f ( x) 
h0 h cos x cos(x  h)
lim cos2 x sinh  sin 2 x sinh
f ( x) 
h0 h cos x cos(x  h)

f ( x) 
lim  
cos2 x  sin 2 x sinh
h0 h cos x cos(x  h)
lim h
f ( x)  but cos(x  h)
h  0 h cos x cos(x  h)
lim 1
f ( x) 
h  0 cos x cos x
1
f ( x)  2
 sec2 x
cos x
d
tan x   sec2 x
dx

Example 57
Find the derivative of f ( x)  tan x 3
Solution

502
y  tan x 3 .......... .......... .......... .......... .......... .......... .......( i )
du
Let u  x 3 ,  3x 2 .......... .......... .......... .......... ........( ii)
dx
dy
y  tan u,  sec2 u.......... .......... .......... .......... ........( iii)
du
dy dy du
From chain rule,  
dx du dx
dy
dx
  
 sec2 u 3 x 2 but u  x 3

dy
  3x 2 sec2 x 3
dx

(d) Derivative of secant (sec)


Consider a function f ( x)  sec x
lim f ( x  h)  f ( x )
From first principle f ( x) 
h0 h
f ( x)  sec x and f ( x  h)  sec(x  h)
lim sec(x  h)  sec x
f ( x) 
h0 h
1 1

lim cos(x  h) cos x
f ( x) 
h0 h
lim cos x  cos(x  h)
f ( x) 
h  0 h cos x cos(x  h)
lim cos x  cos x cosh sin x sinh 
f ( x) 
h0 h cos x cos(x  h)
lim cos x  cos x  h sin x 
f ( x)  but cos(x  h)  cos x
h0 h cos x cos(x  h)
lim h sin x
f ( x) 
h  0 h cos x cos x
sin x
f ( x)   sec x tan x
cos x cos x
d
sec x   sec x tan x
dx

503
Example 58
Find the derivative of f ( x)  sec5 x
Solution
y  sec5 x.......... .......... .......... .......... .........( i )
du
Let u  5 x,  5.......... .......... .......... ....( ii)
dx
dy
y  secu ,  secu tan u.......... .......... .....( iii)
du
dy dy du
From chain rule,  
dx du dx
 secu tan u 5 but u  5 x
dy
dx
dy
  5 sec5 x tan 5 x
dx

(e) Derivative of cosecant (cosec)


Consider a function f ( x)  cos ecx
lim f ( x  h)  f ( x )
From first principle f ( x) 
h0 h
f ( x)  cos ecx and f ( x  h)  cos ec( x  h)
lim cos sec(x  h)  cos ecx
f ( x) 
h0 h
1 1

lim sin( x  h) sin x
f ( x) 
h0 h
lim sin x  sin( x  h)
f ( x) 
h  0 h sin x sin( x  h)
lim sin x  sin x cosh cos x sinh 
f ( x) 
h0 h sin x sin( x  h)
lim sin x  sin x  h cos x 
f ( x)  but cos(x  h)  cos x
h0 h sin x sin( x  h)
lim  h cos x
f ( x) 
h  0 h sin x sin x

f ( x) 
 cos x
  cos ecx cot x
d
cos ecx   cos ecx cot x
sin x sin x dx

504
Example 59
Find the slope of f ( x)  cosecx
4

Solution
y  cos ecx 4 .......... .......... .......... .......... .......... .......... .......( i )
du
Let u  x 4 ,  4 x 3 .......... .......... .......... .......... .......... ..(ii)
dx
dy
y  cos ecu,   cos ecu cot u.......... .......... .......... .........( iii)
du
dy dy du
From chain rule,  
dx du dx
dy
dx
 
  cos ecu cot u  4 x 3 but u  x 4

dy
  4 x 3 cos ecx 4 cot x 4
dx

(f) Derivative of cotangent (cot)

505
d
cot x    cos ec2 x
dx

Example 60
Differentiate f x   cot2 x  1
Solution
y  cot2 x  1

 2 cosec 2 2 x  1
dy

dx

Shortly, we can summarize the above derivatives as shown below;


d
(a) (sin x)  cos x
dx
d
(b) (cos x)   sin x
dx
d
(c) (tan x)  sec2 x
dx
d
(d) (cosecx)   cosecx cot x
dx
d
(e) (sec x)  sec x tan x
dx
d
(f) (cot x)   cosec 2 x
dx
506
Exercise 9.2
1. Find the derivative of the following;
sin x
(a) y 
2  3x
(b) y  xsin x 10
1 x
(c) y 
x
(d) y  e cos 2 x
(e) y  1  1x  tan x

2. Find the derivative of the following;


(a) y  x sec x cos x
(b) y  x 2 tan x sin x
(c) y  xe
x

(d) y  xIn x
(e) y  xIncos x 

9.7 DERIVATIVE OF INVERSE TRIGONOMETRIC FUNCTIONS

(i) Derivative of sin 1 x


Let y  sin 1 x
sin y  x
Differentiate both side w.r.t x
dy
cos y 1
dx
dy 1 1
  but sin y  x
dx cos y 1  sin y
2

dy 1

dx 1 x2
d
 
sin 1 x 
1
dx 1 x2

507
(ii) Derivative of cos1 x
Let y  cos1 x
cos y  x
Differentiate both side w.r.t x
dy
 sin y  1
dx
dy 1 1
  but cos y  x
dx sin y 1  cos2 y
dy 1

dx 1 x2
1
d
cos1 x  
dx 1  x2

(iii)Derivative of tan 1 x
Let y  tan 1 x
tan y  x
dy
sec2 y  1
dx
dy 1 1
 2  but tan y  x
dx sec y 1  tan 2 y
dy 1

dx 1  x 2

d
dx
 
tan 1 x 
1
1  x2

508
(iv) Derivative of cosec 1 x
Let y  cosec 1 x

1
d

cos ec1 x  
dx x x2  1

(v) Derivative of sec1 x


Let y  sec1 x
sec y  x
Differentiate both side w.r.t x
dy
sec y tan y 1
dx
dy 1 1
  but sec y  x
dx sec y tan y sec hy sec2 y  1
dy 1

dx x x 2  1

d

sec1 x  1
dx x x2 1

509
(vi) Derivative of cot 1 x
Let y  cot 1 x
cot y  x
Differentiate both side w.r.t x
dy
 cosec 2 y  1
dx
dy 1 1
  but cot y  x
dx cosec y cot 2 y  1
2

dy 1

dx 1  x 2
1
d
dx

cot1 x 
1  x2

Example 61
Find the derivative of y  sin 1 x 3 
Solution
y  sin 1 x 3 .......... .......... .......... .......... .......... .........( i )
du
Let u  x 3 ,  3x 2 .......... .......... .......... .......... ........( ii)
dx
dy 1 1
y  sin 1 u,   .......... .......... ....( iii)
du 1 u 2 1 x6
dy dy du
From chain rule  
dx du dx
dy 3x 2
 
dx 1 x6

Example 62
Find the derivative of y  sin 1 cos x 
Solution

510
y  sin 1 cos x .......... .......... .......... .......... .......... .......... .......... (i )
du
Let u  cos x,   sin x.......... .......... .......... .......... .......... .....( ii)
dx
dy 1 1 1
y  sin 1 u,    .......... .......... ....( iii)
du 1 u 2
1  cos x 2 sin x

From chain rule


dy dy du
  
1
 sin x   1
dx du dx sin x
dy
  1
dx

Example 63
Find the derivative of y  tan 1 2 x  1
Solution
y  tan 1 2 x  1.......... .......... .......... .......... .......... .......... .......... ...( i )
du
Let u  2 x  1,  2.......... .......... .......... .......... .......... .......... ....( ii)
dx
dy 1 1 1
y  tan 1 u ,    2 .......... .......( iii)
du 1  u 1  2 x  1 4x  4x  2
2 2

dy dy du
From chain rule  
dx du dx
dy 2
 2
dx 4 x  4 x  2
dy 1
  2
dx 2 x  2 x  1

Example 64
Find the derivative of y  sec x
1 2
 
Solution
y  sec1 x 2 .......... .......... .......... .......... .......... .......... .....( i )
du
Let u  x 2 ,  2 x.......... .......... .......... .......... .......... ......( ii)
dx
dy 1 1
y  sec1 u,   .......... .......... ......( iii)
du u u 2  1 x 2 x 4  1
dy dy du 2x 2
From chain rule    
dx du dx x 2 x 4  1 x x 4  1
dy 2
 
dx x x 4  1

511
Example 65
Find the derivative of y  sec1 tan x 
Solution
y  sec1 tan x .......... .......... .......... .......... .......... .......... .......... ..(i )
du
Let u  tan x,  sec2 x.......... .......... .......... .......... .......... .....( ii)
dx
dy 1 1
y  sec1 u,   .......... .......... ......( iii)
du u u 2  1 tan x tan 2 x  1
dy dy du sec2 x
From chain rule   
dx du dx tan x tan 2 x  1
dy sec2 x
 
dx tan x tan 2 x  1

9.8 DERIVATIVE OF HYPERBOLIC FUNCTIONS


(a) Derivative of basic hyperbolic functions

(i) Derivative of sinh x


e x  ex 1 x
Let y  sinh x   e  e  x 
2 2
x
e e
 e  e  x  
x
dy 1 x
 cosh x
dx 2 2
d
sinh x   cosh x
dx

(ii) Derivative of cosh x


e x  ex 1 x
Let y  cosh x   e  e  x 
2 2
e x  e x
 e  e  x  
dy 1 x
 sinh x
dx 2 2
d
cosh x   sinh x
dx

512
(iii)Derivative of tanh x
e x  ex
Let y  tanh x  x
e  e x
dy (e x  e  x )(e x  e  x )  (e x  e  x )(e x  e  x )

dx (e x  e  x ) 2
dy (e x  e  x ) 2  (e x  e  x ) 2

dx (e x  e  x ) 2
2
dy  e x  e x 
 1   x x

dx e e 
dy
 1  tanh 2 x  sec h 2 x
dx
d
tanh x   sec h 2 x
dx

(iv) Derivative of cos echx


Let y  cos echx
1 2
y  x
sinh x e  e  x



dy  2 e x  e  x
 


 2  e  e

 x
x x


dx 
e x  ex
2
 e e
x x
 e  e
x

dy
  cos echx coth x
dx
d
cos echx   cos echx coth x
dx

513
(v) Derivative of sec hx
Let y  sec hx
1 2
y  x
cosh x e  e  x



dy  2 e x  e  x 
  x
2   e  e
x
 x
x


dx 
e x  ex
2

e e
x
 e  e
x

dy
  sec hx tanh x
dx
d
sec hx   sec hx tanh x
dx

(vi) Derivative of coth x


e x  ex
Let y  coth x 
e x  e x
dy (e x  e  x )(e x  e  x )  (e x  e  x )(e x  e  x )

dx (e x  e  x ) 2
dy (e x  e  x ) 2  (e x  e  x ) 2

dx (e x  e  x ) 2
2
dy  e x  e x 
 1   x x

dx e e 
dy
 1  coth 2 x   cos ech 2 x
dx
d
coth x    cos ech 2 x
dx

Example 66
Find the derivative of y  coshx 5
Solution
y  cosh x 5 .......... .......... .......... .......... .......... .......... .......... ...( i )
du
Let u  x 5 ,  5 x 4 .......... .......... .......... .......... .......... ........( ii)
dx
dy
y  coshu ,  sinh u  sinh x 5 .......... .......... .......... .......... (iii)
du
dy dy du
From chain rule  
dx du dx
dy
  5 x 4 sinh x 5
dx

514
Example 67
Find the derivative of y  sinh x
Solution
y  sinh x .......... .......... .......... .......... .......... .......... .......... ...(i )
du 1
Let u  x ,  .......... .......... .......... .......... .......... ....( ii)
dx 2 x
dy
y  sinh u,  coshu  cosh x .......... .......... .......... .......... (iii)
du
dy dy du
From chain rule  
dx du dx
dy cosh x
 
dx 2 x

Example 68
Find the derivative of y  tanh sin x 
Solution
y  tanh sin x .......... .......... .......... .......... .......... .......... .......... .....( i )
du
Let u  sin x,  cos x.......... .......... .......... .......... .......... .......... (ii)
dx
 sec h 2 u  sec h 2 sin x .......... .......... .......... .........( iii)
dy
y  tanh u,
du
dy dy du
From chain rule  
dx du dx
  cos x sec h 2 sin x 
dy
dx

515
Example 69
Find the derivative of y  sech 1 x 
Solution
y  sec h 1 x .......... .......... .......... .......... .......... .......... .......... ......( i )
du
Let u  1 x ,   1 x 2 .......... .......... .......... .......... .......... .......... .(ii)
dx
  sec hu tanh u   sec h 1 x  tanh  1 x .......... .....( iii)
dy
y  sec hu,
du
dy dy du
From chain rule  
dx du dx
dy sec h 1 x  tanh  1 x 
 
dx x2

Example 70
Find the derivative of y  cothx 2 
Solution
y  cothx 2 .......... .......... .......... .......... .......... .......... .......... ...( i )
du
Let u  x 2 ,  2 x 3 .......... .......... .......... .......... .......... .......( ii)
dx
  cosech 2 u   cosech 2 x 2 .......... .......... ..(iii)
dy
y  cothu,
du
dy dy du
From chain rule  
dx du dx
  2 x 3 cosech 2 x 2 
dy
dx

516
(b) Derivative of inverse hyperbolic functions

(i) Derivative of sinh 1 x


Let y  sinh 1 x
sinh y  x
Differentiate both side w.r.t x
dy
cosh y 1
dx
dy 1 1
  but sinh y  x
dx cosh y 1  sinh 2 y
dy 1

dx 1 x2

d
 
sinh 1 x 
1
dx x 2 1

(ii) Derivative of cosh 1 x


Let y  cosh1 x
cosh y  x
Differentiate both side w.r.t x
dy
sinh y  1
dx
dy 1 1
  but cosh y  x
dx sinh y cosh y  1
2

dy 1

dx x2 1
d
cosh1 x  
1
dx x2 1

517
(iii) Derivative of tanh 1 x
Let y  tanh 1 x
tanh y  x
dy
sec h 2 y  1
dx
dy 1 1
  but tanh y  x
dx sec h y 1  tanh 2 y
2

dy 1

dx 1  x 2

d
dx

tanh 1 x  1
1 x2

(iv)Derivative of cos ech 1 x


Let y  cos ech 1 x
cos echy  x
Differentiate both side w.r.t x
dy
 cos echy coth y 1
dx
dy 1 1
  but cos echy  x
dx cos echy coth y cos echy 1  cos ech 2 y
dy 1

dx x 1  x 2
1
d
cos ech1 x  
dx x 1 x2

518
(v) Derivative of sec h 1 x
Let y  sec h 1 x
sec hy  x
Differentiate both side w.r.t x
dy
 sec hy tanh y 1
dx
dy 1 1
  but sec hy  x
dx sec hy tanh y sec hy 1  sec h 2 y
dy 1

dx x 1  x 2

1
d
sec h 1 x  
dx x 1 x2

(vi) Derivative of coth 1 x


Let y  coth 1 x
coth y  x
Differentiate both side w.r.t x
dy
 cos ech 2 y 1
dx
dy 1 1
  but coth y  x
dx cos ech 2 y coth 2 y  1
dy 1 1
 2 
dx x  1 1  x 2
d
dx

coth1 x  
1
1 x2

519
Example 71
Find the derivative of y  sinh 1 3x 
Solution
y  sinh 1 3x .......... .......... .......... .......... .......... .......... .......... .(i )
du
Let u  3 x,  3.......... .......... .......... .......... .......... .......... ....( ii)
dx
dy 1 1
y  sinh 1 u ,   .......... .......... .......... (iii)
du u 1
2
9x 12

dy dy du
From chain rule  
dx du dx
dy 3
 
dx 9x 2 1

Example 72
Find the derivative of y  sinh 1 x 5  
Solution
y  sinh 1 x 5 .......... .......... .......... .......... .......... .......... .......... .(i )
du
Let u  x 5 ,  5 x 4 .......... .......... .......... .......... .......... .......... (ii)
dx
dy 1 1
y  sinh 1 u,   .......... .......... .......... (iii)
du u 2 1 x10  1
dy dy du
From chain rule  
dx du dx
dy 5x 4
 
dx x10  1

520
Example 73
Find the derivative of y  cosh1 x 2  4
Solution
y  cosh1 x 2  4.......... .......... .......... .......... .......... .......... .......... .(i )
du
Let u  x 2  4,  2 x.......... .......... .......... .......... .......... .......... .(ii)
dx
dy 1 1
y  cosh1 u ,   .......... .......... ........( iii)
du u 1
2
x  8 x  15
4 2

dy dy du
From chain rule  
dx du dx
dy 2x
 
dx x 4  8 x 2  15

Example 74
Find the derivative of y  tanh 1 x  
Solution
 
y  tanh 1 x .......... .......... .......... .......... .......... .......... .......... .(i )
du 1
Let u  x ,  .......... .......... .......... .......... .......... ........( ii)
dx 2 x
dy 1 1
y  tanh 1 u,   .......... .......... .......... .......... ..(iii)
du 1  u 2
1 x
dy dy du
From chain rule  
dx du dx
dy 1
 
dx 21  x  x

521
Example 75
Find the derivative of y  sec h 1 x 1 
Solution
y  sec h 1 x 1 .......... .......... .......... .......... .......... .......... .......... .(i )
du 1
Let u  x 1 ,   x 2  2 .......... .......... .......... .......... .......... ........( ii)
dx x
1 dy 1  x2
y  sec h u,   .......... .......... .......... .......... ..(iii)
du u 1  u 2 x2 1
dy dy du
From chain rule  
dx du dx
dy 1
 
dx x 2 1

Exercise 9.3
1. Find the derivative of the following;
(a) y  tan 1 tanh x 
(b) y  cosh x

2. Find the derivative of the following;


(a) y  sinh x sec x
(b) y  coth cos x

3. Show that
d
dx
 
tan 1 e x  sec hx

522
9.9 DERIVATIVE OF NATURAL LOGARITM AND EXPONENTIAL
FUNCTIONS
(a) Derivative of Inx
Consider the function f ( x)  Inx
lim f ( x  h)  f ( x )
From first principle f ( x) 
h0 h
f ( x)  Inx and f ( x  h)  In ( x  h)
lim In ( x  h)  Inx
f ( x)  .......... .......... .......... ......( i )
h0 h
  h   h
But In ( x  h)  In  x1    Inx  In1  .......... .....( ii)
  x   x
Substitute equation (ii) into equation (i)
 h
Inx  In1    Inx
f ( x) 
lim  x
h0 h
 h
In1  
f ( x) 
lim  x  .......... .......... .......... .......... ....( iii)
h0 h
 h h h
2
h3 h4
But the series of In1     2  3  4  ........( iv)
 x  x 2x 3x 4 x
Substitute equation (iv) into equation (iii)
h h2 h3 h4
lim     ...
f ( x)  x 2 x 2 3x 3 4 x 4
h0 h
lim 1 h1 h2 h3
f ( x)      ...
h  0 x 2 x 2 3x 3 4 x 4
1
f ( x) 
x
d
Inx   1
dx x

523
Example 76
Find the derivative of y  Inx 3
Solution
y  Inx 3 .......... .......... .......... .......... .......... .......... ...(i)
du
Let u  x 3 ,  3x 2 .......... .......... .......... .......... ..(ii)
dx
dy 1 1
y  Inu,   .......... .......... .......... .......... .(iii)
du u x 3
dy dy du
From chain rule  
dx du dx
dy 3x 2
  3
dx x

Example 77
Find the derivative of y  Insin x 
Solution
y  In sin x .......... .......... .......... .......... .......... .......... ...(i )
du
Let u  sin x,  cos x.......... .......... .......... .......... ..(ii)
dx
dy 1 1
y  Inu,   .......... .......... .......... .......... ..(iii)
du u sin x
dy dy du 1
From chain rule     cos x
dx du dx sin x
dy
  cot x
dx

Example 78
Find the derivative of y  In tanh 1 x 
Solution

524
 
y  In tanh 1 x .......... .......... .......... .......... .......... .......... ...(i )
du 1
Let u  tanh 1 x,  .......... .......... .......... .......... ..(ii)
dx 1  x 2
dy 1 1
y  Inu,   .......... .......... .......... .......... ..(iii)
du u tanh 1 x
dy dy du
From chain rule  
dx du dx
dy 1
 

dx 1  x tanh 1 x
2

Example 79
Find the derivative of y  In x  
Solution
 
y  In x .......... .......... .......... .......... .......... .......... ...(i )
du 1
Let u  x ,  .......... .......... .......... .......... ..(ii)
dx 2 x
dy 1 1
y  Inu,   .......... .......... .......... .......... ..(iii)
du u x
dy dy du
From chain rule  
dx du dx
dy 1
 
dx 2 x

Example 80
Find the derivative of y  xInx
Solution
y  xInx
du
Let u  x, 1
dx
dv 1
v  Inx, 
dx x
dy dv du
From Product rule u v
dx dx dx
dy
  1  Inx
dx

525
(b) Derivative of log x
let y  log x  log10
x

y  log10 (Change into exponential function)


x

x  10 y (apply In both sides)


Inx  In10 y
Inx  yIn10 (Differentiate both sides w.r.t x)
1 dy
 In10
x dx
dy 1

dx xIn10

d 1
(log x) 
dx xIn10

Example 81
Find the derivative of y  log x 2
Solution
y  log x 2 change into exponential
x 2  10 y apply In both sides
Inx 2  In10 y
yIn10  2 Inx
dy 2
In10 
dx x
dy 2
 
dx xIn10

526
x
(c) Derivative of e
Consider the function f ( x)  e x
lim f ( x  h)  f ( x )
From first principle f ( x) 
h0 h
f ( x)  e x and f ( x  h)  e ( x  h )
lim e ( xh)  e x
f ( x) 
h0 h
lim e xeh  e x
f ( x) 
h0 h

f ( x) 
lim 
e x eh 1 
but e  1  h 
h h 2 h3 h 4
   ...
h0 h 2! 3! 4!
 h 2 h3 h 4 
e 1  h 
x
   ...  1
f ( x) 
lim  2! 3! 4! 
h0 h
 h 2 h3 h 4 
e  h 
x
   ... 
f ( x) 
lim  2! 3! 4! 
h0 h
 h h 2 h3 
e x 1     ... 
f ( x) 
lim  2! 3! 4! 
h0 h
lim x h h 2 h3 
f ( x)  e 1     ... 
h0  2! 3! 4! 
f ( x)  e x

d x
dx
 
e  ex

527
Alternative Derivation
Let the function y  e x apply In both sides
Iny  Ine x
Iny  x (differentiate both sides w.r.t x)
1 dy
1
y dx
dy
 y but y  e x
dx
d x
(e )  e x
dx

Example 82
Find the derivative of y  e
2x

Solution
y  e 2 x .......... .......... .......... .......... .......... .......... .......... (i )
du
Let u  2 x,  2.......... .......... .......... .......... .........( ii)
dx
dy
y  eu ,  e u  e 2 x .......... .......... .......... .......... ......( iii)
du
dy dy du
From chain rule  
du du dx
dy
  2e 2 x
du

Example 83
Find the derivative of y  e
x3

Solution
y  e x .......... .......... .......... .......... .......... .......... .......... (i )
3

du
Let u  x 3 ,  3 x 2 .......... .......... .......... .......... .......( ii)
dx
dy
y  eu ,  e u  e x .......... .......... .......... .......... ......( iii)
3

du
dy dy du
From chain rule  
du du dx
dy
  3x 2 e x
3

du
528
Example 84
Find the derivative of y  e
sin x

Solution
y  e sin x .......... .......... .......... .......... .......... .......... .......... (i )
du
Let u  sin x,  cos x.......... .......... .......... .......... .......( ii)
dx
dy
y  eu ,  e u  e sin x .......... .......... .......... .......... ......( iii)
du
dy dy du
From chain rule  
du du dx
dy
  cos xesin x
du

Example 85
Find the derivative of y  cos xesin x
Solution
y  cos xesin x .......... .......... .......... .......... .......... .......... ...( i )
du
Let u  cos x,   sin x.......... .......... .......... .......... ...( ii)
dx
dv
v  e sin x ,  cos xesin x .......... .......... .......... .......... ....( iii)
dx
dy dv du
From product rule u v
du dx dx
 cos2 x  sin x e sin x
dy

du

Exercise 9.4
1. Find the derivative of y  4e x1
2. Find the derivative of y  xex e 2
e2x
3. Find the derivative of y  2
x
4. Find the derivative of y  4
ex

5. Find the derivative of y  e xcos x

529
9.10 DERIVATIVE OF IMPLICITY FUNCTIONS
Implicity functions are function in which x and y are mixed in the equations.
Implicity functions are solved by differentiating each variable with respect to x
dy
and make the subject.
dx
Example 86
Find the derivative of x 2  y 2  xy  0
Solution
x 2  y 2  xy  0
dy dy
2x  2 y  y  x  0
dx dx
dy  2 x  y 
 
dx 2y  x

Example 87
Find the derivative of y  cosx  y   x
Solution
y  cosx  y   x
dy  dy 
 1   sinx  y   1
dx  dx 

 sinx  y   sinx  y   1
dy dy
dx dx
1  sinx  y  dy  1  sinx  y 
dx
dy 1  sinx  y 
 
dx 1  sinx  y 

Example 88
Find the derivative of x 2 y  xy 2  2 x
Solution
x 2 y  xy 2  2 x
dy dy
2 xy  x 2  2 xy  y 2  2
dx dx
x 2  2 xy dy
dx
 2  2 xy  y 2

dy 2  2 xy  y 2
 
dx x 2  2 xy

530
Example 89
Find the derivative of xy  sinxy  1
Solution
xy  sin xy   1
 dy 
 y   x  y  cos xy   0
dy
x
dx  dx 
x  y  x cos xy   y cos xy   0
dy dy
dx dx
x  x cos xy     y  y cos xy 
dy dy
dx dx
x  x cosxy dy    y  y cosxy
dx
dy y  y cos xy 
 
dx x  x cos xy 

9.11 HIGHER DERIVATIVE


Higher Derivative is the derivative which contains differential coefficient
greater than one.
d2y
 2 .......... .second derivative
dx
d3y
 3 .......... .third derivative
dx
d4y
 4 .......... .fourth derivative
dx
.
.
.
dny
 n .......... .n th derivative
dx

531
Example 90
Find the second derivative of y  x 4  2x 3
Solution
y  x 4  2x3
dy
 4x3  6x 2
dx
d2y
 2  12x 2  12x
dx

Example 91
Find the second derivative of y  e cos x
Solution
y  e cos x
dy
  sin xecos x
dx
d2y
 2  sin 2 xecos x
dx

Example 92
Find the second derivative of y  Inx
Solution
y  Inx
dy 1

dx x
d2y 1
 2  2
dx x

Example 93
Find the second derivative of y  tan 1 x
Solution
y  tan 1 x
dy 1

dx 1  x 2
Apply Quotient rule
d2y  2x
 
dx 2

1 x2 
2

532
9.12 DERIVATIVE OF PARAMETRIC FUNCTIONS
A parametric equation defines a group of quantities as functions of one or more
independent variables called parameters. Parametric equations are commonly
used to express the coordinates of the points that make up a geometric object
such as a curve or surface, in which case the equations are collectively called a
parametric representation or parameterization.
Parametric functions are functions which has a pair of equations in which
x and y are defined in another parameter. We can find the derivative of
parametric functions by computing the derivative of each equation with respect
to the given parameter and the individual derivative are connected by the
formula below;
dy dy dx

dx dt dt

dy dy dt
 .
dx dt dx

Second Derivative of Parametric equation


d 2 y d  dy 
  
dx 2 dx  dx 

d 2 y d  dy  dt
  .
dx2 dt  dx  dx

Example 94
Find the derivative of the parametric equations x  t 2  1 and y  2t  1 .
Solution
dx
x  t 2  1,  2t.......... .......... .......... .......... ......( i )
dt
dy
y  2t  1,  2.......... .......... .......... .......... .......( ii)
dt
dy dy dt 1 1
From    2  
dx dt dx  2t  t
dy 1
 
dx t

533
Example 95
Find the derivative of the parametric equations x  2 cost and y  3 sin t .
Solution
dx
x  2 cos t ,  2 sin t.......... .......... .......... .......... ......( i )
dt
dy
y  3 sin t ,  3 cos t.......... .......... .......... .......... .......( ii)
dt
dy dy dt  1  3
From    3 cost     cot t
dx dt dx   2 sin t  2
dy 3
   cot t
dx 2

Example 96
Find the derivative of the parametric equations x  sin 2 t and y  cost .
Solution
dx
x  sin 2 t ,  2 sin t cost.......... .......... .......... .......... .....( i )
dt
dy
y  cost ,   sin t.......... .......... .......... .......... .......... ..(ii)
dt
dy dy dt  1  1 1
From     sin t     sect
dx dt dx  2 sin t cost  2 cost 2
dy 1
   sect
dx 2

Example 97
Find the derivative of the parametric equations x  b sin t and y  a cost .
Solution
dy
y  a cost ,  a sin t.......... .......... .......... .......... ........( i )
dt
dx
x  b sin t ,  b cost.......... .......... .......... .......... .......... .(ii)
dt
dy dy dt  1  a
From    a sin t     tan t
dx dt dx  b cost  b
dy a
   tan t
dx b

534
Example 98
Find the derivative of the parametric equations x  e t and y  e 2t .
Solution
dx
x  et ,  e t .......... .......... .......... .......... ......( i )
dt
dy
y  e 2t ,  2e 2t .......... .......... .......... .......... ..(ii)
dt
dy dy dt 1
From    2e 2t  t   2e t
dx dt dx e 
dy
  2e t
dx

Example 99
1
Find the derivative of the parametric equations x  Int  2 and y  where
t 1
t  1 .
Solution

x  In t  2,
dx 1
 .......... .......... .......... .......... ......( i )
dt t  2
1 dy 1
y ,  .......... .......... .......... .......... .......( ii)
t  1 dt t  12
1 t2
dy dy dt
From    t  2   
dx dt dx t  12 t  12
dy t2
 
dx t  12

Exercise 9.5
5
1. Given that y  t 3  t 2 and x  t for t  0 . Use parametric
2
dy
differentiation to express in term of t in simplified form.
dx
dy
2. Given that x 2  xy  y 2  k 2 where k  R . Find
dx

535
9.13 PARTIAL DERIVATIVE
Partial derivative is the derivative of the function z  f x, y  where x and y
are independent variables and z is depending variable.
The partial derivative of z  f x, y  is done by taking derivative of z with
respect to one of the two variables x and y and vice versa is true.

The partial derivative of function z or f with respect to x is denoted by one of


the symbol below;
z f
, , f x  x, y  and f x
x x
The partial derivative of function z or f with respect to y is denoted by one
of the symbol below;
z f
, , f y  x, y  and f y
y y

Example 100
Find the first order partial derivatives of f x, y   x 2  y 2  2 xy
Solution
f  x, y   x 2  y 2  2 xy
 f x  x, y   2 x  2 y
 f y  x, y   2 y  2 x

Example 101
Find the first order partial derivatives of f x, y   x y  xy
2 2

Solution
f x, y   x 2 y 2  xy
 f x x, y   2 xy 2  2 y
 f y x, y   2 yx 2  2 x

536
Example 102
Find the first order partial derivatives of z  x 2 y  3 y 2  tan 4 y
Solution
z  x 2 y  3 y 2  tan 4 y
z
  2 xy
x
z
  x 2  6 y  4 sec2 4 y
y

HIGHER ORDER PARTIAL DERIVATIVE


Consider a case of a function of two variables f ( x, y ) since both of the first
order partial derivatives are also functions of x and y , we could in turn
differentiate each with respect to x or y . This mean that for the case of a
function of two variables there are will be a total of four possible second order
derivatives.
Notation:
  z   2 z
(i )  f x x  f xx or  
x  x  x 2
  z   2 z
(ii)  f x y  f xy or  
y  x  yx
  z   2 z
(iii)  f y y  f yy or  
y  y  y 2
  z   2 z
(iv)  f y x  f yx or  
x  y  xy
Example 103
Find the second order partial derivatives of f x, y   x 2 y 2  xy
Solution
f  x, y   x 2 y 2  xy
f x  2 xy 2  2 y
f y  2 yx 2  2 x
f xx  2 y 2
f xy  4 xy  2
f yy  2 x 2
f yx  4 yx  2

537
Example 104
Find the second order partial derivatives of f x, y   y 2 Inx
Solution
f  x, y   y 2 Inx
y2
fx 
x
f y  2 yInx
y2
f xx  
x2
2y
f xy 
x
f yy  2 Inx
2y
f yx 
x

Example 105
Find the second order partial derivatives of f x, y   Inxy
Solution
f  x, y   xInxy 
f x  In  xy   1
x
fy 
y
1
f xx 
x
1
f xy 
y
x
f yy  
y2
1
f yx 
y

538
9. 14 APPLICATION OF DIFFERENTIATION
Differentiation is the subfield of Calculus and there are various application of
differentiation in real world. The differentiation is very important part of
Mathematics as it is used in many scientific fields. The most common
application are Rate of change, Small change, Series expansion, Maximum,
minimum points and point of inflexion. These applications are explained below
as follows;

(a) Rate of change


The rate of change of substance is defined as change of quantity of a substance
per time taken.
The change per time can be either a decrease or increase of a given quantity.

Example 106
The radius of sphere increases at the rate of 0.25 m s . Find rate of changes of its
area if it has a radius of 3.5m
Solution
Let r be radius of a sphere and A be area of a sphere
dr
 0.25 m s
dt
dA
Required when r  3.5m
dt
From A  4r 2
dA dr
 8r
dt dt
 8 3.5 0.25  22 m s
dA 2

dt
dA
  22 m s
2

dt

Example 107
An inverted right circular cone of semi-vertical angle 45 is collecting water
from a tap at a steady of rate 18cm 3 / s . Find the rate at which the depth h of
water is rising when h  3cm
Solution

539
Semi - vertical angle  45
dv
 18cm 3 / s
dt
dh
Required when h  3cm
dt
Consider the figure below

Volume of a cone, v  13 r 2 h.......... .......... .......... .......... .(i )


r
tan 45  , r  h tan 45.......... .......... .......... .......... ........( ii)
h
Substitute equation (ii) into equation (i)

v  13  h tan 45 h 2

v  13 h 3 tan 2 45
dv dh
 h 2 tan 2 45
dt dt
dh dv 18
 2 dt2    2cm / s
dt h tan 45  32 12
 The depth of water is rising at a rate of 2cm / s

Example 108
A 15m ladder is resting against the wall. The bottom is initially 10m away from
the wall and is being pushed towards the wall at a rate of 0.25 m sec . How fast
is the top of the ladder moving up the wall 12 seconds after we start pushing?
Solution
Length of a ladder  15m

540
The ladder is pushed towards the wall at a rate of 0.25m / s
How fast is a top of a ladder after 12 seconds
Distance
From, Rate 
Time
Distance  Rate  Time  0.25 12  3m
The distance moved in 12 seconds is 3m

Apply Pythagoras theorem


x 2  y 2  152
7 2  y 2  152
y  176
Differentiate x 2  y 2  152 to acquire the rate with respect to time
dx dy
2x  2 y  0
dt dt
  0.25 
dy x dx 7 7
  m/s
dx y dt 176 4 176
7
 The rate of change of top is m/ s
4 176

541
Example 109
Two people are 50m apart. One of them starts walking north at a rate so that
the angle is changing at a constant rate of 0.01 rad/min. At what rate is distance
between the two people changing when   0.5 radians ?
Solution
Distance apart, s   50m
d
 0.01rad/min
dt
ds
Required when   0.5 radians
dt
Consider the figure below;

50
cos 
s
s  50 sec
ds d
 50 sec tan 
dt dt
 50 sec0.5 tan 0.5 0.01  0.311m / s
ds
dt
 The rate at which t he distance is changing is 0.311m / s

Example 110
A trough of water is 8m deep and its ends are in the shape of isosceles triangles
whose width is 5m and height is 2m. If water is being pumped in at a constant
rate of 6 m 3 sec . At what rate is the height of water changing when water has a
height of 120 cm?
Solution
Consider the sketch below;

542
L  8m
B  5m
H  2m
dv
 6m 3 / s
dt
dh
Required when h  120cm  1.2m
dt
1
From, V  bhL.......... .......... .......... .........( i )
2
B H Bh
 , b .......... .......... .......... .........( ii)
b h H
Substitute equation (ii) into equation (i)
1 Bh 2 L
V
2 H
dv BhL dh

dt H dt
5 1.2  8 dh
6
2 dt
dh
 0.25m / s
dt
 The rate at which t he height is changing is 0.25m / s

Example 111
The period T of a simple pendulum is calculated by using the formula
l
T  2 where l is length of the pendulum and g is constant. Find the
g
percentage in the period if the pendulum is lengthened by 4% .
Solution
l
Consider T  2
g
4 2
T2  l.......... .......... .......... .......... .......... ..(i )
g

543
dT 4 2
2T 
dl g
2 2
TdT  dl.......... .......... .......... .......... .......( ii)
g
Divide equation (ii) by equation (i)
2 2
TdT dl
 g
4 2
T2 g
l

dT 1 dl dl
 but  4%
T 2 l l
dT 1
  4%  2%
T 2
 The percentage in the period is 2%

Example 112
A hemispherical bowl is being filled with water at a uniform rate. When the
 1 
height of water is hcm the volume is   rh 2  h 3 cm 3 , rcm being the radius
 3 
of the hemisphere. Find the rate at which the water level is rising when it is half
way to the top, given that r  6cm and that the bowl fills in 1 min .
Solution
Consider the figure of hemisphere

 1 
Given that V    rh2  h3 
 3 
dV
dt

  2rh  h 2
dh
dt

.......... .......... .......... .......... .......... .......... .......... .......... .......... (i )

544
2 r 6
But volume of hemisphere, V  r 3 at r  6, h    3 and the bowl fills in 1min
3 2 2
 63
2
dV 3 12
  cm 3 / s.......... .......... .......... .......... .......... .......... .......... .......... ...(ii)
dt 60 5
Substitute equation (ii) into equation (i)
12
5

  2rh  h 2
dh
dt

dh 12 12 4
  
dt 5 2rh  h  2
 
5 2(6)(3)  3 2

45
cm / s

4
 The rate at which t he water level is rising is cm / s
45

Example 113
A ladder 15m long resting against a vertical wall. Its top slides down the wall
while its bottom moves away along the level ground at a speed of 2 m s . How
fast is the angle between the top of the ladder and the wall changing when the
angle is  3 radians?
Solution
Consider the figure below;

Length of a ladder  15m


dx
 2m / s
dt
d 
Required when  
dt 3
x
From sin  , x  15sin
15
dx d    d
 15 cos , 2  15 cos 
dt dt  3  dt
d 4
  rad / s
dt 15

545
Example 114
A boy starts walking north at a speed of 1.5 m/s, and a girl starts walking west
at the same point P at the same time at a speed of 2 m/s. At what rate is the
distance between the boy and the girl increasing 6 seconds later?
Solution
Consider the figure below;

db dg
Given that  1.5m / s and  2m / s
dt dt
Determine the distance covered by each after 6 seconds
Distance  Speed  Time
Distance  1.5  6  9m Boy
Distance  2  6  12m Girl
Apply Pythagoras theorem g 2  b 2  c 2
c 2  122  9 2
c  15
From g 2  b 2  c 2
dg db dc
2g  2b  2c
dt dt dt
dc dg db
c g b
dt dt dt
15  122  91.5
dc
dt
dc
 2.5m / s
dt
 The rate at which t he distance between the boy and girl is increasing is 2.5m / s

546
Example 115
A police car, approaching right-angled intersection from the north, is chasing a
speeding SUV that has turned the corner and is now moving straight east. When
the police car is 0.6km north of intersection and the SUV is 0.8km east of
intersection, the police determine with radar that the distance between them and
the SUV is increasing at 20 km/hr. If the police car is moving at 60km/ hr at
the instant of measurement, what is the speed of the SUV?
Solution
Consider the figure below;

 60km / hrThe distance is decreasing toward the right - angled 


dc dp
Given  20km / hr and
dt dt
By Pythagoras theorem
c2  p2  s2
c 2  0.6  0.8
2 2

c 1
Also from c 2  p 2  s 2
dc dp ds
2c  2 p  2s
dt dt dt
dc dp ds
c  p s
dt dt dt
20  0.6 60  0.8
ds
dt
ds
 70km / hr
dt
 The speed of the SUV is 70km / hr

547
(b) Small change
From first Principle formula
f x  x   f x 
f x  
x
xf x   f x  x   f x 
f x  x   f x   xf x 

Small change formula f x  x   f x   xf x 

Example 116
Without using calculator or mathematical tables evaluate the value of 4.01
Solution
From f  x  x   f x   xf  x 
Given that f  x  x   4.01
f  x  x   4  0.01
x  4 and x  0.01
f x   x  4  2

f x  
1 1
  0.25
2 x 2 4
Then f  x  x   2  0.010.25  2.0025
 4.01  2.0025

Example117
3
Without using calculator or mathematical tables evaluate the value of 27.002
Solution
From f  x  x   f  x   xf  x 
Given that f  x  x   3 27.002
f  x  x   3 27  0.002
x  27 and x  0.002
f  x   3 x  3 27  3

f  x  
1 1 1
 
 
33 x
2

3 3 27  2
27
Then f  x  x   f  x   xf  x 
f  x  x   3  0.002 1 27 
 3 27.002  3.000074

548
Example 118
Without using calculator or mathematical tables evaluate the value of 4 16.012
Solution
From f x  x   f  x   xf  x 
Given that f x  x   4 16.012
f x  x   4 16  0.012
x  16 and x  0.012
f x   4 x  4 16  2

f x  
1 1 1
 
 
44 x
3
 
4 4 16
3
32
Then f x  x   f  x   xf  x 
 1 
f x  x   2  0.012 
 32 
 4 16.012  2.000375

Example 119
5
Without using calculator or mathematical tables evaluate the value of 32.1
Solution
From f  x  x   f  x   xf  x 
Given that f  x  x   5 32.1
f  x  x   5 32  0.1
x  32 and x  0.1
f  x   5 x  5 32  2

f  x  
1 1 1
 
 
55 x
4
 
5 5 32
4
80
Then f  x  x   f x   xf  x 
 1 
f  x  x   2  0.1   2.00125
 80 
 5 32.1  2.00125

549
(c) Maximum, Minimum and Point of Inflexion
The maxima and minima (the respective plurals of maximum and minimum) of
a function, known collectively as extrema (the plural of extremum), are the
largest and smallest value of the function, either within a given range (the local
or relative extrema) or on the entire domain of a function (the global or absolute
extrema).

fig 9. 3

The first derivative test examines a function's monotonic properties (where the
function is increasing or decreasing) focusing on a particular point in its
domain. If the function "switches" from increasing to decreasing at the point,
then the function will achieve a highest value at that point. Similarly, if the
function "switches" from decreasing to increasing at the point, then it will
achieve a least value at that point. If the function fails to "switch", and remains
increasing or remains decreasing, then no highest or least value is achieved.
i.e.

550
After establishing the critical points of a function, the second derivative test
uses the value of the second derivative at those points to determine whether
such points are a local maximum or a local minimum. If the function f is twice
differentiable at a critical point x (i.e. f′(x) = 0), then:
(i) If f x   0 then f has a local maximum at x
(ii) If f x   0 then f has a local minimum at x
(iii)If f x   0 then f has an inflexion at x

Example 120
Find the stationary point of the curve y  x 2  2 x and determine whether its
maximum or minimum point.
Solution
Consider y  x 2  2 x
dy
 2x  2
dx
dy
But at stationary point 0
dx
2x  2  0
x 1
When x  1, y  1
By using first derivative test;
L x 1 R
dy
 2x  2  0 
dx

 Point x, y   1,1 is minimum point

551
Alternatively
By using second derivative test;
Consider y  x 2  2 x
dy
 2x  2
dx
dy
But at stationary point 0
dx
2x  2  0
x 1
When x  1, y  1
d2y
 2  0 indication of minimum point 
dx 2
 Point x, y   1,1 is minimum point

Example 121
The profit PT .shs on the sale of a certain car and the time t (hours) of its
 250 2 
manufacturer are related by the formula P  10100   t  , where t  3
 t 
Find the maximum possible profit.
Solution
 250 2 
P  10100  t 
 t 
dP 250 dP
 2  2t but at stationary point 0
dt t dt
250
0  2  2t
t
2t  250
3

t 3  125
t 5
d 2P
  3  2   3  2  6  0 Indication of max. point 
500 500
2
dt t 5
 250 2 
P  10100   5   250
 5 
 The maximum possible profit is 250 / 

552
Example 122
Find the turning points and point of inflexion of the curve y  x 5  5 x 4  5 x 3  1 .
Solution
Consider the curve y  x 5  5 x 4  5 x 3  1
dy
 5 x 4  20x 3  15x 2
dx
dy
But at stationary points 0
dx
5 x 4  20x 3  15x 2  0
 
x2 x2  4x  3  0
x  0, x  1, x  3
When x  0, y  1
When x  1, y  0
When x  3, y  28
Stationary points are 0,1, 1,0 and 3,28

By using first derivative test;


dy
 5 x 4  20 x 3  15x 2
dx

L x0 R L x 1 R L x3 R
dy  0   0   0 
dx

 Point 0,1 is inflexion point


 Point 1,0 is maximum point
 Point 3,28 is minimum point

553
Alternatively
By using second derivative test;
Consider the curve y  x 5  5 x 4  5 x 3  1
dy
 5 x 4  20x 3  15x 2
dx
dy
But at stationary points 0
dx
5 x 4  20x 3  15x 2  0
 
x2 x2  4x  3  0
x  0, x  1, x  3
When x  0, y  1
When x  1, y  0
When x  3, y  28
Stationary points are 0,1, 1,0  and 3,28
By 2 nd derivative test;
d2y
2
 20x 3  60x 2  30x
dx
d2y
When x  0, 0 indicator of inflexion point 
dx 2
d2y
When x  1,  10  0 indicator of maximum point 
dx 2
d2y
When x  3,  90  0 indicator of minimum point 
dx 2
 0,1 is inflexion point
 1,0  is maximum point
 3,28 is minimum point

554
Example 123

A wire of length L  100 m units is to be cut into pieces. One piece is to be bent
to form a circle and the other piece is to be bent to form a square. If the sum of
the areas of the circle and the square is to be a minimum. What length of the
wire should be used to form a circle?
Solution
Given length L  100m
Let the length to be bent to form a circle be x m and that to be bent to form a square be 100  x m

x
From x  2r , r  .......... .......... .......... .......... .......... .......... .........( i )
2
100  x 1
Length of a square is  25  x.......... .......... .......... .......... .......( ii)
4 4
2 2 2
 1   x   1 
Total Area  A  r   25  x    
2
   25  x 
 4   2   4 
2
1 2  1 
A x   25  x 
4  4 
dA 1 1 1 
 x   25  x 
dx 2 2 4 
1 1 1 
0 x   25  x 
2 2 4 
100
x .......... .......... .......... .......... .......... .......... .......... .......... .........( iii)
4
d2A 1 1
   0 indication of minimum point 
dx 2 2 8
d2A 100
Since  0 then x  give minimum value
dx 2 4
 100 
 The length to be cut to form a circleis  .m
 4   

555
Example 124
A wire of length L  100 m units is to be cut into pieces. One piece is to be bent
to form a circle and the other piece is to be bent to form a square. If the sum of
the areas of the circle and the square is to be a maximum. What length of the
wire should be used to form a circle?
Solution
Given length L  100m
Let the length to be bent to form a circle be x m and that to be bent to form a square be 100  x m
x
From x  2r , r  .......... .......... .......... .......... .......... .......... .........( i )
2
100  x 1
Length of a square is  25  x.......... .......... .......... .......... .......( ii)
4 4
2 2 2
 1   x   1 
Total Area  A  r   25  x        25  x 
2

 4   2   4 
2
1 2  1 
A x   25  x 
4  4 
dA 1 1 1 
 x   25  x 
dx 2 2 4 
1 1 1 
0 x   25  x 
2 2 4 
100
x .......... .......... .......... .......... .......... .......... .......... .......... .........( iii)
4
d2A 1 1
   0 indication of minimum point 
dx 2 2 8
d2A 100
Since  0 then x  give minimum value
dx 2 4
Check other possibilities
When we use the whole wire L  100m  to form a square the total area is 25  25  625m 2

When we use the whole wire L  100m to form a square the total area is below

556
x 100 50
From x  2r , r    m
2 2 
2
 50 
Area of a circle A  r      795.8m 2
2

 
 In order to maximize the area the whole wire should be used to form a circle.

Example 125
Find the coordinate of the minimum point of the catenary curve given by the
 x
equation y  a cosh 
a
Solution
 x
Given that y  a cosh 
a
dy  x dy
 sinh  but 0
dx  
a dx
 x
sinh   0
a
x
 sinh 1 0
a
x
0
a
x0
d2y 1
When x  0, y  a and   0 indication of minimum 
dx 2 a
 The minimum point is 0, a 

557
Example 126
A piece of wire which forms the circumference of a circle of radius of 0.12m
is cut and bent so as to form two new circles. Find the radius of each circle in
order that the sum of their areas of the two circles shall be as small as possible.
Solution
The length of the wire is given by circumference of a circle with radius, r  0.12m
c  2r  2  0.12  0.24
Length of a wire  0.24
Let length x to be cut to form a first circle and 0.24  x  to form a second circle
c1  2r1  x
x
r1  .......... .......... .......... .......... .......... .......... .......... .......... .......... .......... ..(i )
2
c2  2r2  0.24  x
0.24  x
r2  .......... .......... .......... .......... .......... .......... .......... .......... .......... ..(ii)
2
Total area, A  A1  A2
A  r1  r2   r1  r2
2 2
 2 2

  x  2  0.24  x  2  1 2
A    
  2   2
   
x  0.24  x 
2

   4
dA 1
 2 x  20.24  x   1 x  0.24  x  but dA  0
dx 4 2 dx
0
1
x  0.24  x 
2
x  0.24  x   0
2 x  0.24
x  0.12 .......... .......... .......... .......... .......... .......... .......... .......... .......... .......... (iii)
x 0.12
From r1    0.06m
2 2
0.24  x 0.24  0.12 0.12
Also from r2     0.06m
2 2 2
Now,
dA 1
 x  0.24  x   1 2 x  0.24 
dx 2 2
d2A 1
  0 indication of minimum value 
dx 2 
 In order to obtain minimum area the radii of each circleshould be 0.06m

558
Example 127
Find the dimensions of a rectangle with perimeter 1000m so that the area of the
rectangle is a maximum.
Solution
Let the Perimeter,P  1000m
Let l and w be length and width of the rectangle respectively
Consider the figure below;

P  2l  w
1000  2l  w
500  l  w
l  500  w.......... .......... .......... .......... .......... ...(i )
A  lw.......... .......... .......... .......... .......... ........( ii)
Substitute equation (ii) into equation (i)
A  500  ww

 500  w  w  500  2w but


dA dA
0
dw dw
0  500  2w
w  250m.......... .......... .......... .......... .......... .....( iii)
Substitute equation (iii) into equation (i)
l  500  250  250
l  250m
dA
From  500  2w
dw
d2A
 2  0 indication of maximum 
dw2
 In order to maximize area the rectangular shape should be a square with length 250m

559
Example 128
Find the value of constant 'a' and 'b' of a point of inflexion on the graph
y  ax 3  bx 2  2 is 1,0 . Hence find the maximum and minimum points of the
graph.
Solution
y  ax3  bx 2  2 inflexion point is 1,0
dy
 3ax 2  2bx
dx
d2y d2y
 6ax  2b at point of inflexion 2  0
dx 2 dx
6ax  2b  0 but x  1
6a  2b  0.......... .......... .......... .......... .......... .......... .......... .....( i )
Also point 1,0 satisfy the given equation y  ax3  bx 2  2
0  ab2
a  b  2.......... .......... .......... .......... .......... .......... .......... .......( ii)
Solve equations (i) and (ii) simulteneously
6a  2b  0

a  b  2
 a  1, b  3

Now, y  x 3  3x 2  2
dy dy
 3 x 2  6 x at stationary point 0
dx dx
3x 2  6 x  0
x  0, x  2
d2y
 6x  6
dx 2
d2y
When x  0, y  2 and  6  0 indication of maximum point 
dx 2
d2y
When x  2, y  2 and  6  0 indication of minimum point 
dx 2
 The maximum point is 0,2 and minimum point is 2,2 

560
Example 129
Find the stationary points of y  4 x 3  3x 4 and determine its nature.
Solution
Consider the curve y  4 x 3  3 x 4
dy dy
 12x 2  12x 3 but at stationary points 0
dx dx
12x 2  12x 3  0
x  0, x  1
When x  0, y  0
When x  1, y  1
The stationary points are 0,0 and 1,1
By 2 nd derivative test;
d2y
2
 24x  36x 2
dx
d2y
When x  0, 0 indicator of inflexion point 
dx 2
d2y
When x  1,  12  0 indicator of maximum point 
dx 2
 0,0  is inflexion point
 1,1 is maximum point

Example 130
Find the values of x at which maximum, minimum values of y and the point of
inflexion occurs on the curve y  12Inx  x 2  10x .
Solution
y  12Inx  x 2  10x
dy 12 dy
  2 x  10 but at stationary points 0
dx x dx
12
 2 x  10  0
x
x 2  5x  6  0
 x  3, x  2

561
(d) Series Expansion
Taylor’s and Maclaurin’s Theorem

Taylor’s Theorem
If f x  is any function and suppose that f x  can be expanded in ascending
power of x then the series of f x  can be formed as follows;
f x   c0  c1 x  a   c2 x  a   c3 x  a   c4 x  a     .......... (i )
2 3 4

Determine all constants by substituting x  a


f a   c0
f x   c1  2c2 x  a   3c3 x  a   4c4 x  a     
2 3

f a   c1
f x   2c2  6c3 x  a   12c4 x  a     
2

f a 
f a   2c2 , c2 
2!
f x   6c3  24c4 x  a     
f a 
f a   6c3 , c3 
3!



Substitute all constants obtained above into equation (i)

f x   f a   x  a  f a  
x  a 2 f a   x  a 3 f a  
x  a 4 f ( 4) a     .......... (ii)
2! 3! 4!
Let x  a  h
h2 h3 h 4 ( 4)
f x   f a  h   f a   hf a   f a   f a   f a     .......... .......... ....( iii)
2! 3! 4!
Taylor’s series/expansion/theorem is given by;
h2 h3 h 4 ( 4)
f x   f a  h   f a   hf a   f a   f a   f a     
2! 3! 4!

Example 131
 
Use Taylor’s series to expand sin  h  in ascending power of h as far as the
6 
4 
term of h and hence find sin 31 into four significant figures.
Solution

562
 
Given that f  x   sin  h .......... .......... .......... .......... .......... .......... .......... ........( i )
6 
h2 h3 h 4 ( 4)
From, f  x   f a  h   f a   hf a   f a   f a   f a     .........( ii)
2! 3! 4!
 
Equate  h  a  h, a 
6 6
Also x  a  h

x h
6

In Taylor' s theorem x  a 
6
  1
f  x   sin x f a   sin  
6 2
 
f x   cos x f a   cos  
3
6 2
 
f x    sin x f a    sin   
1
6 2
 
f x    cos x f a    cos   
3
6 2
  1
f ( 4 ) x   sin x f ( 4 ) a   sin  
6 2
Substitute the above values into Taylor' s series
h2 h3 h 4 ( 4)
f  x   f a  h   f a   hf a  
f a   f a   f a     
2! 3! 4!
  1
f  x   sin  h   
3 1 3 3 1 4
h  h2  h  h  
6  2 2 4 12 48
  1 3 1 3 3 1 4
 sin  h    h  h2  h  h  
6  2 2 4 12 48
Hence evaluate

sin 31  sin30  1   sin  1  but h  1  0.01745

6 
  1
sin 31  sin  0.01745  
3
0.01745  1 0.017452  3 0.017453  1 0.017454
6  2 2 4 12 48
 sin 31  0.5150

563
Example 132
Obtain the Taylor’s expansion of the function x 2 Inx in ascending powers of
x  1 as far as the term in x  14 .
Solution
Given that f x   x 2 Inx.......... .......... .......... .......... .......... .......... .......... .......... ....( i )

From, f x   f a   x  a  f a  
x  a 2 f a  
x  a 3 f a  
x  a 4 f ( 4) a     
2! 3! 4!
Required a seriesin ascending power of x  1 which imply that " a  1"
In Taylor' s theorem x  a  1
f  x   x 2 Inx f a   x 2 Inx  0
f x   x  2 xInx f a   x  2 xInx  1
f x   3  2 Inx f a   3  2 Inx  3

f x   f a    2
2 2
x x
2
f ( 4) x   2 f ( 4 ) a   2
x
Substitute the above values into Taylor' s series

f  x   f a    x  a  f a  
 x  a
2
f a  
 x  a
3
f a  
 x  a
4
f ( 4 ) a     
2! 3! 4!
 x 2 Inx   x  1  x  12  1 x  13  1 x  14    
3
2 3 12

Example 133
Apply Taylor’s theorem to expand Inx in ascending power of x  e as far as
the term in x  e
4

Solution

564
Substitute the above values into Taylor' s series

f  x   f a    x  a  f a  
 x  a
2
f a  
 x  a
3
f a  
 x  a
4
f ( 4 ) a     
2! 3! 4!
 x e 1 x e 1 x e 1 x e
2 3 4

 Inx  1             
 e  2 e  3 e  4 e 

Maclaurin’s theorem
Consider f x  where a  0
f  x   c0  c1 x  a   c2 x  a   c3 x  a   c4 x  a      but a  0
2 3 4

f  x   c0  c1 x  c2 x 2  c3 x 3  c4 x 4    .......... .......... .......... .......... .......... ......( i )


Determine all constants by substituting x  a
f 0  c0
f x   c1  2c2 x  3c3 x 2  4c4 x 3    
f 0  c1
f x   2c2  6c3 x  12c4 x 2    
f 0
f 0  2c2 , c2 
2!
f  x   6c3  24c4 x    
f 0
f 0  6c3 , c3 
3!



Substitute all constants obtained above into equation (i)
x2 x3 x 4 ( 4)
f  x   f 0  xf 0  f 0  f 0  f 0    .......... .......... ...(ii)
2! 3! 4!
x2 x3 x 4 ( 4)
f  x   f 0   xf 0   f 0   f 0  f 0    
2! 3! 4!

565
Example 134
Find the series of the function f x   e x
Solution
Given that f x   e x
f x   e x f 0  e 0  1
f  x   e x f 0  e 0  1
f  x   e x f 0  e 0  1
f  x   e x f 0  e 0  1
f ( 4) x   e x f ( 4 ) 0  e 0  1
x2 x3 x 4 ( 4)
From f x   f 0  f 0  f 0  f 0  f 0    
x
1! 2! 3! 4!
x x 2 x3 x 4 x5
e  1       ...
x

1! 2! 3! 4! 5!

Example 135
Find the series of the function f x   e  x
Solution
Given that f  x   e  x
f x   e  x f 0  e 0  1
f  x   e  x f 0  e 0  1
f  x   e  x f 0  e 0  1
f  x   e  x f 0  e 0  1
f ( 4) x   e  x f ( 4 ) 0  e 0  1
x2 x3 x 4 ( 4)
From f  x   f 0  xf 0  f 0  f 0  f 0    
2! 3! 4!
x x x 2 x3 x 4 x5
e  1       ...
1! 2! 3! 4! 5!

566
Example 136
Find the series of the function f x   In1  x 
Solution
Given that f  x   In 1  x 
f x   In 1  x  f 0  0

f  x   f 0  1
1
1 x
1
f  x   f 0  1
1  x 2
f  x   f 0  2
2
1  x 3

6
f ( 4) x   f ( 4 ) 0  6
1  x  3

x2 x3 x 4 ( 4)
From f  x   f 0  xf 0  f 0 
 f 0 
 f 0    
2! 3! 4!
x 2 x3 x 4
In 1  x   x     
2 3 4

Example 137
Find the series of the function f x   In1  x 
Solution
Given that f  x   In 1  x 
f x   In 1  x  f 0  0
1
f  x   f 0  1
1 x
1
f  x   f 0  1
1  x  2

2
f  x   f 0  2
1  x 3
6
f ( 4) x   f ( 4 ) 0  6
1  x 3

x2 x3 x 4 ( 4)
From f  x   f 0  xf 0  f 0  f 0  f 0    
2! 3! 4!
x 2 x3 x 4
In 1  x    x       
2 3 4

567
Example 138
1 x   
If x  1 show that In
x3 x5
  2 x       
1 x   3 5 
Solution
1 x 
Given that f  x   In 
 1 x 
1 x 
f x   In  f 0  0
 1  x 
f  x   f 0  2
2
1 x2
f  x   f 0  0
4x
1  x 
2 2

4  8 x 2  12x 4
f  x   f 0  4
1  x 
2 4

x2 x3 x 4 ( 4)
From f  x   f 0  xf 0  f 0  f 0  f 0    
2! 3! 4!
1 x  2x3
In   2x   
 1 x  3
1 x   x3 x5 
 In   2 x       

1 x   3 5 

Alternative
1 x 
Consider In   In 1  x   In 1  x .......... .......... .......... .......... .......... .......( i )
 1 x 
x 2 x3 x 4 x5 x6
But In 1  x   x         .......... .......... .......... .......... ........( ii)
2 3 4 5 6
x 2 x3 x 4 x5 x6
In 1  x    x         .......... .......... .......... .......... ......( iii)
2 3 4 5 6
1 x     
2
x x3 x 4 x5 x6 x 2 x3 x 4 x5 x6
In    x              x          
1 x   2 3 4 5 6   2 3 4 5 6 
1 x   x3 x5 
 In 
 2 x       
 1 x   3 5 

568
Example 139
 x  1 1 1 1 
If x  1 show that In   2  3  5  .......  and hence compute
 x 1  x 3x 5x 
the value of In 2
Solution
 x 1  1  1x 
Consider In   In 1   In 1  1x   In 1  1x .......... .......... .......... .......... ........( i )
 x 1  1 x 
But In 1  1x  
1 1 1 1 1
 2  3  4  5    .......... .......... .......... .......... .......... ......( ii)
x 2 x 3x 4 x 5 x
In 1  1x     2  3  4  5      .......... .......... .......... .......... .......... .(iii)
1 1 1 1 1
x 2 x 3x 4 x 5 x
 x 1  1 1 1 1 1   1 1 1 1 1 
In     2  3  4  5         2  3  4  5     
 x  1   x 2 x 3x 4 x 5 x   x 2 x 3x 4 x 5 x 
 x 1  1 1 1 
 In   2  3 5     
 x  1   x 3x 5 x 
Hence,
Evaluate In 2
x 1
Equate 2
x 1
x 1  2x  2
x3
 x 1  1 1 1 
From In   2  3 5     
 x  1   x 3x 5 x 
 3  1   1 1 1 

In   2      
 3  1   3 33 53
3 5

 In 2  0.6930

GENERAL EXAMPLES

Example 140
If y  tan k tan 1 x , prove that 1  x 2   
dy
 k 1 y2 .
dx
Solution

569
 
Consider y  tan k tan 1 x .......... .......... .......... .....( i )
du k
Let u  k tan 1 x,  .......... .......... .......( ii)
dx 1  x 2
dy
y  tan u,  sec2 u  1  tan 2 u.......... .......... ....( iii)
du
dy dy du
From,  
dx du dx
dy
  k 
 1  tan 2 u  2 
but u  k tan 1 x
dx 1 x 
dy
   k 
 1  tan 2 k tan 1 x  
2 
but y  tan k tan 1 x 
dx 1 x 
dy

 1 y2  
 k 
2 
dx 1 x 

 1 x2
dy
dx
 k 1 y2  

Example 141
1  sin x dy 1
Given that y  show that 
1  sin x dx 1  sin x
Solution
1  sin x
Consider y  rationalize the numerator
1  sin x
1  sin x 1  sin x 1  sin 2 x
2
cos2 x  cos x  cos x
y       
1  sin x 1  sin x 1  sin x 2 1  sin x 2  1  sin x  1  sin x
cos x
y
1  sin x
dy 1  sin x  sin x   cos x cos x 

dx 1  sin x 2
dy cos2 x  sin 2 x  sin x 1  sin x 1
  
dx 1  sin x 2 1  sin x 2 1  sin x
dy 1
 
dx 1  sin x

570
Alternative
1  sin x
Consider y  rationalize the denominator 
1  sin x

y
1  sin x 1  sin x
 
1  sin x 2 
1  sin x 2  1  sin x 
 
2
1  sin x
 
1  sin x 1  sin x 1  sin 2 x cos2 x  cos x  cos x
1  sin x
y
cos x
dy cos xcos x   1  sin x ( sin x) cos2 x  sin 2 x  sin x 1  sin x
  
dx cos2 x cos2 x cos2 x
dy 1  sin x 1  sin x 1  sin x 1
   
dx cos2 x 1  sin 2 x 1  sin x 1  sin x  1  sin x
dy 1
 
dx 1  sin x

Alternative
1  sin x
Consider y  .......... .......... .......... .......... .......... ..(i )
1  sin x
1  sin x du 2 cos x
Let u  ,  .......... .......... .......... .....( ii)
1  sin x dx 1  sin x 2
dy 1 1  sin x
y  u,  but u 
du 2 u 1  sin x
dy 1  sin x
 .......... .......... .......... .......... .......... .......... ..(iii)
du 2 1  sin x
dy dy du
From chain rule,  
dx du dx
dy 1  sin x 2 cos x 1  sin x 1  sin 2 x
   
dx 2 1  sin x 1  sin x 2 1  sin x 1  sin x 
2

dy 1  sin x 1  sin x 1  sin x 1  sin x 1


   
dx 1  sin x 1  sin x 2 1  sin x 2 1  sin x
dy 1
 
dx 1  sin x

571
Example 142
If y  sine x  k  where k is constant, show that d 2y  dy  ye 2 x  0
2

dx dx
Solution
 
Consider y  sin e x  k .......... .......... .......... .......( i )
dy
dx
 
 e x cos e x  k .......... .......... .......... .......... .....( ii)

d2y
dx 2
   
 e x cos e x  k  e 2 x sin e x  k .......... .......... (iii)
Substitute equations (i) and (ii) into equation (iii)
d 2 y dy
  ye2 x
dx 2 dx
d 2 y dy
 2   ye2 x  0
dx dx

Example 143
dy cos2 a  y 
If cos y  x cosa  y  , show that 
dx sin a
Solution
Consider cos y  x cosa  y .......... .......... .......... .......... .......... .......... .......( i )

 sin y  cosa  y   x sina  y 


dy dy
dx dx
dy  cosa  y  
  .......... .......... .......... .......... .......... .......... .......... ..(ii)
dx  x sina  y   sin y 
Make x the subject from equation (i) then substitute into equation (ii)
dy  cosa  y   cos y
   but x 
dx  x sina  y   sin y  cosa  y 
 
 
dy  cosa  y   dy  cos2 a  y  

   dx  sina  y cos y  cosa  y sin y 
  
dx  
  cosa  y   sina  y   sin y 
 cos y 
  
dy cos2 a  y  cos2 a  y 
 
dx sina  y  y  sin a
dy cos2 a  y 
 
dx sin a

572
Example 144
d2y b
If x  a cos and y  b sin  , prove that   2  cosec 3
a 
2
dx
Solution
dx
x  a cos ,  a sin .......... .......... .......... .......... .......... ........( i )
d
dy
y  b sin ,  b cos .......... .......... .......... .......... .......... .......... (ii)
d
dy dy d b cos
From   
dx d dx  a sin
dy b
  cot .......... .......... .......... .......... .......... .......... .......... .....( iii)
dx a
d2y d  dy  d d  b  1
From       cot  
dx 2
d  dx  dx d  a   a sin
d2y b  1  b
  cosec 2     2 cos ec 
3

a   a sin 
2
dx a
d2y b
 2   2 cos ec3
dx a

Example 145
x 1 dy  2x 2
If y   tan x , then show that 
1 x2 dx 1  x 2 2  
Solution
x
Consider y   tan 1 x
1 x 2

dy 1  x   x2 x 
2
1
 
dx 1  x 2 2
1 x2
dy 1 x2 1
 

dx 1  x 2 2
1 x2
dy 1  x 2  1  x 2

 
dx 1 x2
2
 
dy  2x2
 

dx 1  x 2 2 

573
Example 146
d2y
If y  A cosh(nx)  B sinh( nx) , show that 2
 n2 y .
dx
Solution
Consider y  A coshnx  B sinhnx

 nAsinhnx  nB coshnx
dy
dx
d2y
2
 n 2 A coshnx  n 2 B sinhnx
dx
d2y
2
 n 2 A coshnx  B sinhnx
dx
d2y
 2  n2 y
dx

Example 147
If x y  x  y 
p q pq dy y
. Prove that 
dx x
Solution
Consider x p y q  x  y 
pq

Apply natural logarithm both sides


In x p y q   In x  y 
pq

Inx p  Iny q   p  q In  x  y 


pInx  qIny   p  q In x  y 
p q dy  p  q  1  dy
  dx  
x y dx x y
py  qx dy
dx

 
p  q  1  dy
dx 
xy x y

x  y  py  qx dy   xy p  q 1  dy 


 dx    dx 

x  y  py  x  y qx dy   p  q xy   p  q xy dy


dx dx
x  y qx   p  q xy dy   p  q xy  x  y  py
dx

574
dy  p  q xy  x  y  py pxy  qxy  pxy  py 2 qxy  py 2
  
dx x  y qx   p  q xy qx 2  qxy  pxy  qxy qx 2  pxy
dy qxy  py 2 y qx  py y
  
dx qx 2  pxy xqx  py  x
dy y
 
dx x

Example 148

Given that y  e
tan 1 x

show that 1  x
d2y
2
2

 2 x  1  0
dy
dx
dx
Solution
1
Consider y  e tan x
Apply In both sides
1
Iny  Ine tan x

Iny  tan 1 x
1 dy 1

y dx 1  x 2
dy y tan 1 x
 but y  e
dx 1  x 2
1
dy e tan x

dx 1  x 2
By quotient rule
1

 
tan x
2 e
 e tan x 2 x 
1
2 1 x
d y
 1 x 2

dx 2
1 x2
2
 
1

 
2
e tan x
1 x 2 d y
 1  2 x 
dx 2 1 x2  

1 x
d2y
dx
2
2
 1  2 x 
dy
dx

 
2
 2 x  1  0
d y dy
 1 x2 2
dx dx

575
Example 149
dy x
If x 2  y 2  2 y 1  x 2  0 , show that; 
dx 1 x2
Solution
x2  y2  2 y 1 x2  0
dy dy 2 xy
2x  2 y  2 1  x 2  0
dx dx 1 x 2

y  1 x2
dy
dx
  xy
1 x2
x

dy xy  x 1  x 2 dy x y  1 x2  
dx

1 x2 y  1 x2

dx


1 x2 y  1 x2  
dy x
 
dx 1 x2

Example 150
d 2 y 1  sin 
Given y    cos and x  sin . Show that 
dx 2 cos3 
Solution
dy
y    cos ,  1  sin .......... .......... .......... .......... ..(i )
d
dx
x  sin ,  cos .......... .......... .......... .......... .......... .(ii)
d
dy dy d
From  
dx d dx
dy 1  sin
 .......... .......... .......... .......... .......... .......... ....( iii)
dx cos
d 2 y d  dy  d
Now,   
dx 2 d  dx  dx
d 2 y d  1  sin  1  cos2   sin 2   sin  1
     
dx 2 d  cos  cos  cos2   cos
d 2 y 1  sin
 2 
dx cos3 

576
Example 151
dy dy
Given that x  e u where u is a functions of x , prove that x  .
dx du
Solution
Consider x  e u
Differentiate w.r.t y
dx du
 eu
dy dy
dy 1 dy

dx e u du
dy dy
eu  but e u  x
dx du
dy dy
x 
dx du
Example 152
If x is so small that x 3 and higher powers may be neglected, obtain a quadratic
1 ex
approximation of f ( x)  .
1  In (1  x)
Solution
1 ex
Consider f ( x) 
1  In (1  x)
1 ex
f ( x) 
1  In (1  x)
 
 1  e x 1  In (1  x)
1

x2 x2
But e x  1  x   ... and In (1  x)  x   ...
2! 2
1
  
 x

f ( x)  1  e 1  In (1  x)
1
 1  1  x 
x2
 ...  1  x 
x2
 ...
 2!  2 
1
 x2   x 2 
f ( x)   2  x   ...  1   x  
 2!   2 

  x2  
2
 x2 x 2  (1)(2) 
f ( x)   2  x   ...  1   x     x   ....
 2!    2  2!  2  

 x2  3 
f ( x)   2  x   ...  1  x  x 2 ...
 2!  2 
5 2
 f ( x)  2  x  x
2

577
Example 153
 x  y
If P  sin 1    tan 1   , find all partial derivatives of P up to the second
 y  x
derivative
Solution
x  y
P  sin 1    tan 1  
 y x
1 y 1 y2
Px   2
y 1  xy
2
x 1  xy     2
 2 2
y 2  x 2 x y  x 
2
.......... .......... .......... .......... .(i )

x 2 y 2 y 2  2 x 2 
Pxx  
 
.......... .......... .......... .......... .......... .......... .......... (ii)
x y  x 
3 3 2 2 2
y x2 2

y 2y
Pxy  
 
.......... .......... .......... .......... .......... .......... .......... .....( iii)
y x2 2
3
y 2
x 2 2

x 1 x y
Py    
      y y x xy  x 2 
.......... .......... .......... .......( iv)
x 1  xy
2 2
y2 1 x 2 2 2
y

x y  x   xy
2
x  xy
2 2 3 2
 
y  y x 
Pyy .......... .......... .......... .......... .......... .......... .....( v)
2 2 x  y  x 
2
3 2 2 2 2

y 2y
Pyx  
 
.......... .......... .......... .......... .......... .......... .......... ...( vi)
y x2 2
3
y 2
 x2 
2

Example 154
(a) Using Maclaurin’s expansion and differentiation, show that
 x2 x4
In (cos x)    ...
2 12
2 x 
(b) Using cos x  2 cos    1 and results in (a) show that
2
x2 x4
In (1  cos x)  In 2    ...
4 96
Solution

578
a  Consider f x   In cos x 
f x   In cos x  f 0  0
f  x    tan x f 0  0
f  x    sec2 x f 0  1
f  x   2 sec2 x tan x f 0  0
f ( 4 ) x   2 sec4 x  4 sec2 x tan 2 x f ( 4 ) 0  2
x2 x3 x 4 ( 4)
From f  x   f 0  xf 0  f 0  f 0  f 0    
2! 3! 4!
x2 x4
 In cos x        
2 12

b  Consider f x   In 1  cos x  but cos x  2 cos2  2x   1


f x   In 1  2 cos2  2x   1  In2 cos2  2x   In 2  2 Incos 2x 
 x2 x4 
f x   In 2  2     
 8 192 
x2 x4
 In 1  cos x   In 2      
4 96

Example 155
At a given instant the radii of the two concentric circles are 8cm and 12cm . The
radius of the outer circle is increasing at a rate of 1cms 1 and that of the
inner circle is increasing at 2cms 1 . Find the rate of change of an area enclosed
between two circles.
Solution
Consider concentric circles below;

579
dR dr
Given that R  12cm, r  8cm,  1cm / s and  2cm / s
dt dt
Area enclosed( A)  R 2  r 2   R 2  r 2  

A   R2  r 2 
dA  dR dr 
 2  R r 
dt  dt dt 

 2 12(1)  8(2)  8cm 2 / s


dA
dt
 The rate of decreasing of area between the circlesis 8cm 2 / s

Example 156
Determine the area of largest rectangle that can be inscribed in a circle of radius
1.
Solution
Consider the figure below;

Apply Pythagoras theorem


L2  w 2  d 2 but d  2 R  2
L2  w 2  4
w  4  L2 .......... .......... .......... .......... .........( i )
Area  A  Lw.......... .......... .......... .......... .....( ii)
Substitute equation (i) into equation (ii)
A  L 4  L2
dA L2 dA
 4  L2  but 0
dL 4  L2 dL
L2
4  L2  0
4  L2
L2  2
L 2
From w  4  L2  4  2  2
 The rectangular shape is largest when it is a square with length 2 units

580
Example 157
A cylinder of volume Vcm 3 is inscribed in a cone of radius 5cm and height of
15cm . Show that the maximum volume of the cylinder that can be inscribed in
500
the cone is V  cm 3 .
9
Solution
Consider the figure below;

Apply similarity concept


15  h r

15 5
h  15  3r.......... .......... .......... .......... .......... .......... ..(i )
1
V  r 2 h.......... .......... .......... .......... .......... .......... .(ii)
3
Substitute equation (i) into equation (ii)

V  r 2 15  3r   r 2 5  r 
1
3
 2r 5  r   r 2 but
dV dV
0
dr dr
2r 5  r   r 2  0
3r 2  10r  0
r  0, r  103
In order to obtain max. volume r  103 and h  15  3r  5
500
2
1  10 
Now, V  r 2 h     5 
1
cm 3
3 3 3 9
500
 The maximum volume is V  cm 3
9

581
Example 158
dy 1
If x 1  y  y 1  x  0 , prove that 
dx 1  x 2
Solution
Consider x 1  y  y 1  x  0
x 1 y   y 1 x
Square both sides
x 2 1  y   y 2 1  x 
x 2  x 2 y  y 2  xy 2
x 2  y 2  xy 2  x 2 y
x  y x  y   xy y  x 
x  y x  y    xyx  y 
x  y   xy
y  xy   x
y 1  x    x
x
y
1 x
dy 1
 
dx 1  x 2

Example 159
f f
If f x, y   x 3 y  e xy , find
2
and .
x y
Solution
Given that f x, y   x 3 y  e xy
2

f
  3x 2 y  y 2 e xy
2

x
f
  x 3  2 xyexy
2

y

582
Example 160
 y 2z
If z  x 2 tan 1   , find at 1.1 .
 
x xy
Solution
 y
Given that z  x 2 tan 1  
x
z x3

y x 2  y 2
z

 
3x 2 x 2  y 2  x 3 (2 x)
xy 
x2  y2
2

z x 4  3x 2 y 2

xy x2  y2
2

At ( x, y )  (1,1)
z
 1
xy

Example 161
n
 1 x2 
If y   
2 
, show that  1  x 4 dy
 4nxy  0 .
 1  x  dx
Solution
n 1
dy  1  x 2    4 x 
 n 
  1  x 2  
2 
dx  1  x 2

1
 1  x 2   1  x 2    4 x 
n
dy
 n   
 1  x   1  x   1  x 2  
2   2  2
dx

 1  x 2   1  x 2   4 x 
n
dy
 n   
 1  x 2  
2   2 
dx  1  x   1  x 2

n
dy  1  x 2   1   4 x 
 n 
2   2  2 
dx  1  x   1  x  1  x 
dy   4x 
 ny 4 
dx 1 x 

 1 x4
dy
dx
 4nxy  0

583
Example 162
If the minimum value of f x   2 x 3  3x 2  12x  k is one-tenth of its
maximum value. Find the value of k .
Solution
f  x   2 x 3  3 x 2  12x  k .......... .......... ...(i )
f  x   6 x 2  6 x  12.......... .......... .......... .(ii)
f  x   12x  6.......... .......... .......... .........( iii)
At stationary point f x   0
6 x 2  6 x  12  0
x  1, x  2
By second derivative test, f  x   12x  6
f 1  12(1)  6  18  0 (indication of minimum)
f  2   12(2)  6  18  0 (indication of maximum)
Minimum value f 1  2(1) 3  3(1) 2  12(1)  k  7  k
Maximum value f  2  2(2) 3  3(2) 2  12(2)  k  20  k

Minimum value  Maximum value 


1
10
 7  k  20  k , 9k  90
1
10
 k  10

REVISION EXERCISE 9
1. Differentiate the following by first principle;
sin x 1
(a). f ( x )  (b) f ( x)  (c) f ( x)  Inx
x x
, prove that 1  x 2   
dy
2. (a). If 
y  tan k tan 1 x  k 1 y2 .
dx
1  sin x dy 1
(b). Given that y  show that 
1  sin x dx 1  sin x
(c) Show that the volume of the greatest cylinder which can be inscribed in
a cone of height “h” and semi vertical angle  is given by h 3 tan 2  .
4
27

584
3. (a). If y  sine x  k  where k is constant, show that
d 2 y dy
2
  ye 2 x  0
dx dx
d2y
(b). If y  A cosh(nx)  B sinh( nx) , show that 2
 n2 y .
dx
(c)Find the first three terms of the Maclaurian series for In2  x  , and
2 x
show that, if x is very small, then In  x.
 2  x 
dy cos2 a  y 
4. (a). If cos y  x cosa  y  , show that 
dx sin a
dy sec2 x
(b) If y  tan x  tan x  tan x..... then prove that  .
dx 2 y  1
(c) Water is dripping out from the conical funnel at a uniform rate
4 cm 3 sec through the tiny hole at the vertex in the bottom when the
slant height of the water is 3cm , find the rate of decrease of the slant
height of the water cone. Given that the vertical angle of the funnel is
120
5. (a). The period T of a simple pendulum is calculated by using the
l
formula T  2 where l is length of the pendulum and g is
g
constant. Find the percentage in the period if the pendulum is
lengthened by 2% .
(b). Suppose that we have two resistors connected in parallel with
resistances R1 and R2 measured in ohms ( Ω ). The total resistance, R,
then given by, 1  1  1 Suppose that R1 is increasing at a rate of
R R1 R2
0.4  min and R2 is decreasing at a rate of 0.7  min . At what rate is R
changing when R1  80 and R2  105 ?
6. (a). Find all the second order partial derivative for
f x, y   3 y 2  x 2 e 5 y  cos 2 x .
(b). An open box has a square horizontal cross-section. If the box is to
hold a given amount of material and the internal surface area is to be
minimum. Show that the ratio of the height to the length of the sides is
given by 1 : 2
(c). An inverted right circular cone of semi-vertical angle 45 is
collecting water from a tap at a steady of rate 18cm 3 / s . Find the rate at
which the depth h of water is rising when h  3cm
585
7. (a). Obtain the Taylor’s expansion of the function x 2 Inx in ascending
powers of x  1 as far as the term in x  14 .
(b). Apply Taylor’s theorem to expand Inx in ascending power of x  e
as far as the term in x  e4

(c). A 15m ladder is resting against the wall. The bottom is initially 10m
away from the wall and is being pushed towards the wall at a rate of
0.25 m sec . How fast is the top of the ladder moving up the wall 12
seconds after we start pushing?
d2y b
8. (a). If x  a cos and y  b sin  , prove that   2  cosec 3
a 
2
dx
(b). A wire of length l units is to be cut into two pieces. One piece is to be
bent to form a circle and the other piece is to be bent to form a square. If
the sum of the areas of the circle and the square is to be maximum, what
length of the wire should be used to form a circle?
p q

9. (a). If x y  x  y 
pq
. Prove that
dy y

dx x
(b). The profit X(T.Shs) on the sale of a certain car and the time t(hours) of
its manufacturer are related by the formula X  20 200  250  t 2  ,
 t 
where t  3 . Find the maximum possible profit.
x 1 dy  2x 2
10. (a). If y   tan x , then show that 
1 x2 dx 1  x 2 2

(b) Two people are 50m apart. One of them starts walking north at a rate so
that the angle is changing at a constant rate of 0.01 rad/min. At what rate is
distance between the two people changing when θ = 0.5 radians?

11. (a). An error of 2% is made in measuring the radius of a sphere. What is


the resulting percentage error in the calculation of its surface area?
(b) A tank of water in the shape of a cone is leaking water at a constant rate
of 2 m 3 hour . The base radius of the tank is 5m and the height of the tank is
14m .
(i) At what rate is the depth of water in the tank changing when the depth of
water is 6m ?
(ii) At what rate is the radius of the top of water in the tank changing when
the depth of water is 6m ?

586
12. (a). Given that y  e
tan 1 x

show that 1  x2d2y

2
 2 x  1  0
dy
dx
dx
(b) A trough of water is 8m deep and its ends are in the shape of isosceles
triangles whose width is 5m and height is 2m. If water is being pumped in
at a constant rate of 6 m 3 sec . At what rate is the height of water changing
when water has a height of 120 cm?
dy x
13. (a). If x  y  2 y 1  x  0 , show that; 
2 2 2

dx 1 x2

(b). A light is on the top of a 12m tall pole and a 5.5m tall person is
walking away from the pole at a rate of 2 m/sec.
(i) At what rate is the tip of the shadow moving away from the pole when
the person is 25m from the pole?
(ii) At what rate is the tip of the shadow moving away from the person
when the person is 25m from the pole?
d 2 y 1  sin 
14. (a). Given y    cos and x  sin . Show that 2 
dx cos3 
(b). A sport light is on the ground 20m away from the wall and a 6m tall
person is walking towards the wall at a rate of 2.5 m sec . How fast the
height of the shadow changing when the person is 8m from the wall? Is the
shadow increasing or decreasing in height at this time?

15. (a). A piece of wire which forms the circumference of a circle of radius
of 0.12m is cut and bent so as to form two new circles. Find the radius
of each circle in order that the sum of their areas of the two circles shall
be as small as possible.
(b). An isosceles triangle of vertical angle 2 is inscribed in a circle of
radius a .
Show that the area A of the triangle is given by A  4a 2 sin  cos3 
and hence the area is maximum when the triangle is equilateral.

16. (a). Find the dimensions of a rectangle with perimeter 1000m so that the
area of the rectangle is a maximum.
(b). A sealed cylindrical jam tin is of height hcm and radius rcm . The
area of its total outer surface is Acm 2 and its volume is Vcm 3 .Find the
expression for A in term of r and h .
(i). Taking A  24 find an expression for h in term of r and hence
expression for V in term of r .
(ii). The value of r which will make V maximum.

587
17. (a).If R  ar n where a is a constant and an error of x% is made in
measuring r , prove that the resulting error in R is nx%
(b). By using Maclaurian series expansion of a x and b x where b  0 and

x is very small, show that  a  b  = In   .


x x a
 x  b
 
18. (a). Use Taylor’s series to expand sin  h  in ascending power of
6 
h as far as the term of h 4 and hence find sin 31 into four significant
figures.
 
(b). Use Taylor’s series to expand sin  h  in ascending power of
3 
h as far as the term of h 4 and hence find sin 61 into four significant
figures
 
(c). Use Taylor’s series to expand cos  h  in ascending power of
6 
h as far as the term of h 4 and hence find cos31 into four significant
figures
(d).Use Taylor’s series to expand the series of cosx  30  and hence
find the value of cos33

19. (a).The lengths of the sides of a rectangular sheet of metal are 8cm and
3cm . A square of side x is cut from each corner of the sheet and the
remaining piece is folded to make an open box.
(i). Show that the volume V of the box is given by
V  4 x 3  22 x 2  24 x cm 3
(ii). Find the value of x for which the volume of the box is maximum
and calculate this maximum volume.
(b). At a certain instant the volume of a cube is increasing at the rate of
3 cm 3 s while the area of each face is increasing at the rate of 0.2 cm 2 s .
Find the rate of increasing of the length of an edge of the cube at this
instant.
20. A square sheet of cardboard with each side a cm to be used to make
an open-top box by cutting a small square of cardboard from each of the
corners and bending up the sides. What is the side length of the small
squares if the box is to have as large a volume as possible?
21. (a). A hollow cone has a base radius R and height H . What is the
volume of the largest cylinder that can be placed in it?

588
(b). A farmer 8 km of fencing wire, and wishes to fence a rectangular
piece of land. One boundary of the land is the bank of a straight river.
What are the dimensions of the rectangle so that the area is maximized?
22. (a). A stone is dropped into a quite lake and waves move in a circle at a
speed of 3.5 cm sec . At the instant, when the radius of the circular wave
is 7.5cm , how fast is the enclosed area increasing?
(b). The volume of a cube is increasing at a constant rate. Prove that the
increase in surface area varies inversely as the length of the edge of the
cube.
23. A hemispherical bowl is being filled with water at a uniform rate. When
 1 
the height of water is hcm the volume is   rh  h cm , rcm being
2 3 3

 3 
the radius of the hemisphere. Find the rate at which the water level is
rising when it is half way to the top, given that r  6cm and that the
bowl fills in 1 min .
24. (a). A ladder 15m long resting against a vertical wall. Its top slides
down the wall while its bottom moves away along the level ground at a
speed of 2 m s . How fast is the angle between the top of the ladder and
the wall changing when the angle is  3 radians?
(b). Find the values of x at which maximum, minimum values of y and
the point of inflexion occurs on the curve y  12Inx  x 2  10x .
(c). Find the value of constant 'a' and 'b' of a point of inflexion on the
graph y  ax 3  bx 2  2 is 1,0 . Hence find the maximum and
minimum points of the graph.

(d) Determine point(s) on y  x 2  1 that are closest to (0,2) .


(e) Find the coordinate of the minimum point of the catenary curve
x
given by the equation y  a cosh 
a
(f) Find the slope of the curve 2 x 2  3xy  y 2  x  2 y  8  0 at the
point (1,3) .
25. (a). A boy starts walking north at a speed of 1.5 m/s, and a girl starts
walking west at the same point P at the same time at a speed of 2 m/s. At
what rate is the distance between the boy and the girl increasing 6 seconds
later?
(b). A police car, approaching right-angled intersection from the north, is
chasing a speeding SUV that has turned the corner and is now moving
straight east. When the police car is 0.6 km north of intersection and the
SUV is 0.8 km east of intersection, the police determine with radar that the

589
distance between them and the SUV is increasing at 20 km/hr. If the police
car is moving at 60 km/hr at the instant of measurement, what is the speed
of the SUV?
(c). Prove that the maximum volume of a cylinder obtained from a sphere
4a 3
of radius "a" is given by
3 3
26. A rocket R is launched vertically and its tracked from a radar station S
which is 4 km away from the launch site at the same height above sea
level. At a certain instant after launch, R is 5 km away from S and the
distance from R to S is increasing at a rate of 3600 km per hour. Compute
the vertical speed v of the rocket at this instant.
27. (a). If the absolute magnitude of x is greater than a unit, show that
 x  1  1 1 1 
In   2  3  5  .......  and hence compute the value of
 x  1   x 3x 5 x 
In 2
x y u u
(b). If u  arcsin  arctan show that x  y  0
y x x y
28. (a) A wire of length 36cm is cut into two pieces. One piece is bent into a
circle and the other into the Square. Find the length of each piece so
that the combined area is minimum.
(b) Show that, of all rectangles with given perimeter, square has the largest
area.
(c) Find two positive numbers whose sum is 60 and the product of the first
number with cube of second number is maximum.
(d) A circular cylinder open at the top is to be made so as to have a
volume of 1m 3 . If rm is the radius of the base. Prove that the total
 2
outside surface area is  r  m hence prove that this surface area
2 2

 r
is minimum when the height is equal to the radius of the base
29. (a). Find the equation of tangent and normal to the curve x 2  y 2  25 at
point (3,4) .
(b). The volume of a gas expanding adiabatically is related to its pressure
by the law PV   k where k and  are constants. If dp and dv are small
changes in pressure(p) and volume(v) respectively. Show that;
dp dv
i.  
p v
ii. If the volume is increased by 0.3% , estimate the percentage
increase in pressure where by  =1.4

590
(c). A variable rectangular has a fixed area 36cm 2 .

Find the dimension of rectangle for a minimum perimeter.


30. (a) Show that the semi-vertical angle of a cone of given slant height and
maximum volume is tan 1 2 .
(b) Show that the volume of the largest cone that can be inscribed in a
8
given sphere is times the volume of sphere.
27
(c) Prove that the radius of the cylinder with the greatest curved surface
area that can be inscribed in a given cone is half of that of the cone.
31. (a) An open box with a square base is to be made out of a given quantity of
card board of area c 2 square units. Show that the maximum volume of the
c3
box is cubic units.
6 3
(b) A curve of fourth degree passes through (0,4) has got a tangent line
parallel to x  axis at x  1 and x  2 . If it is parallel to y  2 x  3 at
(0,1) and has inflexion point at (3,0) . Find the equation of a curve.
x2 y2 dy
(c) Given the curve 2  2  1 find at a point ( a cos , b sin ) , hence
a b dx
find the equation of tangent and normal at the point.
32. (a) If the sum of lengths of the hypotenuse and a side of a right angled
triangle is given, show that the area is maximum when the angle is between

them is .
3
(b) A window is in the form of a rectangle surmounted by a semi-circle. If
the perimeter of the window is 10m , find its width so as to admit maximum
light through it.
(c) Show that the height of the closed cylinder of given surface area and
maximum volume is equal to the diameter of its base.
(d) Show that the height of cylinder with maximum volume that can be
2R
inscribed in a given sphere of radius R is .
3
33. By using the concepts of differentiation prove that;

x x2 x3 x4 x5
(a) e  1       ...
x

1! 2! 3! 4! 5!

591
x x x 2 x3 x4 x5
(b) e  1       ...
1! 2! 3! 4! 5!

n(n  1) x 2 n(n  1)(n  3) x 3 n(n  1)(n  2)(n  3) x 4


(c) 1  x   1  nx     ...
n

2! 3! 4!

x 3 x 5 x 7 x 9 x11
(d) sin x  x       ...
3! 5! 7! 9! 11!

x 2 x 4 x 6 x 8 x10
(e) cos x  1       ...
2! 4! 6! 8! 10!

x3 2x5
(f) tan x  x    ...
3 15

x2 x3 x4 x5 x6
(g) In (1  x)  x       ...
2 3 4 5 6

x 2 x3 x 4 x5 x6
(h) In (1  x)   x       ...
2 3 4 5 6
dy dy
34. Given that x  e u where u is a function of x , show that x  .
dx du

35. If x is so small that x 3 and higher powers may be neglected, obtain a


1 ex
quadratic approximation of f ( x)  .
1  In (1  x)


f (3)  6 g (3)  5 ,  f 
36. If f (3)  4 , g (3)  2 , and find the values of   .
g
37. Determine the area of largest rectangle that can be inscribed in a circle of
radius 1.

 x  y
38. If P  sin 1    tan 1   , find all partial derivatives of P up to the
 y  x
second derivative.

592
39. At a given instant the radii of the two concentric circles are 8cm and 12cm .
The radius of the outer circle is increasing at a rate of 1cms 1 and that of the
inner circle is increasing at 2cms 1 . Find the rate of change of an area enclosed
between two circles.
40 (a) Using Maclaurin’s expansion and differentiation, show that
 x2 x4
In (cos x)    ...
2 12

 x
(b) Using cos x  2 cos    1 and results in (a) show that
2

2
x2 x4
In (1  cos x)  In 2    ...
4 96

41. Find the maximum and minimum values of the function 2 sin t  cos2t and
hence state the minimum and maximum points.
42. Find the height of the right circular cylinder of greatest volume which can
be cut from a sphere of radius 'a' .

43. 100m of fencing is to be used to make a rectangular enclosure. Find the


greatest possible area and corresponding dimension.
44. Suppose the volume of cylinder disc 54 m 3 . What will be dimension of the
cylinder if the surface area is minimum?
45. Use the Maclaurin’s theorem to find the first three non-zero terms of
In2  x  .

dy y
46. If 3 sin xy  4 cos xy  7 , show that  .
dx x

47. If u  e x  y cos(x  y ) , show that u  u  2u .


x y

 f 
48. If f x, y   sin x sin y sin( x  y) . Show that    2 sinx  y cosx  y  .
 x  y

d 2 y 1 dy y
49. If y  x , show that
x
   0.
dx 2 y dx x

50. Differentiate tan


1
 1  x 1x  w.r.t. x .
2 1

593
 x2  y2  dy y
52. If cos1  2   tan 1 a , then show that
2 
 .
 x  y  dx x

1
53. Find the equation of the normal to the curve y  at the point on the curve
x
where x  2 . Find the coordinates of the point where this normal cuts the curve
again.
1
 1 dy y
54. If x  a and y  a cos 
sin
, prove that 
dx x

dy Inx
55. If x y  e x  y , prove that 
dx 1  Inx 2

x y  y x , prove that dy  Iny  x x


y
56. If
dx Inx  y

57. If y  e x sin x , prove that dy


 sin x  x cos x e x sin x
dx

58. If y  sin e  , prove that dydx  2 xe


x2 x2
 
cos e x
2

dy cos x
59 If y  e
sin x
, prove that  e sin x

dx 2 sin x

dy sec2 x
60. If y  e 
tan x tan x
, prove that e
dx 2 tan x

dy x cos x 2
61. If y  sin x , prove that 
2

dx sin x 2

1 y2
62. If 1  x  1  y  ax  y  , prove that
dy
2 2

dx 1 x2

63. If y  In tan 4  2x  , show that


dy
 sec x  0
dx

64. By using the first principle, find the derivative of f x  cos x

594
Chapter Ten
INTEGRATION
Introduction
Integration is the process of finding the functions from the given slope or
gradient.
On the other hand integration is the reverse process of differentiation and also
integration represent continuous summation.

The international symbol of integration is elongated S i.e. 


Types of Integration
Integration is divided into two categories
(I) INFINITE INTEGRAL

Infinite integral is an integral which does not contain limits eg  f ( x)dx


(II)DEFINITE INTEGRAL
b
Definite integral is an integral which has limits eg  f ( x)dx
a

10.1 GENERAL FORMULA FOR INTEGRATION


Consider the derivative of the following function

dx
 
d n1
x  (n  1) x n

 
d x n1  (n  1) x n dx
Integrate both sides

 d x    (n  1) x dx
n 1 n

x  (n  1)  x dx
n 1 n

x n1
 c   x n dx
n 1
x n1
 x dx  n  1  c
n

x n 1
 x dx  c
n

n 1

595
Example 1
Integrate  3x 2 dx
Solution
 x 21 

2
3 x dx  3   x 3  c
 2 1
  3 x dx  x 3  c
2

Example 2
Integrate  4dx
Solution
 4dx  4 x dx  4 x  c
0

  4dx  4 x  c

Example 3

Integrate  x 4  6 x 2  2 x  7 dx 
Solution
 
  x 4  6 x 2  2 x  7 dx  15 x 5  2 x 3  x 2  7 x  c

Example 4
Integrate  x dx
Solution
 x dx   x 2 dx  23 x 2  c
1 3

  x dx  23 x 2  c
3

Example 5
1
Integrate  x
dx

Solution

596
1 1  12
 x   x 12   dx  2 x 2  2 x  c
1
dx dx x

1
 dx  2 x  c
x

10.2 STANDARD INTEGRAL


(a) Integration of e x
Consider e x   e x
d
dx
d e   e dx
x x

Integrate both sides

 d e    e dx
x x

e  c   e dx
x x

 e dx  e  c
x x

 e dx  e c
x x

Example 6
Integrate  e 2 x dx
Solution
 e dx.......... .......... .......... .......... .......... .......... .........( i)
2x

du du
Let u  2 x,  2, dx  .......... .......... .......... ....( ii)
dx 2
Substitute equation (ii) into equation (i)
u du 1 u 1 u
 e 2  2  e du  2 e  c but u  2 x
1
  e 2 x dx  e 2 x  c
2

Example 7
Integrate  e x3 dx
Solution

597
e
x 3
dx.......... .......... .......... .......... .......... .......... .........( i )
du
Let u  x  3,  1, dx  du.......... .......... .......... ....( ii)
dx
Substitute equation (ii) into equation (i)

 e du e  c but u  x  3
u u

  e dx  e x 3 x 3
c
Example 8
Integrate  e 2 x dx
1

Solution
 e 2 dx.......... .......... .......... .......... .......... .......... .........( i)
1
x

du 1
Let u  12 x,  , dx  2du.......... .......... .......... ..(ii)
dx 2
Substitute equation (ii) into equation (i)

 2e du 2 e du 2e  c but u  12 x


u u u

  e dx  2e  c
1 1
2
x 2
x

1
(b) Integration of x

Consider
d
Inx   1
x
dx
d Inx   1x dx
Integrate both sides

 d Inx    dx 1
x

Inx  c   dx 1
x

 dx  Inx  c
1
x

 1
x dx  Inx  c

Example 9
1
Integrate  2 x dx
598
Solution
1
 2 x dx.......... .......... .......... .......... .......... .......... .......... (i)
du du
Let u  2 x,  2, dx  .......... .......... .......... .....( ii)
dx 2
Substitute equation (ii) into equation (i)
1 1 du 1 1 1
 2 x dx   u 2  2  u du  2 Inu  c but u  2 x
1 1
  dx  In (2 x)  c
2x 2
Example 10
1
Integrate  x  4 dx
Solution
1
 x  4 dx.......... .......... .......... .......... .......... .......... .......... (i)
Let u  x  4, du  dx.......... .......... .......... .......... ........( ii)
Substitute equation (ii) into equation (i)
1
 u dx  Inu  c but u  x  4
1
 dx  In ( x  4)  c
x4

Example 11
3
Integrate  x  5 dx
Solution
3 1
 x  5 dx  3 x  5 dx .......... .......... .......... .......... ....( i)
Let u  x  5, du  dx.......... .......... .......... .......... ....( ii)
Substitute equation (ii) into equation (i)
1
3 dx  3Inu  c but u  x  5
u
3
 dx  3In ( x  5)  c
x 5

599
1
(c) Integration of x in term of common logarithm
Consider
d
log x   1
dx xIn10
d log x  
1
dx
xIn10
Integrate both sides

 d log x   In10  x dx
1 1

In10log x  c    1x dx

 1
x dx  In10 log x  D  1
x dx  In10 log x  D

(d) Integration of a x
Consider
d x
dx
 
a  a x Ina

 
d a x  a x Inadx
Integrate both sides

 d a    a Inadx
x x

a  Ina  a dx
x x

ax
 a dx  Ina  c
x

ax
 a dx  Ina  c
x

Example 12
Integrate  2 x dx
Solution

600
2
x
dx.......... .......... .......... .......... .......... .......... .......... .......( i )
Let u  2 x apply In both sides
1
Inu  xIn2, dx  du .......... .......... .......... .......... .......( ii)
uIn2
Substitute equation (ii) into equation (i)
 1  1 1
    uIn2  In 2 
    u  c but u  2 x
x
2 dx u du du
In 2
2x
  2 dx 
x
c
In 2
Example 13
Integrate 10 x2 dx
Solution
10
x 2
dx.......... .......... .......... .......... .......... .......... .......... .......( i )
Let u  10 x 2 apply In both sides

Inu  x  2In10, dx 
1
du .......... .......... .......... .......... .(ii)
uIn10
Substitute equation (ii) into equation (i)
 1  1 1
10 dx   u  
x 2
du  du  u  c but u  10 x 2
 uIn10  In10 In10
10 x 2
 10 x 2 dx  c
In10

(e) Integration of sin x


Consider cos x    sin x
d
dx
d cos x    sin xdx
Integrate both sides

 d cos x    sin xdx


cos x    sin xdx

 sin xdx   cos x  c


 sin xdx   cos x  c

601
(f) Integration of cos x
Consider sin x   cos x
d
dx
d sin x   cos xdx
Integrate both sides

 d sin x    cos xdx


sin x   cos xdx

 cos xdx  sin x  c  cos xdx  sin x  c


(g) Integration of sec2 x
Consider tan x   sec2 x
d
dx
d tan x   sec2 xdx
Integrate both sides

 d tan x    sec xdx


2

tan x   sec xdx 2

 sec xdx  tan x  c


2

 sec xdx  tan x  c


2

(h) Integration of cosec 2 x


Consider cot x    cosec 2 x
d
dx
d cot x    cosec 2 xdx
Integrate both sides

 d cot x     cosec xdx


2

cot x    cosec xdx 2

 cosec xdx   cot x  c


2

 cosec xdx   cot x  c


2

602
(i) Integration of sec x tan x
Consider sec x   sec x tan x
d
dx
d sec x   sec x tan xdx
Integrate both sides

 d sec x    sec x tan xdx


sec x   sec x tan xdx

 sec x tan xdx  sec x  c


 sec x tan xdx  sec x  c
(j) Integration of cosecxcot x
Consider cos ecx    cos ecx cot x
d
dx
d cos ecx    cos ecx cot xdx
Integrate both sides

 d cosecx     cosecx cot xdx


 cos ecx  c   cos ecx cot xdx

 cosecx cot xdx   cosecx  c


 cosecx cot xdx   cosecx  c
Example 14
Integrate  sin 3xdx
Solution
 sin 3xdx.......... .......... .......... .......... .......... .......... (i)
du
Let u  3 x, du  3dx, dx  .......... .......... ......( ii)
3
du 1 1
 sin u 3 3
  sin udu   cosu  c but u  3 x
3
1
  sin 3 xdx   cos3 x  c
3

603
Example 15
Integrate  2 cos2 xdx
Solution
 2 cos 2 xdx.......... .......... .......... .......... .......... .......... (i)
du
Let u  2 x, du  2dx, dx  .......... .......... ......( ii)
2
du
 2 cosu 2 
 cosudu  sin u  c but u  2 x

  2 cos 2 xdx  sin 2 x  c

Example 16
Integrate  sec2 x 1dx
Solution
 sec x  1dx.......... .......... .......... .......... .......... ...(i)
2

Let u  x  1, du  dx.......... .......... .......... .......... (ii)

 sec udu  tan u  c but u  x  1


2

  sec udu  tan x  1  c


2

Example 17
Integrate  sec2 x tan 2 xdx
Solution
 sec 2 x tan 2 xdx.......... .......... .......... .......... .......... .(i)
Let u  2 x, du  2dx.......... .......... .......... .......... ..(ii)
1 1
2 sec u tan udu 
2
secu  c but u  2 x

1
  sec 2 x tan 2 xdx  sec 2 x  c
2

604
(k) Integration of sinh x
e x  ex e x  ex
1 x 1 x
   
 sinh xdx   2 dx  2  e  e dx  2 e  e  2  cosh x  c
x x

 sinh xdx  cosh x  c

(l) Integration of cosh x


e x  ex e x  ex
1 x 1 x
   
 cosh xdx   2 dx  2  e  e dx  2 e  e  2  sinh x  c
x x

 cosh xdx  sinh x  c

(m) Integration of sec h 2 x


d
From (tanh x)  sec h 2 x
dx
d (tanh x)  sec h 2 xdx
Integrate both sides

 d (tanh x)   sec h xdx


2

tanh x  c   sec h xdx 2

 sec h xdx  tanh x  c


2

(n) Integration of cos ech 2 x


d
From (coth x)   cos ech 2 x
dx
d (coth x)   cos ech 2 xdx
Integrate both sides

 d (coth x)   cos ech xdx


2

coth x  c    cos ech xdx 2

 cos ech xdx   coth x  c


2

605
(o) Integration of sec hx tanh x
From
d
sec hx   sec hx tanh x
dx
d (sec hx)   sec hx tanh xdx
Integrate both sides

 d (sec hx)    sec hx tanh xdx


sec hx  c    sec hx tanh xdx

 sec hx tanh xdx   sec hx  c

(p) Integration of cos echxcoth x


Consider cosechx   cosechx coth x
d
dx
d cosechx   cosechx coth xdx

 d cosechx   cosechx coth xdx


cosechx    cosechx coth xdx

 cosechx coth xdx   cosechx  c


 cosechx coth xdx   cosechx  c

Example 18
Integrate  sinh 4 xdx
Solution

606
 sin 4 xdx.......... .......... .......... .......... .......... .......... (i)
du
Let u  4 x, du  4dx, dx  .......... .......... ......( ii)
4
du 1 1
 sinh u 4  4  sinh udu  4 coshu  c but u  4 x
1
  sinh 4 xdx  cosh 4 x  c
4

Example 19
Integrate  3 cosh3xdx
Solution
 3 cosh3xdx.......... .......... .......... .......... .......... ......( i)
du
Let u  3x, du  3dx, dx  .......... .......... ......( ii)
3
du
 3 coshu 3 
 coshudu  sinh u  c but u  3x

  3 cosh3xdx  sinh 3x  c

Example 20
Integrate  sec h 2 x 1dx
Solution
 sech x  1dx.......... .......... .......... .......... .......... ...(i)
2

Let u  x  1, du  dx.......... .......... .......... .......... (ii)

 sech udu  tanh u  c but u  x  1


2

  sec h udu  tanh x  1  c


2

Example 21
Integrate  2 sech2 x tanh 2 xdx
Solution

607
 2 sech2 x tanh 2 xdx.......... .......... .......... .......... .....( i)
Let u  2 x, du  2dx.......... .......... .......... .......... ..(ii)

 sechu tanh udu  sehcu  c but u  2 x


  2 sec h2 x tanh 2 xdx   sec h2 x  c

Exercise 10.1
1 x
1.  3 dx
sec2 x
2.  x dx
3.  4 cosh4x  5dx
 ax  3bx  cx  d dx
3 2
4.

10.3 INTEGRATION BY USING FACTOR FORMULA


Integration by using factor formula is possible if and only if the integrand
consists of two trigonometric functions which are expressed in product. These
trigonometric functions mostly consist of multiple angles. Therefore we need to
change the product of sine and cosine into sum and difference of these
functions.

608
Nature of integrand which use factor formula are as follows
(i)  2 sin mx cos nxdx   sin(mx  nx)  sin(mx  nx) dx
(ii)  2 cos mx sin nxdx   sin(mx  nx)  sin(mx  nx) dx
(iii)  2 cos mx cos nxdx   cos(mx  nx)  cos(mx  nx) dx
(iv)   2 sin mx sin nxdx   cos(mx  nx)  cos(mx  nx) dx

Example 22
Integrate  2 sin 2 xcos xdx
Solution
 2 sin 2 x cos xdx
From factor formula 2 sin 2 x cos x  sin 3 x  sin x

 2 sin 2 x cos xdx   sin 3 x  sin x dx  


1
cos3x  cos x  c
3
1
  2 sin 2 x cos xdx   cos3 x  cos x  c
3

Example 23
Integrate 12sin 4 x cos 2 xdx
Solution
12sin 4 x cos 2 xdx
From factor formula 2 sin 4 x cos 2 x  sin 6 x  sin 2 x

12sin 4 x cos 2 xdx  6 sin 6 x  sin 2 x dx   cos6 x  3 cos 2 x  c


 12 sin 4 x cos 2 xdx   cos 6 x  3 cos 2 x  c

Example 24
Integrate  2 cos5x sin 3xdx
Solution

609
 2 cos5 x sin 3xdx
From factor formula 2 cos5 x sin 3 x  sin 8 x  sin 2 x

 2 cos5 x sin 3xdx   sin 8 x  sin 2 x dx   8 cos8 x  2 cos 2 x  c


1 1

1 1
  2 cos5 x sin 3 xdx   cos8 x  cos 2 x  c
8 2

Example 25
Integrate  2 cos5x cos3xdx
Solution
 2 cos5 x cos3xdx
From factor formula 2 cos5 x cos3 x  cos8 x  cos 2 x

 2 cos5 x cos3xdx   cos8 x  cos 2 x dx  8 sin 8 x  2 sin 2 x  c


1 1

1 1
  2 cos5 x cos3 xdx  sin 8 x  sin 2 x  c
8 2

Example 26
Integrate  cos3xcos xdx
Solution

1 1
  cos3x cos xdx  sin 4 x  sin 2 x  c
8 4

Example 27
Integrate  cos x sin 5 xdx
Solution

610
 cos x sin 5 xdx   sin 5 x cos xdx
From factor formula sin 5 x cos x 
1
sin 6 x  sin 4 x 
2

 sin 5 x cos xdx   2 sin 6 x  sin 4 x dx   12 cos6 x  8 cos 4 x  c


1 1 1

1 1
  sin 5 x cos xdx   cos6 x  cos 4 x  c
12 8

Example 28
Integrate   4 sin 3xsin x dx
Solution
  4 sin 3x sin x dx  2  2 sin 3x sin x dx
From factor formula,  2 sin 3x sin x  cos 4 x  cos 2 x
Then, 2 cos 4 x  cos 2 x dx  2 14 sin 4 x  12 sin 2 x   c
   4 sin 3x sin x dx  12 sin 4 x  sin 2 x  c

Exercise 10.2
1.  sin 3x cos 2xdx
2.  2 sin x cos5xdx
3.  2 cos 7 x cos xdx
4.  sin 3xcos 4xdx
5.  2 sin 3x sin xdx

611
10.4 INTEGRATION BY CHANGING OF VARIABLES
Integration by changing of variables is possible if and only if the integrand
contains a linear function ax  b
Example 29
1
Integrate  dx
3x  1
Solution
1
 3x  1 dx.......... .......... .......... .......... .......... .......... ....(i)
du
Let u  3 x  1, du  3dx,  dx.......... .......... .........( ii)
3
1 1 du 1 1 1
 3x  1 dx   u 3  3  u du  3 Inu  c but u  3x  1
dx  In 3 x  1  c
1 1

3x  1 3

Example 30
1
Integrate  x  53 dx
Solution
1
 x  53 dx.......... .......... .......... .......... .......... .......... ....( i)
Let u  x  5, du  dx.......... .......... .......... .......... .........( ii)
Substitute equation (ii) into equation (i)
1 1 2 1
u 
3
3
du  u du   u   but u  x  5
2 2u 2
1 1
 dx   c
 x  53
2 x  5
2

Example 31
Integrate  2 x  1dx
Solution
 2 x  1dx.......... .......... .......... .......... .......... .......... ....( i )
du
Let u  2 x  1, du  2dx,  dx.......... .......... .........( ii)
2
2 32
 u dx   u 2 dx  x but u  2 x  1
1

3
  2 x  1dx 
2
2 x  1 2  c
3

3
612
10.5 INTEGRATION OF FUNCTION AND ITS DERIVATIVE
This type of integration is possible if and only if the integrand consists of
function and its derivative.
Standard formats of this type of integration are;
(i )  f ( x) f ( x)dx
f ( x)
(ii)  dx
f ( x)
(iii)  f ( x) P f ( x ) dx where P is any function

Example 32
2x 1
Integrate  2 dx
x x
Solution
2x 1
 x 2  xdx.......... .......... .......... .......... .......... .......... .......... ......( i)
Let u  x 2  x, du  2 x  1dx, dx 
du
.......... .......... .....( ii)
2x 1
Substitute equation (ii) into equation (i)
2x 1  2 x  1  du 1
x2
x
dx    
 u  2x 1
  du  Inu  c but u  x 2  x
u
2x 1
 2
x x

dx  In x 2  x  c 
Example 33
3x 2
Integrate  3 dx
x 4
Solution
3x 2
 x 3  4 dx.......... .......... .......... .......... .......... .......... (i)
du
Let u  x 3  4, du  3x 2 dx, dx  2 .......... ........( ii)
3x
Substitute equation (ii) into equation (i)
3x 2 3x 2 du 1
 x 3  4 dx   u  3x 2   u du  Inu  c
 3
3x 2
x 4

dx  In x 3  4  c 
613
Example 34
x 1
Integrate  x 2  2 x dx
Solution
x 1
x  2x
2
dx.......... .......... .......... .......... .......... .......... .......... ......( i )

Let u  x 2  2 x, du  2 x  2dx, dx 
du
.......... .......... .....( ii)
2 x  1
Substitute equation (ii) into equation (i)
x 1  x  1  du 1 1 1
x  2x
2
dx       du  Inu  c but u  x 2  2 x
 u  2 x  1 2 u 2
x 1
 2
x  2x
1
dx  In x 2  2 x  c
2
 
Example 35
x 2 1
Integrate  3x  x 3 dx
Solution
x 2 1
 3x  x 3 dx.......... .......... .......... .......... .......... .......... .......... .......... .......... ...(i)

Let u  3x  x 3 , du  3  3x 2 dx, dx   du

 3 x 2 1 .......... .......... .......... ..(ii)

Substitute equation (ii) into equation (i)


x 2 1  x 2  1  du 1 1 1
 3x  x 3   u   3 x 2  1   3  u du   3 Inu  c but u  3x  x
 3
dx
 
x 2 1

3x  x 3
1

dx   In 3x  x 3  c
3

Example 36
Integrate  2 xe x dx
2

Solution
 2 xe dx.......... .......... .......... .......... .......... .......... .......... .......... .......... ...(i)
2
x

du
Let u  x 2 , du  2 xdx, dx  .......... .......... .......... .......... .......... .......( ii)
2x
Substitute equation (ii) into equation (i)

 2 xe x2
dx   2 xeu 
du
2x

dx   e u du  e u  c but u  x 2

  2 xe x dx  e x  c
2 2

614
Example 37
Integrate  xe x dx
2

Solution
 dx.......... .......... .......... .......... .......... .......... .......... .......... .......... ...(i)
 x2
xe
du
Let u  x 2 , du  2 xdx, dx  .......... .......... .......... .......... .......... .......( ii)
2x
Substitute equation (ii) into equation (i)

xe  x2

u du
1 u 1 u
 dx   xe 2 x dx  2  e du   2 e  c but u  x
2

1 2
  xe x dx   e  x  c
2

Example 38
Integrate  tan xdx
Solution
sin x
 tan xdx  cos x
dx.......... .......... .......... .......... .......... .......... .......... ...(i )

du
Let u  cos x, du   sin xdx, dx  .......... .......... .......... .......... .(ii)
 sin x
Substitute equation (ii) into equation (i)
sin x  sin x  du 1
 cos x dx    
u   sin x
   du   Inu but u  cos x
u

 tan xdx   Incos x   Insec x   c


  tan xdx   In cos x   c

Example 39
Integrate  cot xdx
Solution
cos x
 cot xdx  sin x
dx.......... .......... .......... .......... .......... .......... .......... ...(i )

du
Let u  sin x, du  cos xdx, dx  .......... .......... .......... .......... ...(ii)
cos x
Substitute equation (ii) into equation (i)
cos x  cos x  du 1
 sin x dx    
u  cos x
  du  Inu but u  sin x
u
  cot xdx  In sin x   c

615
Example 40
sin x  cos x
Integrate  dx
sin x  cos x
Solution
sin x  cos x
 sin x  cos x dx.......... .......... .......... .......... .......... .......... .......... .......... .......... ........( i)
Let u  sin x  cos x, du  cos x  sin x dx, dx 
du
.......... .......... ........( ii)
sin x  cos x
Substitute equation (ii) into equation (i)
sin x  cos x  sin x  cos x  du 1
 sin x  cos x dx    u

 sin x  cos x
  du  Inu but u  sin x  cos x
u
sin x  cos x
 dx  In sin x  cos x   c
sin x  cos x

Example 41
sin x  cos x
Integrate  dx
sin x  cos x
Solution
sin x  cos x
 sin x  cos x dx.......... .......... .......... .......... .......... .......... .......... .......... .......... .......... ..(i)
Let u  sin x  cos x, du  cos x  sin x dx, dx 
du
.......... .......... ........( ii)
 sin x  cos x 
Substitute equation (ii) into equation (i)
sin x  cos x  sin x  cos x  du  1
 sin x  cos x dx    u
     du   Inu but u  sin x  cos x
 sin x  cos x  u
sin x  cos x
 dx   In sin x  cos x   c
sin x  cos x

Example 42
Integrate  cos xesin x dx
Solution
 cos xe dx.......... .......... .......... .......... .......... .......... .......... .......... .......... ...(i)
sin x

du
Let u  sin x, du  cos xdx, dx  .......... .......... .......... .......... .......... (ii)
cos x
Substitute equation (ii) into equation (i)
sin x u

du u

 cos xe dx   cos xe cos x   e du  e  c but u  sin x
u

  cos xesin x dx  e sin x  c

616
Example 43
Integrate 2x
 x 2 1
dx

Solution
2x
 x 2 1
dx.......... .......... .......... .......... .......... .......... .......... .......... ...(i )

du
Let u  x 2  1, du  2 xdx, dx  .......... .......... .......... .......... ......( ii)
2x
Substitute equation (ii) into equation (i)
2x  2 x  du 1
 dx      du   u  2 du  2 u but u  x 2  1
1

x 2 1  u  2x u
2x
 dx  2 x 2  1  c
x 1
2

Example 44
x
Integrate
 2x 2  3
dx

Solution
x
 2x  3
2
dx.......... .......... .......... .......... .......... .......... .......... .......... ...( i )

du
Let u  2 x 2  3, du  4 xdx, dx  .......... .......... .......... .......... ......( ii)
4x
Substitute equation (ii) into equation (i)
x  x  du 1 1 1  12 1
 2x2  3   u  4x 4  u
dx     du 
4  u du 
2
u but u  x 2  1

x 1
 dx  2x2  3  c
2x  3
2 2

Example 45
x
e
Integrate
 x
dx

Solution
x
e
 x
dx.......... .......... .......... .......... .......... .......... .......... .......... ...( i )

dx
Let u  x , du 
, dx  2 x du.......... .......... .......... .....( ii)
2 x
Substitute equation (ii) into equation (i)

 
x
e eu
 x
dx   x
2 x du  2  e u du  2e u  c but u  x

x
e
 dx  2e x
c
x

617
10.6 INTEGRATION WHICH INVOLVES PARTIAL FRACTIONS
These are integration which involve partial fraction before integration process is
carried out.

Example 46
3
Integrate  x  2x  1 dx
Solution
3
 x  2x  1 dx
3 A B
Express in partial fraction  
x  2x  1 x  2 x  1
3  A x  1  Bx  2
A  1, B  1
3  1 1 
 x  2x  1   x  2 x  1 dx
dx   

3 1 1
 x  2x  1  x  2  x  1 dx
dx  dx 

 x  2x  1dx  Inx  2  Inx  1  c


3

3  x2
 dx  In c
x  2x  1  x  1 

Example 47
1
Integrate  x 1x  12
dx

Solution

618
Example 48
1
Integrate x 2
4
dx
Solution
1 1
 x  4  x  2x  2dx
2
dx 

1 A B
Express  
x  2x  2 x  2 x  2
1  A x  2  B x  2
A  14 , B   14
1 1 1 1 1
 x  2x  2dx  4  x  2dx  4  x  2dx
 x  2x  2 4 Inx  2  Inx  2  c 
1 1
dx 

1 1  x2
 dx  In c
x  2x  2 4  x2

Example 49
x3
Integrate  2 dx
x 1
Solution

619
x3  x 
 x 2  1dx    x  x 2  1 dx by long division
x3 x 1 2 x
 x 2  1dx   xdx   x 2  1 dx  2 x   x 2  1 dx.......... .......... .......( i)
x x
Consider  2 dx by expressing 2 into partial fraction
x 1 x 1
x x A B
  
x 2  1  x  1x  1 x  1  x  1
x  A x  1  Bx  1`
A  12 , B  12
1 1 1 1 1
 x  1x  1 2  x  1 2  x  1dx
dx  dx 

1  x 1 
1
 x 2 1 2
dx 
1
In  x  1  In  x  1  In  
2  x 1
1 1  x 1 
x 1
2
dx  In 
2  x 1
.......... .......... .......... .......... .......... .......... .....( ii)

Substitute equation (ii) into equation (i)


x3 1 1  x 1 
  2 dx  x 2  In  c
x 1 2 2  x 1

Example 50
1
Integrate x 2
 x6
dx
Solution
dx but x 2  x  6   x  3 x  2
1
x 2
 x6
1 1
 x  x6
2
dx   x  3x  2 dx
1 A B
Express  
x  3x  2 x  3 x  2
1  A x  2   B x  3
A  15 , B   15 ,
1 1 1 1 1
 x  3x  2 dx  5  x  3dx  5  x  2dx
1 1  x 3
 dx  In  c
x  x6
2
5  x2

620
Example 51
3x  1
Integrate  6 x 2  7 x  2 dx
Solution
3x  1
 6 x 2  7 x  2 dx but 6 x  7 x  2  3x  22 x  1
2

3x  1 3x  1
 6 x  7 x  2  3x  22 x  1 dx
2
dx 

3x  1 A B
Express  
3x  22 x  1 3x  2 2 x  1
3x  1  A2 x  1  B3x  2
A  9, B  5,
3x  1 1 1
 6 x 2  7 x  2 dx  9 3x  2 dx  5 2 x  1 dx
3x  1
dx  3In 3x  2  In 2 x  1  c
5
 2
6x  7x  2 2

10.7 INTEGRATION BY USING TRIGONOMETRIC IDENTITIES


This is applicable if and only if the integrand is a trigonometric function with
higher power  cosn mxdx ,  sin n mxdx etc where n  z  and m  

When n is odd number apply trigonometric identities


(i ) cos2 x  sin 2 x  1
(ii) 1  tan 2 x  sec2 x
(iii) cot 2 x  1  cosec 2 x

When n is even number apply double angle formula


(i ) cos2 x  12 1  cos 2 x 
(ii) sin 2 x  12 1  cos 2 x 
Generally
(i ) cos2 mx  12 1  cos 2mx 
(ii) sin 2 mx  12 1  cos 2mx 
621
NOTE:
(a)  cosn xdx and  sin
n
xdx when n even number use double angle
(b)  cosn xdx and  sin
n
xdx when n odd number use identities
(c)  sin m x cosn xdx when m and n are even numbers use double angle
(d )  sin m x cosn xdx when m and n are odd numbers use identities
(e)  sin m x cosn xdx when m and n are either odd or even but not both
apply identities to odd integrand
( f )  tan m xdx where m is either odd or even use identity 1  tan 2 x  sec2 x
( g )  tan m x secn xdx where m is any number and n even use 1  tan 2 x  sec2 x
(h)  tan m x secn xdx where m is odd and n is any number use 1  tan 2 x  sec2 x
by attaching sec x tan x toward dx
(i )  secn xdx where n even number use 1  tan 2 x  sec2 x
( j )  secn xdx where n odd number apply integration by part
k )  cosec n xdx where n even number use 1  cot 2 x  cosec 2 x
(l )  cosec n xdx where n odd number apply integration by part
(m)  cot m xdx where m even or odd number use 1  cot 2 x  cosec 2 x
(n)  cot m x cosec n xdx where m is any number and n even use 1  cot 2 x  cosec 2 x
(o)  cot m x cosec n xdx where m is odd and n is any number use 1  cot 2 x  cosec 2 x
by attaching cosecx cot x towards dx

622
Example 52
Integrate  cos3 xdx
Solution
 cos xdx   cos x cos xdx.......... .......... .......... .......... ......( i)
3 2

From cos2 x  sin 2 x  1, cos2 x  1  sin x 2 x

 cos
3
 
xdx   1  sin 2 x cos xdx.......... .......... .......... .......... .(ii)
du
Let u  sin x, du  cos xdx, dx 
cos x

 cos 
xdx   1  u 2 cos x
3

 du 

 cos x 
 1 3
 cos xdx   1  u du  u  3 u  c but u  sin x
3 2

1
  cos3 xdx  sin x  sin 3 x  c
3

Example 53
Integrate  sin 3 xdx
Solution
 sin xdx   sin x sin xdx.......... .......... .......... .......... ......( i)
3 2

From cos2 x  sin 2 x  1, sin 2 x  1  cos2 x

 sin
3
 
xdx   1  cos2 x sin xdx.......... .......... .......... .......... (ii)
du
Let u  cos x, du   sin xdx, dx 
 sin x
xdx   1  u 2 sin x
 du 
 sin 
3

  sin x 

 sin 3
xdx    1  u 2
du 
1 3
3
u  u  c but u  cos x

1
  sin 3 xdx  cos3 x  cos x  c
3

623
Example 54
Integrate  cos5 xdx
Solution
5 4 2
 
 cos xdx   cos x cos xdx   cos x cos xdx.......... .......... .......... .......... .......( i)
2

From cos2 x  sin 2 x  1, cos2 x  1  sin x 2 x

 cos
5
 
xdx   1  sin 2 x cos xdx.......... .......... .......... .......... .......... .......... .......... (ii)
2

du
Let u  sin x, du  cos xdx, dx 
cos x

 du 
 cos xdx   1  u cos x cos x 
5 2 2

 cos   1
 2

xdx   1  u 2 du   u 4  2u 2  1 du  u 5  u 3  u  c but u  sin x
5 2

5 3
1 2
  cos5 xdx  sin 5 x  sin 3 x  sin x  c
5 3

Example 55
Integrate  sin 5 xdx
Solution
 sin
5
 
xdx   sin 2 x sin xdx.......... .......... .......... .......... .......... .......... .......... ........( i )
2

From cos2 x  sin 2 x  1, sin 2 x  1  cos2 x


5 2
 
 sin xdx   1  cos x sin xdx.......... .......... .......... .......... .......... .......... .......... ....(ii)
2

du
Let u  cos x, du   sin xdx, dx 
 sin x

 sin
5
  du 
xdx   1  u 2 sin x
2

  sin x 

 sin 5
xdx    1  u 2 2
du    
u 4
 2u 2
 1 
du  
1 5 2 3 

 u  u  u   c but u  cos x
5 3 
1 2
  sin 5 xdx   cos5 x  cos3 x  cos x  c
5 3

624
Example 56
Integrate  cos2 xdx
Solution
 cos xdx.......... .......... .......... .......... .......... ..(i)
2

From cos2 x 
1
1  cos 2 x .......... .......... .....( ii)
2
Substitute equation (ii) into equation (i)

 cos
2 1
1  cos 2 x dx  1  x  1 sin 2 x 
xdx  
2 2 2 
1 1
  cos2 xdx  x  sin 2 x  c
2 4

Example 57
Integrate  sin 2 xdx
Solution
 sin
2
xdx.......... .......... .......... .......... .......... ..(i )

From sin 2 x 
1
1  cos2 x .......... .......... .....( ii)
2
Substitute equation (ii) into equation (i)

 sin
2 1
1  cos 2 x dx  1  x  1 sin 2 x 
xdx  
2 2 2 
1 1
  sin 2 xdx  x  sin 2 x  c
2 4

Example 58
Integrate  cos4 xdx
Solution
 cos
4
 
xdx   cos2 x dx.......... .......... .......... .......... .......... .......... .......... .........( i )
2

From cos2 x  1 2 1  cos 2 x .......... .......... .......... .......... .......... .......... .......... ....( ii)
Substitute equation (ii) into equation (i)

 
2
1 
 cos xdx    2 1  cos2 x  dx  4  1  2 cos2 x  cos 2 x dx.......... .......... ....(iii)
4 1 2

From cos2 2 x  1 2 1  cos 4 x .......... .......... .......... .......... .......... .......... .......... ...(iv)

625
Substitute equation (iv) into equation (iii)
1  
1  2 cos 2 x  1  cos 4 x  dx  x  sin 2 x  1 2 x  1 4 sin 4 x   c
1 1
 cos xdx  
4

4  2  4
1 1 3
  cos4 xdx  sin 4 x  sin 2 x  x  c
16 4 8

Example 59
Integrate  sin 4 xdx
Solution
 sin
4
 
xdx   sin 2 x dx.......... .......... .......... .......... .......... .......... .......... .........( i )
2

From sin 2 x  1 2 1  cos 2 x .......... .......... .......... .......... .......... .......... .......... ....( ii)
Substitute equation (ii) into equation (i)

 
2
1 
 sin xdx    2 1  cos 2 x  dx  4  1  2 cos2 x  cos 2 x dx.......... .......... ..(iii)
4 1 2

From sin 2 2 x  1 2 1  cos 4 x .......... .......... .......... .......... .......... .......... .......... ...(iv)
Substitute equation (iv) into equation (iii)
1  
 sin xdx 
4
 1  2 cos 2 x 
1
1  cos 4 x  dx 
1
x  sin 2 x  1 2 x  1 4 sin 4 x   c
4  2  4
1 1 3
  sin 4 xdx   sin 4 x  sin 2 x  x  c
32 4 8

Example 60
Integrate  cos3 x sin5 xdx
Solution
5 2 5 2 5
 
 cos x sin xdx   cos x sin x cos xdx   1  sin x sin x cos xdx.......... .......( i)
3

du
Let u  sin x, du  cos xdx, dx  .......... .......... .......... .......... .......... .......... (ii)
cos x
Substitute equation (ii) into equation (i)

 cos3
x sin 5
xdx   1  u 2
u5
cos x  
 du 
   1  u u du
2 5
 
 cos x 
3 5 5 7
 1 6 1 8

 cos x sin xdx   u  u du  6 u  8 u  c but u  sin x
 1
6
1

  cos3 x sin 5 xdx   u 5  u 7 du  sin 6 x  sin 8 x  c
8

626
Example 61
Integrate  sin 5 x cos7 xdx
Solution
5 7 2
 7
 2 7
 
 sin x cos xdx   sin x cos x sin xdx   1  cos x cos x sin xdx.......... .....( i)
2 2

du
Let u  cos x, du   sin xdx, dx  .......... .......... .......... .......... .......... .....( ii)
 sin x
Substitute equation (ii) into equation (i)

 sin    du 
x cos7 xdx   1  u 2 u 7 sin x
5 2
 
   1  u u du   u  2u  u du
2 2 7 11 9 7
 
  sin x 
1 12 1 10 1 8
  u  u  u  c but u  cos x
5 7
sin x cos xdx
12 5 8
1 1 1
  sin 5 x cos7 xdx  cos12 x  cos10 x  cos8 x  c
12 5 8

Example 62
Integrate  cos3 x sin 4 xdx
Solution
 cos x sin xdx   cos x sin x cos xdx   1  sin x sin x cos xdx.......... ......( i)
3 4 2 4 2 4

du
Let u  sin x, du  cos xdx, dx  .......... .......... .......... .......... .......... ........( ii)
cos x

 cos x sin xdx   1  u u cos x cos x    1  u u du   u  u du


3 4 2 4  du  2 4 4 6

1 5 1 7
 cos x sin xdx  5 u  7 u  c
3 4

1 1
  cos3 x sin 4 xdx  sin 5 x  sin 7 x  c
5 7

Example 63
Integrate  tan 2 xdx
Solution
 tan xdx   sec x 1dx   sec xdx   dx
2 2 2

  tan 2 xdx  tan x  x  c

627
Example 64
Integrate  tan 4 xdx
Solution
2 2 2
 2 2

 tan xdx   tan x tan xdx   sec x  1 tan xdx   tan x sec xdx   tan xdx
4 2 2

 tan
4
xdx   tan 2 x sec2 xdx   sec 2
x  1dx

 tan
4
xdx   tan 2 x sec2 xdx  tan x  x   c.......... .......... .......... .......... .......... ....( i )
Consider  tan 2 x sec2 xdx
du
Let u  tan x, du  sec2 xdx, dx 
sec2 x
2  du  1 3 1 3
 tan x sec xdx   u sec x sec2 x    u du  3 u  3 tan x.......... .......... .....( ii)
2 2 2 2

Substitute equation (ii) into equation (i)


1
  tan 4 xdx  tan 3 x  tan x  x  c
3

Example 65
Integrate  tan 3 xdx
Solution
3 2 2
 
 tan xdx   tan x tan xdx   sec x  1 tan xdx   tan x sec xdx   tan xdx......( i)
2

Consider  tan x sec2 xdx


du
Let u  tan x, du  sec2 xdx, dx 
sec2 x
2  du  1 2 1 2
        u  tan x.......... .......... .......... .(ii)
2
tan x sec xdx u sec x udu
 
2
sec x 2 2
Also consider  tan xdx   In cos x .......... .......... .......... .......... .......... .......... .......( iii)
Substitute equations (ii) and (iii) into equation (i)

  tan 3 xdx  tan 2 x  In cos x   c


1
2

628
Example 66
Integrate  tan 3 x sec4 xdx
Solution

1 4 1 6
 tan x sec4 xdx 
x  x c
3

4 6
1 1
  tan 3 x sec4 xdx  tan 4 x  tan 6 x  c
4 6
Example 67
Integrate  sec4 xdx
Solution
 sec
4
xdx   sec2 x sec2 xdx   1  tan 2 x sec2 xdx.......... ....( i )
du
Let u  tan x, du  sec2 xdx, dx 
sec2 x

 sec xdx   1  u sec x sec2 x    1  u du  u  3 u  c


4 2 2  du  2 1 3

1
  sec4 xdx  tan 3 x  tan x  c
3
Example 68
Integrate  sec6 xdx
Solution
 sec
6
   
xdx   sec2 x sec2 xdx   1  tan 2 x sec2 xdx.......... ....( i )
2 2

du
Let u  tan x, du  sec2 xdx, dx 
sec2 x
6
 2 
 du 
 
 sec xdx   1  u sec x sec2 x    1  u du   u  2u  1 du
2 2 2 2 4 2
 
1 5 2 3
  u  u u c
6
sec xdx
5 3
1 2
  sec6 xdx  tan 5 x  tan 3 x  tan x  c
5 3

629
Example 69
Integrate  cosec4 xdx
Solution
 cosec xdx   cosec x cosec xdx   1  cot x cosec xdx.......... ....(i)
4 2 2 2 2

du
Let u  cot x, du   cosec 2 xdx, dx 
 cosec 2 x

 cosec xdx   1  u cosec x  cosec2 x    1  u du   u  3 u   c


4 2 2  du  2  1 3

1
  cosec 4 xdx   cot3 x  cot x  c
3

Example 70
Integrate  cosec6 xdx
Solution
 cosec xdx   cosec x  cosec 2 xdx   1  cot 2 x  cosec 2 xdx.......... ....( i )
6 2 2 2

du
Let u  cot x, du   cosec 2 xdx, dx 
 cosec 2 x

 cosec xdx   1  u  cosec x  cosec2 x    1  u  du   u  2u  1du


6 2 2 2  du  2 2 4 2

1 5 2 3 
 cosec xdx   5 u  3 u  u   c
6

1 2
  cosec6 xdx   cot5 x  cot3 x  cot x  c
5 3

Example 71
Integrate  cot 4 xdx
Solution
 cot
4
 
x   cot 2 x cot 2 x   cosec 2 x  1 cot 2 x

 cot x   cot 2 x cosec 2 xdx   cot 2 xdx


4

 cot
4

x   cot 2 x cosec 2 xdx   cosec 2 x  1 dx 
1
  cot 4 x   cot3 x  cot x  x  c
3

630
Example 72
Integrate  cot5 x cosec2 xdx
Solution
 cot
5
x cos ec 2 xdx
du
Let u  cot x, du   cos ec 2 dx, dx 
 cos ec 2 x
2  du  1 6
 u cosec x  cosec2 x    u du   6 u  c
5 5

1
  cot5 x cos ec 2 xdx   cot6 x  c
6

NOTE:
All concepts used in integration by using trigonometric ideas may be switched
to integration by using hyperbolic concepts, for example questions which
involves hyperbolic factor formula, hyperbolic double values, and hyperbolic
identities. Eg  sinh mx cosh nxdx ,  sinh m xdx ,  coshm xdx ,

 sinh x cosh xdx ,  tanh xdx ,  coth xdx ,  sech xdx and
m n m m m

 cosech xdx where m and n are either even or odd integers or both.
m

10.8 INTEGRATION BY USING TRIGONOMETRIC INVERSE CONCEPTS


(a) Integration by using sine or cosine inverse concepts
All integration of the following forms can be solved either by sine
inverse or cosine inverse concepts.
1
(i )  dx
a b x
2 2 2

(ii)  a 2  b 2 x 2 dx
1
(iii)  dx
a  b x  c 
2 2 2

(iv)  a 2  b 2  x  c  dx
2

Solution

631
Integration of the form
1
(i )  dx
a b x
2 2 2

Let x  ba sin , x 2  a2
b2
sin 2 
dx  ba cosd
1
 a b2

2 a2
sin 2

a
b cosd
b2

1
 a  a sin 
2 2 2
a
b cosd

1
 a 1  sin 2  ba cosd
1
 b 1  sin 2  cosd
1
 b cos2  cosd
1
 b cos cosd
1
b
d

  c but   sin 1  ba x 
1
b
1 1 1  b 
 a 2  b 2 x 2 dx  b sin  a x   c
1 1 b 
 a 2  b2 x2
dx 
b
sin 1  x   c
a 

632
Alternative
1
(i )  dx
a b x
2 2 2

Let x  ba cos , x 2  a2
b2
cos2 
dx   ba sind
1
 a b2

2 a2
cos 
2

a
b sind
b2

1
 a  a cos 
2 2 2
a
b sind

1
 a 1  cos2  ba sind
1
 b 1  cos2  sind
1
 b sin 2  sind
1
 b sin sind
1
  d
b
   c but   cos1  ba x 
1
b
1 1 1  b 
 a 2  b 2 x 2 dx   b cos  a x   c
1 1 b 
 a2  b2 x2
dx   cos1  x   c
b a 

633
Example 73
1
Integrate  1 x2
dx

Solution
1
Given  dx.......... .......... .......... .......... .......... .......... ......( i )
1 x2
Let x  sin , dx  cosd .......... .......... .......... .......... .......... ...(ii)
1 1
 1  x 2 dx   1  sin 2  cosd  d    c but   sin x
1

1
 dx  sin 1 x  c
1 x2

Alternative
1
Given  dx.......... .......... .......... .......... .......... .......... .......... ........( i )
1 x 2

Let x  cos , dx   sind .......... .......... .......... .......... .......... .......... .(ii)

 1  cos2   sin d    d    c but   cos x


1 1
 1 x2
1
dx 

1
 dx   cos1 x  c
1 x 2

Example 74
1
Integrate  9  4x2
dx

Solution
1 1
Given  dx   dx.......... .......... .......... .......... .......... .....( i )
9  4x 2 3 1   23 x 
2

Let 2
3 x  sin , dx  32 cosd .......... .......... .......... .......... .......... .......... .......( ii)

 32 cos d  12  d  12   c but   sin 1  23 x 


1 1
3 1   23 x 
2
dx  13 
1  sin 2 

dx  sin 1  23 x   c
1 1

9  4x 2 2

634
Alternative
1 1
Given  dx   dx.......... .......... .......... .......... .......... .......... ...(i )
9  4x2 3 1   23 x 
2

Let 2
3 x  cos , dx   32 sind .......... .......... .......... .......... .......... .......... .......... .(ii)

 32 sin d   12  d  12   c but   cos1  23 x 


1 1
3 1   23 x 
2
dx  13 
1  cos2 

dx   cos1  23 x   c
1 1

9  4x2 2

Example 75
5
Integrate  3  2x 2
dx

Solution
5 5 1
Given 
3  1 2 x 2
dx  dx.......... .......... .......... .......... .......... .....( i)
3  2x2
3  
Let 2
3 x  sin , dx  3
2 cosd .......... .......... .......... .......... .......... .......... .......( ii)
5

1
dx 
5

1
 3

cos d 
5
 d 
5
  c but   sin 1  x2

 
2 3
3 1 2 x 2 3 1  sin 2  2 2
3


5
dx 
5
sin 1  x  c
2
3
3  2x 2
2

Alternative

5 5 1
Given  dx   dx.......... .......... .......... .......... .......... .......... .......... .........( i)
3  2x2 3 1 2 x 2
3  
Let 2
3 x  cos , dx   3
2 sind .......... .......... .......... .......... .......... .......... .......... .......... .......... (ii)
5

1
dx 
5

1
  3

sin d  
5
 d  
5
  c but   cos1  x 2

 
2 3
3 1 2 x 2 3 1  cos2  2 2
3


5
dx  
5
cos1  x  c
2
3
3  2x 2
2

635
Integration of the form
(ii)  a 2  b 2 x 2 dx
Let x  ba sin , x 2  a2
b2
sin 2 
dx  ba cosd

 a 2  b 2 ( ba sin ) 2 ba cosd

 a  a sin  ba cosd
2 2 2

 a 1  sin  cosd
2 a
b

a2
 b cos  cosd
2

a2
b  cos2  d but cos2   12 (1  cos 2 )

a2 1
b 2
(1  cos 2 )d

a2 a2
 (1  cos 2 )d    12 sin 2   c
2b 2b
But   sin 1  ba x 

 a  b x dx 
2 a2
2

2b
2
 
sin 1  ba x   12 sin 2 sin 1 ( ba x)  c 

 a  b x dx 
2 a2
2b
2 2
 
sin 1  ba x   12 sin 2 sin 1 ( ba x)  c 

636
Alternative
(ii)  a 2  b 2 x 2 dx
Let x  ba cos , x 2  a2
b2
cos2 
dx   ba sind

 a 2  b 2 ( ba cos ) 2  ba sin d

 a  a cos   sin d


2 2 2 a
b

 a 1  cos   sin d


2 a
b

a2
  b sin  sind
2

a2
  sin 2  d but sin 2   12 (1  cos 2 )
b
a2 1
  2 (1  cos 2 )d
b
a2 a2
  (1  cos 2 )d     12 sin 2   c
2b 2b
But   cos1  ba x 

 a  b x dx  
2 2 a2
2

2b
 
cos1  ba x   12 sin 2 cos1 ( ba x)  c 

 a  b x dx  
2 a2
2 2

2b
 
cos1  ba x   12 sin 2 cos1 ( ba x)  c 

Example 76
Integrate  1  x 2 dx
Solution
Given  1  x 2 dx.......... .......... .......... .......... .......... .......... .......... .......... .......... ..(i )
Let x  sin , dx  cosd .......... .......... .......... .......... .......... .......... .......... .......( ii)

 1  x 2 dx   1  sin 2  cosd   cos2 d   12 1  cos 2 d

 1  x 2 dx  1
2   12 sin 2   c but   sin 1 x
1
2
1
  1  x 2 dx  sin 1 x  sin 2 sin 1 x  c
4
 
637
Alternative
Given  1  x 2 dx.......... .......... .......... .......... .......... .......... .......... .......... .......... ..(i )
Let x  cos , dx   sind .......... .......... .......... .......... .......... .......... .......... .......( ii)

 1  x 2 dx   1  cos2   sin d    sin 2 d    12 1  cos 2 d

 1  x 2 dx   12   12 sin 2   c but   sin 1 x


1
4
1

  1  x 2 dx  sin 2 sin 1 x  sin 1 x  c
2

Integration of the form


1
(iii)  dx
a  b x  c 
2 2 2

Let x  c  ba sin ,  x  c   ba sin 2 


2 2
2

dx  ba cosd
1
 a b2

2 a2
sin 
2

a
b cosd
b2

1
 a  a sin 
2 2
cosd
2
a
b

1
 a 1  sin 2  ba cosd
1
 b 1  sin 2  cosd
1
 b cos2  cosd
1
 b cos cosd
1
b
d

  c but   sin 1  ba ( x  c) 
1
b

 a 2  b 2 x  c 2 dx  b sin  ba ( x  c)  D
1 1 1

sin 1  ba ( x  c)   D
1 1
 a 2  b 2 x  c 
2
dx 
b

638
Alternative
1
 a 2  b 2 x  c 
2
dx

Let x  c  ba cos ,  x  c   cos2 


2 a2
b2

dx   ba sind


1
 ba sin d
a b 2 2 a2
b2
 cos  2


1
 ba sin d
a  a cos 
2 2 2

 a 1  cos2   ba sin d


1

1
 b 1  cos2  sind
1
 b sin 2  sind
1 1 1
 b sin sin d   
b
d  
b
 c

cos1  ba ( x  c)   D
1 1
 a 2  b 2 x  c 2 dx  
b

dx   cos1  ba ( x  c)   D
1 1
 a 2  b 2 x  c 
2 b

Example 77
1
Integrate  16  9 x  2
2
dx

Solution
1 1
 16  9 x  2
2
dx  
4 1    x  2
3 2
dx.......... .......... .......... .......... .(i )
4

Let 3
4 x  2  sin , dx  43 cosd


1 1 1
 43 cos d  1  d  1   c........( ii)
4  1  sin 2 
dx 
16  9 x  2 3 3
2

dx  sin 1  34  x  2  c
1 1

16  9 x  2  3
2

639
Alternative
1 1
 16  9 x  2 
2
dx  
4 1    x  2 
3 2
dx.......... .......... .......... .......... .......... .(i )
4

Let 3
4 x  2  cos , dx   43 sind


1 1 1
  1 1
4  1  cos2  3
dx   4
sin  d    d      c........( ii)
16  9 x  2 
3
2 3

dx   cos1  34  x  2   c
1 1

16  9 x  2  3
2

Example 78
1
Integrate  5  8x  4 x 2
dx

Solution
1 1 1
 5  8x  4 x 2
dx  
9  4x  1
2
dx  
3 1   x  1
2 2
dx.......... .....( i )
3

Let 2
3 x  1  sin , dx  32 cosd

3
1 1 1
 32 cos d  1  d  1   c........( ii)
3  1  sin 2 
dx 
1   23 x  1 2 2
2

dx  sin 1  23 x  1  c
1 1

5  8x  4 x 2 2

Alternative
1 1 1
 5  8x  4 x 2
dx  
9  4x  1
2
dx  
3 1   x  1
2 2
dx.......... .....( i )
3

Let 2
3 x  1  cos , dx   32 sind

3
1 1 1
  1 1
3  1  cos2  2 2
dx   3
sin  d   d    c........( ii)
1   23 x  1 2
2

dx   cos1  23 x  1  c
1 1

5  8x  4 x 2 2

640
Integration of the form
(iv)  a 2  b 2 x  c  dx
2

Let x  c  ba sin , x  c   ba2 sin 2 


2 2

dx  ba cosd

 a 2  b2  a2
b2
sin 2   a
b cosd

 a  a sin  cosd
2 2 2 a
b

 a 1  sin  cosd
2 a
b

 1  sin  cosd
a2 2
b

 cos  cosd
a2 2
b

 cos d but cos   1  cos2 


a2 2 2 1
b 2

a2
b  1  cos2 d
1
2

a2
2b  1  cos2 d    sin 2   c a2
2b
1
2

But   sin 1  ba ( x  c) 

  
a 2  b 2 x  c  dx  2ab sin 1  ba ( x  c)   12 sin 2 sin 1  ba ( x  c)   c
2 2


 
a 2  b 2 x  c  dx  a2b sin 1  ba ( x  c)   12 sin 2 sin 1  ba ( x  c)   c
2 2
 
Example 79

 1  x  1 dx
2
Integrate
Solution

 1  x  1 dx.......... .......... .......... .......... .......... .......... .......... .......... .(i )


2

Let x  1  sin , dx  cosd .......... .......... .......... .......... .......... .........( ii)

 
1  x  1 dx   1  sin 2  cosd   cos2 d   12 1  cos 2 d
2

 1  x  1 dx 
2 1
2   12 sin 2   12   14 sin 2  c but   sin 1 x  1

  1   x  1 dx  12 sin 1 x  1  14 sin 2 sin 1 x  1  c


2
 
641
Alternative

 a 2  b 2 x  c  dx
2

Let x  c  ba cos ,  x  c   ba2 cos2 


2 2

dx   ba sind

 a2  b2  a2
b2

cos2   ba sin d

 a  a cos   sin d


2 2 2 a
b

 a 1  cos   sin d


2 a
b

  1  cos  sind
a2 2
b

  sin  sind
a2 2
b

  sin d but sin   1  cos2 


a2 2 2 1
b 2

  1  cos 2 d
a2
b
1
2

  1  cos 2 d     sin 2   B


a2
2b
a2
2b
1
2

But   cos1  ba ( x  c) 

  
a 2  b 2 x  c  dx   2ab cos1  ba ( x  c)   12 sin 2 cos1  ba ( x  c)   B
2 2


  
a 2  b 2 x  c  dx   a2b cos1  ba ( x  c)  12 sin 2 cos1  ba ( x  c)   B
2 2


Example 80
Integrate  3  2 x  x 2 dx
Solution

 3  2 x  x 2 dx   4  x  1 dx  2 1  12 x  1 dx.......... .......... .......... .........( i )


2 2

Let 12 x  1  cos , dx  2 sind .......... .......... .......... .......... .......... .......... .........( ii)

  
4  x  1 dx  2 1  cos2   2 sin d  4 sin 2 d  4 12 1  cos 2 d
2

 4  x  1 dx  2  12 sin 2   2  sin 2  c but   cos1 12 x  1


2


  3  2 x  x 2 dx   sin 2 cos1 12  x  1  2 cos1 12 x  1  c 
642
(b) Integration by using tan inverse or cot inverse concepts
All integration of the following forms can be solved either by tan inverse or cot
inverse concepts
1
(i)  2 dx
a  b2 x2
1
(ii)  2 dx
a  b x  c 
2 2

Integration of the form


1
(i )  2 dx
a  b2 x2
Let x  ba tan  , x 2  ba2 tan 2  , dx  ba sec2 d
2

 a 1  tan 2   ba sec  d


1 1 1
 a b x   a 2  b 2 a2 tan 2 
 

2
2 2 2
dx 2 dx 2
b

1
a b x
22 2
dx  ab1  d  ab1   c

1 1 b 
 2 dx  tan 1  x   c
a b x2 2
ab a 

1 1 1  b 
 a 2  b2 x2 dx 
ab
tan  x  c
a 

Alternative
1
(i )  2 dx
a  b2 x2
Let x  ba cot , x 2  ba cot 2  , dx   ba cosec 2d
2
2

a
1
dx   2
1
 dx   2

1
a 1  cot  
 ba cosec 2 d
2
b x
2 2
a  b b2 cot 
2 a2 2 2

1
 a  b 2 x 2 dx   ab1  d   ab1   c
2

1 1 b 
 2 dx   cot 1  x   c
a b x 2 2
ab a 
1 1 1  b 
 a 2  b2 x2 dx  
ab
cot  x  c
a 

643
Example 81
1
Integrate  1 x 2
dx

Solution
1
 1  x 2 dx.......... .......... .......... .......... .......... .......... .......... ...(i)
Let x  tan  , dx  sec2 d .......... .......... .......... .......... .....( ii)
1 1
 1 x2   1  tan 2  sec d   d    c
2
dx

1
 dx  tan 1 x  c
1 x 2

Alternative
1
 1  x 2 dx.......... .......... .......... .......... .......... .......... .......... ...(i)
Let x  cot , dx   cosec 2d .......... .......... .......... .......... .....( ii)
1
 1 x2 dx 
1

 1  cot2   cosec  d   d    c
2

1
 dx   cot 1 x  c
1 x 2

Example 82
1
Integrate  25  16x 2 dx
Solution
1 1
 25  16x 2 dx  1
25  1   x
4 2
dx.......... .......... .......... .......... .(i )
5

Let 4
5 x  tan  , dx  54 sec2 d .......... .......... .......... .......... .....( ii)

 1  tan   
1 1
 1 x 2
dx  1
25 2
5
4 sec2  d  1
20  d  1
20  c

tan 1  54 x   c
1
 dx  1
1 x2
20

644
Alternative
1 1
 25  16x 2
dx  1
25  1   x 4 2
dx.......... .......... .......... .......... .(i )
5

Let 4
5 x  cot , dx   54 cos ec 2d .......... .......... .......... .......... .....( ii)

 1  cot   
1 1
 1 x 2
dx  1
25 2
5
4 cos ec 2 d   201  d   201   c

dx   201 cot 1  54 x   c
1

1 x 2

Integration of the form


1
(ii)  dx
a 2  b 2 x  c 
2

Let x  c  ba tan  , x  c 2  ba 2
2 tan 2  , dx  ba sec2 d

 
1 1 1
a 2
 b x  c 
2 2
dx  
a b 2

2 a2
tan 
2
 dx   2

a 1  tan 2 
a
b sec2  d
b2

1
a  b x  c 
2 2
 c 2
dx  1
ab  d  1
ab

tan 1  ba ( x  c)   A
1 1
 2 dx 
a  b x  c 
2 2
ab

tan 1  ba ( x  c)  A
1 1
a 2
 b 2 x  c 
2
dx 
ab

Alternative
1
(ii)  dx
a  b x  c 
2 2 2

Let x  c  ba cot , x  c 2  ba 2
2 cot 2  , dx   ba cosec 2d

 
1 1 1
a 2
 b x  c 
2 2
dx  
a b
2 2 a2
 cot 
2
 dx   2

a 1  cot 
2
 ba cosec 2 d
b2

1
a  b x  c 
2 2 2
dx   ab1  d   ab1   c

cot 1  ba ( x  c)   A
1 1
 2 dx  
a  b x  c 
2 2
ab

cot1  ba ( x  c)  A
1 1
a 2
 b x  c 
2 2
dx  
ab

645
Example 83
1
Integrate  x 2  4 x  5 dx
Solution
1
 x 2  4 x  5 dx by completing the square
1 1
 x  4 x  5  1  x  22 dx.......... .......... .......... ......( i)
2
dx 

Let x  2  tan  , dx  sec2 d .......... .......... .......... .......( ii)


1 1
 1  x  22   1  tan 2  sec d   d    c
2
dx

dx  tan 1  x  2  c
1
 2
x  4x  5

Alternative
1
 x 2  4 x  5 dx by completing the square
1 1
 x 2  4 x  5 dx   1  x  22 dx.......... .......... .......... .......... .......( i)
Let x  2  cot , dx   cosec 2d .......... .......... .......... .......... ...(ii)

 1  x  22 dx   1  cot2   cosec  d   d    c


1 1 2

dx   cot1 x  2  c
1
 2
x  4x  5

Example 84
1
Integrate  2x 2
 4x  5
dx
Solution
1
 2x 2
 4x  5
dx by completing the square

1 1 1
 2 x 2  4 x  5 dx   3  2x  12 dx  13  1  2 x  12 dx.......... ......( i)
3

Let 2
3 x  1  tan  , dx  32 sec2 d .......... .......... .......... .......... .......( ii)

 2x
1
 4x  5
2
dx  13 
1
1  tan  2
3 2

2 sec  d   1
2  d  1
2  c

dx  12 tan 1  23  x  1  c
1
 2
2x  4x  5
646
Alternative
1
 2 x  4 x  5 dx by completing the square
2

1 1 1
 2 x 2  4 x  5 dx   3  2x  12 dx  13  1  2 x  12 dx.......... .......... ..(i)
3

Let 2
3 x  1  cot , dx   32 cos ec 2d .......... .......... .......... .......... .......( ii)

 2x
1
 4x  5
2
dx  13 
1
1  cot 
2
 
 32 cos ec 2 d   12  d   12   c

dx   12 cot 1  23  x  1  c
1
 2
2x  4x  5

(c) Integration by using sec inverse or cosec concepts


The following form of integration can solved by either sec or cosec concepts
1
(i )  dx
x b2 x2  a2
(ii)  x b 2 x 2  a 2 dx
1
(iii)  dx
x  c  b x  c   a
2 2 2

(iv)  x  c  b 2  x  c   a 2 dx
2

Evaluation of the above Integral


1
(i )  dx
x b x a
2 2 2

Let x  ba sec , x 2  ba2 sec2  , dx  ba sec tan d


2

1 a
sec tan d tan 
x dx    d
b

b x a
2 2 2 a
b sec b 2 a2
b2
 2

sec   a 2
a sec   a
2 2 2

tan  1 tan  1 1
 a sec2   1 a  tan 2  d  d 
a
d 
a
 c

 x b 2 x 2  a 2 a sec  a x   c
1 1 1 b
dx 

 x b2 x 2  a 2 a sec  a x  c
1 1 1 b
dx 

647
Alternative
1
x b2 x2  a2
dx

Let x  ba cosec , x 2  ba2 cosec 2 , dx   ba cosec cotd


2

1  ba cosec cotd  cot


x b2 x2  a2
dx  
a
cosec b 2  a2

cosec 2  a 2

a 2 cosec 2  a 2
d
b b2

 cot 1 cot 1 1
a cosec 2  1
d   
a cot2 
d    d     c
a a

 x b 2 x 2  a 2 dx   a cosec  ba x   c
1 1 1

cos ec1  ba x   c
1 1
 x b2 x2  a 2 dx  
a

(ii)  x b 2 x 2  a 2 dx
Let x  ba sec , x 2  a2
b2
sec2  , dx  ba sec tan d

 x b x  a dx   sec b  sec   a  sec tan  d


2 2 2 a 2 a2 2 2 a
b b2 b

 sec a sec   a  sec tan  d


a 2 2 2 a
b b

 sec sec   1sec tan  d


a3 2
b2

 sec tan  sec tan  d   sec tan  sec tan  d
a3 a3
2
b2 b2

 sec  tan d   sec  tan  d .......... .......... .......... ....( i)


a3 a3
2 2 2 2
b2 b2

1
Let u  tan  , du  sec2 d , d  du
sec2 
Substitute the values of u and d in equation (i)
1
2  sec  u  2  u du  u3  c
a3 2 2 a3 2 a3
du
b
sec 
2 b 3b 2

x b 2 x 2  a 2 dx  a3
3b 2

tan 3 sec1  ba x   c 
 x b x  a dx 
2 2 2 a3
3b 2

tan 3 sec1  ba x   c 

648
Alternative

x b 2 x 2  a 2 dx
Let x  ba cos ec , x 2  a2
b2
cos ec 2 , dx   ba cos ec cotd

 x b x  a dx   cosec b  cosec    a  cosec cot d


2 2 2 a 2 a2 2 2 a
b b2 b

 cosec a cosec   a  cosec cot d


a 2 2 2 a
b b

  cosec cosec   1cosec cot d


a3 2
b2

  cosec cot  cosec cot d    cosec cot cosec cot d
a3 a3
2
b2 b2

  cosec  cot  d    cosec  cot  d .......... .......... .......... ....(i)


a3 a3
2 2 2 2
b2 b2

1
Let u  cot , du   cos ec 2d , d  du
 cos ec 2
Substitute the values of u and d in equation (i)
1
 ba2  cos ec 2 u 2 du  ba2  u 2 du  3ab 2 u 3  c
3 3 3

cos ec 
2

x b 2 x 2  a 2 dx  a3
3b 2

cot3 cos ec 1  ba x   c
 x b x  a dx  3ab2 cot cosec ba x  c
2 2 2 3 1 3
 

Example 85
1
Integrate x x 2 1
dx

Solution
1
x x 2 1
dx.......... .......... .......... .......... .......... .........( i )

Let x  sec , dx  sec tan d .......... .......... .........( ii)


1 sec tan d
 x x 2  1dx   sec sec2   1   d    c
1
 dx  sec1 x  c
x x 12

649
Alternative
1
x x 2 1
dx.......... .......... .......... .......... .......... .......... ........( i )

Let x  cosec , dx   cosec cotd .......... .......... .........( ii)


1  cosec cotd
 x x 2  1dx   cosec cosec2  1   d    c
1
 dx   cosec 1 x  c
x x 2 1

Example 86
1
Integrate x 4 x  49
2
dx

Solution
1 1
x 4 x 2  49
dx  
7x  x
2 2
1
dx.......... .......... .......... ......( i )
7

Let 2
7 x  sec , dx  72 sec tan d .......... .......... .......... ......( ii)
1 7
sec tan d
x 4 x  49
2
dx   2

7 sec  sec   1
7 2
 1
7  d  1
7  c
2

dx  17 sec1  72 x   c
1

x 4 x  492

Alternative
1 1
x 4 x  49
2
dx  
7x  x
2 2
1
dx.......... .......... .......... .......... ....( i )
7

Let 2
7 x  cos ec , dx   72 cos ec cotd .......... .......... .......... .....( ii)
1  72 cos ec cotd
x 4 x 2  49
dx  
7 72 cos ec  cos ec2  1
  17  d   17   c

dx   17 cosec1  72 x   c
1

x 4 x 2  49

650
Example 87
Integrate  x 4 x 2  9dx
Solution
x 4 x 2  9dx   3 x  23 x 2  1.......... .......... .......... .......... .......... .......... .......... ..(i)
Let 2
3 x  sec , dx  32 sec tan d .......... .......... .......... .......... .......... .......... ......( ii)

4 x 2  9dx   3 32 sec  sec2   1 32 sec tan  d 


27
x 4  sec2  tan 2 d

27
x 4 
4 x 2  9dx  sec2  tan 2 d .......... .......... .......... .......... .......... .......... ....( iii)

du
Let u  tan  , du  sec2 d , d  .......... .......... .......... .......... .......... ...( iv)
sec2 
27 2 du 27 2 27  u 3  27
           tan 3   c
2 2
x 4 x 9 dx u sec u du
4 sec 
2
4 4  3  12

  x 4 x 2  9dx 
27
12

tan 3 sec1  23 x   c 

Alternative
x 4 x 2  9dx   3x  23 x 2  1.......... .......... .......... .......... .......... .......... .......... .......... ..(i)
Let 2
3 x  cosec , dx   32 cosec cotd .......... .......... .......... .......... .......... .......... ......( ii)

4 x 2  9dx   3 32 cosec  cosec 2  1 32 cosec cot d  


27
x 4  cosec 2 cot2 d

27
x 4 x 2  9dx  
4 
cosec 2 cot 2 d .......... .......... .......... .......... .......... .......... .......... ..(iii)

du
Let u  cot , du   cossec2 d , d  .......... .......... .......... .......... .......... ...(iv)
 cosec 2
27 2 2  du  27 2 27  u 3  27 3
   
4 
         cot   c
2
x 4 x 9 dx u cos ec u du
  cosec   4
2
4  3  12

  x 4 x 2  9dx 
27 3
12

cot cosec 1  23 x   c 
Alternative
x 4 x 2  9dx.......... .......... .......... .......... .......... .......... .......... .......... (i )
udu
Let u  4 x 2  9 , u 2  4 x 2  9, 2udu  8 xdx, dx  .......... ....( ii)
4x
 udu 
 xu  u du  12 u3  c
1 2 1
4
4x 

  x 4 x 2  9dx 
1
12
 4x 2  9  c
3

651
10.9 INTEGRATION BY USING HYPERBOLIC INVERSE CONCEPTS

(a) Integration by using sinh inverse concepts


All integration of the following forms can be solved by sinh inverse concepts.
1
(i )  dx
a 2  b2 x2
(ii)  a 2  b 2 x 2 dx
1
(iii)  dx
a  b x  c 
2 2 2

(iv)  a 2  b 2  x  c  dx
2

Integration of the above concepts


1
(i )  dx
a2  b2 x2
Let x  ba sinh  , x 2  a2
b2
sinh 2  , dx  ba coshd
1
 a b2

2 a2
sinh  2

a
b coshd
b2

1
 a  a sinh 
2 2
coshd
2
a
b

1
 a 1  sinh 2  ba coshd
1
 b 1  sinh 2  coshd
1
 b cosh2  coshd
1
 b cosh coshd
d    c but   sinh 1  ba x 
1 1
b  b
1 1 1  b 
 a 2  b 2 x 2 dx  b sinh  a x   c
1 1 b 
 a 2  b2 x2
dx 
b
sinh 1  x   c
a 

652
(ii)  a 2  b 2 x 2 dx
Let x  ba sinh  , x 2  a2
b2
sinh 2  , dx  ba coshd

 a 2  b 2 ( ba sinh  ) 2 ba coshd

 a  a sinh  ba coshd
2 2 2

 a 1  sinh  coshd
2 a
b

a2
 b cosh  coshd
2

a2
b  cosh2  d but cosh2   12 (1  cosh 2 )

a2 1
b 2
(1  cosh 2 )d

a2 a2
 (1  cosh 2 )d    12 sinh 2   c but   sinh 1  ba x 
2b 2b

 a 2
 b 2 2
x dx 
a2
2b

sinh 1  ba x   12 sinh 2sinh 1 ( ba x)   c 

 a  b x dx 
2 a2
2b
2 2

sinh 1  ba x   12 sinh 2 sinh 1 ( ba x)  c  
1
(iii)  dx
a 2  b 2 x  c 
2

Let x  c  ba sinh  ,  x  c   sinh 2  , dx  ba coshd


2 a2
b2

1
 a b2

2 a2
sinh  2

a
b coshd
b2

1
 a  a sinh 
2
coshd
2 2
a
b

1 1
 a 1  sinh 2  ba coshd   b 1  sinh 2  coshd
1 1 1 1
 b cosh2  coshd   b cosh coshd  b  d  b   c
But   sinh 1  ba ( x  c) 

sinh 1  ba ( x  c)   D
1 1
 a b2 2
x  c  2
dx 
b

sinh 1  ba ( x  c)   D
1 1
 a 2  b 2 x  c 
2
dx 
b

653
(iv)  a 2  b 2  x  c  dx
2

Let x  c  ba sinh  ,  x  c   sinh 2  , dx  ba coshd


2 a2
b2

 a2  b2  a2
b2
sinh 2   a
b coshd

 a  a sinh  coshd
2 2 2 a
b

 a 1  sinh  coshd
2 a
b

 1  sinh  coshd
a2 2
b

 cosh  coshd
a2 2
b

 cosh d but cosh   1  cosh 2 


a2 2 2 1
b 2

a2
b  1  cosh 2 d
1
2

a2
2b  1  cosh 2 d    sinh 2   c a2
2b
1
2

But   sinh 1  ba ( x  c) 

 a 2  b 2 x  c  dx 
2 a2
2b sinh  1 b
a  
( x  c)   12 sinh 2 sinh 1  ba ( x  c)   A

 a 2  b 2 x  c  dx 
2 a2
2b sinh  1 b
a  
( x  c)   12 sinh 2 sinh 1  ba ( x  c)   A

Example 88
1
Integrate  1 x 2
dx

Solution
1
Given  dx.......... .......... .......... .......... .......... .......... .......... .(i )
1 x2
Let x  sinh  , dx  coshd .......... .......... .......... .......... .......... .....( ii)
1 1
 1  x 2 dx   1  sinh 2  coshd  d    c but   sinh x
1

1
 dx  sinh 1 x  c
1 x2

654
Example 89
1
Integrate  9  4x2
dx

Solution
1 1
Given  dx   dx.......... .......... .......... .......... .......... .......... .(i )
9  4x 2 3 1   x 2 2
3

Let 2
3 x  sinh  , dx  32 coshd .......... .......... .......... .......... .......... .......... .........( ii)

 32 cosh d  12  d  12   c but   sinh 1  23 x 


1 1
3 1   23 x 
2
dx  13 
1  sinh  2

dx  sinh 1  23 x   c
1 1

9  4x 2 2

Example 90
1
Integrate  2  3x 2
dx

Solution
1 1 1
Given  dx   dx.......... .......... .......... .......... .......... .....( i )
3  2x 2
3 1  23 x  2

Let 2
3 x  sinh  , dx  3
2 coshd .......... .......... .......... .......... .......... .......... .......( ii)
1

1
dx 
1

1
 3

cosh d 
1
 d 
1
 c
 
2
3 1 2 x 2 3 1  sinh 2  2 2
3


1
dx 
1
sinh 1  x  c
2
3
3  2x 2 2

Example 91
Integrate  1  x  12 dx
Solution
 1  x  1 dx.......... .......... .......... .......... .......... .......... .......... .......... .......( i )
2

Let  x  1  sinh  , dx  coshd .......... .......... .......... .......... .......... .........( ii)

  
1  x  1 dx   1  sinh 2  coshd   cosh2 d   12 1  cosh 2 d
2

 1  x  1 dx 
2 1
2   12 sinh 2   12   14 sinh 2  c but   sinh 1 x  1

  1  x  1 dx  12 sinh 1  x  1  14 sinh 2 sinh 1 x  1  c


2
 
655
(b) Integration by using cosh inverse concepts
All integration of the following forms can be solved by cosh inverse concepts
1
(i )  dx
b2 x2  a2
(ii)  b 2 x 2  a 2 dx
1
(iii)  b x  c   a
2 2 2
dx

 b 2  x  c   a 2 dx
2
(iv)

Evaluate the above concepts


1
(i )  dx
b x a
2 2 2

x  ba cosh , x 2  ba2 cosh2  , dx  ba sinh d


2


1
dx  
1
 ba sinh d
b x a
2 2 2
b 
2 a2
b2
cosh   a
2
 2


1
 ba sinh d
a cosh   a
2 2 2

a
b
sinh d  1  1
sinh d  1  d  1   c
cosh2   1 b sinh 2  b b
But   cosh1  ba x 

dx  cosh1  ba x   c
1 1
 b2 x2  a2 b

dx  cosh1  ba x   c
1 1
 b2 x2  a 2 b

656
(ii)  b 2 x 2  a 2 dx
x  ba cosh , x 2  a2
b2
cosh2  , dx  ba sinh d

 b 2 x 2  a 2 dx   b 2  a2
b2

cosh2   a 2  ba sinh  d

 a 2 cosh2   a 2  ba sinh  d

 a2
b cosh2   1sinh  d  a2
b  sinh 2  sinh  d  a2
b  sinh
2
d
But sinh 2   12 cosh 2  1
a2
 sinh d   cosh 2  1d  2ab  12 sinh 2     c
a2
2 2
1
b b 2

But   cosh1  ba x 

 b x  a dx  2ab 12 sinh 2cosh  ba x   cosh  ba x   c


2 2 2 1 2 1

 b2 x 2  a 2 dx  a2b
2
 1
2
 
sinh 2 cosh1  ba x   cosh1  ba x   c 

1
(iii)  b x  c   a
2 2 2
dx

x  c  ba cosh , x  c   cosh2  , dx  ba sinh d


2 a2
b2


1
dx  
1
 ba sinh d
b x  c   a
2 2 2
b 
2 a
b2
2
cosh   a
2
 2


1
 ba sinh d
a cosh   a
2 2 2

a
b
sinh d  1  1
sinh d  1  d  1   c
cosh2   1 b sinh 2  b b
But   cosh1  ba ( x  c) 

dx  cosh1  ba ( x  c)   D
1 1
 b 2 x  c   a 2
2 b

dx  cosh1  ba ( x  c)   D
1 1
 b 2 x  c   a 2
2 b

657
 b 2  x  c   a 2 dx
2
(iv)
Let x  c  ba cosh , x  c   cosh2  , dx  ba sinh d
2 a2
b2

 b 2  x  c   a 2 dx   b 2
2
 a2
b2

cosh2   a 2  ba sinh  d

 a 2 cosh2   a 2  ba sinh  d


a2 a2
 a cosh   1 sinh d  b  sinh  sinh  d   sinh 2 d
2 a 2
b
b
But sinh 2   12 cosh 2  1
a2
 sinh d   cosh 2  1d  2ab  12 sinh 2     c
a2
2 2
1
b b 2

But   cosh1  ba x 

 b x  c   a dx  2ab 12 sinh 2cosh  ba x   cosh  ba x   c


2 2 2 1 2 1

 b 2 x  c   a 2 dx 
2 a2
2b  1
2  
sinh 2 cosh1  ba x   cosh1  ba x   c 

Example 92
1
Integrate  3x 2  5
dx

Solution
1 1 1
 dx   dx.......... .......... .......... .......... .......... .......... .(i )
3x 2  5 5  x 1
3
5
2

Let 3
5 x  cosh , dx  5
3 sinh d .......... .......... .......... .......... .......... .......... (ii)
1

1
dx 
1 1
 cosh2   1  5

sinh  d 
1
 d 
1
 c
 x 1 2 3
5 3 5 3 3
5


1
dx 
1
cosh1  x  c 3
5
3x  5 2
3

658
Example 93
1
Integrate  x2  4
dx

Solution
1 1 1
 x2  4
dx 
2   x 2
dx.......... .......... .......... .......... .......... .......... .(i )
  1
2
x
Let  cosh , dx  2 sinh d .......... .......... .......... .......... .......... .......... (ii)
2
1 1 1 1
2 sinh d   d    c
2
dx  
 x
2 2 cosh   1
2

  1
2
1  x
 dx  cosh1    c
x2  4 2

Example 94
1
Integrate  16x 2  1
dx

Solution
1 1
 16x 2  1
dx  
4 x 2  1
dx.......... .......... .......... .......... .......... .......... .(i )

Let 4 x  cosh , dx  14 sinh d .......... .......... .......... .......... .......... .......... (ii)


1
dx  
1
 14 sinh d  14  d  14   c
16x  1
2
cosh   1
2

dx  14 cosh1 4 x   c
1

16x 2  1

659
(c) Integration by using tanh inverse or coth inverse concepts
1
(i)  2 dx
a  b2 x2
1
(ii)  2 dx
a  b 2 x  
2

Evaluation of the above Integral


1
(i )  2 dx
a  b2 x2
Let x  ba tanh  , x 2  ba2 tanh 2  , dx  ba sec h 2d
2

 sec h 2d 
1 1
a 2
b x
2 2
dx   2
a  b b2 tanh 2 
2 a2

a
b

sec h 2
a
b sec h 
a 2

 a 2  a 2 tanh 2  d   a 2 1  tanh 2  d   d   c


b 1 1
ab ab

 a 2  b 2 x 2 dx  ab1 tanh  ba x   c
1

tanh 1  ba x   c
1
a 2
 b2 x2
dx  1
ab

Alternative
1
 a 2  b 2 x 2 dx
Let x  ba coth , x 2  a2
b2
coth2  , dx   ba cosech 2d

a
1
dx   2
1
 ba cosech 2d 
 
2
b x
2 2
a  b b 2 coth 
2 a2 2

 ba cosech 2  ba cosech 2
 a 2  a 2 coth2  d    a 2 coth2   1d  1
ab  d  1
ab  c

 a 2  b 2 x 2 dx  ab1 coth ba x   c
1

coth1  ba x   c
1
a 2
 b2 x2
dx  1
ab

660
Example 95
Integrate  1
dx
16  9 x 2
Solution
1 1
 16  9 x 2 dx  1
16  1   3 x 2 dx.......... .......... .......... .......... .......... ..(i)
4
Let 3
4 x  tanh  , dx  43 sec h 2d .......... .......... .......... .......... ........( ii)

 16  9 x
1 1

1
dx 
16 1  tanh 
2 2 3 sec h  d 
4 2 1
12 
1
d    c
12
 
1 1 3 
 dx  tanh 1  x   c
16  9 x 2
12 4 

1
(ii) a 2
 b x  c 2 2
dx

Let x  c  ba tanh  ,  x  c   tanh 2  , dx  ba sec h 2d


2 a2
b2

a
1
dx  

1
 sec h 2d 

a

 b x  c  a b
2 2 2 2 2 a2 2 b
b2
tanh
sec h 2
a
b sec h 
a 2

 a 2  a 2 tanh 2  d   a 2 1  tanh 2  d     d   c
b 1 1
ab ab

 a 2  b 2 x  c 2 dx  ab1 tanh  ba ( x  c)  c


1

tanh  ba ( x  c)   c
1
a 2
 b x  c 2 2
dx  1
ab

Alternative
1
a 2
 b 2 x  c 
2
dx

Let x  c  ba coth ,  x  c   coth2  , dx   ba cosech 2d


2 a2
b2

 coth   
1 1
a 2
 b x  c 
2 2
dx  
a b
2 2 a2 2
a
b cosech 2d
b2

 ba cosech 2  ba cosech 2
 a 2  a 2 coth2  d    a 2 coth2   1 d   
1
ab  d  1
ab A

 a 2  b 2 x  c 2 dx  ab1 coth ba ( x  c)  A


1

dx  ab1 coth ba ( x  c)   A
1
a 2
 b x  c 2 2

661
(d) Integration by using sech inverse concepts
1
(i )  dx
x a2  b2 x2
(ii)  x a 2  b 2 x 2 dx
1
(iii)  dx
x a 2  b 2 ( x  c) 2
(iv)  x a 2  b 2 ( x  c) 2 dx

Evaluation of the above Integral


1
(i )  dx
x a 2  b2 x2
Let x  ba sec h , x 2  a2
b2
sec h 2 , dx   ba sec hx tanh d

x
1
dx  
1
 ba sec h tanh d 
a b x
2 2 2 a
b sec h a  b 2

2 a2
b2
sec h 
2

 ba sec h tanh   tanh   tanh 
 a
sec h a  a sec h 
2 2 2
d  
a  a sec h 
2 2 2
d  
a 1  sec h 2
d
b

 tanh  1 1
 d   d    c but   sec h 1  ba x 
a tanh 2  a a
1
sec h 1  ba x   c
1
x a 2  b2 x2
dx 
a

1
sec h 1  ba x   c
1
x a 2  b2 x2
dx 
a

662
(ii)  x a 2  b 2 x 2 dx
Let x  ba sec h , x 2  a2
b2
sec h 2 , dx   ba sec hx tanh d

 x a  b x dx   ba sec h a  b
2 2 2 2 2
 a2
b2

sec h 2  ba sec h tanh d 

  sec h
a
b 
a 2  a 2 sec h 2  ba sec h tanh  d    ba2 sec h 2 tanh 2 d
3

du
Let u  tanh  , du  sec h 2d , d 
sec h 2
du
  ba2 sec h  tanh d   ba2  sec h  u sec h 2   ba2  u du   3ab2 u  c
3 2 2 3 2 2 3 2 3 3

 x a  b x dx   3ab2 tanh   c but   sec h  ba x 


2 2 2 3 3 1

 x a  b x dx   3ab2 tanh sec h  ba x   c


2 2 2 3 31

 x a  b x dx   3ab2 tanh sec h ba x  c


2 2 2 3 1 3
 
1
(iii)  dx
x a 2  b 2 ( x  c) 2
Let x  c  ba sec h , ( x  c) 2  a2
b2
sec h 2 , dx   ba sec hx tanh d

x
1
dx  
1
 ba sec h tanh d 
a  b ( x  c)
2 2 2 a
b sec h a  b 2

2 a2
b2
sec h  2

 ba sec h tanh   tanh   tanh 
 a
sec h a 2  a 2 sec h 2
d  
a 2  a 2 sec h 2
d  
a 1  sec h 2
d
b

 tanh  1 1
 d   d    A but   sec h 1  ba ( x  c) 
a tanh 2  a a
1
sec h 1  ba ( x  c)   A
1
x a 2  b 2 ( x  c) 2
dx 
a

1
sec h 1  ba ( x  c)   A
1
x a 2  b 2 ( x  c) 2
dx 
a

663
(iv)  x a 2  b 2 ( x  c) 2 dx
Let x  c  ba sec h , ( x  c) 2  a2
b2
sec h 2 , dx   ba sec hx tanh d

 x a b
2 2
( x  c) 2 dx   ba sec h a 2  b 2  a2
b2

sec h 2  ba sec h tanh d 

  sec h
a
b 
a 2  a 2 sec h 2  ba sec h tanh  d    ba2 sec h 2 tanh 2 d
3

du
Let u  tanh  , du  sec h 2d , d 
sec h 2
du
 b
 a3
b 
2 sec h  tanh d   2
a3
   ba2  u 2 du   3ab2 u 3  c
2 2 2 2 3 3
sec h u
sec h 
2

x a 2  b 2 ( x  c) 2 dx   3ab2 tanh 3   c but   sec h 1  ba ( x  c) 


3

x a 2  b 2 ( x  c) 2 dx   3ab2 tanh 3 sec h 1  ba ( x  c)   B


3

 x a  b ( x  c) dx   3ab2 tanh sec h ba ( x  c)  B


2 2 2 3 1 3
 
(e) Integration by using cosech inverse concepts
1
(i )  dx
x a2  b2 x2
(ii)  x a 2  b 2 x 2 dx
1
(iii)  dx
x a  b 2 ( x  c) 2
2

(iv)  x a 2  b 2 ( x  c) 2 dx
Integration of the above concepts
1
(i )  dx
x a  b2 x2
2

Let x  ba cos ech , x 2  a2


b2
cos ech2 , dx   ba cos ech cothd

x
1
dx  
1
 ba cosech cothd 
a b x
2 2 2 a
b
cos ech a  b 2

2 a2
b2
cos ech  2

 ba cos ech coth  coth  coth
 a
cos ech a 2  a 2 cos ech2
d  
a 2  a 2 cos ech2
d  
a 1  cos ech2
d
b

 coth 1 1
 d   d    c but   cos ech1  ba x 
a coth  2 a a
1
cos ech1  ba x   c
1
x a 2  b2 x2
dx 
a
1
cos ech1  ba x   c
1
x a b x
2 2 2
dx 
a

664
(ii)  x a 2  b 2 x 2 dx
Let x  ba cos ech , x 2  a2
b2
cos ech2 , dx   ba cos echx cothd

 x a  b x dx   ba cos ech a  b
2 2 2 2 2
 a2
b2

cos ech2  ba cos ech cothd 

 a
b 
cos ech a 2  a 2 cos ech2  ba cos ech coth d    ba2 cos ech2 coth2 d
3

du
Let u  coth , du   cos ech2d , d 
 cos ech2
du
 b
 a3
b 
2 cos ech  coth d   2
a3
  ba2  u 2 du  3ab2 u 3  c
2 2 2 2 3 3
cos ech u
 cos ech 
2

x a 2  b 2 x 2 dx  3ab2 coth3   c but   cos ech1  ba x 


3

x a 2  b 2 x 2 dx  3ab2 coth3 cos ech1  ba x   c


3

 x a  b x dx  3ab2 coth cosech ba x  c


2 2 2 3 1 3
 

1
(iii)  dx
x a  b x  c 
2 2 2

Let x  c  ba cos ech , ( x  c) 2  ba2 cos ech2 , dx   ba cos ech cothd


2

x
1
dx  
1
 ba cos ech cothd 
a 2  b 2 x  c 
2 a
b cos ech a  b 2 2 a2
b2
 cos ech 
2

 ba cos ech coth  coth  coth
 a
cos ech a  a cos ech 
2 2 2
d  
a  a cos ech 
2 2 2
d  
a 1  cos ech  2
d
b

 coth 1 1
 d   d    c but   cos ech1  ba ( x  c) 
a coth2  a a
1
cos ech1  ba ( x  c)   A
1
x a 2  b 2 x  c 
2
dx 
a

1
cos ech 1  ba ( x  c)   A
1
x a 2  b 2 x  c 
2
dx 
a

665
(iv)  x a 2  b 2 x  c  dx
2

Let x  c  ba cosech , ( x  c) 2  a2
cosech2 , dx   ba cosech cothd

 x a  b x  c  dx    cosech a  b  cosech  


b2

cosech cothd 
2 2 2 a 2 2 a2 2 a
b b2 b

  cosech a  a cosech   cosech coth d


a 2 2 2 a
b b

  cosech  coth d    cosech  coth  d


a3 a3
2 2 2 2
b2 b2

du
Let u  coth , du   cosech2d , d 
 cosech2
du
Therefore,  ba 2  cosech2coh2 d   ba 2  cosech2 u 2
3 3

 cosech2
 a3
b2  u du 
2 a3
3b 2
u 3  D but u  coth and   cosech1  ba ( x  c) 

 x a  b x  c  dx  coth3   D
2 2 2 a3
3b 2

x a 2  b 2 x  c  dx 
2 a3
3b 2

coth3 cosech1  ba ( x  c)   D 
 x a  b x  c  dx 
2 2 2 a3
3b 2

coth3 cos ech1  ba ( x  c)   D 
10.10 INTEGRATION BY USING DIFFERENT TECHNIQUES AND
SUBSTITUTION
(a) Integration by using half angle formula.
An integrand of trigonometric functions can be transformed into algebraic
integrand by means of substitution,
i.e. Consider the forms below
1
(i)  dx
a  b sin mx
1
(ii)  dx
a  b cos mx
1
(iii)  dx
a  b cos mx  c sin mx
1 t 2
By using half angle formula replace t  tan  x  , sin mx 
2t
and cos mx m
1 t 2 1 t 2
2

From t  tan  m2 x , dt  m2 sec2  m2 x dx, dx  m


dt 2dt 2dt
 
2 sec  2 x  m(1  tan  2 x ) m(1  t 2 )
2 m 2 m

2dt
dx  where m  
m(1  t 2 )

666
Example 96
Integrate  sec xdx
Solution
1
 sec xdx   cos x
dx.......... .......... .......... .......... .......... .......... .......... .......... .......... .(i )

1 t 2 1 t 2
Let t  tan  2 , dx 
2
x
dt and cos x  , sec x 
1 t 2 1 t 2 1 t 2
1 1 t 2  2  2
 cos x dx   sec xdx   1  t 2  1  t 2 dt   1  t 2 dt
2
 sec xdx   1  t 2 dt.......... .......... .......... .......... .......... .......... .......... .......... .........( ii)
2 2 A B
Express in partial fraction   
1 t 2
1  t 1  t  1  t 1  t
2  A1  t   B1  t 
A  1, B  1
2  1 1  1 1
 1  t 2   1  t 1  t   1  t  1  t dt
dt     dt  dt 

1 t 
 sec xdx  In1  t   In1  t   In 1  t  but t  tan  2x 

 1  tan  2x    cos 2x   sin 2x  


 sec xdx  In 
 1  tan  x  
 2 
  In 
 cos x
2   sin  
x 
2 

Rationalize the denominator


 cos 2x   sin 2x  cos 2x   sin 2x    1  2 sin 2x cos 2x    1  sin x 
 sec xdx  In cos 2x   sin 2x   cos 2x   sin 2x    In cos2  2x   sin 2  2x    In cos x 
 1 sin x 
 sec xdx  In cos x  cos x   Insec x  tan x   c
  sec xdx  In sec x  tan x   c

667
Example 97
Integrate  sec 2 xdx
Solution
1
 sec2 xdx   cos 2 x
dx.......... .......... .......... .......... .......... .......... .......... .......... .......... .(i )

1 1 t 2 1 t 2
Let t  tan x, dx  dt and cos x  , sec x 
1 t 2 1 t 2 1 t 2
1 1 t 2  1  1
 cos2 x dx   sec2 xdx   1  t 2  1  t 2 dt   1  t 2 dt
1
 sec xdx   1  t 2 dt.......... .......... .......... .......... .......... .......... .......... .......... .........( ii)
1 1 A B
Express in partial fraction   
1 t 2
1  t 1  t  1  t 1  t
1  A1  t   B1  t 
A  12 , B  12
1  12 1
2  1 1
 1  t 2   1  t 1  t dt  12  1  t dt  12  1  t dt
dt   

1 1 t 
 sec2 xdx  2 In1  t   2 In1  t   2 In 1  t  but
1 1
t  tan x

1  1  tan x  1  cos x  sin x 


 sec2 xdx  2 In 1  tan x   2 In cos x  sin x 
Rationalize the denominator
1  cos x  sin x cos x  sin x  1  1  2 sin x cos x  1  1  sin 2 x 
 sec 2 xdx  In    In   In
2  cos x  sin x cos x  sin x  2  cos2 x  sin 2 x  2  cos 2 x 

1  sin 2 x  1
 sec2 xdx  2 In cos2 x  cos2 x   2 Insec2 x  tan 2 x   c
1

  sec 2 xdx  In sec 2 x  tan 2 x   c


1
2

668
Example 98
Integrate  sec3xdx
Solution
1
 sec3xdx   cos3 x
dx.......... .......... .......... .......... .......... .......... .......... .......... .......... .(i )

1 t 2 1 t 2
Let t  tan  2 x , dx 
2
dt and cos3 x  , sec3 x 
 
3
3 1 t 2 1 t 2 1 t 2
1 1 t 2  2  2 1
 cos3x dx   sec3xdx   1  t 2  3 1  t 2   dt  
 3 1 t 2
dt

2 1
 sec3xdx  3  1  t 2
dt.......... .......... .......... .......... .......... .......... .......... .......... .........( ii)

1 2 A B
Express in partial fraction   
1  t 2 1  t 1  t  1  t 1  t
1  A1  t   B1  t 
A  12 , B  12
2 1 2  12 1
2  1 1 1 

3 1 t 2
dt     dt   
3 1 t 1 t  3  1 t
dt   dt
1  t 
1 1 t 
 sec3xdx  3 In1  t   In1  t   3 In 1  t  but t  tan  32 x 
1

1  1  tan  32x   1  cos 32x   sin 32x  


 sec 3 xdx  In   In 
3  1  tan  32x   3  cos 32x   sin 32x  
Rationalize the denominator
 cos 32x   sin 32x  cos 32x   sin 32x    1  2 sin 32x cos 32x    1  sin 3x 
 sec3xdx  In cos32x   sin32x   cos32x   sin32x    In cos2 32x   sin 2 32x    In cos3x 
 

 1 sin 3x  1
 sec3xdx  In cos3x  cos3x   3 Insec3x  tan 3x   c
  sec3xdx  In sec3x  tan 3x   c
1
3

669
Example 99
Integrate  sec 4 xdx
Solution
1
 sec 4 xdx   cos 4 x dx.......... .......... .......... .......... .......... .......... .......... .......... .......... .(i)
1 1 t 2 1 t 2
Let t  tan 2 x, dx  dt and cos 4 x  , sec 4 x 
2 1 t 2  
1 t 2 1 t 2
1 1 t 2  1  1 1
 cos 4 x dx   sec 4 xdx   1  t 2  2 1  t 2   dt  
 2 1  t 2
dt

1 1
 sec4 xdx  2  1  t 2
dt.......... .......... .......... .......... .......... .......... .......... .......... .........( ii)

1 1 A B
Express in partial fraction   
1 t 2
1  t 1  t  1  t 1  t
1  A1  t   B1  t 
A  12 , B  12
1 1 1  12 1
2  1 1 1 1
2  1 t 2  1 t 1 t  4  1 t 4  1 t
2
dt     dt  dt  dt

1 1 t 
 sec4 xdx  4 In1  t   4 In1  t   4 In 1  t  but
1 1
t  tan 2 x

1  1  tan 2 x  1  cos 2 x  sin 2 x 


 sec4 xdx  4 In 1  tan 2 x   4 In cos2 x  sin 2 x 
Rationalize the denominator
1  cos 2 x  sin 2 x cos 2 x  sin 2 x  1  1  2 sin 2 x cos 2 x  1  1  sin 4 x 
 sec 4 xdx  In    In   In
4  cos 2 x  sin 2 x cos 2 x  sin 2 x  4  cos2 2 x  sin 2 2 x  4  cos 4 x 

1  1 sin 4 x  1
 sec4 xdx  4 In cos4 x  cos4 x   4 Insec4 x  tan 4 x   c
  sec 4 xdx  In sec 4 x  tan 4 x   c
1
4

Example 100
Integrate  cosecxdx
Solution

670
1
 cosecxdx   sin x dx
1 t 2
Let t  tan  , dx 
2 2t
x
dt and sin x  , cosecx 
1 t 2 1 t 2
2
2t
1 1 t  2 
2
1
 sin x dx   cosecxdx   2t  1  t 2 dt   t dt
 cosecxdx   t dt  Int but t  tan  2x 
1

 sin 2x    sin 2x  sin 2x   sin 2  2x  


 cosecxdx  Intan  2   In  cos 2x   In  cos 2x   sin 2x   In  sin 2x cos 2x 
x

 2 sin 2  2x   1  cos x 
 cosecxdx  In  2 sin 2x cos 2x   In  sin x   Incosecx  cot x   c
  cosecxdx  In cosecx  cot x   c

Example 101
Integrate  cosec2 xdx
Solution
1
 cosec2 xdx   sin 2 x dx
1 2t 1 t 2
Let t  tan x, dx  dt and sin 2 x  , cos ec2 x 
1 t 2 1 t 2 2t
1 1 t  1 
2
1 1
 sin 2 x dx   cosec2 xdx   2t  1  t 2 dt  2  t dt
1 1 1
 cosec2 xdx  2  t dt  2 Int but t  tan x
1  sin x  1  sin x sin x  1  sin 2 x 
 
1
    In   In
2  cos x  2  cos x sin x  2  sin x cos x 
cos ec 2 xdx In tan x In
2
1  2 sin 2 x  1 1  cos 2 x  1
 cosec2 xdx  2 In  2 sin x cos x   2 In  sin 2 x   2 Incosec2 x  cot 2 x   c
In cosec2 x  cot 2 x   c
1
  cos ec2 xdx 
2

671
Example 102
Integrate  cosec4 xdx
Solution
1
 cosec4 xdx   sin 4 x
dx

1 2t 1 t 2
Let t  tan 2 x, dx  dt and sin 4 x  , cos ec4 x 
1 t 2 1 t 2 2t
1 1 t  1 
2
1 1
 sin 4 x dx   cosec4 xdx   2t  21  t 2  dt  4  t dt
1 1 1
 cosec4 xdx  4  t dt  4 Int but t  tan 2 x

1  sin 2 x  1  sin 2 x sin 2 x  1  sin 2 2 x 


 cosec4 xdx  4 Intan 2 x  4 In  cos 2 x   4 In  cos 2 x  sin 2 x   4 In  sin 2 x cos 2 x 
1

1  2 sin 2 2 x  1 1  cos 4 x  1
 cosec4 xdx  4 In  2 sin 2 x cos 2 x   4 In  sin 4 x   4 Incosec4 x  cot 4 x   c
  cos ec4 xdx  In cos ec4 x  cot 4 x   c
1
4

Example 103
1
Integrate  1  cos x dx
Solution
1
 1  cos x dx.......... .......... .......... .......... .......... .......... .......... .......... .......... .(i)
1 t 2
Let t  tan  2 , dx 
2
x
dt, cos x  .......... .......... .......... .......... ....( ii)
1 t 2 1 t 2
Substitute equation (ii) into equation (i)
1 1  2 
 1  cos x dx   1 t 2

 1  t 2 

dt   dt  t  c
1
1 t 2
dx  tan  2x   c
1

1  cos x

672
Alternative
1
 1  cos x dx.......... .......... .......... .......... .......... .......... .......... .......... .......... .......... .......... .....(i)
 x  x  x  x
But 1  cos2    sin 2   and cos x  cos2    sin 2  .......... .......... .......... .......... ...(ii)
2 2 2 2
Substitute equation (ii) into equation (i)
1 1 1 1  x
 1  cos x dx    x  x  x  x
dx  
 x
dx 
2  sec2  dx
2
cos2    sin 2    cos2    sin 2   2 cos2  
2 2 2 2 2
1  x
 dx  tan   c
1  cos x 2

Example 104
1
Integrate  1  cos x dx
Solution
1
 1  cos x dx.......... .......... .......... .......... .......... .......... .......... .......... .....( i)
1 t 2
Let t  tan  2x , dx 
2
dt , cos x  .......... .......... .......... .......( ii)
1 t 2 1 t 2
Substitute equation (ii) into equation (i)
1 1  2  1 1
 1  cos x dx    t2 
2 1
  dt  dt  t dt  t  
1 t 2 1 t 2  t
1
1 t 2

 1  cos x dx   tan  2x    cot 2x   c


1 1

dx   cot 2x   c
1

1  cos x

673
Alternative
1
 1  cos x dx.......... .......... .......... .......... .......... .......... .......... .......... .......... .......... .......... ....(i)
 x  x  x  x
But 1  cos2    sin 2   and cos x  cos2    sin 2  .......... .......... .......... .......... .....( ii)
2 2 2 2
Substitute equation (ii) into equation (i)
1 1 1 1  x
 1  cos x dx    x  x   x  x 
dx  
 x
dx   cosec 2  dx
2 2
cos2    sin 2    cos2    sin 2   2 sin 2  
2 2  2  2  2
1  x
 dx   cot   c
1  cos x 2

Example 105
1
Integrate  1  sin x dx
Solution
1
 1  sin x dx.......... .......... .......... .......... .......... .......... .......... .......... .......... ...(i)
Let t  tan  2x , dx 
2 2t
dt, sin x  .......... .......... .......... .......... .........( ii)
1 t 2
1 t 2
Substitute equation (ii) into equation (i)
1 1  2  2 1
 1  sin x dx   
2t  1  t  2 
dt   2
t  2 t  1
dt  2
t  12
dt
1
1 t 2
Let u  t  1, du  dt
1 1 2 2 2
 1  sin x  u2 
2 1
dx  2 du  2 u du  2u   
u t  1 1  tan  2x 
1 2
 dx  c
1  sin x 1  tan  2x 

674
Example 106
1
Integrate  1  sin x dx
Solution
1
 1  sin x dx.......... .......... .......... .......... .......... .......... .......... .......... .......... ...(i)
Let t  tan  2x , dx 
2 2t
dt , sin x  .......... .......... .......... .......... .........( ii)
1 t 2 1 t 2
Substitute equation (ii) into equation (i)
1 1  2  2 1
 1  sin x dx    2 
2t  1  t 
dt   2
t  2 t  1
dt  2 
t  12
dt
1
1 t 2
Let u  t  1, du  dt
1 1 2 2 2
 1  sin x  u2 
2 1
dx  2 du  2 u du  2u   
u t  1 tan  2x   1
1 2
 dx  c
1  sin x 1  tan  2x 

Example 107
1
Integrate  3  2 cos x dx
Solution
1
 3  2 cos x dx.......... .......... .......... .......... .......... .......... .......... .......... .......... .....( i)
1 t 2
Let t  tan  2 , dx 
2
x
dt, cos x  .......... .......... .......... .......... .......... ...(ii)
1 t 2 1 t 2
Substitute equation (ii) into equation (i)
1 1  2  2 2 5  t 
 3  2 cos x dx    2 
1 t  1 t 
2
dt  
5t 2
dt 
5
tan 1  .......... .(iii)
 5
3  2 
2 
1 t 
1 2 5  tan  2x 
 dx  tan 1  c
3  2 cos x 5  5 

675
Example 108
1
Integrate  2  cos x  sin x dx
Solution
1
 2  cos x  sin x dx.......... .......... .......... .......... .......... .......... .......... .......... ........( i)
1 t 2
Let t  tan  , dx 
2
x
dt, cos x  .......... .......... .......... .......... .......... ...(ii)
1 t 2 1 t 2
2

Substitute equation (ii) into equation (i)


1  2  1 2
 2  cos x  sin x dx   1 t

2t  1  t  22 
dt   2
t  2t  3
dt
2 
1 t 2 1 t 2
1 2 1 1  t  1 
 2  cos x  sin x dx   t  12  2 dt    t  1  2 dt  2 tan  2 
1  
 2
1  tan  2   1 
x
1
 dx  2 tan  c
2  cos x  sin x  2 

Examples 109
1
Integrate  a  b cos x dx
Solution
1
 a  b cos x dx.......... .......... .......... .......... .......... .......... .......... .......... .......... .......... .......... (i)
1 t 2
Let t  tan  , dx 
2
x
dt, cos x  .......... .......... .......... .......... .......... .......... ........( ii)
1 t 2 1 t 2
2

Substitute equation (ii) into equation (i)


1 1  2  2 2  a b 
 a  b cos x dx    a  b  a  bt
1
  dt  dt  tan  t
 1 t 2  1 t 
2 2
a b
2 2
 a  b 
a  b 
2 
1 t 
1 2  a b  x 
 dx  tan 1  tan    c
a  b cos x a2  b2  a  b  2 

676
p cos mx  q sin mx
(b) Integration of the form  a  b cos mx  c sin mx dx
Express,Numerator  A(Denominator)  B dxd (Denominator)  C

Example 110
cos x  sin x
Integrate  dx
2 cos x  3 sin x
Solution
cos x  sin x
 2 cos x  3 sin x dx.......... .......... .......... .......... .......... .......... .......... .........( i)
Express cos x  sin x  A2 cos x  3 sin x   B 2 cos x  3 sin x 
d
dx
cos x  sin x  A2 cos x  3 sin x   B 2 sin x  3 cos x 
Equate
2 A  3 B  1

3 A  2 B  1
5 1
A ,B 
13 13
cos x  sin x 
5
2 cos x  3 sin x   1  2 sin x  3 cos x .......... .......... .......( ii)
13 13
Substitute equation (ii) into equation (i)
cos x  sin x 5 2 cos x  3 sin x   1  2 sin x  3 cos x 

 2 cos x  3sin x   13 13
dx dx
2 cos x  3 sin x
cos x  sin x 5 2 cos x  3 sin x 1  2 sin x  3 cos x
 2 cos x  3sin x 13  2 cos x  3sin x 13  2 cos x  3sin x dx
dx  dx 

cos x  sin x
dx  x  In 2 cos x  3 sin x   c
5 1

2 cos x  3 sin x 13 13

677
Example 111
5 cos x  sin x
Integrate  dx
cos x  3 sin x
Solution
5 cos x  sin x
 cos x  3sin x dx.......... .......... .......... .......... .......... .......... .......... .........( i)
Express 5 cos x  sin x  Acos x  3 sin x   B cos x  3 sin x 
d
dx
5 cos x  sin x  Acos x  3 sin x   B sin x  3 cos x 
Equate
 A  3B  5

3 A  B  1
A  15 , B  8 5
5 cos x  sin x  15 cos x  3 sin x   8 5  sin x  3 cos x .......... .......... .......... (ii)
Substitute equation (ii) into equation (i)
5 cos x  sin x 1 cos x  3 sin x   8  sin x  3 cos x 

 cos x  3sin x dx  
5 5
dx
cos x  3 sin x
5 cos x  sin x 1 cos x  3 sin x 8  sin x  3 cos x
 cos x  3sin x 5  cos x  3sin x 5  cos x  3sin x dx
dx  dx 

5 cos x  sin x
dx  x  In cos x  3 sin x   c
1 8

cos x  3 sin x 5 5

Example 112
cos 2 x  sin 2 x
Integrate  dx
cos 2 x  3 sin 2 x
Solution
cos 2 x  sin 2 x
 cos 2 x  3 sin 2 x dx.......... .......... .......... .......... .......... .......... .......... .......... ....(i)
Express cos 2 x  sin 2 x  Acos 2 x  3 sin 2 x   B cos 2 x  3 sin 2 x 
d
dx
cos 2 x  sin 2 x  Acos 2 x  3 sin 2 x   B 2 sin 2 x  6 cos 2 x 

678
Equate
 A  6B  1

3 A  2 B  1
A   15 , B  15
cos x  sin x   1 5 cos 2 x  3 sin 2 x   1 5  2 sin 2 x  6 cos 2 x .......... .......... ........( ii)
Substitute equation (ii) into equation (i)
cos 2 x  sin 2 x  15 cos 2 x  3 sin 2 x   15  2 sin 2 x  6 cos 2 x 
 cos2 x  3sin 2 x  dx 
cos 2 x  3 sin 2 x
dx

cos 2 x  sin 2 x 1 cos 2 x  3 sin 2 x 1  2 sin 2 x  6 cos 2


 cos2 x  3sin 2 x dx  
5  cos 2 x  3 sin 2 x
dx 
5  cos 2 x  3 sin 2 x
dx

cos 2 x  sin 2 x
dx   x  In cos 2 x  3 sin 2 x   c
1 1

cos 2 x  3 sin 2 x 5 5

Example 113
cos x  sin x
Integrate  6  cos x  2 sin x dx
Solution
cos x  sin x
 6  cos x  2 sin x dx.......... .......... .......... .......... .......... .......... .......... .......... .......... .......... .......... ..(i)
Express cos x  sin x  A6  cos x  2 sin x   B 6  cos x  2 sin x   C
d
dx
cos x  sin x  A6  cos x  2 sin x   Bsin x  2 cos x   C
Equate
6 A  C  0

 A  2 B  1
2 A  B  1

A   3 5 , B  15 , C  18 5
cos x  sin x   3 5 6  cos x  2 sin x   15 sin x  2 cos x   18 5 .......... .......... .......... .......... .......... .....( ii)
Substitute equation (ii) into equation (i)
cos x  sin x  3 5 6  cos x  2 sin x   15 sin x  2 cos x   18 5
 6  cos x  2 sin x dx 
6  cos x  2 sin x
dx

cos x  sin x 3 6  cos x  2 sin x 1 sin x  2 cos x 18

 6  cos x  2 sin x       
5
dx dx dx dx
5 6  cos x  2 sin x 5 6  cos x  2 sin x 6  cos x  2 sin x
cos x  sin x
 6  cos x  2 sin x
3 1
  18 1
5  6  cos x  2 sin x
dx   x  In 6  cos x  2 sin x  dx.......... .......... .....( iii)
5 5

679
1
Consider the integral  dx
6  cos x  2 sin x
 x 2 1 t 2 2t
Let t  tan , dx  dt , cos x  , sin x 
2 1 t 2 1 t 2 1 t 2
1  2 
1 2 1
 6  cos x  2 sin x dx   1 t
 2 
 2t   1  t 
2
dt   2
7t  4t  5
dt  72 
t  72   49
2 31
dt
6  2 
1 t 2 1 t 2 

 6  cos x  2 sin x 31  1  7 t  2  2 dt, let 731 t  72   tan 


1 14 1

dx
31
7 
 6  cos x  2 sin x dx  31  1  tan  
1 14 1
2
31
7 
sec2  d 
2
31
 d 
2
31

2
31
tan 1  7
31
t  72 
1 2 1  7   x  2 
 6  cos x  2 sin x dx 
31
tan      .......... .......... .......... .......... .......... .......... .(iv)
 tan
 31   2  7 
cos x  sin x  7   x  2 
dx   x  In 6  cos x  2 sin x  
3 1 36
 tan 1   tan     c
6  cos x  2 sin x 5 5 5 31  31   2  7 

Exercise 10.3
Integrate the following;
cos x  sin x  2 cos x  sin x  8 2 cos x  3 sin x  7
1.  cos x  sin x dx 3.  cos x  sin x dx 5.  cos x  5 sin x dx
cos x  sin x 3 sin x cos 4 x
2.  dx 4.  dx 6.  cos 4 x  sin 4 x  10dx
cos x  sin x  6 cos x  2 sin x

(c) Integration of the form below,


px  q
(i )  2 dx
ax  bx  c
px  q
(ii)  dx
ax 2  bx  c
(iii)   px  q  ax 2  bx  c dx
For evaluation of any of these integral,

express px  q  A
d
dx
 
ax 2  bx  c  B where A and B are constants

to be evaluated by comparing the coefficients.

680
Example 114
3x  1
Evaluate x 2
 x 1
dx

Solution
3x  1
Consider  dx.......... .......... .......... .......... .......... .......... .......... .......... ..(i )
x2  x  1
3x  1  A
d 2
dx

x  x 1  B 
3x  1  A2 x  1  B
By equating A  32 , B  1
2

Then, 3 x  1  3
2 2 x  1  12 .......... .......... .......... .......... .......... .......... .......... .......( ii)
Substitute equation (ii) into equation (i)
3x  1 2 2 x  1  2 2 2 x  1
3 1 3 1

 x2  x  1  x2  x  1
dx  dx   x 2  x  1  x 2 2x  1dx
dx 

3x  1 2x  1 1
 x 2  x  1 2  x 2  x  1 2  x 2  x  1dx
dx  3
dx  1

3x  1
 1

 x 2  x  1dx  32 In x  x  1  12  x 2  x  1dx.......... .......... .......... .......... ...(iii)
2

1
Consider 12  2 dx
x  x 1
By completing the square
1 1
2  2 dx  12 
1
dx
x  x 1 x  12 2  34
Let x  12  2
3
tan  , dx  2
3
sec2 d
1
2 x 2
1
 x 1
dx  1
2  3
tan  
1
2 3
 2
3
sec2 d  3
3
 3
3
tan 1  2
3
x  12 .......( iv)
4 4

Substitute equation (iv) into equation (iii)

 2
3x  1
x  x 1
dx  32 In x 2  x  1  33 tan 1    2
3
x  12   c

681
1
(d) The form   px  q ax2  bx  c
dx

In order to solve the above integral let a linear function x  k  1t

1
(e) Integration of the form  ax 2

 b cx 2  d
dx

In order to solve the above integral let x  1t


1
(f) Integration of the form f g
dx where f and g are functions of x

If f is a linear or quadratic functions and g is linear function let g  t

ax  b
(g) Integration of the form  cx  d
dx

In order to solve the above integral, rationalize the numerator

px  q
(h) Integration of the form  m ax  b  n cx  d dx
In order to solve the above integral, rationalize the denominaor

1
(i) Integration of the form  p cos mx  q sin 2 mx
2
dx

In order to solve the above integral, divide by cos2 mx or sin 2 mx to each term

Exercise 10.4
Integrate the following;
1 x 1
1.  x  1 x 2  4x  2
dx 4.  x 1
dx

1

1
2.  x 2

1 x2  2
dx 5.
2x 1  x  3
dx

1
 x  2
1
3.
x3
dx 6.  4 cos2 x  5 sin 2 x dx
682
10.11 INTEGRATION BY PART
This is a method of integrating the product of two functions of a variable by
expressing in terms of two parts both of which are differentiable functions of
the same variable.
This method is normally applicable when the other techniques fails. The
method reduces two functions into one function to facilitate integration
computation.

Derivation of integration by part formula


Integration by part is derived from product rule
d
uv  u dv  v du
dx dx dx
Integrate both sides w.r.t x
d
 dx uv dx  u
dv
 dx  dx dx
dx  v
du

 d uv   u dxdx   v dx dx
dv du

du dv
uv   v dx   u dx
dx dx
dv du
 dx
u dx  uv   dx dx or  udv  uv   vdu
v

dv du
u dx
dx  uv   v dx or
dx  udv  uv   vdu
NOTE:
(i) U is easily differentiated function and dv is easily integrated function
(ii) The order of functions which should be taken as first function (U ) and which
should be taken as second function (dv). Let as U the function which appear firstly
from the word ILATE and the other termlet as dv,
I - stands for Inverse trigonometric functions
L - stands for Logarithmic functions
A - stands for Algebraic functions
T - stands for Trigonometric functions
E - stands for Exponential functions

683
Example 115
Integrate  xsin xdx
Solution

 x sin xdx.......... .......... .......... .......... .......... .......... .......... (i)


du
Let u  x,  1.......... .......... .......... .......... .......... .......( ii)
dx
dv
dx
 sin x,  dv   sin xdx, v   cos x.......... .......... .........( iii)
dv du
From Integration by part  u dx  uv   v dx
dx dx
 x sin xdx   x cos x    cos x dx
 x sin xdx   x cos x   cos xdx
  x sin xdx   x cos x  sin x  c

Alternative
 x sin xdx.......... .......... .......... .......... .......... .......... .......... ..(i)
Let u  x, du  dx.......... .......... .......... .......... .......... .......( ii)
dv  sin xdx,  dv   sin xdx, v   cos x.......... .......... .....( iii)
From Integration by part  udv  uv   vdu

 x sin xdx   x cos x    cos x dx


 x sin xdx   x cos x   cos xdx
  x sin xdx   x cos x  sin x  c

684
Example 116
Integrate  xcos xdx
Solution
 x cos xdx.......... .......... .......... .......... .......... .......... .......... ...(i)
Let u  x, du  dx.......... .......... .......... .......... .......... ........( ii)
dv  cos xdx,  dv   cos xdx, v  sin x.......... .......... .........( iii)
From Integration by part  udv  uv   vdu

 x cos xdx  x sin x   sin xdx


  x cos xdx  x sin x  cos x  c

Example 117
Integrate  xe x dx
Solution
 xe dx.......... .......... .......... .......... .......... .......... .......... ...(i)
x

Let u  x, du  dx.......... .......... .......... .......... .......... .....( ii)

 dv   e dx, v  e .......... .......... .......... ......( iii)


dv  e x dx, x x

From Integration by part  udv  uv   vdu

 xe dx  xe   e dx
x x x

  xe dx  xe  e  c
x x x

685
Example 118
Integrate  xe x dx
Solution
 xe dx.......... .......... .......... .......... .......... .......... .......... ...(i)
x

Let u  x, du  dx.......... .......... .......... .......... .......... ......( ii)


dv  e  x dx,  dv   e
x
dx, v  e  x .......... .......... .......... ..(iii)
From Integration by part  udv  uv   vdu

 xe dx   xe    e dx
x x x

 xe dx   xe   e
x x x

  xe dx   xe  e  c
x x x

Example 119
Integrate  x 2 cos xdx
Solution
 x cos xdx.......... .......... .......... .......... .......... .......... .......... ...(i)
2

Let u  x 2 , du  2 xdx.......... .......... .......... .......... .......... ......( ii)


dv  cos xdx,  dv   cos xdx, v  sin x.......... .......... .......... ..(iii)
From Integration by part  udv  uv   vdu

 x cos xdx  x sin x  2 x sin xdx.......... .......... .......... .......... (iv)


2 2

Consider  x sin xdx


Let u  x, du  dx
dv  sin xdx,  dv   sin xdx, v   cos x
From Integration by part  udv  uv   vdu

 x sin xdx   x cos x    cos x dx


 x sin xdx   x cos x   cos xdx
 x sin xdx   x cos x  sin x  c.......... .......... .......... .......... ......( v)
Substitute equation (v) into equation (iv)
  x 2 cos xdx  x 2 sin x  2 x cos x  2 sin x  c

686
Example 120
Integrate  xInxdx
Solution
 xInxdx.......... .......... .......... .......... .......... .......... .....( i)
Let u  Inx, du  1x dx.......... .......... .......... .......... ....( ii)
x2
dv  xdx,  dv   xdx, v  .......... .......... .......... (iii)
2
From Integration by part  udv  uv   vdu
1 1 2 1 
 xInxdx  2 x 2  x
Inx 
x  dx
2

1 2 1
 xInxdx  2 x Inx  2  xdx
1 1
  xInxdx  x 2 Inx  x 2  c
2 4

Example 121
Integrate  x 2 Inxdx
Solution
 x Inxdx.......... .......... .......... .......... .......... .......... .......... (i)
2

Let u  Inx, du  1x dx.......... .......... .......... .......... .........( ii)


1
dv  x 2 dx,  dv   x dx, v  x 3 .......... .......... .......... (iii)
2

3
From Integration by part  udv  uv   vdu
1 1 1
x Inxdx  x 3 Inx   x 3  dx
2

3 3  x
1 3 1 2
  
3
2
x Inxdx x Inx x dx
3
1 1
  x 2 Inxdx  x 3 Inx  x 3  c
3 12

687
Example 122
Integrate  xInx2 dx
Solution
 xInx dx.......... .......... .......... .......... .......... .......... .....( i)
2

Let u  Inx 2 , du  2x dx.......... .......... .......... .......... ....( ii)


x2
dv  xdx,  dv   xdx, v  .......... .......... .......... (iii)
2
From Integration by part  udv  uv   vdu
1 2 2 x2  2 
 xInx dx  2 x Inx   2  x dx
2

1 2 2
  x Inx   xdx
2
xInx dx
2
1 1
  xInx 2 dx  x 2 Inx 2  x 2  c
2 2

Example 123
Integrate  Inxdx
Solution
 Inxdx.......... .......... .......... .......... .......... .......... .......( i)
Let u  Inx, du  1x dx.......... .......... .......... .......... ....( ii)
dv  dx,  dv   dx, v  x.......... .......... .......... ......( iii)
From Integration by part  udv  uv   vdu
1
 Inxdx  xInx   x x dx
 Inxdx  xInx   dx
  Inxdx  xInx  x  c

688
Example 124
Integrate  Inx 2
dx
Solution
  Inx 2
dx.......... .......... .......... .......... .......... .......... ....( i )

Let u  Inx  , du 
2 2 Inx
dx.......... .......... .......... ......( ii)
x
dv  dx,  dv   dx, v  x.......... .......... .......... .........( iii)
From integration by part  udv  uv   vdu
 2 Inx 
 Inx  dx  xInx    x dx
2 2

x 

 Inx  dx  xInx   2  Inxdx.......... .......... .......... ...( iv)


2 2

Consider  Inxdx
Let u  Inx, du  1x dx
dv  dx,  dv   dx, v  x
From integration by part  udv  uv   vdu
1
 Inxdx  xInx    x dx
x

 Inxdx  xInx   dx
 Inxdx  xInx  x.......... .......... .......... .......... .......... ...( v)
Substitute equation (v) into equation (iv)

 Inx  dx  xInx   2xInx  x   c


2 2

  Inx  dx  xInx   2 xInx  2 x  c


2 2

Example 125
 Inx dx
3
Integrate
Solution

689
 Inx dx.......... .......... .......... .......... .......... .......... .........( i)
3

3Inx 
2
Let u  Inx  , du 
3
dx.......... .......... .......... ........( ii)
x
dv  dx,  dv   dx, v  x.......... .......... .......... .......... ...(iii)
From integration by part  udv  uv   vdu
 3Inx 2 
 Inx dx  xInx   x x dx
3 3

 Inx dx  xInx  3 Inx  dx.......... .......... .......... .......( iv)


3 3 2

Consider

 Inx  dx
2

Let u  Inx  , du 
2 2 Inx
dx
x
dv  dx,  dv   dx, v  x
From integration by part  udv  uv   vdu
 2 Inx 
  Inx 2
dx  x  Inx 2
   dx
x
 x 
 Inx  dx  xInx   2 Inxdx.......... .......... .......... ...( v)
2 2

Substitute equation (v) into equation (iv)

  Inx 3
dx  x  Inx 3
 3x  Inx 2
 2 Inxdx

 Inx  dx  xInx   3xInx  6 Inxdx.......... ........( vi)


3 3 2

690
Example 126
Integrate  x 3e x dx
2

Solution

 x e dx   x xe dx
3 x 2
2 x
  2

Let u  x 2 , du  2 xdx
dv  xe x dx,  dv   xe x dx, v  12 e x
2 2 2

From integration by part  udv  uv   vdu

 x e dx  12 x e   xe dx
2 2 2
3 x 2 x x

  x 3e x dx  12 x 2 e x  12 e x  c
2 2 2

Example 127
Integrate x
3
 
cos x 2 dx
Solution
x
3
 
cos x 2 dx   x 2 x cos x 2 dx.......... .......... .......... .......... ..(i )
Let u  x 2 , du  2 xdx.......... .......... .......... .......... .......... ...( ii)

 dv   x cos x dx, v  sin x


dv  x cos x 2 dx, 2 1 2
2 .......... ......( iii)
From integration by part  udv  uv   vdu

 x cos x dx  x sin x   x sin x dx


3 2 1 2 2 2
2

  x cos x dx  x sin x  cos x  c


3 2 1
2
2 2 1
2
2

691
Example 128
Integrate  x sin x cos xdx
Solution
 x sin x cos xdx   x sin 2 xdx.......... .......... .......... .......... ..(i)
1
2

Let u  x du  dx.......... .......... .......... .......... .......... .........( ii)


dv  sin 2 xdx,  dv   sin 2 xdx, v   cos 2 x.......... .........( iii)
1
2

From integration by part  udv  uv   vdu

 x sin x cos xdx   x cos 2 x   cos 2 xdx


1
2
1
2
1
2

 x sin x cos xdx   x cos 2 x  sin 2 x   c


1
2
1
2
1
4

  x sin x cos xdx  sin 2 x  x cos 2 x  c


1
8
1
4

Alternative
 x sin x cos xdx.......... .......... .......... .......... .......... .......... .......... ....(i)
Let u  x du  dx.......... .......... .......... .......... .......... .......... .........( ii)
dv  sin x cos xdx,  dv   sin x cos xdx, v   sin 1 2
2 x.......... .....( iii)
From integration by part  udv  uv   vdu

 x sin x cos xdx   x sin x   sin xdx


1 2 1 2
2 2

 x sin x cos xdx   x sin x   1  cos 2 x dx


1 2 1 1
2 2 2

 x sin x cos xdx   x sin x   x  sin 2 x   c


1 2 1 1 1
2 2 2 2

  x sin x cos xdx  x  x sin x  sin 2 x  c


1
4
1
2
2 1
8

Example 129
Integrate  x  3sec2 xdx
Solution
 x  3sec
2
xdx.......... .......... .......... .......... .......... .......... ..(i )
Let u  x  3, du  dx.......... .......... .......... .......... .......... ...( ii)
dv  sec2 xdx,  dv   sec xdx, v  tan x.......... .......... .(iii)
2

From integration by part  udv  uv   vdu

 x  3sec xdx  x  3 tan x   tan xdx


2

   x  3sec xdx  x  3 tan x  In cos x   c


2

692
Alternative
 x  3sec xdx   x sec2 xdx   3 sec2 xdx
2

 x  3sec xdx   x sec2 xdx  3 tan x.......... .......... .........( i )


2

Consider the integral  x sec2 xdx


Let u  x du  dx
dv  sec2 xdx,  dv   sec2 xdx, v  tan x
From integration by part  udv  uv   vdu

 x sec xdx  x tan x   tan xdx


2

 x sec
2
xdx  x tan x  In cos x .......... .......... .......... .......( ii)
Substitute equation (ii) into equation (i)

 x  3sec xdx  x tan x  Incos x   3 tan x  c


2

   x  3sec xdx  x tan x  3 tan x  In cos x   c


2

Example 130
Inx
Integrate  5 dx
x
Solution
Inx
 x 5 dx.......... .......... .......... .......... .......... .......... .......... .(i)
1
Let u  Inx, du  dx.......... .......... .......... .......... ........( ii)
x
1 1
dv  5 dx,  dv   x 5 dx, v   4 .......... .......... ......( iii)
x 4x
From integration by part  udv  uv   vdu
Inx Inx 1 1
 x5 dx  
4x4 4  x5
 dx

Inx Inx 1 5
 x 5 dx   4 x 4  4  x dx
Inx Inx 1
  5 dx   4  c
x 4 x 16x 4

693
FORMS UNDER INTEGRATION BY PARTS
(a) Integration of the form  e ax sin bxdx and e
ax
cosbxdx

Example 131
Integrate  e x sin xdx
Solution
e
x
sin xdx.......... .......... .......... .......... .......... .......... .......... .(i )
Let u  sin x, du  cos xdx.......... .......... .......... .......... .......( ii)
dv  e x dx,  dv   e dx, v  e x .......... .......... .......... .........( iii)
x

From integration by part  udv  uv   vdu

e sin xdx  e x sin x   e x cos xdx.......... .......... .......... .......( iv)


x

Consider  e x cos xdx


Let u  cos x, du   sin xdx
dv  e x dx,  dv   e dx, v  ex
x

From integration by part  udv  uv   vdu

e cos xdx  e x cos x   e x sin xdx.......... .......... .......... .......( v)


x

Substitute equation (v) into equation (iv)

 e sin xdx  e sin x  e cos x   e sin xdx


x x x x

 e sin xdx  e sin x  e cos x   e sin xdx


x x x x

 e sin xdx   e sin xdx  e sin x  e cos x


x x x x

2 e sin xdx  e sin x  e cos x


x x x

  e sin xdx  e sin x  e cos x   c


x 1
2
x x

694
Example 132
Integrate  e 3 x cos 2 xdx
Solution
e
3x
cos 2 xdx.......... .......... .......... .......... .......... .......... .......... .(i )
Let u  cos 2 x, du  2 sin 2 xdx
dv  e 3 x dx,  dv   e dx, v  13 e 3 x
3x

From integration by part  udv  uv   vdu

e cos 2 xdx  13 e 3 x cos 2 x  23  e 3 x sin 2 xdx.......... .......... .......... .......( ii)


3x

Consider  e 3 x sin 2 xdx


Let u  sin 2 x, du  2 cos 2 xdx
dv  e 3 x dx,  dv   e dx, v  13 e 3 x
3x

From integration by part  udv  uv   vdu

e sin 2 xdx  13 e 3 x sin x  23  e 3 x cos 2 xdx.......... .......... .......... .......( iii)


3x

Substitute equation (iii) into equation (ii)

 e cos2 xdx  e cos2 x   e sin 2 x   e cos2 xdx


3x 1 3x 2 1 3x 2 3x
3 3 3 3

 e cos2 xdx  e cos2 x  e sin 2 x   e cos2 xdx


3x 1 3x 2 3x 4 3x
3 9 9

 e cos2 xdx   e cos2 xdx  e cos2 x  e sin 2 x


3x 4 3x 1 3x 2 3x
9 3 9

 e cos2 xdx  e cos2 x  e sin 2 x


13 3x 1 3x 2 3x
9 3 9

 e cos2 xdx   e cos2 x  e sin 2 x   c


3x 9 1 3x 2 3x
13 3 9

  e cos 2 xdx  e cos 2 x  e sin 2 x  c


3x 3
13
3x 2
13
3x

695
Example 133
Integrate  e x cos xdx
Solution
e
x
cos xdx.......... .......... .......... .......... .......... .......... .......... .(i )
Let u  cos x, du   sin xdx
dv  e x dx,  dv   e dx, v  ex
x

From integration by part  udv  uv   vdu

e cos xdx  e x cos x   e x sin xdx.......... .......... .......... .......( ii)


x

Consider  e x sin xdx


Let u  sin x, du  cos xdx
dv  e x dx,  dv   e dx, v  ex
x

From integration by part  udv  uv   vdu

e sin xdx  e x sin x   e x cos xdx.......... .......... .......... .......( iii)


x

Substitute equation (iii) into equation (ii)

 e cos xdx  e cos x  e sin x   e cos xdx


x x x x

 e cos xdx   e cos xdx  e cos x  e sin x


x x x x

2  e cos xdx  e cos x  e sin x


x x x

  e cos xdx  e cos x  e sin x   c


x 1
2
x x

(b) Integration of the form  secn xdx and  cosec xdx where n is odd number
n

Example 134
Integration  sec3 xdx
Solution

696
 sec xdx   sec x sec2 xdx.......... .......... .......... .......... .......... ......( i )
3

Let u  sec x, du  sec x tan xdx.......... .......... .......... .......... .......( ii)
dv  sec2 xdx,  dv   sec xdx, v  tan x.......... .......... .......... ..(iii)
2

From  udv  uv   vdu

 sec xdx  sec x tan x   tan x sec xdx but tan x  sec x  1
3 2 2 2

 sec xdx  sec x tan x   sec x  1sec xdx


3 2

 sec xdx  sec x tan x   sec xdx   sec xdx


3 3

 sec xdx   sec xdx  sec x tan x   sec xdx


3 3

2 sec xdx  sec x tan x  In sec x  tan x 


3

  sec xdx  sec x tan x  In sec x  tan x   c


3 1
2

Example 135
Integration  sec3 2 xdx
Solution
 sec 2 xdx   sec2 x sec 2 xdx.......... .......... .......... .......... .......... .......... ..(i)
3 2

Let u  sec 2 x, du  2 sec 2 x tan 2 xdx.......... .......... .......... .......... .......... .(ii)
dv  sec2 2 xdx,  dv   sec 2 xdx, v  12 tan 2 x.......... .......... .......... ......( iii)
2

From  udv  uv   vdu


 sec 2 xdx  sec2 x tan 2 x   tan 2 x sec2 xdx but tan 2 x  sec2 2 x  1
3 1 2 2
2

 sec 2 xdx  sec2 x tan 2 x   sec 2 x  1sec2 xdx


3 1 2
2

 sec 2 xdx  sec2 x tan 2 x   sec 2 xdx   sec2 xdx


3 1 3
2

 sec 2 xdx   sec 2 xdx  sec2 x tan 2 x   sec2 xdx


3 3 1
2

2 sec 2 xdx  sec 2 x tan 2 x  In sec 2 x  tan 2 x 


3 1
2
1
2

  sec 2 xdx  sec 2 x tan 2 x  In sec 2 x  tan 2 x   c


3 1
4
1
4

697
Example 136
Integration  cosec3 xdx
Solution
 cosec xdx   cosecx cosec xdx.......... .......... .......... .......... .......... ......( i)
3 2

Let u  cosecx, du   cosecx cot xdx.......... .......... .......... .......... .......( ii)
dv  cosec 2 xdx,  dv   cosec xdx, v   cot x.......... .......... .......... ..(iii)
2

From  udv  uv   vdu

 cosec xdx   cosecx cot x   cot x cosecxdx but cot x  cosec x  1


3 2 2 2

 cosec xdx   cosecx cot x   cosec x  1cosecxdx


3 2

 cosec xdx   cosecx cot x   cosec xdx   cosecxdx


3 3

 cosec xdx   cosec xdx   cosecx cot x   cosecxdx


3 3

2 cosec xdx   cosecx cot x  In cosecx  cot x 


3

  cosec xdx   cosecx cot x  In cosecx  cot x   c


3 1
2

 f x dx
1
(c) Integration of the form
where f are inverse trigonometric functions or hyperbolic functions

Example 137
Integrate  sin 1 xdx
Solution

698
Example 138
Integrate  cos1 xdx
Solution
 cos
1
xdx.......... .......... .......... .......... .......... .......... .......... .......( i )
1
Let u  cos1 x, du  dx.......... .......... .......... .......... ....( ii)
1 x 2

dv  dx,  dv   dx, v  x.......... .......... .......... .......... .......... ..(iii)


From  udv  uv   vdu
x
 cos xdx  x cos1 x  
1
dx.......... .......... .......... .......... ..(iv)
1 x 2

x
Consider  1 x2
dx

Let u  1  x 2 , u 2  1  x 2 , 2udu  2 xdx, dx   ux du


x x u 
 1  x 2  u   x du   du  u   1  x
 2
dx

x
 1  x 2 dx   1  x .......... .......... .......... .......... .......... .......( v)
2

Substitute equation (v) into equation (iv)


  cos1 xdx  x cos1 x  1  x 2  c

699
Example 139
Integrate  sin 1 3xdx
Solution
 sin
1
3xdx.......... .......... .......... .......... .......... .......... .......... .......( i )
3
Let u  sin 1 3 x, du  dx.......... .......... .......... .......... ....( ii)
1 9x2
dv  dx, dv   dx, v  x.......... .......... .......... .......... .......... ..(iii)
From  udv  uv   vdu
3x
 sin 3xdx  x sin 1 3x  
1
dx.......... .......... .......... .......... ..(iv)
1 9x2
3x
Consider  1  9x 2
dx

Let u  1  9 x 2 , u 2  1  9 x 2 , 2udu  18xdx, dx   9ux du


3x 3x  u 
 dx     du   13  du   13 u   13 1  9 x
2

1  9x 2 u  9x 
3x
 1  9x 2
dx   13 1  9 x 2 .......... .......... .......... .......... .......... .......( v)

Substitute equation (v) into equation (iv)


  sin 1 3 xdx  x sin 1 3 x  13 1  9 x 2  c

Example 140
Integrate  tan 1 xdx
Solution
 tan
1
xdx.......... .......... .......... .......... .......... .......... .......... .......( i )
1
Let u  tan 1 x, du  dx.......... .......... .......... .......... ....( ii)
1 x2
dv  dx,  dv   dx, v  x.......... .......... .......... .......... .......... ..(iii)
From  udv  uv   vdu
x
 tan xdx  x tan x  
1 1
dx
1 x2
  tan 1 xdx  x tan 1 x  12 In 1  x 2   c

700
Example 141
Integrate  cot1 xdx
Solution
 cot
1
xdx.......... .......... .......... .......... .......... .......... .......... .......( i )
1
Let u  cot 1 x, du  dx.......... .......... .......... .......... ....( ii)
1 x2
dv  dx, dv   dx, v  x.......... .......... .......... .......... .......... ..(iii)
From  udv  uv   vdu
x
 cot xdx  x cot 1 x  
1
dx
1 x2
  cot 1 xdx  x cot 1 x  12 In 1  x 2   c

Example 142
Integrate  sec1 xdx
Solution
 sec
1
xdx.......... .......... .......... .......... .......... .......... .......... ...( i )
1
Let u  sec1 x, du  dx.......... .......... .......... .........( ii)
x x 2 1
Let dv  dx,  dv   dx, v  x.......... .......... .......... .......... ..(iii)
From integration by part  udv  uv   vdu
x
 sec xdx  x sec1 x  
1
dx
x x 2 1
1
 sec xdx  x sec x   x 2  1 dx.......... .......... .......... .......( iv)
1 1

1
Consider  dx
x 2 1
Let x  cosh , dx  sinh d
1 1
 x 2  1 dx   cosh2   1 sinhd   d    cosh x
1

1
 x 2  1 dx  cosh x.......... .......... .......... .......... .......... ........( v)
1

Substitute equation (v) into equation (iv)


  sec1 xdx  x sec1 x  cosh1 x  c

701
Example 143
Integrate  sinh 1 xdx
Solution
 sinh
1
xdx.......... .......... .......... .......... .......... .......... .......... ...(i )
1
Let u  sinh 1 x, du  dx.......... .......... .......... .........( ii)
x2 1
Let dv  dx,  dv   dx, v  x.......... .......... .......... .......... ..(iii)
From integration by part  udv  uv   vdu
x
 sinh xdx  x sinh x  
1 1
dx
x2 1
  sinh 1 xdx  x sinh 1 x  x 2  1  c

Example 144
Integrate  cosh1 xdx
Solution
 cosh xdx.......... .......... .......... .......... .......... .......... .......... ...(i)
1

1
Let u  cosh1 x, du  dx.......... .......... .......... .........( ii)
x 2 1
Let dv  dx,  dv   dx, v  x.......... .......... .......... .......... ..(iii)
From integration by part  udv  uv   vdu
x
 cosh xdx  x cosh x  
1 1
dx
x 2 1
  cosh1 xdx  x cosh1 x  x 2  1  c

702
Example 145
Integrate  tanh 1 xdx
Solution
 tanh xdx.......... .......... .......... .......... .......... .......... .......... ...(i)
1

1
Let u  tanh 1 x, du  dx.......... .......... .......... .......... ...(ii)
1 x2
Let dv  dx,  dv   dx, v  x.......... .......... .......... .......... ....( iii)
From integration by part  udv  uv   vdu
x
 tanh xdx  x tanh x  
1 1
dx
1 x2
  tanh 1 xdx  x tanh 1 x  12 In 1  x 2   c

Example 146
Integrate  tanh 1 4 xdx
Solution
 tanh
1
x 4dx.......... .......... .......... .......... .......... .......... .......... ...( i )
4
Let u  tanh 1 4 x, du  dx.......... .......... .......... .......... (ii)
1  16x 2
Let dv  dx,  dv   dx, v  x.......... .......... .......... .......... ....( iii)
From integration by part  udv  uv   vdu
4x
 tanh 4 xdx  x tanh 1 4 x  
1
dx
1  16x 2
  tanh 1 4 xdx  x tanh 1 4 x  18 In 1  16x 2   c

703
10.12 INTEGRATION BY REDUCTION FORMULA
Reduction formula are formula used to integrate trigonometric functions of
higher power.

(a) Reduction formula for  sin n xdx


Let I n   sin n xdx.......... .......... .......... .......... .......... .......... .........( i )
I n   sin n1 x sin xdx
Let u  sin n1 x , du  (n  1) sin n2 x cos xdx
dv
 sin x, dv  sin xdx,  dv   sin xdx, v   cos x
dx
From integration by part formula I n  uv   vdu
I n   sin n1 x cos x   ( cos x)(n  1) sin n2 x cos xdx
I n   sin n1 x cos x  (n  1)  cos2 x sin n2 xdx

I n   sin n1 x cos x  (n  1)  1  sin 2 x sin n2 xdx 

I n   sin n1 x cos x  (n  1)  sin n2 xdx   sin n xdx 
But  sin n2 xdx  I n2 and  sin
n
xdx  I n
I n   sin n1 x cos x  (n  1)I n2  I n 
I n   sin n1 x cos x  (n  1) I n2  (n  1) I n
I n  (n  1) I n   sin n1 x cos x  (n  1) I n2
nI n   sin n1 x cos x  (n  1) I n2
In  1
n  sin n 1

x cos x  (n  1) I n2 .......... .......... .......... .......... ...( ii)
Equate equation (i) and equation (ii)

 sin
n
xdx  1
n  sin n 1
x cos x  (n  1) I n2  c
 sin
n

xdx  1n  sin n1 x cos x  (n  1) I n2  c 

704
Example 147
By using reduction formula integrate  sin 2 xdx
Solution
 sin
2
xdx.......... .......... .......... .......... .......... .......... ........( i )
From I n   sin n xdx  1
n
 sin n 1
x cos x  (n  1) I n  2 
 sin
2
xdx  1
2  sin 2 1
x cos x  (2  1) I 2 2 
 sin
2
xdx  1
2
 sin x cos x  I 0 .......... .......... .......... ........( ii)
From I n   sin n xdx
I 0   sin 0 xdx   sin x  dx   dx  0.......... .......... ........( iii)
0

Substitute I 0  x into equation (ii)

 sin xdx   sin x cos x  x   c


2 1
2

  sin xdx  x  sin x cos x  c


2 1
2
1
2

Example 148
By using reduction formula integrate  sin 4 xdx
Solution
 sin
4
xdx.......... .......... .......... .......... .......... .......... .......... ..(i )
From I n   sin n xdx  1
n  sin n 1
x cos x  (n  1) I n2 
 sin
4
xdx  1
4  sin 41
x cos x  (4  1) I 42 
 sin
4
xdx  1
4  sin 3

x cos x  3I 2 .......... .......... .......... .........( ii)
From I n   sin n xdx

705
(b) Reduction formula for  cosn xdx
Let I n   cosn xdx.......... .......... .......... .......... .......... .......... .........( i )
I n   cosn1 x cos xdx
Let u  cosn1 x , du  (n  1) cosn2 x sin xxdx
dv
 cos x, dv  cos xdx,  dv   cos xdx, v  sin x
dx
From integration by part formula I n  uv   vdu
I n  cosn1 x sin x    sin x(n  1) cosn2 x sin xdx
I n  cosn1 x sin x  (n  1)  sin 2 x cosn2 xdx

I n  cosn1 x sin x  (n  1)  1  cos2 x cosn2 xdx 

I n  cosn1 x sin x  (n  1)  cosn2 xdx   cosn xdx 
But  cosn2 xdx  I n2 and  cos
n
xdx  I n
I n  cosn1 x sin x  (n  1)I n2  I n 
I n  cosn1 x sin x  (n  1) I n2  (n  1) I n
I n  (n  1) I n  cosn1 x sin x  (n  1) I n2
nI n  cosn1 x sin x  (n  1) I n2
In  1
n cos n 1

x sin x  (n  1) I n2 .......... .......... .......... .......... ...(ii)
Equate equation (i) and equation (ii)

 cos
n
xdx  1
n cos
n 1
x sin x  (n  1) I n2  c 
 cos
n

xdx  1n cosn1 x sin x  (n  1) I n2  c 
706
Example 149
Integrate by reduction formula  cos2 xdx
Solution
 cos xdx.......... .......... .......... .......... .......... .........( i)
2

From I n   cosn x  1
n cos n 1
x sin x  (n  1) I n2 
 cos
2

x  12 cos21 x sin x  (2  1) I 22 
 cos
2
x  12 cos x sin x  I 0 
But I 0   cos0 xdx   dx  x

 cos x  cos x sin x  x 


2 1
2

  cos x  cos x sin x  x  c


2 1
2
1
2

(c) Reduction formula for  tan n xdx


Let I n   tan n xdx.......... .......... .......... .......... .......... .......... (i )
I n   tan n xdx   tan n2 x tan 2 xdx   tan n2 x sec2 x  1 dx  
 tan x sec2 xdx   tan n2 xdx   tan n2 x sec2 xdx  I n2
n2

I n   tan n2 x sec2 xdx  I n2 .......... .......... .......... .......... ........( ii)
du
Let u  tan x, du  sec2 xdx, dx 
sec2 x
du
I n   u n2 sec2 x  I n2
sec2 x
1 n1
I n   u n2 du  I n2  u  I n2
n 1
1
In  tan n1 x  I n2 .......... .......... .......... .......... .......... ......( iii)
n 1
Equate equation (i) and equation (iii)
1
  tan n1 x  I n2  C
n
tan xdx
n 1
1
 tan xdx  tan n 1 x  I n  2  C
n

n 1

707
Example 150
By using reduction formula integrate  tan 2 xdx
Solution

 tan
2
xdx
From  tan n xdx  1
n 1 tan n1 x  I n2

 tan xdx  tan 21 x  I 22


2 1
21

 tan xdx  tan x  I 0


2

But I 0  x
  tan 2 xdx  tan x  x  c

(d) Reduction formula for  cotn xdx


Let I n   cot n xdx.......... .......... .......... .......... .......... .......... (i )
I n   cotn xdx   cot n2 x cot 2 xdx   cot n2 x cosec 2 x  1 dx  
 cot x cosec 2 xdx   cot n2 xdx   cot n2 x cosec 2 xdx  I n2
n2

I n   cotn2 x cosec 2 xdx  I n2 .......... .......... .......... .......... ........( ii)
du
Let u  cot x, du   cosec 2 xdx, dx 
 cosec 2 x
 du
I n   u n2 cosec 2 x  I n2
cosec 2 x
1 n1
I n    u n2 du  I n2  u  I n2
n 1
1
In  cot n1 x  I n2 .......... .......... .......... .......... .......... ......( iii)
n 1
Equate equation (i) and equation (iii)
1
  cot n1 x  I n2  C
n
cot xdx
n 1
1
 cot xdx  cotn 1 x  I n  2  C
n

n 1

708
Example 151
By using reduction formula integrate  cot2 xdx
Solution
 cot
2
xdx

 cot xdx  1
cot n 1 x  I n  2
n
From n 1

 cot xdx  1
cot 21 x  I 2 2
2
2 1

 cot xdx   cot x  I 0


2

But I 0   cot0 xdx   dx  x


  cot 2 xdx   cot x  x  c

(e) Reduction formula for  secn xdx


Let I n   secn xdx.......... .......... .......... .........( i )
I n   secn2 x sec2 xdx
Let u  secn2 x, du  (n  2) secn3 x sec x tan xdx  (n  2) secn2 x tan xdx
dv
 sec2 x , dv  sec2 xdx ,  dv   sec2 xdx, v  tan x
dx
From integration by parts I n  uv   vdu

I n  tan x secn2 x   tan x (n  2) secn2 x tan x dx 
I n  tan x secn2 x  (n  2)  tan 2 x secn2 xdx
I n  tan x secn2 x  (n  2)  sec2 x  1secn2 xdx

I n  tan x secn2 x  (n  2)  secn xdx   secn2 xdx 
I n  tan x secn2 x  (n  2)I n  I n2 
I n  tan x secn2 x  (n  2) I n  (n  2) I n2
I n  (n  2) I n  tan x secn2 x  (n  2) I n2
(n  1) I n  tan x secn2 x  (n  2) I n2

In 
1
(n  1)
 
tan x secn2 x  (n  2) I n2 .......... .......... .......... ......( ii)

Equate equation (i) and equation (ii)


1

 sec xdx  (n  1) tan x sec x  (n  2) I n2  C
n n2

 sec
n
xdx 
1
(n  1)

tan x secn  2 x  (n  2) I n  2  C 
709
Example 152
By using reduction formula integrate  sec2 xdx
Solution

 sec
2
xdx

From  secn xdx  n11 tan x secn2 x  n  2I n2 
 sec xdx  tan x sec x  2  2I 22 
n 1 2 2
21

  sec xdx  tan x  c


2

Example 153
By using reduction formula integrate  sec4 xdx
Solution
 sec
4
xdx.......... .......... .......... .......... .......... .......... ......( i )
From  secn xdx  1
n 1 tan x sec n2
x  n  2 I n2 
 sec
4
xdx  1
41 tan x sec 4 2
x  4  2 I 42 
 sec
4
 
xdx  13 tan x sec2 x  2 I 2 .......... .......... .......... .....( ii)
I 2   sec2 xdx  tan x.......... .......... .......... .......... .........( iii)
Substitute equation (iii) into equation (ii)

 sec xdx  tan x sec x  2 tan x c


4 1 2
3

  sec xdx  tan x sec x  tan x  c


4 1
3
2 2
3

710
(f) Reduction formula for  cosecn xdx
Let I n   cosec n xdx.......... .......... .......... .........( i )
I n   cosec n2 x cosec 2 xdx
Let u  cosec n2 x, du  (n  2) cosec n3 x cosecx cot xdx  (n  2) cosec n2 x cot xdx
dv
 cosec 2 x , dv  cosec 2 xdx ,  dv   cosec 2 xdx, v   cot x
dx
From integration by parts I n  uv   vdu

I n   cot x cosec n2 x    cot x   (n  2) cosec n2 x cot xdx dx 
I n   cot x cosec n2 x  (n  2)  cot2 x cosec n2 xdx
I n   cot x cosec n2 x  (n  2)  cosec 2 x  1cosec n2 xdx

I n   cot x cosec n2 x  (n  2)  cosec n xdx   cosec n2 xdx 
I n   cot x cosec n2 x  (n  2)I n  I n2 
I n   cot x cosec n2 x  (n  2) I n  (n  2) I n2
I n  (n  2) I n   cot x cosec n2 x  (n  2) I n2
(n  1) I n   cot x cosec n2 x  (n  2) I n2

In 
1
(n  1)
 
 cot x cosec n2 x  (n  2) I n2 .......... .......... .......... ......( ii)

Equate equation (i) and equation (ii)

 cos ec n
xdx 
1
(n  1)

 cot x cosec n2 x  (n  2) I n2  C 

 cos ec xdx 
n 1
(n  1)

 cot x cos ecn 2 x  (n  2) I n2  C 

711
(II) DEFINITE INTEGRALS

Definite integrals are integrals defined in a certain limit (range) for instance
integral from point ‘a’ to point ‘b’.
b
Definite integral is denoted by  f ( x)dx where ‘a’ is lower limit and ‘b’ is
a

upper limit in most cases a  b .

THE FORMULA USED TO EVALUATE DEFINITE INTEGRAL


b
(a)  f ( x)dx  F (b)  F (a)
a
b b
(b)  kf ( x)dx  k  f ( x)dx  k F (b)  F (a)
a a
b b b
(c)   f ( x)  g ( x)dx   f ( x)dx   g ( x)dx
a a a
b a
(d)  f ( x)dx   f ( x)dx
a b
a
(e)  f ( x)dx  F (a)  F (a)  0
a
a 0 a a
(f)  f ( x)dx   f ( x)dx   f ( x)dx  2 f ( x)dx
a a 0 0

Example 154

 3x 
2
Evaluate 2
 2 dx
1

Solution

 3x   
2
2
2
 2 dx  x 3  2 x 1  12  3  9
1

 
2
  3 x 2  2 dx  9
1

712
Example 155
2x 1
3
Integrate x
1
2
x
dx

Solution
2x 1
3

1 x 2  x dx.......... .......... .......... .......... .......... .......... .......... ......( i)


Let u  x 2  x, du  2 x  1dx, dx 
du
.......... .......... .....( ii)
2x 1
Substitute equation (ii) into equation (i)
2x 1  2 x  1  du
3
1
x
1
2
x
dx    
 u  2x 1
  du  Inu  c but u  x 2  x
u
2x 1
   
3

1 x 2  x  In10  In 2  In 102   In 5
3
dx  In x 2
 x 1

2x 1
3
 dx  In 5
1
x2  x

Example 156

2

Integrate  2 cos2 xdx



4

Solution

2

 2 cos2 xdx.......... .......... .......... .......... .......... .......... (i)



4

du
Let u  2 x, du  2dx, dx  .......... .......... ......( ii)
2
du
 2 cosu 2 
 cosudu  sin u but u  2 x

  2 cos 2 xdx  sin 2 x  24  sin   sin  2  1


2


4

713
10.13 APPLICATION OF INTEGRATION

10.13.1 AREA
(a) Area under the curve
Consider the area under the curve with x  axis below,

From the figure above subdivide the given area under the curve into very small equal
rectangles with length y  f ( x) and width dx as shown in the figure above
Area of rectangle  length  width
A1  f ( x)dx, A2  f ( x)dx, A3  f ( x)dx ... A6  f ( x)dx
Sum the above areas by integrating from x1 and x2
x2

Area   f ( x)dx
x1

x2

Area   f ( x)dx
x1

Example 157
Find the area under the curve y  x 2 from x  1 to x  2 .
Solution

714
Consider the sketch below

 
x2 2
1 32 7
Area( A)   ydx   x 2 dx  x 1  sq.units
x1 1
3 3
7
 Area( A)  sq.units
3

Example 158
Find the area enclosed between the two curves y  4  x 2 and y  x 2  2 x
Solution
Consider the sketch below

At points of intersection the curves share the same coordinates


i.e. x 2  2 x  4  x 2
2x 2  2x  4  0
x  1, x  2

 4  x   x 
 2 x  dx   4  2 x  2 x dx  4 x  x 
x2 2 2
Area  A    y1  y 2 dx 
2
2 2 2 2
 23 x 3 1
x1 11 11

 Area  A  9 sq.units

715
Example 159
Find the area bounded by y  2 x  x 2 and x axis.
Solution

x  intercepts occurs when y  0


0  2x  x 2
x  0, x  2
2
 2 x3 
 
x2 2
4
 Area   ydx   2 x  x dx   x    sq.units
2

x1 0  3 0 3

Also consider the area under the curve with y-axis below

716
From the figure above subdivide the given area under the curve into very small equal
rectangles with length x  f ( y ) and width dy as shown in the figure above
Area of rectangle  length  width
A1  f ( y )dy, A2  f ( y )dy, A3  f ( y )dy ... A6  f ( y )dy
Sum the above areas by integrating from y1 and y 2
y2

Area   f ( y)dy
y1

y2

Area   f ( y)dy
y1

Example 160
Find the area under the curve y  x 2 for which x-is positive from y  1
to y  4 .
Solution
Consider the figure below

y2

Area   xdy
y1

From, y  x 2 , x  y

 y   143 sq.units
4
2
Area   y dy 
4
1
1
3
14
 Area  sq.units
3

717
Example 161
Find the area bounded by y 2  2 y  x and y  axis.
Solution

y  intercepts occurs when x  0


0  2y  y2
y  0, y  2
2
 2 y3 
 
y2 2
4
 Area   ydx   2 y  y dx   y    sq.units
2

y1 0  3 0 3

Example 162
Find the area under the curve y  x 2  1 , x  0 from y  1 to y  0 .
Solution
Consider the figure below;

y  x 2  1, x   y  1 but x  0

 
y2 0
2 
0
2
A   xdy  
3
y  1dy   y 1  
y1 1 3  1 3
 Area  23 sq.units

718
(b) Area enclosed between two curves
Consider the area enclosed by the curves f ( x) and g ( x) from x1 to x2 as shown
below,

From the figure above the area enclosed between the curves f ( x) and g ( x)
where f ( x) is upper function and g ( x) is lower function is given by
x2 x2 x2

Area    f ( x)  g ( x)dx or
x1
Area   f ( x)dx   g ( x)dx
x1 x1

The area enclosed between curves is x2

Area    f ( x)  g ( x)dx
x1

Example 163
Find the area enclosed by the curves f ( x)  x 2  1 and g ( x)  x  1 .
Solution

Required the area enclosed between curves f x  x  1 and g x   x  1
2

At point of contact the two curves share the same coordinates f x   g x ;


x 2 1  x  1
x2  x  2  0
x  1, x  2

719
Consider the figure below;

 g x   f x dx   x  1  x   
x2 2 2
Area  A  2
 1 dx  2  x  x 2 dx  4.5 sq.units
x1 1 1

 The area enclosed between curves is 4.5 sq.units

Example 164
Find the area bounded by y 2  16x and x  4 .
Solution
Consider y 2  16x
y  4 x y  4 x and y  4 x 
Consider the figure below;

Consider y  4 x

 x   643 .......... .......... .......... ...(i)


4
8
A1   4 x dx 
4
0
0
3
Also consider

   x
4
8 64
A2    4 x dx  
4
0  .......... .......... ......( ii)
0
3 3
64 64 128
A  A1  A2    sq.unit
3 3 3
128
 The area enclosed between curves is sq.unit
3

720
Example 165
Draw a rough sketch and find the area bounded by y  sin x
and x  axis between x  0 and x  2
Solution
Consider the sketch below;

 2
A   sin xdx   sin xdx   cos x  0   cos x    4 sq.units
 2

0 

 The area is 4 sq.units

Example 166
Calculate the area of the plane region bounded by the circle x 2  y 2  9 leaving
your answer in form of  .
Solution
Consider the figure below;

721
Consider x 2  y 2  9
y   9  x2
Consider y  9  x 2
3
A1  
3
9  x 2 dx  4.5 sq.units.......... .......... ......( i )

Also consider y   9  x 2

  
3
A2  9  x 2 dx  4.5 sq.units.......... .......... .(ii)
3

The total area  4.5  4.5  9 sq.units


 The area enclosed by the curve x 2  y 2  9 is 9 sq.units

Consider the area enclosed by the curves f ( y) and g ( y) from y1 to y2 as shown


below,

From the figure above the area enclosed between the curves f ( y ) and g ( y )
where f ( y ) is right hand side function and g ( y ) is left hand side function
y2 y2 y2

is given by Area    f ( y)  g ( y)dy or


y1
Area   f ( y)dy   g ( y)dy
y1 y1

y2
The area enclosed by the curves is Area    f ( y)  g ( y)dy
y1

722
Example 167
Find the area enclosed by the curves x  y 2  1 and x  y  1 .
Solution
At point of contact all curves share the same coordinates
y2 1  y 1
y2  y  2  0
y  2, y  1
When y  2, x  3
When y  1, x  0

y2

A   f  y   g  y dx
y1

  y  1  y 
2
A 2
 1 dx
1

 2  y  y dx  2 y  
2
2
A 2 1
2 y 2  13 y 3 1
 4.5sq.units
1

 The area enclosedis 4.5sq.units

723
(c) Area of a sector in polar form
Consider the figure below which show a sector from 1 to  2

Consider a sector above with small angle d approaching to zero,


If the angle is small an arc PQ approaches line PQ this is why we consider
the sector OPQ as a triangle OPQ.
Area of triangle OPQ  12 r r  dr sin d but sin d  d
dA  12 r r  dr d
dA  12 r rd  drd  but drd  0
dA  12 r 2 d this is a small area for a small sector cut from the large sector.
Integrate from1 to  2 to obtain the total area of a large sector
2 2

 dA  

1
2 r 2 d
1 1

2
A 

1
2 r 2 d
1

Area of the sector in polar form is 2


A   12 r 2 d
1
Example 168
Find the area swept out by radius vector of the equiangular spiral r  aek as 
increases from   to  .
Solution
2
From A  

1
2 r 2 d but r  aek
1

  
 e 2 k 
 ae  d  e
k 2 2 k
A 1
2
1
2 a 2
d  a 
1
2
2

   2k  
a 2  e 2k  e 2k  a 2
A
a 2 2 k
e  
 
a 2 2k
e e
  2k
 
2k 

  2k sinh2k 
4k 4k 2 
2
sinh2k  sq.units
a
 The area swept is
2k

724
(d) Area of a sector in parametric form
Consider the figure below which show a sector from t1 to t 2

Consider a triangle OPQ above, determine small change of area


0 0 1
1   xdy  ydx
1 1
dA  x y
2 2
x  dx y  dy 1
1
dA  xdy  ydx divide by dt to each term
2
dA 1  dy dx 
 x  y 
dt 2  dt dt 
1  dy dx 
dA   x  y dt
2  dt dt 
Sum the area from t1 and t 2
t2 t2
1  dy dx 
 dA   2  x dt  y dt dt
t1 t1

1  d  y 
t2 t
1  dy dx  2

A    x  y dt  A   x 2    dt
t1 
2 dt dt  t1
2  dt  x 
t2
1  dy dx 
The area of a sector in parametric form is A  2  x dt  y dt dt
t1

725
Example 169
Show that the area of a sector in parametric form is given by
t2
1  dy dx 
A x  y dt
t1
2  dt dt 
Solution
Consider the figure below;

Consider a triangle OPQ above, determine small change of area


0 0 1
1  xdy  ydx
1 1
dA  x y
2 2
x  dx y  dy 1
1
dA  xdy  ydx divide by dt to each term
2
dA 1  dy dx 
 x  y 
dt 2  dt dt 
1  dy dx 
dA   x  y dt
2  dt dt 
Sum the area from t1 and t 2
t2 t2
1  dy dx 
 dA   2  x dt  y dt dt
t1 t1

1  d  y 
t2 t
1  dy dx  2

A    x  y dt  A   x 2    dt
t1 
2 dt dt  t1
2  dt  x 

726
1 2 d  y 
t2 t
2
1  dy dx 
A   x    dt    x  y dt
t1
2  dt  x   t1
2  dt dt 
t2
1  dy dx 
 A    x  y dt
t1
2  dt dt 

Example 170
Show that the area of a sector in parametric form is given by
1 2 d  y 
t2

A  x    dt
t1
2  dt  x  
Solution
Refer example 169

10.13.2 LENGTH
(a) Length of the curve (arc length) in Cartesian form
Consider the figure below

727
From the figure above PNQ is very small where length of arc PQ is approximat e
equal to the length of line PQ
Apply Pythagoras Theorem to PNQ
2 2 2
PQ  PN  NQ
ds2  dx2  dy2 divide by dx2 to each term
ds2  dx2  dy2
dx2 dx2 dx2
2 2
 ds   dy 
   1  
 dx   dx 
2
 dy 
ds  1    dx
 dx 
Integrate both sides from x1 to x2 to obtain the total length
x2 x2 2
 dy 
 ds  
x1 x1
1    dx
 dx 
x2 2
 dy 
S 1    dx
x1  dx 
x2 2
 dy 
The length of the curve is given by
S 
x1
1    dx
 dx 

Example 171
Find the length of the curve y  cosh x between x  0 and x  In 2 .
Solution
dy
Given that y  cosh x,  sinh x
dx
x2 2
 dy 
Length, L    1    dx
x1  dx 
In 2 In 2
L  1  sinh x dx   cosh xdx  sinh x 0  0.75 unit
2 In 2

0 0

 The length of the curve is 0.75 unit

728
Example 172
Find the length of the curve y  12 x from x  0 to x  1
2

Solution
Given that y  12 x 2
dy
x
dx
x2 2
 dy 
1
L 
x1
1    dx   1  x 2 dx
 dx  0

By using calculator
1
L   1  x 2 dx  1.3333
0

 The length of the curve is 43 units

Example 173
Find the length of the curve x  12 y from y  0 to y  1
2

Solution
Given that x  12 y 2
dx
y
dy
2
 dx 
y2 1
L   1    dy   1  y 2 dy
y1  dy  0

By using calculator
1
L   1  y 2 dy  1.3333
0

 The length of the curve is 43 units

729
(b) Length of a curve in parametric form
Consider the figure below,

From the figure above PNQ is very small where length of arc PQ is approximat e
equal to the length of line PQ
Apply Pythagoras Theorem to PNQ
2 2 2
PQ  PN  NQ
ds2  dx2  dy2 divide by dt 2 to each term
ds2  dx2  dy2
dt 2 dt 2 dt 2
2 2 2
 ds   dx   dy 
     
 dt   dt   dt 
2 2
 dx   dy 
ds       dt
 dt   dt 
Integrate both sides from t1 to t 2 to obtain the total length
t2 t2 2 2
 dx   dy 
 ds  
t1 t1
   
 dt   dt 
dt

t2 2 2
 dx   dy 
S     dt
 dt   dt  t2 2 2
 dx   dy 
t1

The length of the curve in parametric form S      dt


t1  dt   dt 

730
Example 174
Find the length of a curve given by x  a cos t and y  a sin t from t  0 to
t 3.
Solution
dx
x  a cost ,  a sin t
dt
dy
y  a sin t ,  a cost
dt
t2 2 2
 dx   dy 
From length formula, L        dt
t1  dt   dt 

 
3 3 3
L  a sin t   a cost  dt   a sin t  cos t dt  a  dt  at 0  3a unit
2 2 2 2 2 3

0 0 0

 The length of the curve is 3a unit

Example 175
Show that the length of a curve in parametric form is given by
t2 2 2
 dx   dy 
t  dt    dt  dt
1

Solution
Consider the figure in the previous page;

731
(c) Length of a curve in polar form

Consider the figure below

732
From the figure above PNQ is very small where length of arc PQ is approximat e
equal to the length of line PQ
Apply Pythagoras Theorem to PNQ
2 2 2
PQ  NP  NQ
ds2  NP  dr  .......... .......... .......... .......... .......... .......... .......... ......(i)
2 2

NP
From sin d  , NP  r sin d but sin d  d  NP  rd .......... ..(ii)
r
Substitute equation (ii) into equation (i)
ds2  rd 2  dr 2
ds2  r 2 d 2  dr 2
2  d 
2 2 2
 ds   dr 
  r    
 d   d   d 
2 2
 ds   dr 
   r  
2

 d   d 
2
 dr 
ds  r    d
2

 d 
Integrate both sides from1 to  2 to obtain the total length
2 2 2
 dr 
 ds   r   d
2

1 1
 d 
2 2
 dr 
S r   d
2

1  d 
2 2
 dr 
S  r   d
2
Length of the curve in polar form 1  d 

Example 176
Find the length of the spiral r  a from   0 to    .
Solution

733
dr
Given that , r  a , a
d
2 2
 dr 
From, L   r    d
2

1  d 

 
  
a
L   a   a d  a 
2 2 2
1   d   1   2  sinh 1 
2
0
0 0
2

 The length of the spiral is


a
2
 
 1   2  sinh 1  units

10.13.3 VOLUME OF SOLIDS OF REVOLUTION


The volume of solids revolution is the figure formed after rotating the curve
along the given axis (or line) through a certain angle.

(a) Volume of solid of revolution along x-axis


Consider the area between the curve y  f x  and x  axis under the interval
x1  x  x2 is rotated completely about 360 along x  axis tend to generate

solid of revolution as shown below;

The volume of solid revolution formed after rotating the curve y is obtained as follow;
x2 x2

 dv   y
2
dx
x1 x1
x2

V   y 2 dx x2
x1 V   y 2 dx
The volume of solid revolution along x  axis x1

734
Example 177
The region defined by x 2  y 2  9 , y  0 is rotated completely about the x-
axis. Find the volume of the sphere generated.
Solution
Consider x 2  y 2  32
Consider the figures below which show region before and after rotation

From x 2  y 2  9, y 2  9  x 2

 
x2 3
V   y dx    9  x 2 dx
2

x1 3


V   9 x  x3
3

3
3
 12
V  12 cubic units
 The volume is less than or equal to 12 cubic units

(b) Volume of solid of revolution along y-axis


Consider the area between the curve x  f  y  and y  axis under the interval
y1  y  y2 is rotated completely about 360 along y  axis tend to generate

solid of revolution as shown below;

735
The volume of solid revolution formed after rotating the curve y is obtained as follow;
y2 y2

 dv   x dy
2

y1 y1
y2

V   x 2 dy
y2
y1
V   x 2 dy
The volume of solid revolution along x  axis y1

Example 178
The region defined by x 2  y 2  9 , x  0 is rotated completely about the y-
axis. Find the volume of the sphere generated.
Solution
Consider x 2  y 2  32
Consider the figures below which show region before and after rotation

From x 2  y 2  9, x 2  9  y 2

 
y2 3
V   x dy    9  y 2 dy
2

y1 3

V   9y  
y3
3
3 3  12
V  12 cubic units
 The volume is less than or equal to 12 cubic units

736
(c) Volume of solid of revolution along the horizontal line.
Consider the rotation of solid of revolution along the horizontal line y  y1

The volume of solid revolution formed after rotating the curve y about the horizontal line;
x2 x2

 dv     y  y  dx
2
1
x1 x1
x2

V     y  y1  dx
2

x1
x2

V     y  y1  dx
2

The volume is x1

Example 179
The region enclosed by the curve y  x 2  2 x and y  3 is rotated about y  3 .
Find the resulting volume.
Solution
Determine the point of intersection
x2  2x  3
x2  2x  3  0
x  1, x  3
Consider the sketch below;

737
x2

V     y1  y 2  dx
2

x1

512
  dx    9  12x  2 x 
3 3
V   3  2x  x 2 2 2
 4 x 3  x 4 dx 
1 1
15
512
 The volume is cubic units
15

(d) Volume of solid of revolution along the vertical line.


Consider the rotation of solid of revolution along the vertical line x  x1

The volume of solid revolution formed after rotating the curve x about the vertical line;
y2 y2

 dv    x  x  dy
2
1
y1 y1
y2

V    x  x1  dy
2

y1
y2

V    x  x1  dy
2
The volume is
y1

Example 180
The region enclosed by the curve x  y 2 and x  2  y 2 is rotated about x  3 .
Find the resulting volume.
Solution

738
Equate the curve y 2  2  y 2
y 2  1, y  1
The region is rotated about x  3
Consider the sketch below;

y2 y2

V    x  x1  dy    x  x2  dy
2 2

y1 y1

  dy    3  2  y  dy
1 1
V   3  y 2 2 2 2

1 1

  dy    1  y  dy
1 1
V   3  y 2 2 2 2

1 1

32
 
1
V  8  1  y 2 dy 
1
3
32
 The volume is cubic units
3

739
(e) Rotation of solid of revolution between two curves along x-axis
When two curves such that one at the lower and other at the top are rotated
completely about x  axis tend to produce an enclosed area between the
curves.

x2

Volume of upper curve V2    y 2 dx.......... .......... .......... ..(i )


2

x1
x2

Volume of lower curve V1    y1 dx.......... .......... .......... ....( ii)
2

x1

Volume of solid of revolution between two curves V   V2  V1


x2 x2

V   y 2 dx   y1 dx
2 2

x1 x1

 
x2

V    y 2  y1 dx
2 2

x1

Example 181
Find the volume of revolution formed when the area bounded by the graph
y  x 2  3 and y   x 2  5 is rotated through 2 radian about the x  axis .
Solution
Given that y  x 2  3 and y   x 2  5
Points of contacts x 2  3   x 2  5
x 2  1, x  1
Consider the figure below which is rotated completely about x  axis

740
          64
1 1 1
V    y1  y 2 dx     x  5  x  3 dx     x 2  5  x 2  3 dx 
2 2 2 2 2 2 2 2

1 1 1
3
64
 The volume of revolution formed is cubic units
3

(f) Rotation of solid of revolution between two curves along y-axis


When two curves such that one at the lower and other at the top are rotated
completely about y  axis tend to produce an enclosed area between the
curves.

y2

Volume of outer curve V2    x 2 dx.......... .......... .......... ..(i )


2

y1
y2

Volume of inner curve V1    x1 dx.......... .......... .......... ....( ii)
2

y1

Volume of solid of revolution between two curves V   V2  V1


y2 y2

V   x1 dx   x1 dx
2 2

y1 y1

 
y2

V    x 2  x1 dx
2 2

y1

(g) Rotation of solid of revolution between two curves along a line.


Consider two given curves y 3 and y 2 along y1
x2

Volume of upper curve V2      y3  y1  dx.......... .......... .......... ..(i )


2

x1
x2

Volume of lower curve V1      y 2  y1  dx.......... .......... .......... ....( ii)


2

x1

Volume of solid of revolution between two curves V   V2  V1

 
x2

V     y3  y1    y 2  y1  dx
2 2

x1

741
Example 182
Find the volume of revolution formed when the area bounded by the graph
x  y 2  3 and x   y 2  5 is rotated through 2 radian about the y  axis .
Solution
At points of intersections the curves share the same coordinate
y2  3   y2  5
2 y 2  2  0
y  1
When y  1, x  4
When y  1, x  4

      64
y1 1
V   x1  x2 dy    y 2  5  y 2  3 dy 
2 2 2 2
cubic units
y1 1
3
64
 The volume of revolution formed is cubic units
3

GENERAL EXAMPLES

Example 183
x sin 1 x 2
Evaluate  dx
1 x4
Solution
x sin 1 x 2
 1  x 4 dx
2x 1 x4
Let u  sin 1 x 2 , du  dx, dx  du
1 x4 2x
x sin 1 x 2 xu 1 x4 1 1 2
 1  x 4 dx   1  x 4  2 x du  2  udu  4 u  c
x sin 1 x 2
dx  sin 1 x 2   c
1

2

1 x4 4

742
Example 184
4x5  x2 1
Show that  4 dx  2 x  In 2
2
  c
x 1  x  1 
Solution
Consider L.H.S
4x5  4x  4x 4x
 x 4 1   x 4 1  
       x 2 1 x 2 1    x 2  1 x 2  1 dx
2
dx 4 x dx 4 xdx dx 2 x
     
4x5 4x
 x 4 1    x 2  1 x 2  1 dx.......... .......... .......... .......... .......... .......... .......... ..(i)
2
dx 2 x
  
4x
Consider  2

x 1 x 2  1 
dx

4x 4x
 x 2  1 x 2  1 dx   x  1x  1 x 2  1 dx
    
4x A B Cx  D
  
Partial fraction

x  1x  1 x  1 x  1 x  1 x 2  1
2

   
4 x  Ax  1 x 2  1  Bx  1 x 2  1  Cx  D x  1x  1
A  1, B  1, C  2, D  0
4x 1 1 2x
Then,  dx   dx   dx   2 dx
x  1x  1 x  1
2
 x 1  x 1 x 1

 x  1x  1 x 2  1 dx  Inx  1  Inx  1  In x  1  c


4x
 
2
 

 x  1x  1 x 2  1 dx  Inx  1x  1  In x  1  c


4x
 
2
 
4x

 x  1x  1 x 2  1 dx  In x  1  In x  1  c

2


2
  
4x  x 2 1 
 x  1x  1 x 2  1 dx  In x 2  1   c.......... .......... .......... .......... .......... .......... .......... ....(ii)
 
Substitute equation (ii) into equation (i)
4x5  x 2 1 
  4 dx  2 x  In 2   c
2

x 1  x 1

743
Example 185
1  1  ex  1 
Show that  dx  In c
1 e  
 1 e 1
x x

Solution
Consider L.H.S
1
 1  e x dx.......... .......... .......... .......... .......... .......... .......... .......... .(i)
2udu
Let u  1  e x , u 2  1  e x , dx  x .......... .......... .......... .......... ....( ii)
e
Substitute equation (ii) into equation (i)
1 1 2udu 2 2 2
 1 ex dx   u e x  e x  u 2  1  u  1u  1 du
  du  du 

2 A B
Express in partial fraction  
u  1u  1 u  1 u  1
2  Au  1  Bu  1
A  1, B  1
2 1 1
 
u  1u  1 u  1 u  1
2 1 1
 u  1u  1  u  1  u  1du
du  du 

 u  1u  1du  Inu  1  Inu  1  c


2

2  u 1 
 u  1u  1     c but u  1  e
x
du In
 u 1
1  1  e x 1 
 dx  In c
1 e  
 1 e 1
x x

744
Example 186
1 1 1 1 x 
Using 1 x dx, show that tanh x  In 
2
2 1 x 
Solution
1
Consider  1  x 2 dx
By partial fraction concept
1 1
 1  x 2 dx   1  x 1  x  dx
1 A B
Express in partial fraction  
1  x 1  x  x  1 x  1
1  A( x  1)  B( x  1)
A  12 , B  1
2
1 1
1
Then,  2  2
1  x 1  x  x  1 x  1
1  x 1 
1
dx
1 1
dx
1 1
 1  x 1  x  2  x  1 2  x  1 2
  dx 
1
In  x  1  In  x  1  In 
2  x 1
1 1  x 1 
 1 x 2
dx 
In
2  x 1
  c.......... .......... .......... .......... .......... .......... .......... .......... .(i )

1
Also consider  dx
1 x2
By hyperbolic substitution
Let x  tanh  , dx  sec h 2d
1 1
 1 x2  1  tanh 2  sech d  d  but   tanh x
1
dx  2

1
 1  x 2 dx  tanh x  c.......... .......... .......... .......... .......... .......... .......... .......... ......( ii)
1

Equate equations (i) and (ii)


1  x 1 
 tanh 1 x  In 
2  x 1

745
Example 187
x sin 1 x
Evaluate  dx
1 x 2

Solution
x sin 1 x
 1  x 2 dx
1
Let u  sin 1 x, du  dx, dx  1  x 2 du
1 x 2

x sin 1 x xu
 1 x 2
dx  
1 x 2
 1  x 2 du   xudu   u sin udu.......... ..(i )

By integration by parts  u sin udu  u cosu  sin u  c.......... ........( ii)


Substitute equation (ii) into equation (i)
x sin 1 x
 1 x 2
dx  u cosu  sin u  c but u  sin 1 x

x sin 1 x
   
dx   sin 1 x cos sin 1 x  sin sin 1 x  c 
1 x 2

x sin 1 x
 
dx  x  sin 1 x cos sin 1 x  c 
1 x 2

Example 188
1  Inx
Evaluate  dx
xInx
Solution
1  Inx  1 1 1 1 1
 xInx dx    xInx  x dx   xInx dx   x dx  Inx   xInx dx.......... .......... (i)
1
Consider  dx
xInx
1
Let u  Inx, du  dx, dx  xdu
x

 u du Inu  InInx   c.......... .......... .......... .......... ........( ii)


1 1 1
 xInx  xu
dx  xdu 

Substitute equation (ii) into equation (i)


1  Inx
 dx  Inx  In Inx   c
xInx

746
Example 189
1
Evaluate  xx 5
1 dx
Solution
1
 xx 5  1 dx
du
Let u  x 5  1, du  5 x 4 dx, dx 
5x 4
1 1 du 1 1 1
 xx 5
 1
dx    4 
xu 5 x 5ux 5
du  
5 u u  1
du.......... .......... .......... .......... .(i )

1 1 
 uu  1   u u  1   u  u  1du  Inu  Inu  1
1 1 1
By partial fraction du     du  du 

1  u 
 uu  1 du In u  1 .......... .......... .......... .......... .......... .......... .......... .......... ......( ii)
Substitute equation (ii) into equation (i)
1 1  u  1  x5 1 
 xx 5  1 dx  5 In u  1   5 In x 5   c
1 1  x5 1 
 dx  In 5   c
xx 5  1 5  x 

Example 190
 x  x

Evaluate sin  cos  dx
2

3 6
Solution
 sin cos  dx but cos  dx  cos   1
x 2 x 2 x 1 x
3 6 6 2 3

 sin cos  dx   sin  cos   1dx   sin cos dx   sin dx
x 2 x x 1 x 1 x x x
3 6 3 2 3 2 3 3 3

 sin cos  dx    sin dx   sin dx   cos   3 cos   c


x 2 x 1 1 2x x 1 3 2x x
3 6 2 2 3 3 2 4 3 3

  sin cos  dx   cos   cos   c


x
3
2 x
6
3
8
2x
3
3
2
x
3

747
Example 191
1  1 
Evaluate  x Inx 1  Inx dx
2

Solution
1  1  1  Inx
 x Inx  Inx   x 2 Inx2 dx
2  1  dx 

1
Let u  Inx, du  dx, dx  xdu
x
1  Inx 1 u 1 u 1 u 1 1
 x 2 Inx2 dx   x 2u 2 xdu   xu 2 du   u 2eu du   u 2eu du   ueu du   u e du   u e du....(i)
 2 u 1 u

Consider  u 2 e u du
By integration by parts  u 2 e u du  u 1e u   u 1e u du.......... .......... .......... .......... .......... .......... ...(ii)
Substitute equation (ii) into equation (i)
1  Inx
 x 2 Inx2 dx  u e   u e du   u e du
1 u 1 u 1 u

1  Inx 1 1
 x 2 Inx2 dx  u e  ueu  xInx  c
1 u

1  1  1
 1  dx  c
x Inx  Inx 
2
xInx

Example 192
1  2x
Evaluate  dx
1 x2
Solution

748
1  2x 1 2x 1
 1 x2
dx  
1 x2
dx  
1 x2
dx  
1 x2
dx  2 1  x 2 .......... ......( i )

1
Consider  dx
1 x 2

Let x  sinh  , dx  coshd


1 1
 1  x 2 dx   1  sinh 2  coshd   d   sinh x.......... .......... .........( ii)
1

Substitute equation (ii) into equation (i)


1  2x
 dx 2 1  x 2  sinh 1 x  c
1 x2

Example 193
x 
Show that  sec xdx  In tan    c
2 4
Solution
Consider L.H.S
1
 sec xdx   cos x dx
1 t 2
Let t  tan  2 , dx 
2dt
x
, cos x 
1 t 2 1 t 2
1  t 2 2dt 2 2
 sec xdx   1  t 2  1  t 2   1  t 2 dt   1  t 1  t  dt
2 A B
Express in partial fraction  
1  t 1  t  1  t 1  t
2  A1  t   B1  t 
A  1, B  1
2 1 1
Then,  
1  t 1  t  1  t 1  t
 1  t 1  t   1  t  1  t dt  In1  t   In1  t   c
2 1 1
dt  dt 

1 t 1  tan  2x  tan 4   tan  2x 


 sec xdx In 1  t  In 1  tan  2x   In 1  tan4 tan  2x   In tan 4  2x   c
  sec xdx  In tan 4  2x   c

749
Example 194
tan x
Evaluate  sin x cos xdx
Solution
tan x
 sin x cos xdx
Let u  tan x , u 2  tan x, 2udu  sec2 xdx, dx  2u cos2 xdu
tan x u 2u 2 cos x 2u 2 2u 2
 sin x cos xdx   sin x cos x  2u cos xdu   sin x du   tan x du  u 2 du  2 du  2u  c
2

tan x
 dx  2 tan x  c
sin x cos x

Example 195
1
Evaluate  xInxInInx dx
Solution
1
 xInxInInx dx.......... .......... .......... .......... .......... .......... .....( i)
1
Let u  Inx, du  dx, dx  xdu
x
1 1 1
 xInxInInx   xuInu
dx   xdu   uInudu.......... .......... ......( ii)
1
Let p  Inu, dp  du, du  udp
u
1 1 1
 uInu  up
du  udp   p dp  Inp  c but p  Inu
 xInxInInx dx  InInu   c but u  Inx
1

dx  InIn Inx   c
1

xInxInInx 

750
Example 196
cosInx 
Evaluate  x
dx

Solution
cosInx 
 x dx
1
Let u  Inx, du  dx, dx  xdu
x
cosInx  cosu
 x dx   x xdu   cosudu  sin u  c
cosInx 
 dx  sinInx   c
x

Example 197
2e x
Evaluate  x dx
e 1
Solution
2e x
 e x  1 dx
Let u  e x  1, du  e x dx
2e x du 1
 u e x  2 u du  2Inu  c

2e x du
u e x

 2 In e x  1  c

751
Example 198
1 x
Evaluate  x
dx

Solution
1 x
 dx Rationalize the numerator
x
1 x 1  x 1  x dx  1  x dx.......... .......... .......... .......... .......... .......... ....(i)
 x
dx  
x1  x   x  x2
1 x  A
d
dx
x  x2  B 
1  x  A1  2 x   B
Equate
1 A B
 1  2 A
A  12 , B  12
Then, 1  x  12 1  2 x   12
1 x 1
1  2 x   12 1  1 2x 1  1 1 
 x  x2
dx   2
x  x2
dx   
2  x  x2
dx   dx  2 x  x 2  
x  x2  2 
dx...(ii)
x  x2 
1
Consider  dx
x  x2
By completing the square x  x 2  14  x  12 
2

1 1 1 1
 x  x2
dx  
1
 x  12 
2
dx  
1
1  2x  12 
2
dx  2
1  2 x  1
2
dx
4 2

Let 2 x  1  tanh  , 2dx  sec h 2d


sec h 2d
  d    tanh 1 2 x  1.......... .......... ........( iii)
1 1
2 dx  2 
1  2 x  1 1  tanh  2
2 2

Substitute equation (iii) into equation (ii)


1 x

 x  x 2 dx  2 2 x  x  tanh 2 x  1  c
1 2 1

1 x
 dx  x  x 2  12 tanh 1 2 x  1  c
xx 2

752
Example 199
e2x
Evaluate  x dx
e 1
Solution
e2x ex   2

 e x  1 dx   e x  1 dx
Let u  e x  1, du  e x dx
e 
x 2
du e 
ex x 2

 e x  1 dx   u e x   u du but e  u  1
x

ex u 1  1 1
 u du  u du  1  u du  du   u du  u  Inu  c
e2x
  x dx  e x  1  In e x  1  c
e 1
 
Example 200
Evaluate  1  1x cosx  Inx dx
Solution
 1  cosx  Inx dx
1
x

Let u  x  Inx, du  1  1x dx

 1  cosu  1     cosudu  sin u  c


1 du
x 1
x

  1  1x cosx  Inx dx  sinx  Inx   c

Example 201
Evaluate  cos x 3sin x dx  
Solution
 cos x3 dx
sin x

Let u  sin x, du  cos xdx

 sin x
 du
 
3u 3sin x
 cos x 3 dx   cos x 3  cos x   3 du  In3  In3  c
u u


  cos x 3 dx 
sin x 3sin x
In3

c

753
Example 202
cos x
Evaluate  x
dx

Solution
cos x
 x dx
Let u  x , u 2  x, 2udu  dx
cos x cosu
 x
dx  
u
 2udu  2 cosudu  2 sin u  c

cos x
 dx  2 sin x  c
x

Example 203
e
x
Evaluate dx
Solution
e
x
dx
Let u  x , u 2  x, 2udu  dx

e dx   e u  2udu  2 ueu du.......... .......... .....( i )


x

Consider  ueu du apply integration by part 

 ue du ue  e u  c.......... .......... .......... .......... .(ii)


u u

Substitute equation (ii) into equation (i)

 e dx  2ue  e   c
x u u

 e dx  2 xe  e  c
x x x

  e dx  2 x  1e  c
x x

Example 204
xex
Evaluate  dx
1  x2

Solution

754
xe x
 1  x 2 dx
Let u  1  x, du  dx
xe x xe x u  1eu 1 du   1  1 eu e 1du  e 1 u 1eu du  u 2eu du  
 1  x 2  u 2
dx  du   u2   u u 2   
xe x
 
 1  x 2 dx  e  u e du   u e du .......... .......... .......... .......... .......... .......... .......... (i)
1 1 u 2 u

Consider  u  2 eu du and then integrate by part

u e du  u 1eu   u 1eu du.......... .......... .......... .......... .......... .......... .......... .......... ......( ii)
2 u

Substitute equation (ii) into equation (i)


xe x

 1  x 2 dx  e  u e du   u e   u e du
1 1 u 1 u 1 u
 
xe x 1 1 u

 1  x 2 dx  e  u e du  u e   u e du
1 u 1 u

 1  x 2 dx  e u e   u e  1  x  e  c
xe x 1 1 u 1 u 1 1 x

xe x ex
 dx  c
1  x 2 1 x

Example 205
e 2t
Evaluate  dt
1  e 4t
Solution
e 2t e 2t
 1  e 4t dt   1  e 2t 2 dt
 
Let u  e 2t , du  2e 2t dt
e 2t e 2t du 1 1 1
 1  e 2t 2  1  u 2 2e 2t 2  1  u 2 2 tan u  c
1
dt    du 
 

e 2t
1 e 4t
1
dt  tan 1 e 2t  c
2
 

755
Example 206
Evaluate  e t e dt
t

Solution
  e dt
t  et
e dt  e t et

Let u  e t , du  e t dt
u du u du
  et  u   e du  e  c
t  et
e dt  ue  ue u u

  e t e dt  e e  c
t t

Example 207
sin x  cos x
Evaluate  dx
1  sin 2 x
Solution
sin x  cos x sin x  cos x sin x  cos x sin x  cos x
 1  sin 2x dx   sin 2 x  2 sin x cos x  cos2 x dx   sin x  cos x2 dx   sin x  cos x dx
sin x  cos x
 dx   Insin x  cos x   c
1  sin 2 x
Example 208
x  12
Integrate  x2  x  5
dx

Solution
x  12
 x  x5
2
dx

Let u  x 2  x  5 , u 2  x 2  x  5, 2udu  2 x  1dx, 2udu  2x  12 dx


u
dx  du
x  12
x  12 x  12 u
 x2  x  5
dx   
u x  12 
du   du  u  c

x  12
 dx  x 2  x  5  c
x2  x  5

Example 209
sin x
Evaluate  sin 4 x dx
756
Solution
sin x sin x 1
 sin 4 xdx   4 sin x cos x cos2 xdx   4 cos x cos2 xdx.......... .......... .......... .......... .........( i)
Multiply by cos x to the numerator and denominator
1 cos x 1 cos x
 4 cos x cos2 x  4 cos2 x cos2 x 4  1  sin 2 x 1  2 sin 2 x dx
dx  dx 

Let u  sin x, du  cos xdx


1 cos x du 1 1 1 1
 
4 1  u 2 1  2u 2  cos x 4  1  u 2 1  2u 2  4  1  u 1  u 1  2u 2 
  du  du

 

sin x 1 1
 sin 4 xdx  4  1  u 1  u  1  2u 1  2u du.......... .......... .......... .......... .......... .......( ii)
  
Express in partial fractions then integrate
sin x 1   12  12 1 1 
 sin 4 x 4  1  u   1  u du   1  2u du   1  2u du
dx  du 
   
sin x 1 1 1 1  1 1 
 sin 4 x 4  2   1  u   1  u    1  2u  1  2u 
dx     du  du   du  du
   
sin x 1  1 u  2  1  2u 
 sin 4 xdx  8 In 1  u  In
8  1  2u 
  c but u  sin x

sin x 1  1  sin x  2  1  2 sin x 


 dx  In  In c
sin 4 x 8  1  sin x  8  1  2 sin x 

Example 210
1
Evaluate  sin x  sin 2 x dx
Solution
1 1 1 sin x
 sin x  sin 2 x dx   sin x  2 sin x cos x dx   sin x1  2 cos x  dx   sin x1  2 cos x  dx
2

1 sin x
 sin x  sin 2 x dx   1  cos x 1  2 cos x  dx
2

Let u  cos x, du   sin xdx


1 sin x sin x du 1
 sin x  sin 2 x dx   1  cos2 x 1  2 cos x  dx   1  u 2 1  2u    sin x   1  u 2 1  2u  du
     
1 1
 sin x  sin 2 x dx   1  u 1  u 1  2u  du

757
Express in partial fractions then integrate
1 1
 12 4

 sin x  sin 2 x dx   1  u du   1  u du   1 32u du


6

1 1 1 1
 sin x  sin 2 x dx  1
6  1 u du  1
2  1 u du  4
3  1  2u du
 sin x  sin 2 x dx   16 In1  u   12 In1  u   23 In1  2u   c
1

dx  23 In 1  2 cos x   16 In 1  cos x   12 In 1  cos x   c


1

sin x  sin 2 x

Example 211
If c   e ax cos bxdx and s   e ax sin bxdx , show that ac  bs  e ax cos bx .
Solution
Consider
c   e ax cosbxdx
By integration by parts
Let u  cosbx, du  b sin bxdx
dv  e ax dx,  dv   e ax dx, v  1a e ax
From  udv  uv   vdu
c  1a e ax cosbx  ba  e ax sin bxdx
ac  e ax cosbx  b  e ax sin bxdx but s   e ax sin bxdx
ac  e ax cosbx  bs
 ac  bs  e ax cosbx

Example 212
If c   e ax cos bxdx and s   e ax sin bxdx , show that as  bc  e ax sin bx .
Solution

758
Example 213
1
If the gradient of the certain function is

7 ex  1  , find the function.
Solution
dy 1

Given that

dx 7 e  1
x

1
dy 

7 ex 1
dx

Integrate both sides
1
 dy   7 e x  1 dx
 
1
y

7 e 1
x
dx

Let u  e x  1, du  e x dx
1 du 1 1
y  x   du
7u e 7 u u  1
du express in partial fractions then integrate
1 1
y 
7 u u  1
1  1 1 1  u 1 
 du  In u  1  Inu   In 
1 1 1
y  du    c
7 u u  1 7  u 1 u  7 7  u 
1  ex 
y  In x c
7  e  1 
1  ex 
 f x   In c
7  e x  1 

759
Example 214
 x sin x 2
Prove that 
0 1  cos2 x
dx 
4
Solution
Consider L.H.S

x sin x
Let I n   dx.......... .......... .......... .......... .......... .......... ..(i )
0
1  cos2 x

In  

  x sin  x dx     x sin x dx     x sin x dx
0
1  cos2   x  0 1  cos2   x  0 1  cos2 x

  x sin x dx    sin x dx   x sin x dx
In  
0
1  cos2 x  1  cos
0
2
x  1  cos
0
2
x
 
 sin x x sin x
In   dx   dx.......... .......... .......... .......... .....( ii)
0
1  cos 2
x 0
1  cos2 x
Add equations (ii) and (i)
  
x sin x  sin x x sin x
2I n   dx   dx   dx
0
1  cos x
2
0
1  cos x
2
0
1  cos2 x

 sin x
2I n   dx
0
1  cos2 x
Let u  cos x, du   sin xdx
 
 sin x du 1
2I n       du
0
1 u 2  sin x 0
1  u 2


2 I n   tan 1 u 
0 
  tan 1 cos x 

0

   
  tan 
2

2I n 1
( 1)  tan 1 1 0      
 4 4 2
2
2I n 
2
 2
In 
4

x sin x 2
 dx 
0
1  cos2 x 4

Example 215
Find the volume if the area bounded by the straight line y  3 x , x  1 and the
x  axis when is rotated 360 about x  axis .
Solution
Consider the figure below;

760
The figure after rotation

x2 1
Volume, V   y dx    3x  dx  3 cubic units
2 2

x1 0

 The volume is 3 cubic units

Example 216
Find the volume if the area bounded by the curve y  x 3  1 , the x  axis and the
limits of x  0 and x  3 is rotated around x  axis .
Solution
Consider the sketch below;

761
And here is a volume generated when we rotate the region around the x-axis;

4983
   
x2 3 3
Volume, V   y dx    x  1 dx    x 6  2 x 3  1 dx 
2 3 2
cubic units
x1 0 0
14
4983
 The volume is cubic units cubic units
14

Example 217
Find the volume of the solid of revolution generated by rotating the curve
y  x 3 between y  0 and y  4 about the y  axis.
Solution
Consider the sketch below;

And here is a volume generated when we rotate the region around the y-axis;

762
y2 4
Volume, V   x dy   y 3 dy  18.999 cubic units
2
2

y1 0

 The volume is 18.999 cubic units cubic units

Example 218
A cup-like object is made by rotating the area between y  2x 2 and
y  x  1 with x  0 around the x  axis . Find the volume of the material
needed to make the cup.
Solution
Determoine the point of intersection
2x 2  x 1
2x 2  x 1  0
x  0.5, x  1  Take the uper value x  1 since x  0
The figure obtained after rotation

763
 
x2

V    y1  y 2 dx
2 2

x1

   dx    x   
1 1
V     x  1  2 x 2 2
 2 x  1  4 x 4 dx
2 2

0 0
1
 x3 4  23
V     x 2  x  x 5   cubic units
 3 5 0 15

Example 219
Determine the length of y  Insec x  between 0  x  4 .
Solution
Given that y  In sec x 
dy
 tan x
dx
 
x2 2
 dy 
4 4

From L   1    dx   1  tan 2 x dx   sec xdx


x1  dx  0 0

 
L  Insec x  tan x0 4  In 1  2

 Length of the curve is In 1  2 units

Example 220
Determine the length of x 
2
 y  1 2 between 1  y  4 .
3

3
Solution
Given that x 
2
 y  1 2
3

3
  y  1 2
dx 1

dy

 
2
 dx 
y2 4 4 4 4

From L   1    dy   1   y  1 dy   1   y  1dy   y dy  y 2 
2 2 3 14
 
1
2

y1  dy  1 1 1
3 1 3
14
 Length of the curve is units
3

764
Example 221
 x1  x dx
n
Integrate
Solution
 x1  x  dx
n

u  1  x, x  1  u, dx  du

 x1  x  dx   1  u u dx   u 
 u n1 dx
n n n

u n1 u n 2
 u dx 
n 1
n
u n 1
 n 2
c

  x1  x  dx 
1  x
n
n 1

1  x
n 2
c
n 1 n 2

Example 222
1
1 
Show that 0 1  x 2
dx 
4
Solution
Consider L.H.S

  
1
1
0 1  x 2
1 1 1 1
dx  tan x 0  tan 1  tan 0 
4
1
1 
 dx 
0
1 x 2
4
Example 223
b
1 
If 0 1  x 2
dx 
2
find the value of b .

Solution
Consider L.H.S

  
b
1
0 1  x 2
1 b 1 1
dx  tan x 0  tan b  tan 0 
2

tan 1 b 
2
 
b  tan    
2
 The possible value of b is 

765
Example 224
8
3
bx
Find b if x
2
2
1
dx  2 In  .
 3
Solution
8
3
bx
2 x 2  1dx  2 In  .......... .......... .......... .......... .......... .......... .......... ........( i)
 3
Consider L.H.S
3 3
bx x
2 x 2  1dx  b2 x  1x  1dx
x
Express into partial fractions
x  1x  1
x A B
 
x  1x  1 x 1 x 1
A  12 , B  1
2
1 1
x
 2
 2

x  1x  1 x 1 x 1
b  1 1 
3 3

dx  In ( x  1)  In ( x  1)2


x b
b dx    
3

2
x  1x  1 2 2  x 1 x 1 2

 
3
bx
2 x 2  1 2
dx 
b
In ( x  1)( x  1) 3
2 
b
In ( x 2
 1)
3
2
2
b 8
3
bx
2 x 2  1 2
dx 
b
In 8  In 3  In 
2  3
b 8
3
bx
x
2
2
1
dx  In .......... .......... .......... .......... .......... .......... .......... .......... (ii)
2  3
Equate equations (i) and (ii)
b 8 8
In   2 In 
2 3  3
b
2
2
b  4

766
Example 225
b
14
If 
1
x dx 
3
find the value of b .

Solution
b
14
Consider 
1
x dx 
3
b
14
 x 2 dx 
1

1
3
2 32
3
x    143
b
1

 
b
 x 3  7
 1
 b  1  7
3

 b  8 3

b 2
b  4

Example 226
Integrate  2  x  dx
4

Solution
 2  x  dx
4

Let u  2  x , x  u  2
dx
du  , 2 x du  dx
2 x

 2  x  dx   u  2 xdu  2 u u  2du  2 u 
4 4 4 5
 2u 4 du

 2  x  dx  2 u  u   c
4 6 5
1 2
6 5

  2  x  dx  2  x   2  x   c
4 6 5
2 4
6 5

767
Example 227
1
b 2
If
0 1 x  x
dx 
3
find the value of b .

Solution
1
b 2

0 1 x  x
dx 
3
Rationalize the denominator
1 x  x
1
1 2
b  dx 
0 1 x  x 1 x  x 3
1 x  x
1
2
b dx 
0
1 x  x 3

 
1
2
b  1  x  x dx 
0
3
1
2 2 3 
b  1  x  2  x 2  
3 2
3 3 0 3
2b
3
 3

1  x  2  x 3 2
1
0
3
2

2b1  x   x  
3 3 1
2 2
0 2
2b2  1  1  2
3
2

b2   2
5
2

2 1 1
b  
 2  2 5 4
4
1
b 
4

768
Example 228
Integrate  tan x dx
Solution
Given that  tan x dx.......... .......... .......... .......... .......... .......... .......... .......... .......... .....( i )
Let u  tan x , u 2  tan x, 2udu  sec2 xdx
2udu 2udu 2u
dx    du.......... .......... .......... .......... .......... .......... .......... (ii)
sec2 x 1  tan 2 x 1  u 4
Substitute equation (ii) into equation (i)
2u 2u 2
 tan xdx   u  1  u 4 du   1  u 4 du
Divide by u 2 to each term
2u 2 2 1 1 1 1

 1  u 4 du   1  u2
u2
du   u2
1  u2
u2
u2
du

1  1u 2 1  1u 2
 tan x dx  
1  u2
u2
du  
1  u2
u2
du.......... .......... .......... .......... .......... .......... ....( iii)

By completing the square to each integral


1  1u 2 1  1u 2
 tan x dx   u 2  2  1u 2  2  u 2  2  1u 2  2 du
 du 
  
1 1 1 1

 tan x dx  
u  1u 
u2
2
2
du  
u  1u 
u2
2
2
du.......... .......... .......... .......... .......... ....( iv)

dp
Let p  u  1u , du 
1  1u 2
dt
Also let t  u  1u , du 
1  1u 2
1  1u 2 dp 1  1u 2 dt
 tan x dx  
p 2  2 1  1u 2  t 2  2 1  1u 2
  

1 1
 tan x dx  
p 2
dp   2
2
t 2
dt

2  1  1  2  1  1 
  tan x dx  tan 1   tan x    tanh 1   tan x    c
2  2 tan x  2  2 tan x 

769
Example 229

x tan x
Integrate  sec x  tan x dx
0

Solution

x tan x
Let I n   dx.......... .......... .......... .......... .......... .......... .......... .......... (i )
0
sec x  tan x
Take x    x

In  

  x  tan   x  
dx  
  x  tan x dx
0
sec  x   tan   x  0
sec x  tan x
 
 tan x x tan x
In   dx   dx.......... .......... .......... .......... .......... ..(ii)
0
sec x  tan x 0
sec x  tan x
Add equations (i) and (ii)

 tan x
2I n   dx
0
sec x  tan x

dx rationalize the denominator 
sin x
2I n   
0
1  sin x
  
sin x 1  sin x sin x  sin 2 x sin x  sin 2 x
2I n     dx    dx    dx
0
1  sin x 1  sin x 0
1  sin 2
x 0
cos 2
x


2 I n    sec x tan x  tan 2 x dx 
0

2 I n   sec x  tan x  x0


2
In  
2

x tan x 2
 dx  
0
sec x  tan x 2

770
Example 230
Find the area enclosed by the lines x  2 y  7 , y  x  1 and 2 x  y  7 .
Solution
Consider the sketch below;

7
2
A   x  1  12 7  x dx   7  2 x   12 7  x dx
3

5
3
2
7
2
A    32 x  52 dx   72  32 x dx
3

5
3
2

    1
7
2
A 3
4 x 2  52 x 5  7
2 x  34 x 2 3
2

3
6
1
 The area enclosed between lines is sq.units
6

Example 231
x
 1  x 2 dx where x  sint
Solution
x
 1  x 2 dx
x  sin t , dx  costdt
x sin t sin t
 1 x2
dx  
1  sin 2 t
 costdt  
cost
 costdt   sin tdt

x
 dx   cost  c
1 x 2

771
Example 232
x2
Evaluate  
x2 1 x2  4
dx

Solution
x2
 x 2  1x 2  4dx let x  t
2

x2 t

 
x  1 x  4 t  1t  4
2 2

t A B
Partialize  
t  1t  4 t  1 t  4
A   13 , B  43
Then,
t 1 1  4 1 
    
t  1t  4 3  t  1  3  t  4 
x2 1 1 4 1
 x 2  1 x 2  4   3  x 2  1 dx  3  x 2  4dx
  
x2 1 1 1  x 
   1
  c

x2 1 x2  4 3
tan x
3 
tan
2

Example 233
x2
Evaluate  
1 x2 4  x2
dx

Solution
x2
 1  x 2 4  x 2 dx let x  t
2

x2 t

 
x  1 x  4 1  t 4  t 
2 2

t A B
Partialize  
1  t 4  t  1  t 4  t
A  13 , B   43
Then,
t 1 1  4 1 
    
1  t 4  t  3  1  t  3  4  t 

772
x2 1 1 4 1
 
1 x2 4  x2
dx 
3  1 x2
 dx 
3  4  x2
dx

x2 1 1 1  x 
  1
  c

1 x2 4  x2 
dx
3
tanh
x
3
tanh
2

Example 234
Integrate  sin x dx
Solution
 sin xdx
2udu 2udu
Let u  sin x , u 2  sin x, 2udu  cos xdx, dx  , dx 
cos x 1  sin 2 x
2udu
dx 
1 u 4
2u 2u 2
 sin x dx   u.
1 u4
du  
1 u 4
du

du   2u 2 1  u 4  2 du.......... .......... .......... .......... .......... ..(i )


2u 2 1
 1 u4
nn  1 2 nn  1n  2 3
From, 1  x   1  nx  x  x  
n

2! 3!
1 4  12  32   12  32  52  2n !u 4 n
1  u   u   3!  u        n 2

4  2
1
4 2 4 3
 1 u 
2 2! n 0 4 n!

2u 2  2n !u 4 n   2n !u 4 n 2  2n !u 4 n 2


 du   2u  n 2 du   2 2 n 2 du   2 n1 2 du
2

1 u  4 n!   2 n!  2 n!


4

2u 2 2n !u 4 n3  C


 1 u4
du  2 n1 2
2 n! 4n  3
2n !sin x 
2 n 3
2u 2 2

 1  u 4 du  2 2n1 n!2 4n  3  C


2n !sin 2 n x
  sin x dx  2 n1 2 C
3
2

2 n! 4n  3

773
Example 235
Integrate  1  sin xdx
Solution
1  sin x 1  sin x
 1  sin x dx  
1

1  sin x
dx

1  sin 2 x cos2 x cos x


 1  sin x dx   1  sin x dx   1  sin x
Let u  sin x, du  cos xdx
cos x du 1
 1  sin x cos x  1  u du
. 

p  1  u , p 2  1  u, 2 pdp  du
1 1
 1  u du   p  2 pdp  2 p  C
 1  sin x dx  2 1  u  2 1  sin x  c
  1  sin x dx  2 1  sin x  c

Alternative

 1  sin x dx   cos2  2x   2 sin 2x cos 2x   sin 2  2x dx  cos 2x   sin 2x 2


 1  sin x dx   cos 2x   sin 2x dx  2sin 2x   cos 2x   c
  1  sin x dx  2sin 2x   cos 2x   c

Alternative
 1  sin xdx   1  cos2  x dx   2 sin 2  x dx  2  sin2  x dx
2

  1  sin x dx  2 cos2  x   C

774
Example 236

2
sin x a

0 sin x  cos x
1
Integrate dx , hence evaluate 0 x  a 2  x 2 dx .
Solution

2
sin x
 sin x  cos x dx
0

Express, sin x  A
d
sin x  cos x   Bsin x  cos x 
dx
sin x  Acos x  sin x   Bsin x  cos x 
Equate coefficient of cos x and sin x
1   A  B

0  A  B
A   12 , B  1
2

Then, sin x   12 cos x  sin x   12 sin x  cos x 


 12 cos x  sin x   12 sin x  cos x 
 
2 2
sin x
0 sin x  cos x dx  0 sin x  cos x
dx 
  
2
sin x 1 2 cos x  sin x 1 2 cos x  sin x
0 sin x  cos x dx  
2 0 sin x  cos x
dx 
2 0 sin x  cos x
dx
 
 1 1  2 1
 
2
sin x
0 sin x  cos x  2    

dx   In sin x  cos x  x  x  In sin x  cos x 2

2  0
0
2


 sin x  cos x dx  2   Insin   cos    0  Insin 0  cos0  4
2
sin x 1
2 2 2
0

2
sin x 
 sin x  cos x dx  4
0

Hence evaluate :
a
1
 x
0 a2  x2
dx

Let x  a cos , dx  a sind , when x  0,  2 and when x  a,  0


 a sin d sin 
a 0 0
1
 x
0 a2  x2
dx  
 a cos  a 2  a 2 cos2 
 
 cos  sin 
d
2 2

775

sin sin 
0 2

But   d   d 
 cos  sin 0
cos  sin 4
2

a
1 
 dx 
0 x  a2  x2 4

Example 237
Integrate  sec2 x cot x dx
Solution
 sec x cot x dx
2

1
 sec
2
x dx
tan x
sec2 x
 tan x
dx

Let u  tan x , u 2  tan x, 2udu  sec2 xdx


sec2 x sec2 x 2udu
 tan x
dx  
u
 2   2du 2u  c
sec x
  sec2 x cot x dx  2 tan x  c

Example 238
Find the volume of the solid of revolution obtained by revolving about the
y-axis the region bounded by the parabola y  4x 2 and the lines x  0 and
y  16
Solution
Consider the figure below;

16
 16   y2   2
  
16 
y2
 y
16
V   x dy     dy   ydy     0  32
16
2
y 0  2

y1 0  4  4 0
4  2  0
8 8
Volume  32 cubic.units

776
Example 239
1  2 x 
  1  x 2 dx
sin
Solution
1  2 x 
 sin  1  x 2 dx.......... .......... .......... .......... ..(i)
 2x 
Let , u  sin 1  2 
1 x 

du 
 
2 1  x 2  2 x2 x 

1
dx

1 x2
2
1  12 xx2   2

du 

2 1 x2  1 x2
dx
1  x  1  x 
2 2 2 2
 4x 2

21  x  1 2
du   dx
1 x2
1 2x  x 2 4

du 
2 1 x 

1 2

dx
1 x  
2
2 2
1 x

du 
2 1 x2 

1 dx
1 x2 1 x2
2
du  dx.......... .......... .......... .......... ......( ii)
1 x2
dv  dx,  dv   dx, v  x
from,  udv  uv   vdu
 2x   2x  2x
 sin 
1
 2 
dx  x sin 1  2 
dx
1 x  1 x  1 x
2

 2x 
  sin 1   dx  x sin 1  2 x 
 2 
 In 1  x 2  c  
1 x  1 x 
2

777
Example 240
x cos x 2  1
Integrate  x2 1
dx

Solution
x cos x 2  1
 x2 1
dx

Let u  x 2  1, u 2  x 2  1 , 2udu  2 xdx


x cos x 2  1 x cos u u
 x2 1
dx  
u
 du
x

 cosudu  sin u  c
x cos x 2  1
 dx  sin x 2  1  c
x 1
2

Example 241
8
Integrate  xx 8
1dx
Solution
8
 x x 8  1 dx
 
du
u  x 8  1, u  1  x 8 , du  8 x 7 dx, dx 
8x 7
8 8 du 1
 xx 18
dx  
  7 
xu 8 x u u  1
du

By partial fractions
1 1 1
 u u  1  u  1  u du
du  du 

 u u  1du  Inu  1  Inu  c


1

8
 
 x x 8  1 dx  In x  In x  1  c
 
8 8
 
8  x8 
 dx  In 8   c

x x8  1  
x  1 

778
Example 242

2

Show that cos x


0 cos x  sin x  1 dx 
1
  2In 2
4
Solution
Express, cos x  A
d
cos x  sin x  1  Bcos x  sin x  1  C
dx
cos x  Acos x  sin x   Bcos x  sin x  1  C
Equates;
1  A  B

0   A  B
0  B  C

A  12 , B  12 , C   12 ,
Then, cos x  1 2 cos x  sin x  1  1 2 cos x  sin x  1  1 2
cos x  sin x   12 cos x  sin x  1  12 dx
 
2 2
cos x 1

0 cos x  sin x  1 dx  
2

0
cos x  sin x  1
cos x  sin x  cos x  sin x  1 dx  1
  
2 2 2
1
 1
2  cos x  sin x  1
0
dx  1 2 
0
cos x  sin x  1
2 0 cos x  sin x  1 dx
 
2 2

In cos x  sin x  1 
cos x 1
0 cos x  sin x  1 dx  1
2
1
2 x 1
2 0 cos x  sin x  1 dx.......( i)

dx and then integrate by t - formular where t  tan  2x 


2
1
Consider 
0
cos x  sin x  1
1 1 2dt 1 t 2 2dt
 cos x  sin x  1 dx   1  t 2 2t  1  t 2   1  t 2  2t  1  t 2  1  t 2
 1
1 t 2 1 t 2

 cos x  sin x  1 dx   1  t dt  In1  t   In1  tan 2x .......... .......... .......... .....( ii)
1 1

Substitute equation (ii) into equation (i)


 1 In cos x  sin x  1  1 x  1 In 1  tan  x  2


2
cos x
0 cos x  sin x  1

dx  2 2 2 2 0


2

dx    2 In 2
cos x 1

0
cos x  sin x  1 4

779
Alternatively
Consider L.H.S
cos2  2x   sin 2  2x 
 
2 2
cos x
0 cos x  sin x  1 dx  0 cos2  2x   sin 2  2x   2 sin 2x cos 2x   cos2  2x   sin 2  2x  dx
cos2  2x   sin 2  2x 
 
2 2
cos x
0 cos x  sin x  1 dx  0 2 cos2  2x   2 sin 2x cos 2x  dx

2
cos x cos 2x   sin 2x cos 2x   sin 2x  dx

2

0 cos x  sin x  1 dx  0 2 cos 2x cos 2x   sin 2x 


1 cos 2x   sin 2x 
  

0 cos x  sin x  1 dx  2 0 cos 2x  dx  2 0 1  tan  2x dx


2 2 2
cos x 1

  
2
cos x 1
0 cos x  sin x  1 2

dx  x  2 In cos x
2 0
2


1   
 
2
cos x
0 cos x  sin x  1 2  2
dx   2 In cos 4 

1   2 

2
cos x
0 cos x  sin x  1 2  2
dx   2 In  
 2 
 


2
cos x 1  1 
0 cos x  sin x  1 2  2  2
dx   2 In 2  In 2 


2
1  
 cos x  sin x  1 dx  2  2  In 2  2 In 2
cos x
0

2
cos x 1  
0 cos x  sin x  1 2  2  In 2
dx 

2

dx    2 In 2
cos x 1

0
cos x  sin x  1 4

780
Example 243
The area enclosed by the curve y  x x  1 4 and the lines x  1 , x  3 and
1

y  0 is rotated through 360 about x  axis . Show that the volume generated is
44 2
.
15
Solution
x2 3 3
From V   y dx    xx  1 dx    x x  1dx
1
2 2

x1 11 11

Let u  x  1, u 2  x  1, x  u 2  1, 2udu  dx

 
3
V    x x  1dx    u 2  1 u  2udu
11

 u5 u3 

V  2  u  u du  2   
4 2

 5 3

   
3
 x 1
5
x 1 
3
44 2
V  2    
 5 3  1 15

44 2
 The volume generated is cubic units
15

Alternatively;
x2 3 3
From V   y dx    xx  1 dx    x x  1dx
1
2 2

x1 11 11

Let u  x  1, u 2  x  1, x  u 2  1, 2udu  dx, limits from 0 to 2

 
3
V    x x  1dx    u 2  1 u  2udu
11

2
u5 u3  44 2
 
2
V  2 u  u du  2    
4 2

0 5 3 0 15
44 2
 The volume generated is cubic units
15

781
Example 244
Find the length of the arc given by x  acos   sin  and
y  asin   cos  between   0 and   2 .
Solution
x  acos   sin  ,  a sin   sin    cos   a cos
dx
d
y  asin    cos ,  acos  cos   sin    a sin 
dy
d
2 2 2 2
 dy   dx 
L      d   a sin   a cos  d
2 2

1
 d   d  0

2 2 2
 2 
L 
2 2
 2 2

a  sin   cos  d   a d  a    2a 2
0 0  2 0
 The length of the arc is 2a 2 units

REVISION EXERCISE 10
1. Evaluate the following;
4x5  x2 1
 x4 1    2   c
2
a) Show that dx 2 x In
 x  1 
1  1  ex  1 
b) Show that  dx  In c
1 e  
 1 e 1
x x

1 1 1 1 x 
c) Using  1 x2 dx, show that tanh x  In 
2 1 x 
x sin 1 x 1
d)  1 x 2
dx (e).  xx 5
1 dx
2. Evaluate the following;
x  xInx  x  2 x 
a)  2
x Inx
dx
3 6

(c). sin  cos  dx (e).  sec3 xdx

1  2x
b)  21 1  1 dx (d).  dx (f)  Inx 2 dx
x Inx  Inx  1 x2
Inx
(g).  (1  x) 2 dx

782
3. Evaluate the following;
mx  n
a)  x 2  px  qdx (c). Show that  sec xdx  In sec x  tan x  c

mx  n x 
b)  dx (b). Show that  sec xdx  In tan    c
x 2  px  q 
r
2 4

4. Evaluate the following;


tan x cosInx  x sin1 x 2
a)  dx (c).  dx (e).  dx
sin x cos x x 1  x4
2e x
 x Inx dx (f).  x dx
1

2 2
b) dx (d).
xInxInInx  e 1
5. Evaluate the following;
1 x e2x
a)  dx (e)  x dx
x e 1
 1
b)  1   cos(x  Inx)dx (f)  e x dx
 x

 cos x3 dx


xe x
c) sin x
(g)  dx
1  x 2
e 2t
(h) 
cos x
d)  (i)  e t e dt
t
dx dt
x 1  e 4t
6. Evaluate the following;
sin x  cos x
a)  1  sin 2 x dx
sin x
b)  sin 4 x dx
1
c)  sin x  sin 2 x dx
1
7. If the gradient of the certain function is

7 ex  1  , find the function.
8. If c   e ax cos bxdx and s   e ax sin bxdx , show that ac  bs  e ax cos bx .
 x sin x 2
9. Prove that 
0 1  cos2 x
dx 
4
.

783
10. Show that;
x2
a) The area under the curve f ( x ) is given by x1
f ( x)dx .
y2
b) The area under the curve f ( y ) is given by  y1
f ( y )dy .
c) The area enclosed by two curves f ( x ) and g ( x) , where f ( x ) is
upper and g ( x) is lower functions respectively is given by

  f ( x)  g ( x)dx .
x2

x1

d) The area enclosed by two curves f ( x ) and g ( x) , where f ( x ) is


right and g ( x) is left functions respectively is given by

  f ( x)  g ( x)dx .
x2

x1
2 1 2
e) Area of a sector in polar form is given by 
1 2
r dx

1  dy dx 

t2
f) Area of a sector in parametric form is given by  x  y dt
t1 2  dt dt 
t2 1 2 d 
or 
t1
x
2  dt
( y x) dt .

11. Find the area under the curve y  x 2 from x  1 to x  2 .
12. Find the area enclosed between the two curves y  4  x 2 and
y  x2  2x .
13. Find the area enclosed by the functions f ( x)  e x and g ( x)  x from
x  1 to x  2 .
14. Find the area under the curve y  x 2 for which x-is positive from
y  1 to y  4 .
15. Find the area enclosed by the curves f ( x)  x 2  1 and g ( x)  x  1 .
16. Find the area enclosed by the curves f ( x)  x 2  1 and g ( x)  x  1
from x  0 to x  3 .
17. Find the area bounded by y 2  16x and x  4 .
18. Find the area bounded by y  2 x  x 2 and x-axis.
19. Draw a rough sketch and find the area bounded by y 2  2 y  x and y-
axis.
20. Draw a rough sketch and find the area bounded by y  x 2  2 , y  x ,
x  0 , and x  3 .
21. Find the area swept out by radius vector of the equiangular spiral
r  ae k as  increase from   to  .

784
22. Draw a rough sketch and find the area bounded by y  sin x and x-axis
between x  0 and x  2 .
23. Draw a rough sketch and find the area bounded by y  sin x and x-axis
3
between x  0 and x  .
2
24. Draw a rough sketch and find the area bounded by y  sin x and x-axis
 3
between x  and x  .
2 2
25. A curve has parametric equation on x  at 2 , y  2at . Find the area
bounded by the curve, x-axis and the coordinates at t  1 and t  2 .
26. Find the area of triangle whose sides are x  2 y  7 , y  x  1 and
2x  y  7 .
27. Find the area enclosed by the lines x  2 y  7 , y  x  1 and 2 x  y  7 .
28. Show that;
a) The length of a curve (arc length in Cartesian form) is given by
2
x2  dy 
x1
1 
 dx
 dx .

b) The length of a curve (arc length in parametric form) is given by
2 2
t2  dx   dy 
t1
     dt .
 dt   dt 
c) The length of a curve (arc length in polar form) is given by
2
2  dr 
 r   d .
2
1  d 

1 2
29. Find the length of a curve y  x from x  0 to x  1 .
2
30. Find the length of a curve defined by y  t 2  1 and x  2t from t  0
to t  1 .
31. Find the length of the curve 6 xy  3  x between the points whose
4

abscissa are 1 and 3.


32. Find the length of the spiral r  a from   0 to    ’
33. Find the length of the curve y  cosh x between x  0 and x  In 2 .
34. Find the length of the curve y  e x  e  x  between x  0 and
1
2
x  In 2 .
35. Use the arc length formula to verify that the length of the straight line
joining the points A(2,1) and B (4,5) is 2 10 unit.

785
36. Find the length of a curve y  x 2 from P (0,0) to Q(2,4) .
37. Find the length of a curve given by x  a cos t and y  a sin t from
t  0 to t  2 .
38. Show that the volume of solids revolution is given by the following
expressions;
x2
(a) Volume(V )   y 2 dx along x-axis from x1 to x2 .
x1

y2
(b) Volume(V )   x 2 dy along y-axis from y1 to y2 .
y1

(c) Volume(V )     y  y1 2 dx along


x2
y  y1 from x1 to x2 .
x1

(d) Volume(V )     x  x1 2 dy along


y2
x  x1 from y1 to y2 .
y1

x2

x1
 
(e) Volume(V )    y 2 2  y12 dx along x-axis from x to x . 1 2

Volume(V )    x  dy along y-axis from y to y .


y2
 x1
2 2
(f) 2 1 2
y1

39. Find the volume of the solid generated by rotating about x-axis the area
3
under y  x from x  0 to x  4 .
4
40. Find the volume of the solid generated about by rotating about the y-
axis area in the first quadrant enclosed by y  x 2 , y  1 , y  4 and
the y  axis .
41. Calculate the area of the plane region bounded by the circle x 2  y 2  9
leaving your answer in form of  .
42. Find the area enclosed by the curve y  x( x  1)( x  2) and the x  axis
between x  0 and x  4.
43. Find the area bounded by the curves y 2  4 x and x 2  4 y .
44. The area included between the parts of the two curves x2  y2  1 and
4x 2  y 2  4for which y is positive is rotated about the x-axis. Find
the volume of the solid thus formed.
45. Find the area between the curves a 7 y  x 8 and a 7 x  y 8 .
46. The area enclosed by the two curves and y 2  8x 13 is rotated
y 2  x3
about the x  axis Find the volume of the solid thus formed.

786
47. The area enclosed by the curves y  2  x 2 , y  3  x 2 and abscissa at
x  0, x  a is rotated about x-axis. Find the volume of the solid so
formed.
48. Find the volume of revolution formed when the area bounded by the
graph y  x 2  3 and y   x 2  5 is rotated th3rough 2 radian about
the x  axis .
49. Integrate  tan xdx

x tan x
50.  sec x  tan x dx
0

51. The region defined by x 2  y 2  a 2 , y  0 is rotated completely about


the x-axis. Find the volume of the sphere generated.
2x 2  1
52. Evaluate  2 2
x x 4
dx
 
53. A certain vessel is formed by revolution of the area bounded by the
curve 5 y
2
 2 x , x  0 about y  axis with its height of 0.2m . Find its
volume.
54. Integrate  sin x dx
sin x
55. Integrate
x
dx

56. Integrate  1  sin x dx


sin 3 x
57. Integrate  cos x  2
dx

58. Integrate  x sin xdx



2
sin x 
59. Prove that 
0 sin x  cos x
dx 
4
b
1
60. By writing x  a cos2   b sin 2  , show that  x  a  b  x 
a
1
2
1
2
dx  

sin 2
61.  cos   sin 4 
4
d

 x Inx dx
2 2
62. Evaluate
63. Find the volume if the area bounded by the straight line y  3 x , x  1
and the x  axis when is rotated 120 about x  axis .

787
ANSWERS
SET

Exercise 2.1
1. (a) B (b)  (c) B  A or B  A

2. (a) B  A (b) A  B (c) B  A (d) A

Exercise 2.2
1. (a) 11customers (b) 78 customers
2. (b) 7 students (c) 8 Boys

LOGIC
Exercise 3.1
1. ~ q  p
2. T
Exercise 3.3
1. Electrical diagram

2.  p  q   r  s   t   w
3. Electrical diagram

788
COORDINATE GEOMETRY I
Exercise 4.1

1
3. 2 or 1
4. 2 x  8 y  11  0

5. 3x 2  4 y 2  14x  11  0

Exercise 4.2
3. x 2  y 2  4 x  6 y  4  0

3. 4 x  3 y  25

REVESION EXERCISE 4

1. Show
2. 4 x  7 y  19  0 and 7 x  4 y  48  0
3. Area  1sq.units
4. B(5,8)
5. 9 x 2  9 y 2  28x  16 y  65  0

1
6. y  3x and y  x
3
7. The question is inconsistent.
8. Circles does not intersect.
9.
65
(a) Dis tan ce  unit
17

rp
(b) Dis tan ce  unit
n 2

789
2
10. tan  
5
11. Lines are 2 x  y  1  0 and 2 x  y  4  0 , Dis tan ce  5 unit
12.  5618  123 41
13. Blue region represent the locus 52  x  12   y  22  102

14. Show
15. Show
16. Prove
17. The equations are x  3 y  3  0 and 3 x  y  11  0
18. 53 7

19. 45 

20. Show
 x2  x  2 
21. Mindpoint   x, 
 2 
22. x 2  y 2  4x  4 y  4  0

23. x 2  y 2  28x  4 y  100  0

24. (3,2)
25. x 2  y 2  22x  4 y  0

26. Show
27. x 2  y 2  3x  3 y  0

28. 5 x 2  5 y 2  60x  126 y  212  0

790
29. Prove
30. Prove
27
31. 4 x  3 y  9  0 , 4 x  3 y  11  0 , Area  sq.units ,
8
121
Area  sq.units
24
32. ax  by  a 2
33. Show
34. Show
35. x 2  4 xy  y 2  0

36. ab( x 2  y 2 )  (b 2  a 2 ) xy  0

37. Show
38. 4 x  3 y  2  0
39. x 2  y 2  10 y  0

40. x  5 y  2  0
41. x 2  y 2  5x  y  4  0

42. x 2  y 2  5x  5 y  0

43. Show
44. Show
45. x 2  y 2  4x  7 y  5  0

46. x 2  y 2  20x  0 , x 2  y 2  4 x  16 y  64  0

47. Show
9
48. y  x
40
40
49. m  
9
50. 3 x 2  3 y 2  8ay  4a 2  0

791
24
51.
5
52. Show
53. 2 x  5 y  18
1
54. units
12
55. centre  (1,3) , radius  13 and Area  13 sq. units
4
56. y  0 and y  x
3
57. 4 x  3 y  25  0 and 24x  7 y  125  0
58. prove
59. show
60. 2 x  y  1  0
61. A line and circle does not intersect.
62. 724sq.units
63. show
 4 8
64. Angle  63 265 , equation of chord is x  2 y  0 ,  ,  and
 5 5
4 8 
 , 
 5 5
65. 3 x  4 y  20  0 and 4 x  3 y  35  0
78. The ratio is 3:1
79. 3x  3 y  9  0

80. x  y  4 x  5 y  4  0 , y  0 and y   9 x .
2 2 40

81. x  4 y  8  0
82. x  3 y  18
135
83. Distance  units
13
84. x  y  4 x  16 y  64  0 and x  y  20x  0
2 2 2 2

85. x  y  2 x  2 y  7  0
2 2

86. t  1 and area 10 sq.units


87. A 1,3 , B2,3

792
FUNCTIONS
Revision Exercise 5
7. x  3

9. x  1, x  0, x  1, x  2

10. (a) x  1, x  4

22. (a) Domain  x : x   32 and x  3


2 
(b) Domain  x : x  1

23. g x   2 x  3 or g x   2 x  6

ALGEBRA
Exercise 6.1
8. 1.504

Exercise 6.2
1 1
1. 
1 x 1 x
5 7
2. x  6   2
x x
1 3
3. 
x  18 x  19
Exercise 6.3
4. (a) n 2 (b) nn  1 (c) 1
4 
n n 2  6n  1  (d) 1
12 n  1n  23n  1
2
5. (a) nn  3 (b)  n  1 (c) 13 nn  1n  2 (d) 16 nn  12n  7 
1 n
2
2 
6. (a) 16 nn  12n  1  2n (b) 23 nn  12n  1 (c) 13 n 4n2  1  
 
7. (a) 14 nn  1 n 2  5n  2 (b) 320
8. (a) 3025 (b) 44064 (c) 2931

793
REVISION EXERCISE 6
01. Show
02. Prove
03. Show
04. q 3  27r 2
05. 2
06. Show
07. p  10 , q  31
08. q  5
09. k  1 , s  1
10. Prove
3 6 4
11.  
x  1 x  1 x  13
2

8 15 6
12.  
x 13 x  14 x 15
4 21 22
13.  
x  26 x  27 x  28
14. Prove
15. Prove
16. Prove
17. Prove
18. Prove
19.
(a) x  1
1
(b) x  or x  16
2
9
(c) x  
10
(d) x  3
(e) x  38 or x  52

20. Prove
21. Prove
22. Prove
23. Sum  1.9990234
24. 75,600

794
x2 x2
25.  4 and 0
x 1 x 1
26. 0  p  4
9
27. Show
28. Show
29.
(a) 5
9
16
(b)
9
(c) 80
27
80
(d)
9
30.
(a) 56
(b) 3
7
(c)
8
(d)  32
31. 2 x 2  4 x  1  0
35
32. q 
4
33. Show
34. Prove
35. Prove
36. k  12
37. p 2  4 p  4  0
38. q2  4 p
39. 4k  12 cannot be negative for whatever values of k .
40. 7 x 3  4 x 2  6 x  3  0
41. Show
42. x 3  4 x 2  25x  28  0
43. Show
44. k  1 , other factors are ( x  2) and ( x  3)

795
45. a  5 and b  8
46. k  5 , x  2 and x  3
47. ( x  2 y )( x  y )( x  3 y )
48. Show
49. Show
50. a  3 , b  7 , c  2
51. p  1 , q  5
52. k   10
3
53. a  13
54. (a) 4 x 2  5 x  11, 29
(b) 2 x 2  3x  9,  32
(c) x 2  3x  28, 0
55. c  5
4
56. x   4 , x   , x   5
3 3 2
57. Prove
58. Prove
59. Prove
60. Prove
61. 2  x
62. Pr ove
63. The condition is a  1 and k  5
64.
(a) For no solution a  3
(b) For more than one solution a  2
(c) For unique solution a  2 , a  3
65. The condition is 5a  2b  c  0

796
TRIGONOMETRY
EXERCISE 7.2
9. (a) 4 x  y  4 (b) xy  a b 2  y 2
2 2
(c) 25x 2  4 y 2  100
(d) 2 x 2  y  1
10. (a) x 2 y 2  4 x 2  36 (b) y  2 x  1 (c) y 1  x 2   2 x
(d) bx  ay   ax  by   a  b
2 22 2
 2

1  x2
11. (a) y  (b) x  y  4  x  y   2x  y 
2
(c)
1 x 2

nx  m y 2  n 2
2 22

(d) nx  my   mx  ny   m  n
2
 2

 1 (b) x 1  y   2a (c) y 1  x   1  x (d)


p2 q2 2 2 2 2 2
12. (a) 2  2
x y
 
y 2 1  x 2  1  x
2

EXERCISE 7.7
1. (a) max. 5 , min  5 (b) max. 13 , min  13 (c) max. 5 2, min
5 2
(d) max. 2 , min  2 (e) max. 5 , min  5
2. (a) max is 5 at 143 7 , min is  5 at  36 53 (d) max is 5 at
126 52 , min is  5 at  538

(b) max is 2 at 30  , min is  2 at 210 (e) max is 7 at

 3218 , min is  7 at 147 42

(c) max is 2 3 at 60  , min is  2 3 at  120


3. (a) max is 2 at 120 , min is  2 at  60
(b) max is 2 3 at  60 , min is  2 3 at  105
4. (a) max. 12 , min 2 (b) max. 23 , min  3
(c) max. 0 , min  26 (d) max. 11, min 1

797
EXERCISE 7.8
1.
(a) 2 sin 5x cos x
(b) 2 cos 2x cos x
(c) 2 sin 5x cos 4x
(d) 2 cos3xsin x
(e)  2 sin 5x sin 2x
2.
(a) 2 sin 2 x cos 2x
(b) 2 cos10x cos 2x
(c)  2 sin 8x sin 3x
  
(d) 2 sin 2 x  15 cos 2 x  15 
(e) 2 cos3x  5 cos2 x  25 
3.
(a) 2 cos2 A  B sin4 A  B 
(b) 2 sin 45 cos2 x  45 
(c) 2 cos2 x  15 cos2 x  15 
  
(d) 2 sin 45  x cos 45  x 
4.
(a) sin 4x  sin 2x
(b) sin 6x  sin 2x
(c) cos9x  cos x
(d) cos10x  cos 4x
(e) sin 9x  sin 7 x
5.
(a) 12 sin10x  sin 2 x 
(b) 12 sin 6 x  sin 4 x 
(c) 12 cos15x  cos 5 x 
(d) 12 cos11x  cos 7 x 
(e)  12 cos 7 x  cos x 
6.
(a)  sin 5x  sin x 
(b)  1
2cos12x  cos 2 x 
(c)  12 cos 5 x  cos 3x 
(d) cos 2x  cos 2x

7.
(a) sin 2 A  sin 2B
798
(b) sin 2 A  sin 2B
(c)  cos 2 x  cos 2 y 
(d) 12 cos A  cos B 
(e) cos   cos 
M P
 2  2

EXERCISE 7.11
1. (a) 0 , 120 , 240 , 360
(b) 60 , 120 , 240 , 300
(c) 45 , 90 , 135 , 225 , 270 , 315
(d) 30 , 150 , 270
2. (a) 43 , 137 , 19032, 349 28
(b) 60 , 120 , 240 , 300
(c) 60 , 300
(d) 5618, 23618
(e) 5032, 23032, 15114, 33114
  n 
3. (a) n   1   and n   1  
n

2 6

(b) 2n  and 2n  
3
  n 
(c) n   1   and n   1  
n

6 2
n n 
(d)   1  
2 4
 n 
4. (a) 2n  and n   1  
2 6

(b) n and 2n 
3
(c) n   1 1738
n

(d) 180 n  63 26 and 180 n  63 26


 
5. (a) n   1  
n

6
 n 
(b) 2n  and n   1  
2 4
(c) 2n, 2n  72 and 2n  14359
799
(d) n
n n 
(e) and 
2 2 6
 
6. (a) 2n  and 2n 
2 6
n
(b) and 90 n  31 43
2
(c) 180 n  35 45 and 180 n  5412

(d) n 
4

(e) n  and 180 n  2633
4
7. (a) 2n

(b) 2n 
3
(c) 180 n  743 and 180 n  6811

 
(d) n   1  
n

2
2
(e) 2n  and 2n  4459
3
 4n 
8. (a) 2n, 2n  and 
2 5 5
n  
(b) and 2n   
3 3

(c) n  and 180 n  5959
4
9. (a) 180 n   1n 30  60

(b) 180 n   1n 3515  30


(c) 2n  3652
(d) 180 n  54 44
10. (a) 60 n  15
(b) 45 n  7.5
(c) 36 n  6
(d) 180 n and 180 n  5959
11. (a) 180 n   1n 2258
(b) 180 n   1n 39 49

(c) 2n 
3
800
n n 
(d) and 
2 2 8

EXERCISE 7. 12
1
1.
5
2. 
cot1 cot1 3 

3.
4
3
4.
4
5. 0 or 45
9.  2
1
12.
6
2
16. 
3
17.  1
21. x  0

REVISION EXERCISE 7
1  tan 
1.
1  tan 
2.
(a) x  1.789
1
(b) x  
3
3. Prove
4. Show
5. Show
6. Prove
7.     90
1
8. cos3  4 cos3   3 cos , x  1 and x  
2
9. The general solutions are;
  2
(a) x  n   1    or x  2n 
n

 6 3
  4n  
(b) x  2n   , x
2 4 20

801

(c)   n 
4
10.
(a) Maximum  5 , Minimum  5
1 1
(b) Maximum  , Minimum 
2 12
11. k  36 sin 2 
12. Prove
13. Prove
14. Show
15. Prove
16. Prove
 7 5 11
17. , , ,
6 6 6 6
18. sin y  0.2 , cos x  0.2 or y  11 32,168 27 and x  78 27,281 32
19. Prove
20. Show
21. Show
22. Prove
23. Show
24. Show
25. Show
26. Show
27. Show
28. 250
29. Show
30. Prove
31. Show
32. Prove
33. Prove
34. Prove
35. Prove
36. Prove
37. 0

802
LINEAR PROGRAMMING
Exercise 8.1
1. In order to maximize profit, the company should manufacture 37 chairs and
27 tables. The maximum profit is 4271000/ 
2. The patient should take no Feelgood pills and 10 Getbetter pills. The cost is
15,450 / 
3. The store owner has to have 1333 toys of type A and 667 toys of type B in
order to maximize his profit.
4. The company needs to produce 2300 tables of type T1 and 600 tables of
type T2 in order to maximize its profit 273000/ 
5. 3.75 bags of food A and 0.75 bags of food B are needed to satisfy the
minimum daily requirements in terms of proteins, minerals and vitamins at
the lowest possible cost.
6. For maximum return, John has to invest $11333 in fund F1, $5667 in fund
F2 and $3000 in fund F3.

Exercise 8.2
1.
80 tons of cement from D1 to C1
40 tons of cement from D1 to C2
0 tons of cement from D2 to C1
10 tons of cement from D2 to C2
2.
5 units from NYEGEZI to DODOMA
0 units from NYEGEZI to SINGIDA
15 units from BAGAMOYO to DODOMA
15 units from BAGAMOYO to SINGIDA

3. (a)
0 tons of bricks from H1 to P
500 tons of bricks from H1 to Q
400 tons of bricks from H1 to R
500 tons of bricks from H 2 to P
100 tons of bricks from H 2 to Q
0 tons of bricks from H 2 to R

(b) The overall minimum cost is 530,000 / 


803
DIFFERENTIATION
EXERCISE 9.1
3 1
3. (a) (b) 2e 2 x (c) e x  1 (d) 1
x 2 x
 x 1 
2

4. (a) 1 (b)   2 
(c) 2x (d) 3 x 2  2 x
xx 

EXERCISE 9.2
1.
dy 2 cos x  3x cos x  sin x 
(a) 
dx 2  3x2
 sin x   10 x cos x sin 9 x
dy 10
(b)
dx
dy 1
(c) 
dx 2 x x
dy
(d)  2 sin 2 xecos 2 x
dx
dy  1  2 tan x
(e)  1   sec x  2
dx  x  x
2.
dy
(a) 1
dx
(b)
dy
dx

 x sin x 2 tan x  x sec2 x  x 
dy e x 1 x 
(c) 
dx 2 xex
dy 1
(d)   In x
dx 2
 In cos x   x tan x
dy
(e)
dx

EXERCISE 9.3

dy sec h 2 x dy sinh x
5. (a)  (b) 
dx 1  tanh 2 x dx 2 x

804
dy
6. (a)  sinh x sec x tan x  sec x cosh x (b)
dx
  
dy sin x cos ech cos x coth cos x


dx 2 cos x

EXERCISE 9.4
dy
1.  4e x 1
dx
2.
dy
dx

 e 2 e x  xe x 
dy 2 xe2 x  2e 2 x
3. 
dx x3
dy
 4 e .e x In 4
x
4.
dx

EXERCISE 9.5
dy
 6t 2  10t 2
5 3
6.
dx
dy  2x  y 
7.   
dx  x  2y 

REVISION EXERCISE 9
1.
x cos x  sin x
(a) f ( x) 
x2
1
(b) f ( x) 
2 x3
1
(c) f ( x) 
x
2. Prove
x x2
3. (c) Inx  In 2  
2 8
32
4. (c) cm / s
27
5. (a) 1%
(b) 0.002045 / min

805
 2z  2
z  2
z
6. (a) 2  2e  4 cos 2 x , 2  6  25x e ,
5y 2 5y
 10xe5 y ,
x y x y
 2z
 10xe5 y
y x
(c) 2cm/ s

7. (a) f ( x)   x  1  x  12  1 x  13  1 x  14


3
2 3 12
 x e 1 x e 1 x e 1 x e
2 3 4

f ( x)  1           
(b)  e  2  e  3  e  4  e 
(c) 0.1319m/ s

8. (b) L
4
9. (b) 2,500 /  Tshs.
10. (b) 0.3113m / min
11. (a) 4%
98
(b) (i) m / hr
225
5
(ii) m / hr
252
1
12. (b) m / sec
3
13. (b) (i) 3.6923m / sec
(ii) 1.6923m / sec
14. (b) 2.0833m / sec
15. (a) r  0.06m
16. (a) length  250m , width  250m
 12 
(b) 2r  2rh , (i) h    r cm , V  r (12  r 2 ) (ii) r  2cm
2

r 
17. Prove
18.
  1 3 1 3 3 1 4
(a) sin  h    h  h2  h  h …,
6  2 2 4 12 48
sin 31  0.5150

806
  3 1 3 2 1 3 3 4
(b) sin  h    h h  h  h …,
6  2 2 4 12 48
sin 61  08746
  3 1 3 2 1 3 3 4
(c) cos  h    h h  h  h …,
6  2 2 4 12 48
cos31  0.8572


(d) cos x  30 

 3 1
 x
2 2 4
3 2 1 3
x  x 
12
3 4
48
x …,
cos33  0.8387
2
19. (a) (ii)
3
(b) 0.01m / sec
1
20. a
6

4
21. (a) V  H 3 tan 2 
27
(b) 2km and 4km

22. (a) 52.5cm 2 / sec


4
23. 45

24.
4
(a) rad / s
15
(b) 2,3
(c) a  1 , b  3 , max . po int(0,2) , min . po int(2,2)
 1 3  1 3
(d)  ,  and   , 
 2 2   2 2
(e) (0, a )
6
(f)
5
25. (a) 15.133m/ s
(b) 70km/ hr
26. 6000km/ hr
27. (a) 0.693

807
36 144
28. (a) cm and cm
4 4
(a) 15 and 45
29. (a) Equation of tangent is 4 x  3 y  25 , normal is 4 x  3 y  0
(b) ii) 0.42%
(c) 6cm
30. Show
7 4 8 3 45 2
31. (b) y  x  x  x  2x  4
124 31 62
dy b3
(c)   3 tan  , Equation of tangent
dx a
ya3 cos  xb3 sin  ab(a 2  b 2 ) sin cos and equation of the normal
is yb3 sin  xa 3 cos  b 4 sin 2   a 4 cos2 

20
32. (b) Width= m
 4
33. show
34. show
5 2
35. f ( x)  x x2
2

 f 
36.    8
g
37. 2 sq. units
38. Px  x 2 xy

 
,
y x
2 2
3
x 2
y 2 2

x x
Py  
 
,
y x
2 2 x 2
 y2 
1 3x 2 2 xy
Pxx   
    x 
3 5 ,
2 2
y x
2 2
y x2 2 2
y
 3xy 2x 2 xy 2
Pxy   
y  x  x 
5 ,
2 2 2 3
2
x 2 2
y 2
y

808
xy 2 xy
Pyy  

and
 y x
2 2
3
x 2
y 
2 2

1 x2 1 2x 2
Pyx 
 y x
2 2
  
y2  x2 
3
 2
x  y 2   x 2  y 2 2 .
39.  8cm 2 / s
40. Show
45. In2  x   In 2  12 x  18 x
2

dy 1

50.
dx 2 1  x 2  
 1 
53. 8 x  2 y  15,   ,8 
 8 

INTEGRATION
Exercise 10.1
3
1. 1
3 x  92 x 2  c
2. 2 tan x  c
3. sinh4 x  5  c
4. 1
4 ax4  bx3  12 cx 2  dx  c

Exercise 10.2
1.  101 cos 5 x  12 cos x  c
2.  16 cos 6 x  14 cos 4 x  c
3. 18 sin 8 x  16 sin 6 x  c
4. 141 cos 7 x  12 cos x  c
5. 14 sin 4 x  12 sin 2 x  c

Exercise 10.3
 tan  2x   1 
1
12. x  2 2 cosh  c
 2 
13. Incos x  sin x  6  c
 tan  2x   1 
14. Incos x  sin x   8 2 coth1  c
 2 

809
15.  53 In cos x  2 sin x   65 x  c
 tan  2x   5 
16. Incos x  5 sin x   x 
7
26 coth 
7
26
7 26
13
1
c
 26 
 7tan  2x   19  
17. 12 x  12 Incos4 x  sin 4 x  10  5 7 2 tan 1  c
 7 2 

Exercise 10.4
 x 
18. cos1  c

 2  x  1 
 x2  2 
19.  tan  1 c
 x 
 
20.  2 tanh
1
 x  3  c
21. x 2  1  cosh1 x  c
 8x 1   x 5
22. 2 2 x  1  2 5 tan    2 x  3  2 5 tan 1 
1
 c
 5   5 
5 1  5 
23. tan  tan x   c
10  2 

REVISION EXERCISE 10
01. Show
02.
(a) In ( xInx)  c
1
(b)  c
xInx
3  2x  3  x 
(c)  cos   cos   c
8   2 3
3
(d) sinh 1 x  2 cosh(sinh1 x)  c

(e)
1
sec x tan x  In sec x  tan x   c
2
(f) xInx  2 xInx 1  c
2

03. Show
810
04.
(a) 2 tan x  c
(b) InIn( Inx)  c
(c) sin( Inx)  c
2  1
x Inx   x 3  Inx    c
1 3 2
(d)
3 9  3
(e)
1
4

sin 1 x 2  c 
(f) 2 In (e x  1)  c
05.
1
(a) sin 2(sin 1 1  x )  sin 1 x  c
2
(b) sin( x  Inx)  c
(c)
In 3
3   c
1 sin x

(d) 2 sin x  c
(e) (e x  1)  In (e x  1)  c
(f) 2e ( x  1)  c
x

ex
(g) c
1 x
(h)
1
2
 
tan 1 e 2t  c
t
(i) ee +c

06.
(a)  In sin x  cos x  c
1 sin x  1 2 1  2 sin x
(b) In  In c
8 sin x  1 8 1  2 sin x
1 1 2
(c) In 1  cos x  In 1  cos x  In 1  2 cos x  c
6 2 3

1  ex 
07. y  In x   c
7  e 1
08. Show
09. Show
10. Show

811
7
11. sq.units
3
12. 9sq.units
13. 5.521sq.units
14
14. sq.units
3
9
15. sq.units
2
31
16. sq.units
6
128
17. sq.units
3
4
18. sq.units
3
4
19. sq.units
3
20. 10.5sq.units
a2
21. sinh 2k sq.units
2k
22. 4 sq.units
23. 3 sq.units
24. 2 sq.units
7 2
25. a units
3
1
26. sq.units
6
1
27. sq.units
6
28. Show
29. 1.1478units
30. 2.296 units
14
31. units
3
32. 2.6197 unit
33. 0.75 unit
34. 0.75 unit
35. Show
36. 1.473 unit
37. 2a unit
812
38. Show
39. 12 cubic units
15
40.  cubic units
2
41. 9 sq.units
42. 16.5 sq.units
16
43. sq.units
3
44. 4 cubic.units
7a 2
45. sq.units
9
46. 2 cubic.units
 2 3
47.   5a  a  cubic.units
 3 
64
48.  cubic units.
3
2  1  1  2  1  1 
49. tan 1   tan x    tan 1   tan x    c
2  2  tan x  2  2  tan x 


50. (  2)
2
1 7 1  x 
  tan    c
52. 4x 8  2

m3
53. 2500
2n!sin2n x  C
3
2

54. 22 n1 n! 4n  3


2

55.  2 cos x  C
56.  2 1  sin x  C

57. 52 cos x  2  85 5
 
3
cos x  2  6 cos x  2  C
61. tan 1 tan 2    c
1 2
62.  Inx   Inx  e
2
C
2 3 Inx

3 9 27 
63.  cubic units

813
INDEX
Disjunction, 76
A
Distance, 104
Angle, 430 Distributive law, 54, 88
Associative law, 54, 88 domain, 40, 192, 198, 199, 201, 202, 203, 204,
Asymptotes, 194, 196, 197 205, 206, 207, 208, 211, 212, 561
Double angle, 374
B Double implication, 78

Bi-condition, 78 E
binomial theorem, 347
BINOMIAL THEOREM, 328 electrical networks, 94
ELECTRICAL NETWORKS, 94
C Empty set, 44
Excel, 38, 39
calculating device, 13, 458 Exclusive disjunction, 80
calculator, xii, 13, 14, 15, 17, 18, 19, 20, 21, 22,
23, 25, 26, 27, 28, 464, 559, 560 F
Centre, 133
Circle, 103 Factor formula, 403
combination, 13, 74, 91, 329, 368 Fallacy, 85
common chord, 160, 180, 183, 184 Finite series, 230
common roots, 299, 350 first principle, 431, 476, 477, 478, 479, 480,
Commutative law, 54, 88 481, 482, 483, 484, 485, 489, 505, 596, 606
computer, xii, 27, 28, 29, 30, 31, 32, 33, 34, 35, function, 13, 192, 194, 195, 196, 197, 198, 199,
36, 38, 73 201, 210, 211, 212, 220, 348, 352, 360, 361,
Concentric circles, 158 362, 395, 397, 449, 459, 460, 472, 474, 475,
Conditional, 77 476, 491, 496, 497, 498, 499, 500, 507, 508,
Conjunction, 75 541, 547, 548, 561, 562, 574, 576, 578, 579,
Contradiction, 85, 91 598, 604, 605, 623, 624, 632, 695, 771, 796
Contrapositive, 86, 87
Converse, 85, 86, 87 H
cos, 506
Half angle, 379, 380
Horizontal asymptotes, 194
D
hyperbolic functions, 18, 523, 528
De-Morgan law, 54
derivative, 431, 488, 491, 492, 493, 494, 496, I
497, 498, 499, 500, 501, 502, 503, 504, 507,
508, 509, 510, 511, 513, 515, 518, 521, 522, Idempotent law, 53, 54, 88
523, 525, 526, 527, 531, 532, 533, 535, 536, Identity law, 54, 88
537, 539, 540, 541, 542, 543, 544, 545, 546, Identity matrix, 321
547, 561, 562, 563, 564, 565, 590, 597,604, Implication, 77
606, 624 Improper subset, 44
Determinant, 23, 322 Inequalities, 310, 311
Diagonal matrix, 321 infinite series, 230

814
integration, 25, 607, 624, 629, 642, 654, 658, P
663, 668, 695
intercept, 186 Parallel, 94, 95, 107
Intersection, 46 Partial Fraction, 220
Inverse, 16, 17, 23, 86, 87, 326 Pascal Triangle, 328, 329, 330
permutation, 13
J Perpendicular, 108
Power set, 45
Joint denial, 80 product rule, 695
Proper subset, 44
L Proposition, 74

Locus, 124
R
Logarithms, 21
Logic, 36, 37, 73, 92 Radians, 430
Radical axis, 160
M Radius, 133, 145
range, xi, 186, 192, 198, 199, 201, 202, 203,
Maple, 40 204, 205, 206, 207, 208, 211, 212, 457, 561,
Mathematical induction, 253 724
Matlab, 40 rational functions, 194, 198
Matrix, 23, 320, 325 Remainder, 300
Matrix method, 325
maximum, 397, 398, 399, 400, 401, 402, 446,
S
451, 454, 456, 460, 462, 464, 472, 561, 562,
563, 568, 571, 572, 573, 593, 596, 598, 599, Set, 53, 62
600, 601, 602, 603, 605, 816 Shortest distance, 153
Midpoint, 105 Sigma notation, 231
minimum, 397, 398, 399, 400, 401, 402, 451, sin, 500
454, 456, 466, 471, 472, 474, 550, 561, 562, Singular matrix, 321
567, 569, 572, 573, 596, 597, 601, 602, 603, special angles, 458
605, 816 SPECIAL ANGLES, 358
SPSS, 39
N Square matrix, 321
Statement, 41, 74, 86
Normal, 17 Subset, 44
number line, 53, 311, 312, 313 summation, 24, 26, 231, 607
NUMBER LINE, 52 Symmetric difference, 47

O T
Oblique asymptotes, 194 tan, 144, 150, 151, 181, 183, 184, 185, 377,
orthogonal, 108, 109, 155, 156, 157, 166, 172, 380, 382, 383, 512, 654
173, 180, 184 Tangent, 139, 142, 362, 417
Tautology, 85, 91
t-formula, 382, 383, 417, 422
Transpose, 23, 325
815
Triple angle, 377 Venn diagram, 50, 65, 68, 69, 70, 72
Trisection, 122 Vertical Asymptotes, 194
volume, 28, 448, 555, 593, 596, 599, 600, 601,
U 602, 603, 605, 746, 747, 748, 749, 750, 752,
754, 772, 773, 774, 775, 788, 794, 799, 800
Union, 45
Universal set, 45
Z
V Zero matrix, 321

Validity, 91, 92

816
BIBLIOGRAPHY
Backhouse, J. K. (1985). Pure Mathematics 1. London: 90 Tottenham Court Road,
London W1P 9HE.

Backhouse, J. K. (1985). Pure Mathematics 2. London: 90 Tottenham Court Road,


London W1P 9HE.

H. K. DASS, D. R. (2005). S. CHAND'S Mathematics Volume I. New Delhi: Nirja


Publishers and Printers.

H. K. DASS, D. R. (2013). S. CHANDS Mathematics Volume II. New Delhi: S.


CHAND School.

TRANTER, C. J. (1975). ADVANCED LEVEL PURE MATHEMATICS. London: 90


Tottenham Court Road,

817

You might also like